File "markSceme-SL-paper2.html"

Path: /IB QUESTIONBANKS/4 Fourth Edition - PAPER/HTML/Mathematical Studies/Topic 6/markSceme-SL-paper2html
File size: 5.14 MB
MIME-type: text/html
Charset: utf-8

 
Open Back
<!DOCTYPE html>
<html>


<meta http-equiv="content-type" content="text/html;charset=utf-8">
<head>
<meta charset="utf-8">
<title>IB Questionbank</title>
<link rel="stylesheet" media="all" href="css/application-746ec5d03ead8d9e8b3bb7d32173f2b4e2e22a05f0c5278e086ab55b3c9c238e.css">
<link rel="stylesheet" media="print" href="css/print-6da094505524acaa25ea39a4dd5d6130a436fc43336c0bb89199951b860e98e9.css">
<script src="js/application-3c91afd8a2942c18d21ed2700e1bdec14ada97f1d3788ae229315e1276d81453.js"></script>
<script type="text/javascript" async src="https://cdnjs.cloudflare.com/ajax/libs/mathjax/2.7.5/MathJax.js?config=TeX-MML-AM_CHTML-full"></script>
<!--[if lt IE 9]>
<script src='https://cdnjs.cloudflare.com/ajax/libs/html5shiv/3.7.3/html5shiv.min.js'></script>
<![endif]-->
<meta name="csrf-param" content="authenticity_token">
<meta name="csrf-token" content="iHF+M3VlRFlNEehLVICYgYgqiF8jIFlzjGNjIwqOK9cFH3ZNdavBJrv/YQpz8vcspoICfQcFHW8kSsHnJsBwfg==">
<link href="favicon.ico" rel="shortcut icon">

</head>
<body class="teacher questions-show">
<div class="navbar navbar-fixed-top">
<div class="navbar-inner">
<div class="container">
<div class="brand">
<div class="inner"><a href="http://ibo.org/">ibo.org</a></div>
</div>
<ul class="nav">
<li>
<a href="../../index.html">Home</a>
</li>
<li class="active dropdown">
<a class="dropdown-toggle" data-toggle="dropdown" href="#">
Questionbanks
<b class="caret"></b>
</a><ul class="dropdown-menu">
<li>
<a href="../../geography.html" target="_blank">DP Geography</a>
</li>
<li>
<a href="../../physics.html" target="_blank">DP Physics</a>
</li>
<li>
<a href="../../chemistry.html" target="_blank">DP Chemistry</a>
</li>
<li>
<a href="../../biology.html" target="_blank">DP Biology</a>
</li>
<li>
<a href="../../furtherMath.html" target="_blank">DP Further Mathematics HL</a>
</li>
<li>
<a href="../../mathHL.html" target="_blank">DP Mathematics HL</a>
</li>
<li>
<a href="../../mathSL.html" target="_blank">DP Mathematics SL</a>
</li>
<li>
<a href="../../mathStudies.html" target="_blank">DP Mathematical Studies</a>
</li>
</ul></li>
<!-- - if current_user.is_language_services? && !current_user.is_publishing? -->
<!-- %li= link_to "Language services", tolk_path -->
</ul>
<ul class="nav pull-right">

<li>
<a href="https://06082010.xyz">IB Documents (2) Team</a>
</li></ul>
</div>
</div>
</div>

<div class="page-content container">
<div class="row">
<div class="span24">



</div>
</div>

<div class="page-header">
<div class="row">
<div class="span16">
<p class="back-to-list">
</p>
</div>
<div class="span8" style="margin: 0 0 -19px 0;">
<img style="width: 100%;" class="qb_logo" src="images/logo.jpg" alt="Ib qb 46 logo">
</div>
</div>
</div><h2>SL Paper 2</h2><div class="specification">
<p>A function \(f\) is given by \(f(x) = (2x + 2)(5 - {x^2})\).</p>
</div>

<div class="specification">
<p>The graph of the function \(g(x) = {5^x} + 6x - 6\) intersects the graph of \(f\).</p>
</div>

<div class="question" style="padding-left: 20px; padding-right: 20px;">
<p>Find the <strong>exact </strong>value of each of the zeros of \(f\).</p>
<div class="marks">[3]</div>
<div class="question_part_label">a.</div>
</div>
<div class="question" style="padding-left: 20px; padding-right: 20px;">
<p>Expand the expression for \(f(x)\).</p>
<div class="marks">[1]</div>
<div class="question_part_label">b.i.</div>
</div>
<div class="question" style="padding-left: 20px; padding-right: 20px;">
<p>Find \(f’(x)\).</p>
<div class="marks">[3]</div>
<div class="question_part_label">b.ii.</div>
</div>
<div class="question" style="padding-left: 20px; padding-right: 20px;">
<p>Use your answer to part (b)(ii) to find the values of \(x\) for which \(f\) is increasing.</p>
<div class="marks">[3]</div>
<div class="question_part_label">c.</div>
</div>
<div class="question" style="padding-left: 20px; padding-right: 20px;">
<p><strong>Draw </strong>the graph of \(f\) for \( - 3 \leqslant x \leqslant 3\) and \( - 40 \leqslant y \leqslant 20\). Use a scale of 2 cm to represent 1 unit on the \(x\)-axis and 1 cm to represent 5 units on the \(y\)-axis.</p>
<div class="marks">[4]</div>
<div class="question_part_label">d.</div>
</div>
<div class="question" style="padding-left: 20px; padding-right: 20px;">
<p>Write down the coordinates of the point of intersection.</p>
<div class="marks">[2]</div>
<div class="question_part_label">e.</div>
</div>
<h2 style="margin-top: 1em">Markscheme</h2>
<div class="question" style="padding-left: 20px;">
<p>\( - 1,{\text{ }}\sqrt 5 ,{\text{ }} - \sqrt 5 \)     <strong><em>(A1)(A1)(A1)</em></strong></p>
<p> </p>
<p><strong>Note:     </strong>Award <strong><em>(A1) </em></strong>for –1 and each exact value seen. Award at most <strong><em>(A1)(A0)(A1) </em></strong>for use of 2.23606… instead of \(\sqrt 5 \).</p>
<p> </p>
<p><strong><em>[3 marks]</em></strong></p>
<div class="question_part_label">a.</div>
</div>
<div class="question" style="padding-left: 20px;">
<p>\(10x - 2{x^3} + 10 - 2{x^2}\)     <strong><em>(A1)</em></strong></p>
<p> </p>
<p><strong>Notes:     </strong>The expansion may be seen in part (b)(ii).</p>
<p> </p>
<p><strong><em>[1 mark]</em></strong></p>
<div class="question_part_label">b.i.</div>
</div>
<div class="question" style="padding-left: 20px;">
<p>\(10 - 6{x^2} - 4x\)     <strong><em>(A1)</em>(ft)<em>(A1)</em>(ft)<em>(A1)</em>(ft)</strong></p>
<p> </p>
<p><strong>Notes:     </strong>Follow through from part (b)(i). Award <strong><em>(A1)</em>(ft) </strong>for each correct term. Award at most <strong><em>(A1)</em>(ft)<em>(A1)</em>(ft)<em>(A0) </em></strong>if extra terms are seen.</p>
<p> </p>
<p><strong><em>[3 marks]</em></strong></p>
<div class="question_part_label">b.ii.</div>
</div>
<div class="question" style="padding-left: 20px;">
<p>\(10 - 6{x^2} - 4x &gt; 0\)     <strong><em>(M1)</em></strong></p>
<p> </p>
<p><strong>Notes:     </strong>Award <strong><em>(M1) </em></strong>for their \(f’(x) &gt; 0\). Accept equality or weak inequality.</p>
<p> </p>
<p>\( - 1.67 &lt; x &lt; 1{\text{ }}\left( { - \frac{5}{3} &lt; x &lt; 1,{\text{ }} - 1.66666 \ldots  &lt; x &lt; 1} \right)\)     <strong><em>(A1)</em>(ft)<em>(A1)</em>(ft)<em>(G2)</em></strong></p>
<p> </p>
<p><strong>Notes:     </strong>Award <strong><em>(A1)</em>(ft) </strong>for correct endpoints, <strong><em>(A1)</em>(ft) </strong>for correct weak or strict inequalities. Follow through from part (b)(ii). Do not award any marks if there is no answer in part (b)(ii).</p>
<p> </p>
<p><strong><em>[3 marks]</em></strong></p>
<div class="question_part_label">c.</div>
</div>
<div class="question" style="padding-left: 20px;">
<p><img src="images/Schermafbeelding_2018-02-14_om_06.23.51.png" alt="N17/5/MATSD/SP2/ENG/TZ0/05.d/M">     <strong><em>(A1)(A1)</em>(ft)<em>(A1)</em>(ft)<em>(A1)</em></strong></p>
<p> </p>
<p><strong>Notes:     </strong>Award <strong><em>(A1) </em></strong>for correct scale; axes labelled and drawn with a ruler.</p>
<p>Award <strong><em>(A1)</em>(ft) </strong>for their correct \(x\)-intercepts in approximately correct location.</p>
<p>Award <strong><em>(A1) </em></strong>for correct minimum and maximum points in approximately correct location.</p>
<p>Award <strong><em>(A1) </em></strong>for a smooth continuous curve with approximate correct shape. The curve should be in the given domain.</p>
<p>Follow through from part (a) for the \(x\)-intercepts.</p>
<p> </p>
<p><strong><em>[4 marks]</em></strong></p>
<div class="question_part_label">d.</div>
</div>
<div class="question" style="padding-left: 20px;">
<p>\((1.49,{\text{ }}13.9){\text{ }}\left( {(1.48702 \ldots ,{\text{ }}13.8714 \ldots )} \right)\)     <strong><em>(G1)</em>(ft)<em>(G1)</em>(ft)</strong></p>
<p> </p>
<p><strong>Notes:     </strong>Award <strong><em>(G1) </em></strong>for 1.49 and <strong><em>(G1) </em></strong>for 13.9 written as a coordinate pair. Award at most <strong><em>(G0)(G1) </em></strong>if parentheses are missing. Accept \(x = 1.49\) and \(y = 13.9\). Follow through from part (b)(i).</p>
<p> </p>
<p><strong><em>[2 marks]</em></strong></p>
<div class="question_part_label">e.</div>
</div>
<h2 style="margin-top: 1em">Examiners report</h2>
<div class="question" style="padding-left: 20px;">
[N/A]
<div class="question_part_label">a.</div>
</div>
<div class="question" style="padding-left: 20px;">
[N/A]
<div class="question_part_label">b.i.</div>
</div>
<div class="question" style="padding-left: 20px;">
[N/A]
<div class="question_part_label">b.ii.</div>
</div>
<div class="question" style="padding-left: 20px;">
[N/A]
<div class="question_part_label">c.</div>
</div>
<div class="question" style="padding-left: 20px;">
[N/A]
<div class="question_part_label">d.</div>
</div>
<div class="question" style="padding-left: 20px;">
[N/A]
<div class="question_part_label">e.</div>
</div>
<br><hr><br><div class="specification">
<p style="margin: 0.0px 0.0px 0.0px 0.0px; font: 21.0px 'Times New Roman';"><span style="font-family: 'times new roman', times; font-size: medium;">The front view of the edge of a water tank is drawn on a set of axes shown below.</span></p>
<p style="margin: 0.0px 0.0px 0.0px 0.0px; font: 21.0px 'Times New Roman';"><span style="font-family: 'times new roman', times; font-size: medium;">The edge is modelled by \(y = a{x^2} + c\).</span></p>
<p style="font: normal normal normal 21px/normal 'Times New Roman'; text-align: center; margin: 0px;"><span style="font-family: 'times new roman', times; font-size: medium;"><br><img src="images/Schermafbeelding_2014-09-02_om_11.23.28.png" alt><br></span></p>
<p style="font: normal normal normal 21px/normal 'Times New Roman'; text-align: left; margin: 0px;"><span style="font-family: 'times new roman', times; font-size: medium;">Point \({\text{P}}\) has coordinates \((-3,&nbsp;1.8)\), point \({\text{O}}\) has coordinates \((0,&nbsp;0)\) and point \({\text{Q}}\) has coordinates \((3,&nbsp;1.8)\).</span></p>
</div>

<div class="question" style="padding-left: 20px; padding-right: 20px;">
<p><span>Write down the value of \(c\).</span></p>
<div class="marks">[1]</div>
<div class="question_part_label">a.</div>
</div>
<div class="question" style="padding-left: 20px; padding-right: 20px;">
<p><span>Find the value of \(a\).</span></p>
<div class="marks">[2]</div>
<div class="question_part_label">b.</div>
</div>
<div class="question" style="padding-left: 20px; padding-right: 20px;">
<p><span>Hence write down the equation of the quadratic function which models the edge of the water tank.</span></p>
<div class="marks">[1]</div>
<div class="question_part_label">c.</div>
</div>
<div class="question" style="padding-left: 20px; padding-right: 20px;">
<p><span>The water tank is shown below. It is partially filled with water.</span></p>
<p><br><span><img src="images/Schermafbeelding_2014-09-02_om_10.48.45_1.png" alt></span></p>
<p><span>Calculate the value of <em>y </em>when \(x = 2.4{\text{ m}}\).</span></p>
<div class="marks">[2]</div>
<div class="question_part_label">d.</div>
</div>
<div class="question" style="padding-left: 20px; padding-right: 20px;">
<p><span>The water tank is shown below. It is partially filled with water.</span></p>
<p><br><span><img src="images/Schermafbeelding_2014-09-02_om_10.48.45.png" alt></span></p>
<p><span>State what the value of \(x\) and the value of \(y\) represent for this water tank.</span></p>
<div class="marks">[2]</div>
<div class="question_part_label">e.</div>
</div>
<div class="question" style="padding-left: 20px; padding-right: 20px;">
<p><span>The water tank is shown below. It is partially filled with water.</span></p>
<p><br><span><img src="images/Schermafbeelding_2014-09-02_om_10.48.45_3.png" alt></span></p>
<p><span>Find the value of \(x\) when the height of water in the tank is \(0.9\) m.</span></p>
<div class="marks">[2]</div>
<div class="question_part_label">f.</div>
</div>
<div class="question" style="padding-left: 20px; padding-right: 20px;">
<p><span>The water tank is shown below. It is partially filled with water.</span></p>
<p><br><span><img src="images/Schermafbeelding_2014-09-02_om_10.48.45_2.png" alt></span></p>
<p> </p>
<p><span>The water tank has a length of 5 m.</span></p>
<p> </p>
<p><span>When the water tank is filled to a height of \(0.9\) m, the front cross-sectional area of the water is \({\text{2.55 }}{{\text{m}}^2}\).</span></p>
<p><span>(i)     Calculate the volume of water in the tank.</span></p>
<p><span>The total volume of the tank is \({\text{36 }}{{\text{m}}^3}\).</span></p>
<p><span>(ii)     Calculate the percentage of water in the tank.</span></p>
<div class="marks">[2]</div>
<div class="question_part_label">g.</div>
</div>
<h2 style="margin-top: 1em">Markscheme</h2>
<div class="question" style="padding-left: 20px;">
<p><span>\(0\)     <strong><em>(A1)(G1)</em></strong></span></p>
<p><span><strong><em>[1 mark]</em></strong></span></p>
<div class="question_part_label">a.</div>
</div>
<div class="question" style="padding-left: 20px;">
<p><span>\(1.8 = a{(3)^2} + 0\)     <strong><em>(M1)</em></strong></span></p>
<p><span><strong>OR</strong></span></p>
<p><span>\(1.8 = a{( - 3)^2} + 0\)     <strong><em>(M1)</em></strong></span></p>
<p> </p>
<p><span><strong>Note: </strong>Award <strong><em>(M1) </em></strong>for substitution of \(y = 1.8\) or \(x = 3\) and their value of \(c\) into equation. \(0\) may be implied.</span></p>
<p> </p>
<p><span>\(a = 0.2\)   \(\left( {\frac{1}{5}} \right)\)     <strong><em>(A1)</em>(ft)<em>(G1)</em></strong></span></p>
<p> </p>
<p><span><strong>Note: </strong>Follow through from their answer to part (a).</span></p>
<p><span>     Award <strong><em>(G1) </em></strong>for a correct answer only.</span></p>
<p> </p>
<p><span><strong><em>[2 marks]</em></strong></span></p>
<div class="question_part_label">b.</div>
</div>
<div class="question" style="padding-left: 20px;">
<p><span>\(y = 0.2{x^2}\)     <strong><em>(A1)</em>(ft)</strong></span></p>
<p> </p>
<p><span><strong>Note: </strong>Follow through from their answers to parts (a) and (b).</span></p>
<p><span>     Answer must be an equation.</span></p>
<p> </p>
<p><span><strong><em>[1 mark]</em></strong></span></p>
<div class="question_part_label">c.</div>
</div>
<div class="question" style="padding-left: 20px;">
<p><span>\(0.2 \times {(2.4)^2}\)     <strong><em>(M1)</em></strong></span></p>
<p><span>\( = 1.15{\text{ (m)}}\)   \((1.152)\)     <strong><em>(A1)</em>(ft)<em>(G1)</em></strong></span></p>
<p> </p>
<p><span><strong>Notes: </strong>Award <strong><em>(M1) </em></strong>for correctly substituted formula, <strong><em>(A1) </em></strong>for correct answer. Follow through from their answer </span><span>to part (c).</span></p>
<p><span>     Award <strong><em>(G1) </em></strong>for a correct answer only.</span></p>
<p> </p>
<p><span><strong><em>[2 marks]</em></strong></span></p>
<div class="question_part_label">d.</div>
</div>
<div class="question" style="padding-left: 20px;">
<p><span>\(y\) is the height     <strong><em>(A1)</em></strong></span></p>
<p><span>positive value of \(x\) is half the width (<em>or equivalent</em>)     <strong><em>(A1)</em></strong></span></p>
<p><span><strong><em>[2 marks]</em></strong></span></p>
<div class="question_part_label">e.</div>
</div>
<div class="question" style="padding-left: 20px;">
<p><span>\(0.9 = 0.2{x^2}\)     <strong><em>(M1)</em></strong></span></p>
<p> </p>
<p><span><strong>Note: </strong>Award <strong><em>(M1) </em></strong>for setting their equation equal to \(0.9\).</span></p>
<p> </p>
<p><span>\(x =  \pm 2.12{\text{ (m)}}\)   \(\left( { \pm \frac{3}{2}\sqrt 2 ,{\text{ }} \pm \sqrt {4.5} ,{\text{ }} \pm {\text{2.12132}} \ldots } \right)\)     <strong><em>(A1)</em>(ft)<em>(G1)</em></strong></span></p>
<p> </p>
<p><span><strong>Note: </strong>Accept \(2.12\). Award <strong><em>(G1) </em></strong>for a correct answer only.</span></p>
<p> </p>
<p><span><strong><em>[2 marks]</em></strong></span></p>
<div class="question_part_label">f.</div>
</div>
<div class="question" style="padding-left: 20px;">
<p><span>(i)     \(2.55 \times 5\)     <strong><em>(M1)</em></strong></span></p>
<p> </p>
<p><span><strong>Note: </strong>Award <strong><em>(M1) </em></strong>for correct substitution in formula.</span></p>
<p> </p>
<p><span>\( = 12.8{\text{ (}}{{\text{m}}^3}{\text{)}}\)   \(\left( {{\text{12.75 (}}{{\text{m}}^3}{\text{)}}} \right)\)     <strong><em>(A1)(G2)</em></strong></span></p>
<p><span><strong><em>[2 marks]</em></strong></span></p>
<p><span><strong><em> </em></strong></span></p>
<p><span>(ii)     \(\frac{{12.75}}{{36}} \times 100\)     <strong><em>(M1)</em></strong></span></p>
<p> </p>
<p><span><strong>Note: </strong>Award <strong><em>(M1) </em></strong>for correct quotient multiplied by \(100\).</span></p>
<p> </p>
<p><span>\( = 35.4 (\%)\)  \((35.4166 \ldots )\)     <strong><em>(A1)</em>(ft)(<em>G2)</em></strong></span></p>
<p> </p>
<p><span><strong>Note: </strong>Award <strong><em>(G2) </em></strong>for \(35.6 (\%) (35.5555… (\%))\).</span></p>
<p><span>     Follow through from their answer to part (g)(i).</span></p>
<p> </p>
<p><span><strong><em>[2 marks]</em></strong></span></p>
<div class="question_part_label">g.</div>
</div>
<h2 style="margin-top: 1em">Examiners report</h2>
<div class="question" style="padding-left: 20px;">
[N/A]
<div class="question_part_label">a.</div>
</div>
<div class="question" style="padding-left: 20px;">
[N/A]
<div class="question_part_label">b.</div>
</div>
<div class="question" style="padding-left: 20px;">
[N/A]
<div class="question_part_label">c.</div>
</div>
<div class="question" style="padding-left: 20px;">
[N/A]
<div class="question_part_label">d.</div>
</div>
<div class="question" style="padding-left: 20px;">
[N/A]
<div class="question_part_label">e.</div>
</div>
<div class="question" style="padding-left: 20px;">
[N/A]
<div class="question_part_label">f.</div>
</div>
<div class="question" style="padding-left: 20px;">
[N/A]
<div class="question_part_label">g.</div>
</div>
<br><hr><br><div class="specification">
<p><span style="font-size: medium; font-family: times new roman,times;">A function is defined by \(f(x) = \frac{5}{{{x^2}}} + 3x + c,{\text{ }}x \ne 0,{\text{ }}c \in \mathbb{Z}\).</span></p>
</div>

<div class="question" style="padding-left: 20px; padding-right: 20px;">
<p><span>Write down an expression for \(f ′(x)\).</span></p>
<div class="marks">[4]</div>
<div class="question_part_label">a.</div>
</div>
<div class="question" style="padding-left: 20px; padding-right: 20px;">
<p><span>Consider the graph of <em>f</em>. The graph of <em>f</em> passes through the point P(1, 4).</span></p>
<p><span>Find the value of <em>c</em>.</span></p>
<div class="marks">[2]</div>
<div class="question_part_label">b.</div>
</div>
<div class="question" style="padding-left: 20px; padding-right: 20px;">
<p><span>There is a local minimum at the point Q.</span></p>
<p><span>Find the coordinates of Q.</span></p>
<div class="marks">[4]</div>
<div class="question_part_label">c, i.</div>
</div>
<div class="question" style="padding-left: 20px; padding-right: 20px;">
<p><span>There is a local minimum at the point Q.</span></p>
<p><span>Find the set of values of <em>x</em> for which the function is decreasing.</span></p>
<div class="marks">[3]</div>
<div class="question_part_label">c, ii.</div>
</div>
<div class="question" style="padding-left: 20px; padding-right: 20px;">
<p><span>Let<em> T</em> be the tangent to the graph of <em>f</em> at P.</span></p>
<p><span>Show that the gradient of <em>T</em> is –7.</span></p>
<div class="marks">[2]</div>
<div class="question_part_label">d, i.</div>
</div>
<div class="question" style="padding-left: 20px; padding-right: 20px;">
<p><span>Let<em> T</em> be the tangent to the graph of <em>f</em> at P.</span></p>
<p><span>Find the equation of <em>T</em>.</span></p>
<div class="marks">[2]</div>
<div class="question_part_label">d, ii.</div>
</div>
<div class="question" style="padding-left: 20px; padding-right: 20px;">
<p><span><em>T</em> intersects the graph again at R. Use your graphic display calculator to find the coordinates of R.</span></p>
<div class="marks">[2]</div>
<div class="question_part_label">e.</div>
</div>
<h2 style="margin-top: 1em">Markscheme</h2>
<div class="question" style="padding-left: 20px;">
<p><span>\(f'(x) = \frac{{ - 10}}{{{x^3}}} + 3\)     <em><strong>(A1)(A1)(A1)(A1)</strong></em></span></p>
<p><br><span><strong>Note:</strong> Award <em><strong>(A1)</strong></em> for −10, <em><strong>(A1)</strong></em> for <em>x</em><sup>3</sup> (or </span><span> <em>x<sup>−</sup></em><sup>3</sup></span><span>), <em><strong>(A1)</strong></em> for 3, <em><strong>(A1)</strong></em> for no other constant term.</span></p>
<p><span> </span></p>
<p><em><strong><span>[4 marks]</span></strong></em></p>
<div class="question_part_label">a.</div>
</div>
<div class="question" style="padding-left: 20px;">
<p><span>4 = 5 + 3 + <em>c</em>     <em><strong>(M1)</strong></em></span></p>
<p><br><span><strong>Note:</strong> Award <em><strong>(M1)</strong></em> for substitution in <em>f</em> (<em>x</em>).</span></p>
<p><br><span><em>c</em> = −4     <em><strong>(A1)(G2)</strong></em></span></p>
<p><span><em><strong>[2 marks]</strong></em></span></p>
<div class="question_part_label">b.</div>
</div>
<div class="question" style="padding-left: 20px;">
<p><span>\(f '(x) = 0\)     <em><strong>(M1)</strong></em></span></p>
<p><span>\(0 = \frac{{ - 10}}{{{x^3}}} + 3\)     <em><strong>(A1)</strong></em><strong>(ft)</strong></span></p>
<p><span>(1.49, 2.72)   (accept <em>x</em> = 1.49  <em>y</em> = 2.72)     <em><strong>(A1)</strong></em><strong>(ft)</strong><em><strong>(A1)</strong></em><strong>(ft)</strong><em><strong>(G3)</strong></em></span></p>
<p><br><span><strong>Notes:</strong> If answer is given as (1.5, 2.7) award <em><strong>(A0)(AP)(A1)</strong></em>.</span></p>
<p><span>Award at most <em><strong>(M1)(A1)(A1)(A0)</strong></em> if parentheses not included. <strong>(ft)</strong></span> <span>from their (a).</span></p>
<p><span>If no working shown award <em><strong>(G2)(G0)</strong></em> if parentheses are not included.</span></p>
<p><br><strong><span>OR</span></strong></p>
<p><span>Award <em><strong>(M2)</strong></em> for sketch, <em><strong>(A1)</strong></em><strong>(ft)</strong><em><strong>(A1)</strong></em><strong>(ft)</strong> for correct </span><span>coordinates. <strong>(ft)</strong> from their (b).     <em><strong>(M2)(A1)</strong></em><strong>(ft)</strong><em><strong>(A1)</strong></em><strong>(ft)</strong></span></p>
<p><br><span><strong>Note:</strong> Award at most <em><strong>(M2)(A1)</strong></em><strong>(ft)</strong><em><strong>(A0)</strong></em> if parentheses not included.</span></p>
<p><span> </span></p>
<p><em><strong><span>[4 marks]</span></strong></em></p>
<div class="question_part_label">c, i.</div>
</div>
<div class="question" style="padding-left: 20px;">
<p><span>0 &lt; <em>x</em> &lt; 1.49  <strong>OR </strong> 0 &lt; <em>x </em>≤ 1.49     <strong><em>(A1)(A1)</em>(ft)<em>(A1)</em></strong></span></p>
<p><span><span><br> </span><strong>Notes:</strong> Award <strong><em>(A1)</em></strong> for 0, <strong><em>(A1)</em>(ft)</strong> for 1.49 and <strong><em>(A1)</em></strong> for correct inequality signs.</span></p>
<p><span><strong>(ft)</strong> from their <em>x</em> value in (c) (i).</span></p>
<p><span> </span></p>
<p><span><em><strong>[3 marks]</strong></em></span></p>
<div class="question_part_label">c, ii.</div>
</div>
<div class="question" style="padding-left: 20px;">
<p><span>For P(1, 4) \(f '(1) = - 10 + 3\)     <em><strong>(M1)(A1)</strong></em></span></p>
<p><span>\(= -7\)     <em><strong>(AG)</strong></em></span></p>
<p><br><span><strong>Note:</strong> Award <em><strong>(M1)</strong></em> for substituting \(x = 1\) into their \(f '(x)\). <em><strong>(A1)</strong></em> for \(-10 + 3\).</span></p>
<p><span>\(-7\) must be seen for <em><strong>(A1)</strong></em> to be awarded.<br></span></p>
<p><span> </span></p>
<p><em><strong><span>[2 marks]</span></strong></em></p>
<div class="question_part_label">d, i.</div>
</div>
<div class="question" style="padding-left: 20px;">
<p><span>\(4 = -7 \times 1 + c\)     \(11 = c\)     <strong><em>(A1)</em></strong></span></p>
<p><span>\(y = -7 x + 11\)     <strong><em>(A1)</em></strong></span></p>
<p><span><strong><em>[2 marks]</em></strong></span></p>
<div class="question_part_label">d, ii.</div>
</div>
<div class="question" style="padding-left: 20px;">
<p><span>Point of intersection is R(−0.5, 14.5)     <em><strong>(A1)</strong></em><strong>(ft)</strong><em><strong>(A1)</strong></em><strong>(ft)</strong><em><strong>(G2)</strong></em><strong>(ft)</strong></span></p>
<p><br><span><span><strong>Notes:</strong> Award <em><strong>(A1)</strong></em> for the <em>x</em> coordinate, <em><strong>(A1)</strong></em> for the <em>y</em> coordinate.</span></span></p>
<p><span><span>Allow <strong>(ft)</strong> from candidate’s (d)(ii) equation and their (b) even with</span> <span>no working seen.</span></span></p>
<p><span><span>Award <em><strong>(A1)</strong></em><strong>(ft)</strong><em><strong>(A0)</strong></em> if brackets not included and not previously penalised.</span></span></p>
<p><span><span> </span></span></p>
<p><em><strong><span><span>[2 marks]</span></span></strong></em></p>
<div class="question_part_label">e.</div>
</div>
<h2 style="margin-top: 1em">Examiners report</h2>
<div class="question" style="padding-left: 20px;">
<p><span style="font-size: medium; font-family: times new roman,times;">A large number of candidates were not able to answer this question well because of the first</span> <span style="font-size: medium; font-family: times new roman,times;">term of the function&rsquo;s negative index. Many candidates who did make an attempt lacked </span><span style="font-size: medium; font-family: times new roman,times;">understanding of the topic.</span></p>
<p><span style="font-size: medium; font-family: times new roman,times;">Most candidates could score 4 marks.</span></p>
<div class="question_part_label">a.</div>
</div>
<div class="question" style="padding-left: 20px;">
<p><span style="font-size: medium; font-family: times new roman,times;">A large number of candidates were not able to answer this question well because of the first</span> <span style="font-size: medium; font-family: times new roman,times;">term of the function&rsquo;s negative index. Many candidates who did make an attempt lacked </span><span style="font-size: medium; font-family: times new roman,times;">understanding of the topic.</span></p>
<p><span style="font-size: medium; font-family: times new roman,times;">A good number of candidates correctly substituted into the original function.</span></p>
<div class="question_part_label">b.</div>
</div>
<div class="question" style="padding-left: 20px;">
<p><span style="font-size: medium; font-family: times new roman,times;">A large number of candidates were not able to answer this question well because of the first</span> <span style="font-size: medium; font-family: times new roman,times;">term of the function&rsquo;s negative index. Many candidates who did make an attempt lacked </span><span style="font-size: medium; font-family: times new roman,times;">understanding of the topic.</span></p>
<p><span style="font-size: medium; font-family: times new roman,times;"><span style="font-size: medium; font-family: times new roman,times;">Very few managed to answer this part algebraically. Those candidates who were aware</span> <span style="font-size: medium; font-family: times new roman,times;">that they could read the values from their GDC gained some easy marks.</span></span></p>
<div class="question_part_label">c, i.</div>
</div>
<div class="question" style="padding-left: 20px;">
<p style="margin: 0.0px 0.0px 10.0px 0.0px; font: 16.0px Times; color: #3f3f3f;"><span style="font-family: 'times new roman', times; font-size: medium;">A large number of candidates were not able to answer this question well because of the firstterm of the function&rsquo;s negative index. Many candidates who did make an attempt lacked understanding of the topic.</span></p>
<p style="margin: 0.0px 0.0px 10.0px 0.0px; font: 16.0px Times; color: #3f3f3f;">Proved to be difficult for many candidates as increasing/decreasing intervals were not well understood by many.</p>
<div class="question_part_label">c, ii.</div>
</div>
<div class="question" style="padding-left: 20px;">
<p><span style="font-size: medium; font-family: times new roman,times;">A large number of candidates were not able to answer this question well because of the first</span> <span style="font-size: medium; font-family: times new roman,times;">term of the function&rsquo;s negative index. Many candidates who did make an attempt lacked </span><span style="font-size: medium; font-family: times new roman,times;">understanding of the topic.</span></p>
<p><span style="font-size: medium; font-family: times new roman,times;"><span style="font-size: medium; font-family: times new roman,times;">As this part was linked to part (a), candidates who could not find the correct derivative </span><span style="font-size: medium; font-family: times new roman,times;">could not show that the gradient of <em>T</em> was &minus;7. Many candidates did not realize that they </span><span style="font-size: medium; font-family: times new roman,times;">had to substitute into the first derivative. For those who did, finding the equation of <em>T </em></span><span style="font-size: medium; font-family: times new roman,times;">was a simple task.</span></span></p>
<div class="question_part_label">d, i.</div>
</div>
<div class="question" style="padding-left: 20px;">
<p style="margin: 0.0px 0.0px 10.0px 0.0px; font: 16.0px Times; color: #3f3f3f;"><span style="font-family: 'times new roman', times; font-size: medium;">A large number of candidates were not able to answer this question well because of the firstterm of the function&rsquo;s negative index. Many candidates who did make an attempt lacked understanding of the topic.</span></p>
<p style="margin: 0.0px 0.0px 10.0px 0.0px; font: 16.0px Times; color: #3f3f3f;"><span style="font-family: 'times new roman', times; font-size: medium;">As this part was linked to part (a), candidates who could not find the correct derivative could not show that the gradient of <em>T</em> was &minus;7. Many candidates did not realize that they had to substitute into the first derivative. For those who did, finding the equation of <em>T </em>was a simple task.</span></p>
<div class="question_part_label">d, ii.</div>
</div>
<div class="question" style="padding-left: 20px;">
<p><span style="font-size: medium; font-family: times new roman,times;">A large number of candidates were not able to answer this question well because of the first</span> <span style="font-size: medium; font-family: times new roman,times;">term of the function&rsquo;s negative index. Many candidates who did make an attempt lacked </span><span style="font-size: medium; font-family: times new roman,times;">understanding of the topic.</span></p>
<p><span style="font-size: medium; font-family: times new roman,times;"><span style="font-size: medium; font-family: times new roman,times;">For those who had part (d) (ii) correct, this allowed them to score easy marks. For the</span> <span style="font-size: medium; font-family: times new roman,times;">others, it proved a difficult task because their equation in (d) would not be a tangent.</span></span></p>
<div class="question_part_label">e.</div>
</div>
<br><hr><br><div class="question" style="padding-left: 20px; padding-right: 20px;">
<p><span>Sketch the graph of <em>y</em> = 2<sup><em>x</em></sup> for \( - 2 \leqslant x \leqslant 3\). Indicate clearly where the curve intersects the <em>y</em>-axis.</span></p>
<div class="marks">[3]</div>
<div class="question_part_label">A, a.</div>
</div>
<div class="question" style="padding-left: 20px; padding-right: 20px;">
<p><span>Write down the equation of the asymptote of the graph of <em>y</em> = 2<sup><em>x</em></sup>.</span></p>
<div class="marks">[2]</div>
<div class="question_part_label">A, b.</div>
</div>
<div class="question" style="padding-left: 20px; padding-right: 20px;">
<p><span>On the same axes sketch the graph of <em>y</em> = 3 + 2<em>x</em> − <em>x</em><sup>2</sup>. Indicate clearly where this curve intersects the <em>x</em> and <em>y</em> axes.</span></p>
<div class="marks">[3]</div>
<div class="question_part_label">A, c.</div>
</div>
<div class="question" style="padding-left: 20px; padding-right: 20px;">
<p><span>Using your graphic display calculator, solve the equation 3 + 2<em>x</em> − <em>x</em><sup>2</sup> = 2<sup><em>x</em></sup>.</span></p>
<div class="marks">[2]</div>
<div class="question_part_label">A, d.</div>
</div>
<div class="question" style="padding-left: 20px; padding-right: 20px;">
<p><span>Write down the maximum value of the function <em>f</em> (<em>x</em>) = 3 + 2<em>x</em> − <em>x</em><sup>2</sup>.</span></p>
<div class="marks">[1]</div>
<div class="question_part_label">A, e.</div>
</div>
<div class="question" style="padding-left: 20px; padding-right: 20px;">
<p><span>Use Differential Calculus to verify that your answer to (e) is correct.</span></p>
<div class="marks">[5]</div>
<div class="question_part_label">A, f.</div>
</div>
<div class="question" style="padding-left: 20px; padding-right: 20px;">
<p><span>The curve <em>y</em> = <em>px</em><sup>2</sup> + <em>qx</em> − 4 passes through the point (2, –10).</span></p>
<p><span>Use the above information to write down an equation in <em>p</em> and <em>q</em>.</span></p>
<div class="marks">[2]</div>
<div class="question_part_label">B, a.</div>
</div>
<div class="question" style="padding-left: 20px; padding-right: 20px;">
<p><span>The gradient of the curve \(y = p{x^2} + qx - 4\) at the point (2, –10) is 1.</span></p>
<p><span>Find \(\frac{{{\text{d}}y}}{{{\text{d}}x}}\).</span></p>
<div class="marks">[2]</div>
<div class="question_part_label">B, b, i.</div>
</div>
<div class="question" style="padding-left: 20px; padding-right: 20px;">
<p><span>The gradient of the curve \(y = p{x^2} + qx - 4\) at the point (2, –10) is 1.</span></p>
<p><span>Hence, find a second equation in <em>p</em> and <em>q</em>.</span></p>
<div class="marks">[1]</div>
<div class="question_part_label">B, b, ii.</div>
</div>
<div class="question" style="padding-left: 20px; padding-right: 20px;">
<p><span>The gradient of the curve \(y = p{x^2} + qx - 4\) at the point (2, –10) is 1.</span></p>
<p><span>Solve the equations to find the value of <em>p</em> and of <em>q</em>.</span></p>
<div class="marks">[3]</div>
<div class="question_part_label">B, c.</div>
</div>
<h2 style="margin-top: 1em">Markscheme</h2>
<div class="question" style="padding-left: 20px;">
<p><span><img src="data:image/png;base64,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" alt><span>     <em><strong>(A1)(A1)(A1)</strong></em></span></span></p>
<p><br><span><strong>Note:</strong> Award <em><strong>(A1)</strong></em> for correct domain, <em><strong>(A1)</strong></em> for smooth curve,</span> <span><em><strong>(A1)</strong></em> for <em>y</em>-intercept clearly indicated.</span></p>
<p><span> </span></p>
<p><em><strong><span>[3 marks]</span></strong></em></p>
<div class="question_part_label">A, a.</div>
</div>
<div class="question" style="padding-left: 20px;">
<p><span><em>y</em> = 0     <strong><em>(A1)(A1)</em></strong></span></p>
<p><br><span><strong>Note:</strong> Award <em><strong>(A1)</strong></em> for <em>y</em> = constant, <em><strong>(A1)</strong></em> for 0.</span></p>
<p><span> </span></p>
<p><em><strong><span>[2 marks]</span></strong></em></p>
<div class="question_part_label">A, b.</div>
</div>
<div class="question" style="padding-left: 20px;">
<p><span><strong>Note:</strong> Award <em><strong>(A1)</strong></em> for smooth parabola,</span></p>
<p><span><em><strong>(A1)</strong></em> for vertex (maximum) in correct quadrant.</span></p>
<p><span><em><strong>(A1)</strong></em> for all clearly indicated intercepts <em>x</em> = −1, <em>x</em> = 3 and <em>y</em> = 3.</span></p>
<p><span>The final mark is to be applied very strictly.     <em><strong>(A1)(A1)(A1)</strong></em></span></p>
<p><span><em><strong> </strong></em></span></p>
<p><span><em><strong>[3 marks]</strong></em></span></p>
<div class="question_part_label">A, c.</div>
</div>
<div class="question" style="padding-left: 20px;">
<p><span><em>x</em> = −0.857   <em>x</em> = 1.77     <em><strong>(G1)(G1)</strong></em></span></p>
<p><br><span><strong>Note:</strong> Award a maximum of <em><strong>(G1)</strong></em> if <em>x</em> and <em>y</em> coordinates are both given.</span></p>
<p><span> </span></p>
<p><em><strong><span>[2 marks]</span></strong></em></p>
<div class="question_part_label">A, d.</div>
</div>
<div class="question" style="padding-left: 20px;">
<p><span>4     <em><strong>(G1)</strong></em> </span></p>
<p><br><span><strong>Note:</strong> Award <em><strong>(G0)</strong></em> for (1, 4).</span></p>
<p><span> </span></p>
<p><em><strong><span>[1 mark]</span></strong></em></p>
<div class="question_part_label">A, e.</div>
</div>
<div class="question" style="padding-left: 20px;">
<p><span>\(f'(x) = 2 - 2x\)     <em><strong>(A1)(A1)</strong></em></span></p>
<p><br><span><strong>Note:</strong> Award <em><strong>(A1)</strong></em> for each correct term.</span></p>
<p><span>Award at most <em><strong>(A1)(A0)</strong></em> if any extra terms seen.</span></p>
<p><br><span>\(2 - 2x = 0\)     <em><strong>(M1)</strong></em></span></p>
<p><br><span><strong>Note:</strong> Award <em><strong>(M1)</strong></em> for equating their gradient function to zero.</span></p>
<p><br><span>\(x = 1\)     <em><strong>(A1)</strong></em><strong>(ft)</strong></span></p>
<p><span>\(f (1) = 3 + 2(1) - (1)^2 = 4\)     <em><strong>(A1)</strong></em></span></p>
<p><br><span><strong>Note:</strong> The final <em><strong>(A1)</strong></em> is for substitution of <em>x</em> = 1 into \(f (x)\) and subsequent correct </span><span>answer. Working must be seen for final <em><strong>(A1)</strong></em> to be awarded.</span></p>
<p><span> </span></p>
<p><em><strong><span>[5 marks]</span></strong></em></p>
<div class="question_part_label">A, f.</div>
</div>
<div class="question" style="padding-left: 20px;">
<p><span>2<sup>2</sup> × <em>p</em> + 2<em>q</em> − 4 = </span><span><span>−</span>10     <em><strong>(M1)</strong></em></span></p>
<p><br><span><strong>Note:</strong> Award <em><strong>(M1)</strong></em> for correct substitution in the equation.</span></p>
<p><br><span>4<em>p</em> + 2<em>q</em> = </span><span><span>−</span>6     or     2<em>p</em> + <em>q</em> = </span><span><span>−</span>3     <em><strong>(A1)</strong></em></span></p>
<p><br><span><strong>Note:</strong> Accept equivalent simplified forms.</span></p>
<p><span> </span></p>
<p><em><strong><span>[2 marks]</span></strong></em></p>
<div class="question_part_label">B, a.</div>
</div>
<div class="question" style="padding-left: 20px;">
<p><span>\(\frac{{{\text{d}}y}}{{{\text{d}}x}} = 2px + q\)     <em><strong>(A1)(A1)</strong></em></span></p>
<p><br><span><strong>Note:</strong> Award <em><strong>(A1)</strong></em> for each correct term.</span></p>
<p><span>Award at most <em><strong>(A1)(A0)</strong></em> if any extra terms seen.<br></span></p>
<p><span> </span></p>
<p><em><strong><span>[2 marks]</span></strong></em></p>
<div class="question_part_label">B, b, i.</div>
</div>
<div class="question" style="padding-left: 20px;">
<p><span>4<em>p </em>+<em> q</em> = 1     <strong><em>(A1)</em>(ft)</strong></span></p>
<p><span><em><strong>[1 mark]</strong></em></span></p>
<div class="question_part_label">B, b, ii.</div>
</div>
<div class="question" style="padding-left: 20px;">
<p><span>4<em>p</em> + 2<em>q</em> = −6<br></span></p>
<p><span>4<em>p</em> + <em>q</em> = 1     <em><strong>(M1)</strong></em></span></p>
<p><span><br><strong>Note:</strong> Award <em><strong>(M1)</strong></em> for sensible attempt to solve the equations.</span></p>
<p><span><br><em>p</em> = 2, <em>q</em> = </span><span><span>−</span>7     <em><strong>(A1)(A1)</strong></em><strong>(ft)</strong><em><strong>(G3)</strong></em></span></p>
<p><span><em><strong>[3 marks]</strong></em></span></p>
<div class="question_part_label">B, c.</div>
</div>
<h2 style="margin-top: 1em">Examiners report</h2>
<div class="question" style="padding-left: 20px;">
<p><span style="font-family: times new roman,times; font-size: medium;">Undoubtedly, this question caused the most difficulty in terms of its content. Where there was no alternative to using the calculus, the majority of candidates struggled. However, for those with a sound grasp of the topic, there were many very successful attempts.</span></p>
<p><span style="font-family: times new roman,times; font-size: medium;">The most common error was using the incorrect domain.</span></p>
<div class="question_part_label">A, a.</div>
</div>
<div class="question" style="padding-left: 20px;">
<p><span style="font-family: times new roman,times; font-size: medium;">Undoubtedly, this question caused the most difficulty in terms of its content. Where there was no alternative to using the calculus, the majority of candidates struggled. However, for those with a sound grasp of the topic, there were many very successful attempts.</span></p>
<p><span style="font-family: times new roman,times; font-size: medium;">Many had little idea of asymptotes. Others did not write their answer as an equation.</span></p>
<div class="question_part_label">A, b.</div>
</div>
<div class="question" style="padding-left: 20px;">
<p><span style="font-family: times new roman,times; font-size: medium;">Undoubtedly, this question caused the most difficulty in terms of its content. Where there was no alternative to using the calculus, the majority of candidates struggled. However, for those with a sound grasp of the topic, there were many very successful attempts.</span></p>
<p><span style="font-family: times new roman,times; font-size: medium;">The intercepts being inexact or unlabelled was the most frequent cause of loss of marks.</span></p>
<div class="question_part_label">A, c.</div>
</div>
<div class="question" style="padding-left: 20px;">
<p><span style="font-family: times new roman,times; font-size: medium;">Undoubtedly, this question caused the most difficulty in terms of its content. Where there was no alternative to using the calculus, the majority of candidates struggled. However, for those with a sound grasp of the topic, there were many very successful attempts.</span></p>
<p><span style="font-family: times new roman,times; font-size: medium;">Often, only one solution to the equation was given. Elsewhere, a lack of appreciation that the solutions were the <em>x</em> coordinates was a common mistake.</span></p>
<div class="question_part_label">A, d.</div>
</div>
<div class="question" style="padding-left: 20px;">
<p><span style="font-family: times new roman,times; font-size: medium;">Undoubtedly, this question caused the most difficulty in terms of its content. Where there was no alternative to using the calculus, the majority of candidates struggled. However, for those with a sound grasp of the topic, there were many very successful attempts.</span></p>
<p><span style="font-family: times new roman,times; font-size: medium;">The maximum is the<em> y</em> coordinate only; again a common misapprehension was the answer &ldquo;(1, 4)&rdquo;.</span></p>
<div class="question_part_label">A, e.</div>
</div>
<div class="question" style="padding-left: 20px;">
<p><span style="font-family: times new roman,times; font-size: medium;">Undoubtedly, this question caused the most difficulty in terms of its content. Where there was no alternative to using the calculus, the majority of candidates struggled. However, for those with a sound grasp of the topic, there were many very successful attempts.</span></p>
<p><span style="font-family: times new roman,times; font-size: medium;">This was a major discriminator in the paper. Many candidates were unable to follow the analytic approach to finding a maximum point.</span></p>
<div class="question_part_label">A, f.</div>
</div>
<div class="question" style="padding-left: 20px;">
<p><span style="font-size: medium; font-family: times new roman,times;">Undoubtedly, this question caused the most difficulty in terms of its content. Where there was no alternative to using the calculus, the majority of candidates struggled. However, for those with a sound grasp of the topic, there were many very successful attempts.</span></p>
<p><span style="font-size: medium; font-family: times new roman,times;">This part was challenging to the majority, with a large number not attempting the question at all. However, there were a pleasing number of correct attempts that showed a fine understanding of the calculus.</span></p>
<div class="question_part_label">B, a.</div>
</div>
<div class="question" style="padding-left: 20px;">
<p><span style="font-size: medium; font-family: times new roman,times;">Undoubtedly, this question caused the most difficulty in terms of its content. Where there was no alternative to using the calculus, the majority of candidates struggled. However, for those with a sound grasp of the topic, there were many very successful attempts.</span></p>
<p><span style="font-size: medium; font-family: times new roman,times;">This part was challenging to the majority, with a large number not attempting the question at all. However, there were a pleasing number of correct attempts that showed a fine understanding of the calculus.</span></p>
<div class="question_part_label">B, b, i.</div>
</div>
<div class="question" style="padding-left: 20px;">
<p style="margin: 0.0px 0.0px 10.0px 0.0px; font: 16.0px Times; color: #3f3f3f;">Undoubtedly, this question caused the most difficulty in terms of its content. Where there was no alternative to using the calculus, the majority of candidates struggled. However, for those with a sound grasp of the topic, there were many very successful attempts.</p>
<p style="margin: 0.0px 0.0px 10.0px 0.0px; font: 16.0px Times; color: #3f3f3f;">This part was challenging to the majority, with a large number not attempting the question at all. However, there were a pleasing number of correct attempts that showed a fine understanding of the calculus.</p>
<div class="question_part_label">B, b, ii.</div>
</div>
<div class="question" style="padding-left: 20px;">
<p><span style="font-size: medium; font-family: times new roman,times;">Undoubtedly, this question caused the most difficulty in terms of its content. Where there was no alternative to using the calculus, the majority of candidates struggled. However, for those with a sound grasp of the topic, there were many very successful attempts.</span></p>
<p><span style="font-size: medium; font-family: times new roman,times;">This part was challenging to the majority, with a large number not attempting the question at all. However, there were a pleasing number of correct attempts that showed a fine understanding of the calculus.</span></p>
<div class="question_part_label">B, c.</div>
</div>
<br><hr><br><div class="specification">
<p><span style="font-family: times new roman,times; font-size: medium;">Consider the function \(f(x) = {x^3} + \frac{{48}}{x}{\text{, }}x \ne 0\).</span></p>
</div>

<div class="question" style="padding-left: 20px; padding-right: 20px;">
<p><span>Calculate \(f(2)\) .</span></p>
<div class="marks">[2]</div>
<div class="question_part_label">a.</div>
</div>
<div class="question" style="padding-left: 20px; padding-right: 20px;">
<p><span>Sketch the graph of the function \(y = f(x)\) for \( - 5 \leqslant x \leqslant 5\) and \( - 200 \leqslant y \leqslant 200\) .</span></p>
<div class="marks">[4]</div>
<div class="question_part_label">b.</div>
</div>
<div class="question" style="padding-left: 20px; padding-right: 20px;">
<p><span>Find \(f'(x)\) .</span></p>
<div class="marks">[3]</div>
<div class="question_part_label">c.</div>
</div>
<div class="question" style="padding-left: 20px; padding-right: 20px;">
<p><span>Find \(f'(2)\) .</span></p>
<div class="marks">[2]</div>
<div class="question_part_label">d.</div>
</div>
<div class="question" style="padding-left: 20px; padding-right: 20px;">
<p><span>Write down the coordinates of the local maximum point on the graph of \(f\) .</span></p>
<div class="marks">[2]</div>
<div class="question_part_label">e.</div>
</div>
<div class="question" style="padding-left: 20px; padding-right: 20px;">
<address><span>Find the range of \(f\) .</span></address>
<div class="marks">[3]</div>
<div class="question_part_label">f.</div>
</div>
<div class="question" style="padding-left: 20px; padding-right: 20px;">
<p><span>Find the gradient of the tangent to the graph of \(f\) at \(x = 1\).</span></p>
<div class="marks">[2]</div>
<div class="question_part_label">g.</div>
</div>
<div class="question" style="padding-left: 20px; padding-right: 20px;">
<p><span>There is a second point on the graph of \(f\) at which the tangent is parallel to the tangent at \(x = 1\). </span></p>
<p><span>Find the \(x\)-coordinate of this point.</span></p>
<div class="marks">[2]</div>
<div class="question_part_label">h.</div>
</div>
<h2 style="margin-top: 1em">Markscheme</h2>
<div class="question" style="padding-left: 20px;">
<p><span>\(f(2) = {2^3} + \frac{{48}}{2}\)     <em><strong>(M1)</strong></em></span></p>
<p><span>\(= 32\)     <em><strong>(A1)(G2)</strong></em></span></p>
<p><span><em><strong>[2 marks]<br></strong></em></span></p>
<div class="question_part_label">a.</div>
</div>
<div class="question" style="padding-left: 20px;">
<p><span><img src="data:image/png;base64,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" alt></span></p>
<p><span><em><strong>(A1)</strong></em> for labels and some indication of scale in an appropriate window</span><br><span><em><strong>(A1)</strong></em> for correct shape of the two unconnected and smooth branches</span><br><span><em><strong>(A1)</strong></em> for maximum and minimum in approximately correct positions</span><br><span><em><strong>(A1)</strong></em> for asymptotic behaviour at \(y\)-axis     <em><strong>(A4)</strong></em></span></p>
<p><span><strong>Notes:</strong> Please be rigorous.</span><br><span>The axes need not be drawn with a ruler.</span><br><span>The branches must be smooth: a single continuous line that does not deviate from its proper direction.</span><br><span>The position of the maximum and minimum points must be symmetrical about the origin.</span><br><span>The \(y\)-axis must be an asymptote for both branches. Neither branch should touch the axis nor must the curve approach the</span><br><span>asymptote then deviate away later.</span></p>
<p><span><em><strong>[4 marks]</strong></em><br></span></p>
<div class="question_part_label">b.</div>
</div>
<div class="question" style="padding-left: 20px;">
<p><span>\(f'(x) = 3{x^2} - \frac{{48}}{{{x^2}}}\)     <em><strong>(A1)(A1)(A1)</strong></em></span></p>
<p><span><strong>Notes:</strong> Award <em><strong>(A1)</strong></em> for \(3{x^2}\) , <em><strong>(A1)</strong></em> for \( - 48\) , <em><strong>(A1)</strong></em> for \({x^{ - 2}}\) . Award a maximum of <em><strong>(A1)(A1)(A0)</strong></em> if extra terms seen.</span></p>
<p><span><em><strong>[3 marks]</strong></em><br></span></p>
<div class="question_part_label">c.</div>
</div>
<div class="question" style="padding-left: 20px;">
<p><span>\(f'(2) = 3{(2)^2} - \frac{{48}}{{{{(2)}^2}}}\)     <em><strong>(M1)</strong></em></span> </p>
<p><span><strong>Note:</strong> Award <em><strong>(M1)</strong></em> for substitution of \(x = 2\) into their derivative.</span></p>
<p> </p>
<p><span><span>\(= 0\)</span><span>     <em><strong>(A1)</strong></em><strong>(ft)</strong><em><strong>(G1)</strong></em></span></span></p>
<p><span><span><em><strong>[2 marks]<br></strong></em></span></span></p>
<div class="question_part_label">d.</div>
</div>
<div class="question" style="padding-left: 20px;">
<p><span>\(( - 2{\text{, }} - 32)\) or \(x = - 2\), \(y = - 32\)     <em><strong>(G1)(G1)</strong></em></span></p>
<p><span><strong>Notes:</strong> Award <em><strong>(G0)(G0)</strong></em> for \(x = - 32\), \(y = - 2\) . Award at most <em><strong>(G0)(G1)</strong></em> if parentheses are omitted.</span></p>
<p><span><em><strong>[2 marks]</strong></em><br></span></p>
<div class="question_part_label">e.</div>
</div>
<div class="question" style="padding-left: 20px;">
<p><span>\(\{ y \geqslant 32\}  \cup \{ y \leqslant - 32\} \)     <em><strong>(A1)(A1)</strong></em><strong>(ft)</strong><em><strong>(A1)</strong></em><strong>(ft)</strong></span></p>
<p><span><strong>Notes:</strong> Award <strong><em>(A1)</em>(ft)</strong> \(y \geqslant 32\) or \(y &gt; 32\) seen, <strong><em>(A1)</em>(ft)</strong> for \(y \leqslant - 32\) or \(y &lt; - 32\) , <em><strong>(A1)</strong></em> for weak (non-strict) inequalities used in both of the above.</span><br><span>Accept use of \(f\) in place of \(y\). Accept alternative interval notation.</span><br><span>Follow through from their (a) and (e).</span><br><span>If domain is given award <em><strong>(A0)(A0)(A0)</strong></em>.</span><br><span>Award <strong><em>(A0)(A1)</em>(ft)<em>(A1)</em>(ft)</strong> for \([ - 200{\text{, }} - 32]\) , \([32{\text{, }}200]\).</span><br><span>Award <em><strong>(A0)(A1)</strong></em><strong>(ft)</strong><em><strong>(A1)</strong></em><strong>(ft)</strong> for \(] - 200{\text{, }} - 32]\) , \([32{\text{, }}200[\).</span></p>
<p><span><em><strong>[3 marks]</strong></em><br></span></p>
<div class="question_part_label">f.</div>
</div>
<div class="question" style="padding-left: 20px;">
<p><span>\(f'(1) = - 45\)     <em><strong>(M1)(A1)</strong></em><strong>(ft)</strong><em><strong>(G2)</strong></em></span></p>
<p><span><strong>Notes:</strong> Award <em><strong>(M1)</strong></em> for \(f'(1)\) seen or substitution of \(x = 1\) into their derivative. Follow through from their derivative if working is seen.</span></p>
<p><span><em><strong>[2 marks]</strong></em><br></span></p>
<div class="question_part_label">g.</div>
</div>
<div class="question" style="padding-left: 20px;">
<p><span>\(x = - 1\)     <em><strong>(M1)(A1)</strong></em><strong>(ft)</strong><em><strong>(G2)</strong></em></span></p>
<p><span><strong>Notes:</strong> Award <em><strong>(M1)</strong></em> for equating their derivative to their \( - 45\) or for seeing parallel lines on their graph in the approximately correct position.</span></p>
<p><span><em><strong>[2 marks]</strong></em><br></span></p>
<div class="question_part_label">h.</div>
</div>
<h2 style="margin-top: 1em">Examiners report</h2>
<div class="question" style="padding-left: 20px;">
<p><span style="font-family: times new roman,times; font-size: medium;">As usual and by intention, this question caused the most difficulty in terms of its content; however, for those with a sound grasp of the topic, there were many very successful attempts. Much of the question could have been answered successfully by using the GDC, however, it was also clear that a number of candidates did not connect the question they were attempting with the curve that they had either sketched or were viewing on their GDC. Where there was no alternative to using the calculus, many candidates struggled.</span></p>
<p><span style="font-family: times new roman,times; font-size: medium;">The majority of sketches were drawn sloppily and with little attention to detail. Teachers must impress on their students that a mathematical sketch is designed to illustrate the main points of a curve &ndash; the smooth nature by which it changes, any symmetries (reflectional or rotational), positions of turning points, intercepts with axes and the behaviour of a curve as it approaches an asymptote. There must also be some indication of the dimensions used for the &ldquo;window&rdquo;.</span></p>
<p><span style="font-family: times new roman,times; font-size: medium;">Differentiation of terms with negative indices remains a testing process for the majority; it will continue to be tested.</span></p>
<p><span style="font-family: times new roman,times; font-size: medium;">It was also evident that some centres do not teach the differential calculus.</span></p>
<div class="question_part_label">a.</div>
</div>
<div class="question" style="padding-left: 20px;">
<p><span style="font-family: times new roman,times; font-size: medium;">As usual and by intention, this question caused the most difficulty in terms of its content; however, for those with a sound grasp of the topic, there were many very successful attempts. Much of the question could have been answered successfully by using the GDC, however, it was also clear that a number of candidates did not connect the question they were attempting with the curve that they had either sketched or were viewing on their GDC. Where there was no alternative to using the calculus, many candidates struggled.</span></p>
<p><span style="font-family: times new roman,times; font-size: medium;">The majority of sketches were drawn sloppily and with little attention to detail. Teachers must impress on their students that a mathematical sketch is designed to illustrate the main points of a curve &ndash; the smooth nature by which it changes, any symmetries (reflectional or rotational), positions of turning points, intercepts with axes and the behaviour of a curve as it approaches an asymptote. There must also be some indication of the dimensions used for the &ldquo;window&rdquo;.</span></p>
<p><span style="font-family: times new roman,times; font-size: medium;">Differentiation of terms with negative indices remains a testing process for the majority; it will continue to be tested.</span></p>
<p><span style="font-family: times new roman,times; font-size: medium;">It was also evident that some centres do not teach the differential calculus.</span></p>
<div class="question_part_label">b.</div>
</div>
<div class="question" style="padding-left: 20px;">
<p><span style="font-family: times new roman,times; font-size: medium;">As usual and by intention, this question caused the most difficulty in terms of its content; however, for those with a sound grasp of the topic, there were many very successful attempts. Much of the question could have been answered successfully by using the GDC, however, it was also clear that a number of candidates did not connect the question they were attempting with the curve that they had either sketched or were viewing on their GDC. Where there was no alternative to using the calculus, many candidates struggled.</span></p>
<p><span style="font-family: times new roman,times; font-size: medium;">The majority of sketches were drawn sloppily and with little attention to detail. Teachers must impress on their students that a mathematical sketch is designed to illustrate the main points of a curve &ndash; the smooth nature by which it changes, any symmetries (reflectional or rotational), positions of turning points, intercepts with axes and the behaviour of a curve as it approaches an asymptote. There must also be some indication of the dimensions used for the &ldquo;window&rdquo;.</span></p>
<p><span style="font-family: times new roman,times; font-size: medium;">Differentiation of terms with negative indices remains a testing process for the majority; it will continue to be tested.</span></p>
<p><span style="font-family: times new roman,times; font-size: medium;">It was also evident that some centres do not teach the differential calculus.</span></p>
<div class="question_part_label">c.</div>
</div>
<div class="question" style="padding-left: 20px;">
<p><span style="font-family: times new roman,times; font-size: medium;">As usual and by intention, this question caused the most difficulty in terms of its content; however, for those with a sound grasp of the topic, there were many very successful attempts. Much of the question could have been answered successfully by using the GDC, however, it was also clear that a number of candidates did not connect the question they were attempting with the curve that they had either sketched or were viewing on their GDC. Where there was no alternative to using the calculus, many candidates struggled.</span></p>
<p><span style="font-family: times new roman,times; font-size: medium;">The majority of sketches were drawn sloppily and with little attention to detail. Teachers must impress on their students that a mathematical sketch is designed to illustrate the main points of a curve &ndash; the smooth nature by which it changes, any symmetries (reflectional or rotational), positions of turning points, intercepts with axes and the behaviour of a curve as it approaches an asymptote. There must also be some indication of the dimensions used for the &ldquo;window&rdquo;.</span></p>
<p><span style="font-family: times new roman,times; font-size: medium;">Differentiation of terms with negative indices remains a testing process for the majority; it will continue to be tested.</span></p>
<p><span style="font-family: times new roman,times; font-size: medium;">It was also evident that some centres do not teach the differential calculus.</span></p>
<div class="question_part_label">d.</div>
</div>
<div class="question" style="padding-left: 20px;">
<p><span style="font-family: times new roman,times; font-size: medium;">As usual and by intention, this question caused the most difficulty in terms of its content; however, for those with a sound grasp of the topic, there were many very successful attempts. Much of the question could have been answered successfully by using the GDC, however, it was also clear that a number of candidates did not connect the question they were attempting with the curve that they had either sketched or were viewing on their GDC. Where there was no alternative to using the calculus, many candidates struggled.</span></p>
<p><span style="font-family: times new roman,times; font-size: medium;">The majority of sketches were drawn sloppily and with little attention to detail. Teachers must impress on their students that a mathematical sketch is designed to illustrate the main points of a curve &ndash; the smooth nature by which it changes, any symmetries (reflectional or rotational), positions of turning points, intercepts with axes and the behaviour of a curve as it approaches an asymptote. There must also be some indication of the dimensions used for the &ldquo;window&rdquo;.</span></p>
<p><span style="font-family: times new roman,times; font-size: medium;">Differentiation of terms with negative indices remains a testing process for the majority; it will continue to be tested.</span></p>
<p><span style="font-family: times new roman,times; font-size: medium;">It was also evident that some centres do not teach the differential calculus.</span></p>
<div class="question_part_label">e.</div>
</div>
<div class="question" style="padding-left: 20px;">
<p><span style="font-family: times new roman,times; font-size: medium;">As usual and by intention, this question caused the most difficulty in terms of its content; however, for those with a sound grasp of the topic, there were many very successful attempts. Much of the question could have been answered successfully by using the GDC, however, it was also clear that a number of candidates did not connect the question they were attempting with the curve that they had either sketched or were viewing on their GDC. Where there was no alternative to using the calculus, many candidates struggled.</span></p>
<p><span style="font-family: times new roman,times; font-size: medium;">The majority of sketches were drawn sloppily and with little attention to detail. Teachers must impress on their students that a mathematical sketch is designed to illustrate the main points of a curve &ndash; the smooth nature by which it changes, any symmetries (reflectional or rotational), positions of turning points, intercepts with axes and the behaviour of a curve as it approaches an asymptote. There must also be some indication of the dimensions used for the &ldquo;window&rdquo;.</span></p>
<p><span style="font-family: times new roman,times; font-size: medium;">Differentiation of terms with negative indices remains a testing process for the majority; it will continue to be tested.</span></p>
<p><span style="font-family: times new roman,times; font-size: medium;">It was also evident that some centres do not teach the differential calculus.</span></p>
<div class="question_part_label">f.</div>
</div>
<div class="question" style="padding-left: 20px;">
<p><span style="font-family: times new roman,times; font-size: medium;">As usual and by intention, this question caused the most difficulty in terms of its content; however, for those with a sound grasp of the topic, there were many very successful attempts. Much of the question could have been answered successfully by using the GDC, however, it was also clear that a number of candidates did not connect the question they were attempting with the curve that they had either sketched or were viewing on their GDC. Where there was no alternative to using the calculus, many candidates struggled.</span></p>
<p><span style="font-family: times new roman,times; font-size: medium;">The majority of sketches were drawn sloppily and with little attention to detail. Teachers must impress on their students that a mathematical sketch is designed to illustrate the main points of a curve &ndash; the smooth nature by which it changes, any symmetries (reflectional or rotational), positions of turning points, intercepts with axes and the behaviour of a curve as it approaches an asymptote. There must also be some indication of the dimensions used for the &ldquo;window&rdquo;.</span></p>
<p><span style="font-family: times new roman,times; font-size: medium;">Differentiation of terms with negative indices remains a testing process for the majority; it will continue to be tested.</span></p>
<p><span style="font-family: times new roman,times; font-size: medium;">It was also evident that some centres do not teach the differential calculus.</span></p>
<div class="question_part_label">g.</div>
</div>
<div class="question" style="padding-left: 20px;">
<p><span style="font-family: times new roman,times; font-size: medium;">As usual and by intention, this question caused the most difficulty in terms of its content; however, for those with a sound grasp of the topic, there were many very successful attempts. Much of the question could have been answered successfully by using the GDC, however, it was also clear that a number of candidates did not connect the question they were attempting with the curve that they had either sketched or were viewing on their GDC. Where there was no alternative to using the calculus, many candidates struggled.</span></p>
<p><span style="font-family: times new roman,times; font-size: medium;">The majority of sketches were drawn sloppily and with little attention to detail. Teachers must impress on their students that a mathematical sketch is designed to illustrate the main points of a curve &ndash; the smooth nature by which it changes, any symmetries (reflectional or rotational), positions of turning points, intercepts with axes and the behaviour of a curve as it approaches an asymptote. There must also be some indication of the dimensions used for the &ldquo;window&rdquo;.</span></p>
<p><span style="font-family: times new roman,times; font-size: medium;">Differentiation of terms with negative indices remains a testing process for the majority; it will continue to be tested.</span></p>
<p><span style="font-family: times new roman,times; font-size: medium;">It was also evident that some centres do not teach the differential calculus.</span></p>
<div class="question_part_label">h.</div>
</div>
<br><hr><br><div class="specification">
<p><span style="font-family: times new roman,times; font-size: medium;">The diagram shows a sketch of the function <em>f</em> (<em>x</em>) = 4<em>x</em><sup>3</sup> &minus; 9<em>x</em><sup>2</sup> &minus; 12<em>x</em> + 3.</span></p>
<p style="text-align: center;"><span style="font-family: times new roman,times; font-size: medium;"><img src="data:image/png;base64,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" alt></span></p>
</div>

<div class="question" style="padding-left: 20px; padding-right: 20px;">
<p><span>Write down the values of <em>x</em> where the graph of <em>f</em> (<em>x</em>) intersects the <em>x</em>-axis.</span></p>
<div class="marks">[3]</div>
<div class="question_part_label">a.</div>
</div>
<div class="question" style="padding-left: 20px; padding-right: 20px;">
<p><span>Write down <em>f </em>′(<em>x</em>).</span></p>
<div class="marks">[3]</div>
<div class="question_part_label">b.</div>
</div>
<div class="question" style="padding-left: 20px; padding-right: 20px;">
<p><span>Find the value of the local maximum of <em>y</em> = <em>f</em> (<em>x</em>).</span></p>
<div class="marks">[4]</div>
<div class="question_part_label">c.</div>
</div>
<div class="question" style="padding-left: 20px; padding-right: 20px;">
<p><span>Let P be the point where the graph of <em>f</em> (<em>x</em>) intersects the <em>y</em> axis.<br></span></p>
<p><span>Write down the coordinates of P.</span></p>
<div class="marks">[1]</div>
<div class="question_part_label">d.</div>
</div>
<div class="question" style="padding-left: 20px; padding-right: 20px;">
<p><span><span>Let P be the point where the graph of <em>f</em> (<em>x</em>) intersects the <em>y</em> axis.</span></span></p>
<p><span>Find the gradient of the curve at P.</span></p>
<div class="marks">[2]</div>
<div class="question_part_label">e.</div>
</div>
<div class="question" style="padding-left: 20px; padding-right: 20px;">
<p><span>The line, <em>L</em>, is the tangent to the graph of <em>f</em> (<em>x</em>) at P.</span></p>
<p><span>Find the equation of <em>L</em> in the form <em>y</em> = <em>mx</em> +<em> c</em>.</span></p>
<div class="marks">[2]</div>
<div class="question_part_label">f.</div>
</div>
<div class="question" style="padding-left: 20px; padding-right: 20px;">
<p><span>There is a second point, Q, on the curve at which the tangent to <em>f</em> (<em>x</em>) is parallel to <em>L</em>.</span></p>
<p><span>Write down the gradient of the tangent at Q.</span></p>
<div class="marks">[1]</div>
<div class="question_part_label">g.</div>
</div>
<div class="question" style="padding-left: 20px; padding-right: 20px;">
<p><span>There is a second point, Q, on the curve at which the tangent to <em>f</em> (<em>x</em>) is parallel to <em>L</em>.</span></p>
<p><span>Calculate the <em>x</em>-coordinate of Q.</span></p>
<div class="marks">[3]</div>
<div class="question_part_label">h.</div>
</div>
<h2 style="margin-top: 1em">Markscheme</h2>
<div class="question" style="padding-left: 20px;">
<p><span>–1.10, 0.218, 3.13     <em><strong>(A1)(A1)(A1)</strong></em></span></p>
<p><span><em><strong>[3 marks]</strong></em></span></p>
<div class="question_part_label">a.</div>
</div>
<div class="question" style="padding-left: 20px;">
<p><span><em>f </em><span>′</span>(<em>x</em>) = 12<em>x</em><sup>2</sup> – 18<em>x</em> – 12     <em><strong>(A1)(A1)(A1)</strong></em></span></p>
<p><br><span><strong>Note:</strong> Award <em><strong>(A1)</strong> </em>for each correct term and award maximum of<em><strong> (A1)(A1)</strong></em> if other terms seen.</span></p>
<p><span> </span></p>
<p><em><strong><span>[3 marks]</span></strong></em></p>
<div class="question_part_label">b.</div>
</div>
<div class="question" style="padding-left: 20px;">
<p><span><span><em>f </em>′(<em>x</em>)</span> = 0<em><strong>     (M1)</strong></em></span><br><span><em>x</em> = –0.5, 2</span><br><span><em>x</em> = –0.5   <em><strong>  (A1)</strong></em></span></p>
<p><br><span><strong>Note:</strong> If <em>x</em> = –0.5 not stated, can be inferred from working below.</span></p>
<p><br><span><em>y</em> = 4(–0.5)<sup>3</sup> – 9(–0.5)<sup>2 </sup>– 12(–0.5) + 3     <em><strong>(M1)</strong></em></span><br><span><em>y</em> = 6.25   <em><strong>  (A1)(G3)</strong></em></span></p>
<p> </p>
<p><span><strong>Note:</strong> Award <em><strong>(M1)</strong></em> for their value of <em>x</em> substituted into <em>f</em> (<em>x</em>).</span></p>
<p><span>Award <em><strong>(M1)(G2)</strong></em> if sketch shown as method. If coordinate pair given then award <em><strong>(M1)(A1)(M1)(A0)</strong></em>. If coordinate pair given with no working award <em><strong>(G2)</strong></em>.</span></p>
<p><span> </span></p>
<p><em><strong><span>[4 marks]</span></strong></em></p>
<div class="question_part_label">c.</div>
</div>
<div class="question" style="padding-left: 20px;">
<p><span>(0, 3)     <em><strong>(A1)</strong></em></span></p>
<p><br><span><strong>Note:</strong> Accept <em>x</em> = 0,<em> y</em> = 3.</span></p>
<p><span> </span></p>
<p><em><strong><span>[1 mark]</span></strong></em></p>
<div class="question_part_label">d.</div>
</div>
<div class="question" style="padding-left: 20px;">
<p><span><em>f </em>′(0) = –12     <em><strong>(M1)(A1)</strong></em><strong>(ft)</strong><em><strong>(G2)</strong></em></span></p>
<p><span><em><strong> </strong></em></span></p>
<p><span><strong>Note:</strong> Award <em><strong>(M1)</strong> </em>for substituting <em>x</em> = 0 into their derivative.</span></p>
<p><span> </span></p>
<p><em><strong><span>[2 marks]</span></strong></em></p>
<div class="question_part_label">e.</div>
</div>
<div class="question" style="padding-left: 20px;">
<p><span>Tangent: <em>y</em> = –12<em>x</em> + 3     <em><strong>(A1)</strong></em><strong>(ft)</strong><em><strong>(A1)(G2)</strong></em></span></p>
<p><br><span><strong>Note:</strong> Award <strong><em>(A1)</em>(ft)</strong> for their gradient, <em><strong>(A1)</strong></em> for intercept = 3.</span></p>
<p><span>Award<em><strong> (A1)(A0)</strong></em> if <em>y</em> = not seen.</span></p>
<p><span> </span></p>
<p><em><strong><span>[2 marks]</span></strong></em></p>
<div class="question_part_label">f.</div>
</div>
<div class="question" style="padding-left: 20px;">
<p><span>–12   <strong>  <em>(A1)</em>(ft)</strong></span></p>
<p><br><span><strong>Note:</strong> Follow through from their part (e).</span></p>
<p><span> </span></p>
<p><em><strong><span>[1 mark]</span></strong></em></p>
<div class="question_part_label">g.</div>
</div>
<div class="question" style="padding-left: 20px;">
<p><span>12<em>x</em><sup>2</sup> – 18<em>x </em></span><span><span>– 12 = </span></span><span><span><span>–12</span></span>     <em><strong>(M1)</strong></em></span></p>
<p><span>12<em>x</em><sup>2</sup> </span><span><span>– </span>18<em>x</em> = 0     <em><strong>(M1)</strong></em></span></p>
<p><span><em>x</em> = 1.5, 0</span></p>
<p><span>At Q <em>x</em> = 1.5     <em><strong>(A1)</strong></em><strong>(ft)</strong><em><strong>(G2)</strong></em></span></p>
<p><em><strong><span> </span></strong></em></p>
<p><span><strong>Note:</strong> Award <em><strong>(M1)(G2)</strong></em> for 12<em>x</em><sup>2</sup> </span><span><span>– 18</span><em>x </em></span><span><span>– 12 = </span></span><span><span><span>–12</span></span> followed by </span><span><em>x</em> = 1.5.</span></p>
<p><span>Follow through from their part (g).</span></p>
<p><span> </span></p>
<p><em><strong><span>[3 marks]</span></strong></em></p>
<div class="question_part_label">h.</div>
</div>
<h2 style="margin-top: 1em">Examiners report</h2>
<div class="question" style="padding-left: 20px;">
<p><span style="font-family: times new roman,times; font-size: medium;">This question was either very well done &ndash; by the majority &ndash; or very poor and incomplete attempts were seen. This would perhaps indicate a lack of preparation in this area of the syllabus from some centres, though it is recognised that the differential calculus is one of the more problematic topics for the candidature.</span></p>
<p><span style="font-family: times new roman,times; font-size: medium;">It was however disappointing to note the number of candidates who do not use the GDC to good effect; in part (a) for example, the zeros were not found accurately due to &ldquo;trace&rdquo; being used; this is not a suitable approach &ndash; there is a built-in zero finder which should be used. Much of the question was accessible via a GDC approach, a sketch was given that could have been verified on the GDC; this was lost on many.</span></p>
<div class="question_part_label">a.</div>
</div>
<div class="question" style="padding-left: 20px;">
<p><span style="font-family: times new roman,times; font-size: medium;">This question was either very well done &ndash; by the majority &ndash; or very poor and incomplete attempts were seen. This would perhaps indicate a lack of preparation in this area of the syllabus from some centres, though it is recognised that the differential calculus is one of the more problematic topics for the candidature.</span></p>
<p><span style="font-family: times new roman,times; font-size: medium;">It was however disappointing to note the number of candidates who do not use the GDC to good effect; in part (a) for example, the zeros were not found accurately due to &ldquo;trace&rdquo; being used; this is not a suitable approach &ndash; there is a built-in zero finder which should be used. Much of the question was accessible via a GDC approach, a sketch was given that could have been verified on the GDC; this was lost on many.</span></p>
<div class="question_part_label">b.</div>
</div>
<div class="question" style="padding-left: 20px;">
<p><span style="font-family: times new roman,times; font-size: medium;">This question was either very well done &ndash; by the majority &ndash; or very poor and incomplete attempts were seen. This would perhaps indicate a lack of preparation in this area of the syllabus from some centres, though it is recognised that the differential calculus is one of the more problematic topics for the candidature.</span></p>
<p><span style="font-family: times new roman,times; font-size: medium;">It was however disappointing to note the number of candidates who do not use the GDC to good effect; in part (a) for example, the zeros were not found accurately due to &ldquo;trace&rdquo; being used; this is not a suitable approach &ndash; there is a built-in zero finder which should be used. Much of the question was accessible via a GDC approach, a sketch was given that could have been verified on the GDC; this was lost on many.</span></p>
<div class="question_part_label">c.</div>
</div>
<div class="question" style="padding-left: 20px;">
<p><span style="font-family: times new roman,times; font-size: medium;">This question was either very well done &ndash; by the majority &ndash; or very poor and incomplete attempts were seen. This would perhaps indicate a lack of preparation in this area of the syllabus from some centres, though it is recognised that the differential calculus is one of the more problematic topics for the candidature.</span></p>
<p><span style="font-family: times new roman,times; font-size: medium;">It was however disappointing to note the number of candidates who do not use the GDC to good effect; in part (a) for example, the zeros were not found accurately due to &ldquo;trace&rdquo; being used; this is not a suitable approach &ndash; there is a built-in zero finder which should be used. Much of the question was accessible via a GDC approach, a sketch was given that could have been verified on the GDC; this was lost on many.</span></p>
<div class="question_part_label">d.</div>
</div>
<div class="question" style="padding-left: 20px;">
<p><span style="font-family: times new roman,times; font-size: medium;">This question was either very well done &ndash; by the majority &ndash; or very poor and incomplete attempts were seen. This would perhaps indicate a lack of preparation in this area of the syllabus from some centres, though it is recognised that the differential calculus is one of the more problematic topics for the candidature.</span></p>
<p><span style="font-family: times new roman,times; font-size: medium;">It was however disappointing to note the number of candidates who do not use the GDC to good effect; in part (a) for example, the zeros were not found accurately due to &ldquo;trace&rdquo; being used; this is not a suitable approach &ndash; there is a built-in zero finder which should be used. Much of the question was accessible via a GDC approach, a sketch was given that could have been verified on the GDC; this was lost on many.</span></p>
<div class="question_part_label">e.</div>
</div>
<div class="question" style="padding-left: 20px;">
<p><span style="font-family: times new roman,times; font-size: medium;">This question was either very well done &ndash; by the majority &ndash; or very poor and incomplete attempts were seen. This would perhaps indicate a lack of preparation in this area of the syllabus from some centres, though it is recognised that the differential calculus is one of the more problematic topics for the candidature.</span></p>
<p><span style="font-family: times new roman,times; font-size: medium;">It was however disappointing to note the number of candidates who do not use the GDC to good effect; in part (a) for example, the zeros were not found accurately due to &ldquo;trace&rdquo; being used; this is not a suitable approach &ndash; there is a built-in zero finder which should be used. Much of the question was accessible via a GDC approach, a sketch was given that could have been verified on the GDC; this was lost on many.</span></p>
<div class="question_part_label">f.</div>
</div>
<div class="question" style="padding-left: 20px;">
<p><span style="font-family: times new roman,times; font-size: medium;">This question was either very well done &ndash; by the majority &ndash; or very poor and incomplete attempts were seen. This would perhaps indicate a lack of preparation in this area of the syllabus from some centres, though it is recognised that the differential calculus is one of the more problematic topics for the candidature.</span></p>
<p><span style="font-family: times new roman,times; font-size: medium;">It was however disappointing to note the number of candidates who do not use the GDC to good effect; in part (a) for example, the zeros were not found accurately due to &ldquo;trace&rdquo; being used; this is not a suitable approach &ndash; there is a built-in zero finder which should be used. Much of the question was accessible via a GDC approach, a sketch was given that could have been verified on the GDC; this was lost on many.</span></p>
<div class="question_part_label">g.</div>
</div>
<div class="question" style="padding-left: 20px;">
<p><span style="font-family: times new roman,times; font-size: medium;">This question was either very well done &ndash; by the majority &ndash; or very poor and incomplete attempts were seen. This would perhaps indicate a lack of preparation in this area of the syllabus from some centres, though it is recognised that the differential calculus is one of the more problematic topics for the candidature.</span></p>
<p><span style="font-family: times new roman,times; font-size: medium;">It was however disappointing to note the number of candidates who do not use the GDC to good effect; in part (a) for example, the zeros were not found accurately due to &ldquo;trace&rdquo; being used; this is not a suitable approach &ndash; there is a built-in zero finder which should be used. Much of the question was accessible via a GDC approach, a sketch was given that could have been verified on the GDC; this was lost on many.</span></p>
<div class="question_part_label">h.</div>
</div>
<br><hr><br><div class="specification">
<p><span style="font-size: medium; font-family: times new roman,times;">When Geraldine travels to work she can travel either by car (<em>C</em>), bus (<em>B</em>) or train (<em>T</em>). She travels by car on one day in five. She uses the bus 50 % of the time. The probabilities of her being late (<em>L</em>) when travelling by car, bus or train are 0.05, 0.12 and 0.08 respectively.</span></p>
</div>

<div class="specification">
<p><em><span style="font-size: medium; font-family: times new roman,times;">It is <strong>not</strong> necessary to use graph paper for this question.</span></em></p>
</div>

<div class="question" style="padding-left: 20px; padding-right: 20px;">
<p><span>Copy the tree diagram below and fill in all the probabilities, where <em>NL</em> represents not late, to represent this information.</span></p>
<p><img src="data:image/png;base64,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" alt></p>
<div class="marks">[5]</div>
<div class="question_part_label">i.a.</div>
</div>
<div class="question" style="padding-left: 20px; padding-right: 20px;">
<p><span>Find the probability that Geraldine travels by bus and is late.</span></p>
<div class="marks">[1]</div>
<div class="question_part_label">i.b.</div>
</div>
<div class="question" style="padding-left: 20px; padding-right: 20px;">
<p><span>Find the probability that Geraldine is late.</span></p>
<div class="marks">[3]</div>
<div class="question_part_label">i.c.</div>
</div>
<div class="question" style="padding-left: 20px; padding-right: 20px;">
<p><span>Find the probability that Geraldine travelled by train, given that she is late.</span></p>
<div class="marks">[3]</div>
<div class="question_part_label">i.d.</div>
</div>
<div class="question" style="padding-left: 20px; padding-right: 20px;">
<p><span>Sketch the curve of the function \(f (x) = x^3 − 2x^2 + x − 3\) for values of \(x\) from −2 to 4, giving the intercepts with both axes.</span></p>
<div class="marks">[3]</div>
<div class="question_part_label">ii.a.</div>
</div>
<div class="question" style="padding-left: 20px; padding-right: 20px;">
<p><span>On the same diagram, sketch the line \(y = 7 − 2x\) and find the coordinates of the point of intersection of the line with the curve.</span></p>
<div class="marks">[3]</div>
<div class="question_part_label">ii.b.</div>
</div>
<div class="question" style="padding-left: 20px; padding-right: 20px;">
<p><span>Find the value of the gradient of the curve where \(x = 1.7\) .</span></p>
<div class="marks">[2]</div>
<div class="question_part_label">ii.c.</div>
</div>
<h2 style="margin-top: 1em">Markscheme</h2>
<div class="question" style="padding-left: 20px;">
<p><span><img src="data:image/png;base64,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" alt></span></p>
<p><em><span>Award <strong>(A1)</strong> for 0.5 at B, <strong>(A1)</strong> for 0.3 at T, then <strong>(A1)</strong> for each correct pair. Accept fractions or percentages.     <strong>(A5)</strong></span></em></p>
<p><em><span><strong>[5 marks]<br></strong></span></em></p>
<div class="question_part_label">i.a.</div>
</div>
<div class="question" style="padding-left: 20px;">
<p><span>0.06 (<em>accept</em> \(0.5 \times 0.12\) <em>or</em> 6%)     <em><strong>(A1)</strong></em><strong>(ft)</strong></span></p>
<p><span><strong><em>[1 mark]</em><br></strong></span></p>
<div class="question_part_label">i.b.</div>
</div>
<div class="question" style="padding-left: 20px;">
<p><span><em>for a relevant two-factor product, either</em> \(C \times L\) <em>or</em> \(T \times L\)     </span><span><em><strong>(M1)</strong></em></span></p>
<p><span> <em>for summing three two-factor products  </em>   <em><strong>(M1)</strong></em></span></p>
<p><span>\((0.2 \times 0.05 + 0.06 + 0.3 \times 0.08)\)</span></p>
<p><span>0.094     <em><strong>(A1)</strong></em><strong>(ft)</strong><em><strong>(G2)</strong></em></span></p>
<p><span><em><strong>[3 marks]<br></strong></em></span></p>
<div class="question_part_label">i.c.</div>
</div>
<div class="question" style="padding-left: 20px;">
<p><span>\(\frac{{0.3 \times 0.08}}{{0.094}}\)     <em><strong>(M1)(A1)</strong></em><strong>(ft)</strong></span></p>
<p><span><em><em>award <strong>(M1)</strong> for substituted conditional probability formula seen, </em></em></span><span><em><strong>(A1)</strong></em><strong>(ft) </strong></span><span><em><em>for correct substitution</em></em></span></p>
<p><span>= 0.255<em><em>     <strong>(A1)</strong></em></em><strong>(ft)</strong><em><em><strong>(G2)</strong><br></em></em></span></p>
<p><span><em><em><strong>[3 marks]<br></strong></em></em></span></p>
<div class="question_part_label">i.d.</div>
</div>
<div class="question" style="padding-left: 20px;">
<p><img src="data:image/png;base64,iVBORw0KGgoAAAANSUhEUgAAAVkAAADfCAIAAADTDjf9AAAQD0lEQVR4nO3dT4jc5hnH8Tew4ENNBYHF7EmHPRSMiwhpTw7IsOBCIdHJh9JiBQIJ2AcRTCCFggiFmKYHQcEljjG6pDE1KXMIoUlw0KWQQCjCkGRbCAi6tg/uQf4HaTCDengSeXZ2Znb+aB49z/v+Psdm8Cp15rvSI72vTAMA0DSm7wNwTp7nfR8CiFNVVRiG/R4DWsCqKApjTFVVfR8IyBJFke/7/R4DWsAqDENjTBzHfR8ICEK/IQaDQb+HgRbwob9yglMDaAVB0PsFQoMWcKKTAoJTAyBZlgn53YAWMBk9KcCpAZC6rj3PS9O07wNpGrSAzehJAU4NgMRx7Pt+Xdd9H0jToAU8Dp4U4NQAhIwMW2gBh4MnBTg1ACEjwxZasHb0GAnxfd8YQ/8RECHnh8Asz3NjTFmWfR/IE2gBqzRNjTFFUfR9INAnGhkmSdL3geyDFrBCC6BpmiRJPM+TdkqIFrBCC6AsS2OMwGUpaAErtABoTtT3UUyAFrBCCxwncGTYQgtYoQUukzkybKEFrNACl8kcGbbQAlZogbPEjgxbaAErtMBZYRgGQdD3UcyCFrBCC9xEI0Phf+9oASu0wEF1Xfu+L3/tCVrACi1wUJqmkkeGLbSAFVrgmqqqjDFZlvV9IIdDC1ihBa6RPzJsoQWs0AKnDAYDRX/daAErtMAdWkaGLbSAFVrgDi0jwxZawAotcISikWELLWCFFjhC0ciwhRawQgtcoGtk2EILWKEF1lM3MmyhBazQAuupGxm20AJWaIHdNI4MW2gBK7TAbhpHhi20gBVaYDGlI8MWWsAKLbAV7WWocWTYQgtYoQW2Er6X4TzQAlZogZVoL0OlI8MWWsAKLbCS6pFhCy1ghRbYR/LrTxaCFrBCCywj/PUnC0ELZqmqKk3TDpOPFljGgpFhCy2Ypa7rIAiMMWEYDgaD1f9AtMAmRVEIf/3JQtCCwxVFEUWRMcb3/TzPV/klgBbYJAgCmW9MXg5aMK+qquI49jzP87w0TauqWuIPQQuskWWZHSPDFlqwmLqu0zT1fd8YE8fxov8poAV2oJFhmqZ9H0iX0IIl5Xm+xCgBLbBDFEW+79sxMmyhBStZdJSAFliARoadzJJFQQs6MP8oAS2wgO/7URT1fRTdQws6M88oAS3QjrYtWm5yLBxa0L0ZowS0QDXatsiykWELLViXiaMEtEA1O9YgTYMWrNfYKCFJErRAKVqDZPHfHVrAYXSUYIy5cuVK30cEi7FpDdI0aAErumrocIED8LBpDdI0aAErmhdkWdbVAgdgYNkapGnQAlajs8NOFjgAA8vWIE2DFrA6eB9hxQUOsG4WP1AwBi1gNeOe4nILHGCtqqqybw3SNGgBq0OfL+hwrwRYnd0PFIxBC1jN+awRRgkSaH8P0qLQAlYLPXeIUUKPXHigYAxawGq5Z5AxSuBH52VOXaOhBaxWWY+AUQIbW3comA0tYLX62iSMEtatrmtbdyiYDS1g1dU6RYwS1sedBwrGoAWsOl+zjFFCt+x4S+py0AJWa9q/AKOErjjyuPFEaAGrte5lglHCiuhvx9kLLrSAFcO+RhglLMepx40nQgtYce5xhlHCQpx63HgitIAV/36HGCXMg16I5s7jxhOhBaz62vsUo4QZHHzceCK0gFW/+yBjlDCRlS9EWwJawErInugYJbRcW4w4A1rASkgLCEYJuDoYhRawEtUC4vIoAVcHo9ACVgJbQBwcJeDqYAxawEpsC1qOjBLo6sDBxYgzoAWs5LeAWD9KiKLIta1KDoUWsNLSAmLrKIGuDiw+61kOWsBKVwuIZaMEXB1Mgxaw0tiClh2jBFwdTIMWsFLdAqJ6lICrgxnQAlYWtIBoHCXg6mA2tICVNS0gukYJeLJoNrSAlWUtaMkfJeDJokOhBaxsbQERO0qgPYuw7mA2tICV3S0gAkcJYRji6uBQaAErF1pA5IwSsGfRnNACVu60oNXvKAE7ms4PLWDlYAtIX6OEIAgc39F0fmgBK2dbQJhHCY6/72BRaAErx1tAeEYJLr8NbTloASu0YNT6Rgn0rmRn34a2HLSAFVpw0DpGCXQlIuF2piJoASu0YJoORwlYgLQctIAVWjDb6qMEWoAUx/Eajs5yaAErtGBOS48S8Ijh0tACVmjBQhYdJeAm4irQAlZowRLmHCXQTUQ8Yrg0tIAVWrC02aME3ERcHVrACi1Y3cRRAnYxXB1awAot6MroKOGll17CTcTVoQWs0IJuVVX1/PPPP/XUU0eOHBGyV4JeaAErtKBbdV0HQXDixAkheyWohhawQgu6NfassfxtFyVDC1ihBR3K83zimEDstovCoQWs0IKulGU5eztTgdsuCocWsEILOkFjgnk2LJKz7aJ8aAErtKATSyxJxijhUGgBK7Rgdavsa4xRwgxoASu0YEWdLDrAKGEitIAVWrAK2pugq0UHGCWMQQtYoQWrWNPeBBglELSAFVqwtCRJPM9b329vjBLQAlZowXLosaI8z9f9g1weJaAFrNCCJRz6WFHn3BwloAWs0IJFdTsvXJRTowS0gBVasKggCHrfy9SRUQJawAotWAhdugs5Rbd+lIAWsEIL5kfPF0o7M7d4lIAWsFLVgke3//Hi2S1jjDHm9Kk/f/blcMoHHxQfXn3lwrM/On79P0/+x4fvvfXchtnn2VMf/3fOn03vPpL8ZlT7RgloASs9LXj88PPfHH/5evFg2Ay//OzN41tma/Pi53cnfPKbTy/82BhjzPZICx7d+ssLO9e+bj8/3D23s/H6pXvTcrIP3ThQ8e4jm0YJaAErPS345qMzb15/+MNXd3jj6s6G2f7j9ceTPz3cPXd6Xwvu3Lz+1Ug47v/77e2N1z69M8cPphsH86xHlsOOUQJawEpPC8b88/1fH5n1Zb6dnt3Xgv2GN6787Ol5LhDajQk0/oLVPkpAC1hpbcG9S69t/HTWl3lmC+a/QKDXHKj7Fo1ROkpAC1jpbMGjW3/9+ZEX378547s8qwXzXiCIuoO4OnWjBLSAlcYWDPfSM1tvPZkdTDSjBfNdIND/MwwrDpgpGiWgBaz0teDhh3869fqlW98d8rHpLZjnAoFt6VFfVIwS0AJWylowvPnxL35x7qt7h39yagvu/+vtn0y5Gfk9CgHn0qMeSR4loAWsNLVgPASP7vz9V2c+mvylHu6eO222tt7ZHb/n+PC9i5svnNv9dtoPUfQoQYdkjhLQAlZqWjC8eePClhmz9dtL94Z0f9GYV9Lb9MW/+8XFzSef2fcMwuP7n+xs7FwtplwfuBmClrRRAlrASk0L1o9CoOuZonWYNkrgn56gBazQAtKGQMjpsQRjo4T2CoLtANACVmhBM/KUMUJwUDtKaLEVAS1ghRaofsqYTVVVY7MahiKYEBjRZWEQBH0fSD9Onjx59OjRo0ePnjx5su9jkW58cLv+Ikz8iQAg1Obm5prOK00BjOI4NsZkWdb3gXC7du3a9vb29vb2Bx980Pex6HCwAseOHXvmmWfM2p5KwLyAlZvzAtw1WMJoBUYvDdb3VAJawMrBFiAEyzlYgVHrWOCAFrByrQUIwdLmHBN2uMABLWDlVAsGg4HneVEUIQTrVnSxwAEtYOVOC2j1obNrDXqx4igBLWDlSAvoX9ORZcjSLD1KQAtYudACum9q8cYkWiw6SkALWNndAnq+2PM8abt0uGz+UQJawMriFpRl6fu+7/syN/By3DyjBLSAla0tyPMc9w7lmz1KQAtYWdmCJElwy0CXiaMEtICVZS2oqooGBJgUajQ6SqiqCi1gZVML6FGiIAgwIFCtqqo0TcuyRAtY2dGCuq7p90mSJBgQWAMtYGVBC2hMiBuH9kELWKluQVmWtN8OTgeshBawUtoCujtNY2d1Bw9zQgtYqWtBWwHm/bmBH1rASlELih92ZEMFHIEWsJLfgrqu2wdRgiBABdyBFrCS3ILBYEAnAvSOQ5kHCeuDFrCS1oKqqvI8j6LI8zxjTBRFcl77C8zQgieKosjzPE3TKIrCH95rcqj25RZRFKU/oG2tDz6QJ6EFZVnmeR7HMf0L0jZkSAC43oKyLLMsa19T43leGIZxHI9+pWcYDAbt9z+OY4rCwYj4vj/6Ppzz589Pi0XnqqoqiiLLsiRJRt/GE4Yh/Quu+wBAC0dbMDoh830/juPBYNDhVvOEvof0VUzTNEkSysSxY8fGYkENIkmStH2Z/60qdEZDDiaJYpSmaZ7nWD4AEznXAlrCTU/RdrWx/PzGrhHKspz2TZ52ijHNwZpQR/DNhzm51YI8z33fp61derk8ljAvAJjIlRa0z9LHcdzjkAwtALGcaAF9AyWstEcLQCzLW1BVFZ0OpGna97E0DVoAgtncgqIopG28gxaAWNa2gN7hJe1lfmgBiGVnC+grJ+S6YBRaAGJZ2ALJ7/BCC0As21pAe/XLDEGDFoBgVrWAZgR9hODxw6/T157bMMYYc/rUtd27Uz6HFoBY9rRgMBj0NSMY7v1+Z+da8WDYDG/euLBlzJlzu99O/CRaAGJZ0oKqqmh9Qd8H0jS307NoAShkSQuCIJDxYs/H9z/ZOfLi+zeHk/8xWgBi2dACWnco4IGiO7sfnnt184VpJwUNWgCCqW9BWZbGmCzL+j6Qu19c3Px+/fDW+XTvu4kfQgtALPUtCIIgDMN1/5TLly9fvnz55UnefffdJ597UHz0xvEtY4784Yv/Tfpz0AIQS3cL6CZi5/sRHbS3t7e3t/fVJHfu3Nn30eGNqzsb5uzfvpz056AFIJbiFtR17XlekiR9H8iYb268+vTWO7uPJ/0ztADEUtwCGhkKuHcw6tHtj3+5ufm7S7cwLwBltLaATgpkrD56fP+Tna3vx4Ynjr9x/fqUwWGDFoBgWluQ57m8k4LDoQUgltYW0EbmfR/FwtACEEtlC2jpAcPtg86hBSCWyhbQG4r6PoploAUglr4W1HUteYeC2dACEEtfC+j5InVTQ4IWgFj6WhBFURRFfR/FktACEEtZC1RfIDRoAQimrAV0B0HpBUKDFoBgylqg9w4CQQtALGUt8H1fxnPHS0ILQCxNLaiqSvsXCS0AsTS1gO4m9n0UK0ELQCxNX60kSVQPCxq0AATT1IIwDFUPCxq0AATT1AJjzGAw6PsoVoIWgFhqWkD7HWtcmzgKLQCx1LTAgsFhgxaAYGq+XWmaah8cNmgBCKamBWEYatzIaAxaAGKpaUEQBNpvIjRoAQimpgV2fIXQAhBLRwvoJoKAt6euCi0AsXS0oCgKC24iNGgBCKbjC5Zlme/7fR9FB9ACEEtHC+y4odigBSCYjhZEUWTBDcUGLQDBdLTAglVJBC0AsXS0wPO8LMv6PooOoAUglo4WWPP9QQtALAUtoK3NLHi4oEELQDAFLbDm4YIGLQDBFHzH6J0IfR9FN9ACEEvBdyxN0yAI+j6KbqAFIJaOFtjxoFHTNFmWWTP7AMsoaEFd19q3NhuFEIBM/wdysfXsd2u0FwAAAABJRU5ErkJggg==" alt>     <em><strong><span>(G3)</span></strong></em></p>
<p><em><strong><span>[3 marks]<br></span></strong></em></p>
<div class="question_part_label">ii.a.</div>
</div>
<div class="question" style="padding-left: 20px;">
<p><span><em>line drawn with <strong>–ve</strong> gradient and <strong>+ve</strong> y-intercept</em>     <strong><em>(G1)</em></strong></span></p>
<p><span>(2.45, 2.11)     <em><strong>(G1)(G1)</strong></em></span></p>
<p><span><em><strong>[3 marks]<br></strong></em></span></p>
<div class="question_part_label">ii.b.</div>
</div>
<div class="question" style="padding-left: 20px;">
<p><span>\(f ' (1.7) = 3(1.7)^2 - 4(1.7) + 1\)     <em><strong>(M1)</strong></em><br></span></p>
<p><em><span>award <strong>(M1)</strong> for substituting in their</span></em><span> \(</span><span><span>f</span></span><span><span><span>' </span> </span> (x)\)</span></p>
<p><span>2.87     <em><strong>(A1)(G2)</strong></em></span></p>
<p><span><em><strong>[2 marks]<br></strong></em></span></p>
<div class="question_part_label">ii.c.</div>
</div>
<h2 style="margin-top: 1em">Examiners report</h2>
<div class="question" style="padding-left: 20px;">
<p><span style="font-size: medium; font-family: times new roman,times;">This should have been an easy first question but, even so, there were some candidates who were unable to fill in the tree diagram correctly let alone evaluate any probabilities. The majority of candidates were confident with answering parts (a), (b) and (c) but the conditional probability question was not well answered with few candidates managing to recognise that it was a conditional type.</span></p>
<p><span style="font-size: medium; font-family: times new roman,times;">&nbsp;</span></p>
<div class="question_part_label">i.a.</div>
</div>
<div class="question" style="padding-left: 20px;">
<p><span style="font-size: medium; font-family: times new roman,times;">This should have been an easy first question but, even so, there were some candidates who were unable to fill in the tree diagram correctly let alone evaluate any probabilities. The majority of candidates were confident with answering parts (a), (b) and (c) but the conditional probability question was not well answered with few candidates managing to recognise that it was a conditional type.</span></p>
<p><span style="font-size: medium; font-family: times new roman,times;">&nbsp;</span></p>
<div class="question_part_label">i.b.</div>
</div>
<div class="question" style="padding-left: 20px;">
<p><span style="font-size: medium; font-family: times new roman,times;">This should have been an easy first question but, even so, there were some candidates who were unable to fill in the tree diagram correctly let alone evaluate any probabilities. The majority of candidates were confident with answering parts (a), (b) and (c) but the conditional probability question was not well answered with few candidates managing to recognise that it was a conditional type.</span></p>
<p><span style="font-size: medium; font-family: times new roman,times;">&nbsp;</span></p>
<div class="question_part_label">i.c.</div>
</div>
<div class="question" style="padding-left: 20px;">
<p><span style="font-size: medium; font-family: times new roman,times;">This should have been an easy first question but, even so, there were some candidates who were unable to fill in the tree diagram correctly let alone evaluate any probabilities. The majority of candidates were confident with answering parts (a), (b) and (c) but the conditional probability question was not well answered with few candidates managing to recognise that it was a conditional type.</span></p>
<p><span style="font-size: medium; font-family: times new roman,times;">&nbsp;</span></p>
<div class="question_part_label">i.d.</div>
</div>
<div class="question" style="padding-left: 20px;">
<p><span style="font-size: medium; font-family: times new roman,times;">This should have been an easy first question but, even so, there were some candidates who were unable to fill in the tree diagram correctly let alone evaluate any probabilities. The majority of candidates were confident with answering parts (a), (b) and (c) but the conditional probability question was not well answered with few candidates managing to recognise that it was a conditional type.</span></p>
<p><span style="font-size: medium; font-family: times new roman,times;">The curve sketching and straight line were well drawn but not all candidates indicated the intersection points with the axes. In finding the line / curve intersection some candidates did not use the intersection function on the GDC. Few candidates managed the last part. Many just chose two sets of coordinates and used the gradient formula.</span></p>
<div class="question_part_label">ii.a.</div>
</div>
<div class="question" style="padding-left: 20px;">
<p><span style="font-size: medium; font-family: times new roman,times;">This should have been an easy first question but, even so, there were some candidates who were unable to fill in the tree diagram correctly let alone evaluate any probabilities. The majority of candidates were confident with answering parts (a), (b) and (c) but the conditional probability question was not well answered with few candidates managing to recognise that it was a conditional type.</span></p>
<p><span style="font-size: medium; font-family: times new roman,times;">The curve sketching and straight line were well drawn but not all candidates indicated the intersection points with the axes. In finding the line / curve intersection some candidates did not use the intersection function on the GDC. Few candidates managed the last part. Many just chose two sets of coordinates and used the gradient formula.</span></p>
<div class="question_part_label">ii.b.</div>
</div>
<div class="question" style="padding-left: 20px;">
<p><span style="font-size: medium; font-family: times new roman,times;">This should have been an easy first question but, even so, there were some candidates who were unable to fill in the tree diagram correctly let alone evaluate any probabilities. The majority of candidates were confident with answering parts (a), (b) and (c) but the conditional probability question was not well answered with few candidates managing to recognise that it was a conditional type.</span></p>
<p><span style="font-size: medium; font-family: times new roman,times;">The curve sketching and straight line were well drawn but not all candidates indicated the intersection points with the axes. In finding the line / curve intersection some candidates did not use the intersection function on the GDC. Few candidates managed the last part. Many just chose two sets of coordinates and used the gradient formula.</span></p>
<div class="question_part_label">ii.c.</div>
</div>
<br><hr><br><div class="specification">
<p><span style="font-size: medium; font-family: times new roman,times;">The following graph shows the temperature in degrees Celsius of Robert&rsquo;s cup of coffee, \(t\) minutes after pouring it out. The equation of the cooling graph is \(f (t) = 16 + 74 \times 2.8^{&minus;0.2t}\) where \(f (t)\) is the temperature and \(t\) is the time in minutes after pouring the coffee out.</span></p>
<p><span style="font-size: medium; font-family: times new roman,times;"><img src="data:image/png;base64,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" alt></span></p>
</div>

<div class="specification">
<p><span style="font-size: medium; font-family: times new roman,times;">Robert, who lives in the UK, travels to Belgium. The exchange rate is 1.37 euros to one British Pound (GBP) with a commission of 3 GBP, which is subtracted before the exchange takes place. Robert gives the bank 120 GBP.</span></p>
</div>

<div class="question" style="padding-left: 20px; padding-right: 20px;">
<p><span>Find the initial temperature of the coffee.</span></p>
<div class="marks">[1]</div>
<div class="question_part_label">i.a.</div>
</div>
<div class="question" style="padding-left: 20px; padding-right: 20px;">
<p><span>Write down the equation of the horizontal asymptote.</span></p>
<div class="marks">[1]</div>
<div class="question_part_label">i.b.</div>
</div>
<div class="question" style="padding-left: 20px; padding-right: 20px;">
<p><span>Find the room temperature.</span></p>
<div class="marks">[1]</div>
<div class="question_part_label">i.c.</div>
</div>
<div class="question" style="padding-left: 20px; padding-right: 20px;">
<p><span>Find the temperature of the coffee after 10 minutes.</span></p>
<div class="marks">[1]</div>
<div class="question_part_label">i.d.</div>
</div>
<div class="question" style="padding-left: 20px; padding-right: 20px;">
<p><span>Find the temperature of Robert’s coffee after being heated in the microwave for 30 <strong>seconds</strong> after it has reached the temperature in part (d).</span></p>
<div class="marks">[3]</div>
<div class="question_part_label">i.e.</div>
</div>
<div class="question" style="padding-left: 20px; padding-right: 20px;">
<p><span>Calculate the length of time it would take a similar cup of coffee, initially at 20°C, to be heated in the microwave to reach 100°C.</span></p>
<div class="marks">[4]</div>
<div class="question_part_label">i.f.</div>
</div>
<div class="question" style="padding-left: 20px; padding-right: 20px;">
<p><span>Calculate <strong>correct to 2 decimal places</strong> the amount of euros he receives.</span></p>
<div class="marks">[3]</div>
<div class="question_part_label">ii.a.</div>
</div>
<div class="question" style="padding-left: 20px; padding-right: 20px;">
<p><span>He buys 1 kilogram of Belgian chocolates at 1.35 euros per 100 g.</span></p>
<p><span>Calculate the cost of his chocolates in GBP <strong>correct to 2 decimal places</strong>.</span></p>
<div class="marks">[3]</div>
<div class="question_part_label">ii.b.</div>
</div>
<h2 style="margin-top: 1em">Markscheme</h2>
<div class="question" style="padding-left: 20px;">
<p><span><em>Unit penalty <strong>(UP)</strong> is applicable in part (i)(a)(c)(d)(e) and (f)</em></span></p>
<p><span><em><strong>(UP)</strong></em> 90°C     <em><strong>(A1)</strong></em></span></p>
<p><span><em><strong>[1 mark]<br></strong></em></span></p>
<div class="question_part_label">i.a.</div>
</div>
<div class="question" style="padding-left: 20px;">
<p><span><em>y</em> = 16     <em><strong>(A1)</strong></em></span></p>
<p><span><em><strong>[1 mark]<br></strong></em></span></p>
<div class="question_part_label">i.b.</div>
</div>
<div class="question" style="padding-left: 20px;">
<p><span><em><span><em>Unit penalty <strong>(UP)</strong> is applicable in part (i)(a)(c)(d)(e) and (f)</em></span></em></span></p>
<p><span><em><strong>(UP)</strong></em> 16°C <strong>(ft)</strong><em> from answer to part (b)     <strong>(A1)</strong></em><strong>(ft)</strong></span></p>
<p><span><strong><em>[1 mark]</em><br></strong></span></p>
<div class="question_part_label">i.c.</div>
</div>
<div class="question" style="padding-left: 20px;">
<p><span><em><span><em>Unit penalty <strong>(UP)</strong> is applicable in part (i)(a)(c)(d)(e) and (f)</em></span></em></span></p>
<p><span><em><strong>(UP)</strong></em> 25.4°C     <em><strong>(A1)</strong></em></span></p>
<p><span><em><strong><span>[1 mark]</span></strong></em></span><span><em><strong> </strong></em></span>  </p>
<div class="question_part_label">i.d.</div>
</div>
<div class="question" style="padding-left: 20px;">
<p><em><span><span><em><span><em>Unit penalty <strong>(UP)</strong> is applicable in part (i)(a)(c)(d)(e) and (f)</em></span></em></span></span></em></p>
<p><em><span>for seeing </span></em><span>2</span><span><sup>0.75</sup></span><em><span> or equivalent     <strong>(A1)</strong><br></span></em></p>
<p><em><span>for multiplying their (d) by their </span></em><span>2<sup>0.75</sup></span><em><span>     <strong>(M1)</strong></span></em></p>
<p><span><em><strong>(UP)</strong></em> 42.8°C     <em><strong>(A1)</strong></em><strong>(ft)</strong><em><strong>(G2)</strong></em></span></p>
<p><span><em><strong>[3 marks]<br></strong></em></span></p>
<div class="question_part_label">i.e.</div>
</div>
<div class="question" style="padding-left: 20px;">
<p><span><em><em><span>Unit penalty <strong>(UP)</strong> is applicable in part (i)(a)(c)(d)(e) and (f)<br></span></em></em></span></p>
<p><span><em>for seeing </em> \(20 \times 2^{1.5t} = 100\)     <em><strong>(A1)</strong></em></span></p>
<p><span><em>for seeing a value of t between</em> 1.54 <em>and</em> 1.56 <em>inclusive     <strong>(M1)(A1)</strong></em></span></p>
<p><span><em><strong>(UP)</strong></em> 1.55 minutes or 92.9 seconds     <em><strong>(A1)(G3)</strong></em></span></p>
<p><span><em><strong>[4 marks]<br></strong></em></span></p>
<div class="question_part_label">i.f.</div>
</div>
<div class="question" style="padding-left: 20px;">
<p><span><em>Financial accuracy penalty <strong>(FP)</strong> is applicable in part (ii) <strong>only</strong>.</em><br></span></p>
<p><span>\(120 - 3 = 117\)</span></p>
<p><span><em><strong>(FP)</strong></em> \(117 \times 1.37\)     <em><strong>(A1)</strong></em></span></p>
<p><span>= 160.29 euros<em> (correct answer only)     <strong>(M1)</strong></em></span></p>
<p><em><span>first <strong>(A1)</strong> for 117 seen, <strong>(M1)</strong> for multiplying by 1.37     <strong>(A1)(G2)</strong></span></em></p>
<p><em><span><strong>[3 marks]<br></strong></span></em></p>
<div class="question_part_label">ii.a.</div>
</div>
<div class="question" style="padding-left: 20px;">
<p><span><em>Financial accuracy penalty <strong>(FP)</strong> is applicable in part (ii) <strong>only</strong>.</em><br></span></p>
<p><span><em><strong>(FP)</strong></em> \(\frac{{13.5}}{{1.37}}\)     <em><strong>(A1)(M1)</strong></em></span></p>
<p><span>9.85 GBP <em>(answer correct to 2dp only)</em></span></p>
<p><span><em>first <strong>(A1)</strong> is for 13.5 seen, <strong>(M1)</strong> for dividing by 1.37</em>     <em><strong>(A1)</strong></em><strong>(ft)</strong><em><strong>(G3)</strong></em></span></p>
<p><span><em><strong>[3 marks]<br></strong></em></span></p>
<div class="question_part_label">ii.b.</div>
</div>
<h2 style="margin-top: 1em">Examiners report</h2>
<div class="question" style="padding-left: 20px;">
<p><span style="font-size: medium; font-family: times new roman,times;">Many candidates who had not lost a UP in question 2 lost one here. Parts (a), (c) and (d) were reasonably well tackled. Almost everybody had difficulty with the equation of the horizontal asymptote, a common answer being <em>y</em> = 20. Most of the candidates realised that 30 seconds was 0.5 minutes and calculated part (e) correctly. Part (f), solving an exponential equation, was a good discriminator. Trial and error was expected but many students did not think of doing this.</span></p>
<p><span style="font-size: medium; font-family: times new roman,times;">&nbsp;</span></p>
<div class="question_part_label">i.a.</div>
</div>
<div class="question" style="padding-left: 20px;">
<p><span style="font-size: medium; font-family: times new roman,times;">Many candidates who had not lost a UP in question 2 lost one here. Parts (a), (c) and (d) were reasonably well tackled. Almost everybody had difficulty with the equation of the horizontal asymptote, a common answer being <em>y</em> = 20. Most of the candidates realised that 30 seconds was 0.5 minutes and calculated part (e) correctly. Part (f), solving an exponential equation, was a good discriminator. Trial and error was expected but many students did not think of doing this.</span></p>
<p><span style="font-size: medium; font-family: times new roman,times;">&nbsp;</span></p>
<div class="question_part_label">i.b.</div>
</div>
<div class="question" style="padding-left: 20px;">
<p><span style="font-size: medium; font-family: times new roman,times;">Many candidates who had not lost a UP in question 2 lost one here. Parts (a), (c) and (d) were reasonably well tackled. Almost everybody had difficulty with the equation of the horizontal asymptote, a common answer being <em>y</em> = 20. Most of the candidates realised that 30 seconds was 0.5 minutes and calculated part (e) correctly. Part (f), solving an exponential equation, was a good discriminator. Trial and error was expected but many students did not think of doing this.</span></p>
<p><span style="font-size: medium; font-family: times new roman,times;">&nbsp;</span></p>
<div class="question_part_label">i.c.</div>
</div>
<div class="question" style="padding-left: 20px;">
<p><span style="font-size: medium; font-family: times new roman,times;">Many candidates who had not lost a UP in question 2 lost one here. Parts (a), (c) and (d) were reasonably well tackled. Almost everybody had difficulty with the equation of the horizontal asymptote, a common answer being <em>y</em> = 20. Most of the candidates realised that 30 seconds was 0.5 minutes and calculated part (e) correctly. Part (f), solving an exponential equation, was a good discriminator. Trial and error was expected but many students did not think of doing this.</span></p>
<p><span style="font-size: medium; font-family: times new roman,times;">&nbsp;</span></p>
<div class="question_part_label">i.d.</div>
</div>
<div class="question" style="padding-left: 20px;">
<p><span style="font-size: medium; font-family: times new roman,times;">Many candidates who had not lost a UP in question 2 lost one here. Parts (a), (c) and (d) were reasonably well tackled. Almost everybody had difficulty with the equation of the horizontal asymptote, a common answer being <em>y</em> = 20. Most of the candidates realised that 30 seconds was 0.5 minutes and calculated part (e) correctly. Part (f), solving an exponential equation, was a good discriminator. Trial and error was expected but many students did not think of doing this.</span></p>
<p><span style="font-size: medium; font-family: times new roman,times;">&nbsp;</span></p>
<div class="question_part_label">i.e.</div>
</div>
<div class="question" style="padding-left: 20px;">
<p><span style="font-size: medium; font-family: times new roman,times;">Many candidates who had not lost a UP in question 2 lost one here. Parts (a), (c) and (d) were reasonably well tackled. Almost everybody had difficulty with the equation of the horizontal asymptote, a common answer being <em>y</em> = 20. Most of the candidates realised that 30 seconds was 0.5 minutes and calculated part (e) correctly. Part (f), solving an exponential equation, was a good discriminator. Trial and error was expected but many students did not think of doing this.</span></p>
<p><span style="font-size: medium; font-family: times new roman,times;">&nbsp;</span></p>
<div class="question_part_label">i.f.</div>
</div>
<div class="question" style="padding-left: 20px;">
<p><span style="font-size: medium; font-family: times new roman,times;">The financial part was the best done question in the paper and a large majority of candidates gained full marks here.</span></p>
<div class="question_part_label">ii.a.</div>
</div>
<div class="question" style="padding-left: 20px;">
<p><span style="font-size: medium; font-family: times new roman,times;">The financial part was the best done question in the paper and a large majority of candidates gained full marks here.</span></p>
<div class="question_part_label">ii.b.</div>
</div>
<br><hr><br><div class="specification">
<p><span style="font-family: times new roman,times; font-size: medium;">Consider the function \(f:x \mapsto \frac{{kx}}{{{2^x}}}\).</span></p>
</div>

<div class="specification">
<p><span style="font-family: times new roman,times; font-size: medium;">The cost per person, in euros, when \(x\) people are invited to a party can be determined by the function </span></p>
<p style="text-align: center;"><span style="font-family: times new roman,times; font-size: medium;">\(C(x) = x + \frac{{100}}{x}\)</span></p>
</div>

<div class="question" style="padding-left: 20px; padding-right: 20px;">
<p><span>Given that \(f(1) = 2\), show that \(k = 4\).</span></p>
<div class="marks">[2]</div>
<div class="question_part_label">i.a.</div>
</div>
<div class="question" style="padding-left: 20px; padding-right: 20px;">
<p><span>Write down the values of \(q\) and \(r\) for the following table.</span></p>
<p><span><img src="data:image/png;base64,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" alt></span></p>
<div class="marks">[2]</div>
<div class="question_part_label">i.b.</div>
</div>
<div class="question" style="padding-left: 20px; padding-right: 20px;">
<p><span>As \(x\) increases from \( - 1\), the graph of \(y = f(x)\) reaches a maximum value and then decreases, behaving asymptotically.</span></p>
<p><span><span>Draw the graph of \(y = f(x)\) for \( - 1 \leqslant x \leqslant 8\). Use a scale of \({\text{1 cm}}\) to represent 1 unit on both axes. The position of the maximum, </span></span><span><span><span><span>\({\text{</span></span>M}}\), the</span> <span>\(y\)-intercept and the asymptotic behaviour should be clearly shown.</span></span></p>
<div class="marks">[4]</div>
<div class="question_part_label">i.c.</div>
</div>
<div class="question" style="padding-left: 20px; padding-right: 20px;">
<p><span>Using your graphic display calculator, find the coordinates of \({\text{M}}\), the maximum point on the graph of \(y = f(x)\).</span></p>
<div class="marks">[2]</div>
<div class="question_part_label">i.d.</div>
</div>
<div class="question" style="padding-left: 20px; padding-right: 20px;">
<p><span>Write down the equation of the horizontal asymptote to the graph of \(y = f(x)\).</span></p>
<div class="marks">[2]</div>
<div class="question_part_label">i.e.</div>
</div>
<div class="question" style="padding-left: 20px; padding-right: 20px;">
<p><span>(i) Draw and label the line \( y = 1\) on your graph.</span></p>
<p><span>(ii) The equation \(f(x) = 1\) has two solutions. One of the solutions is \(x = 4\). Use your <strong>graph</strong> to find the other solution.</span></p>
<div class="marks">[4]</div>
<div class="question_part_label">i.f.</div>
</div>
<div class="question" style="padding-left: 20px; padding-right: 20px;">
<p><span>Find \(C'(x)\).</span></p>
<div class="marks">[3]</div>
<div class="question_part_label">ii.a.</div>
</div>
<div class="question" style="padding-left: 20px; padding-right: 20px;">
<p><span>Show that the cost per person is a minimum when \(10\) people are invited to the party.</span></p>
<div class="marks">[2]</div>
<div class="question_part_label">ii.b.</div>
</div>
<div class="question" style="padding-left: 20px; padding-right: 20px;">
<p><span>Calculate the minimum cost per person.</span></p>
<div class="marks">[2]</div>
<div class="question_part_label">ii.c.</div>
</div>
<h2 style="margin-top: 1em">Markscheme</h2>
<div class="question" style="padding-left: 20px;">
<p><span><span>\(f(1) = \frac{k}{{{2^1}}}\)</span>     <em><strong>(M1)</strong></em></span></p>
<p><span><strong><br>Note: <em>(M1)</em></strong> for substituting \(x = 1\) into the formula.</span></p>
<p><br><span>\(\frac{k}{2} = 2\)     <em><strong>(M1)</strong></em></span></p>
<p><span><strong><br>Note: <em>(M1)</em></strong> for equating to 2.</span></p>
<p><br><span>\(k = 4\)     <em><strong>(AG)</strong></em></span></p>
<p><span><em><strong>[2 marks]<br></strong></em></span></p>
<div class="question_part_label">i.a.</div>
</div>
<div class="question" style="padding-left: 20px;">
<p><span>\(q = 2\), \(r = 0.125\)     <em><strong>(A1)(A1)</strong></em></span></p>
<p><span><em><strong>[2 marks]<br></strong></em></span></p>
<div class="question_part_label">i.b.</div>
</div>
<div class="question" style="padding-left: 20px;">
<p><span><img src="data:image/png;base64,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" alt><span>     <em><strong>(A4)</strong></em></span></span></p>
<p><strong><span>Notes:</span></strong><span> <em><strong>(A1)</strong></em> for scales and labels.</span><br><span><em><strong>(A1)</strong></em> for accurate smooth curve passing through \((0, 0)\) drawn at least in the given domain.</span><br><span><em><strong>(A1)</strong></em> for asymptotic behaviour (curve must not go up or cross the \(x\)-axis).</span><br><span><em><strong>(A1)</strong></em> for indicating the position of the maximum point.</span></p>
<p><span><em><strong>[4 marks]</strong></em><br></span></p>
<div class="question_part_label">i.c.</div>
</div>
<div class="question" style="padding-left: 20px;">
<p><span><span>\({\text{M}}\)</span> (\(1.44\), \(2.12\))     <em><strong>(G1)(G1)</strong></em></span></p>
<p><span><span><strong>Note: </strong>Brackets required, if missing award</span> <span><em><strong>(G1)(G0)</strong></em>. Accept \(x = 1.44\) and \(y = 2.12\).</span></span></p>
<p><span><span><em><strong>[2 marks]</strong></em><br></span></span></p>
<div class="question_part_label">i.d.</div>
</div>
<div class="question" style="padding-left: 20px;">
<p><span>\(y = 0\)     <em><strong>(A1)(A1)</strong></em></span></p>
<p><span><strong>Note: <em>(A1)</em></strong> for ‘\(y = \)’ provided the right hand side is a constant. <em><strong>(A1)</strong></em> for 0.</span></p>
<p><span><em><strong>[2 marks]</strong></em><br></span></p>
<div class="question_part_label">i.e.</div>
</div>
<div class="question" style="padding-left: 20px;">
<p><span>(i) See graph     <em><strong>(A1)(A1)</strong></em></span></p>
<p><span><strong><br>Note: <em>(A1)</em></strong> for correct line, <em><strong>(A1)</strong></em> for label.</span></p>
<p><br><span>(ii) \(x = 0.3\) <strong>(ft)</strong> from candidate’s graph.     <em><strong>(A2)</strong></em><strong>(ft)</strong></span></p>
<p><span><strong>Notes: </strong>Accept \( \pm 0.1\) from their <em>x</em>. For \(0.310\) award <em><strong>(G1)(G0)</strong></em>. For other answers taken from the GDC and not given correct to 3 significant figures award <em><strong>(G0)(AP)(G0)</strong></em> or <em><strong>(G1)(G0)</strong></em> if <em><strong>(AP)</strong></em> already applied.</span></p>
<p><span><em><strong>[4 marks]</strong></em><br></span></p>
<div class="question_part_label">i.f.</div>
</div>
<div class="question" style="padding-left: 20px;">
<p><span>\(C'(x) = 1 - \frac{{100}}{{{x^2}}}\)     <em><strong>(A1)(A1)(A1)</strong></em></span></p>
<p><span><strong>Note: <em>(A1)</em></strong> for 1, <em><strong>(A1)</strong></em> for \( - 100\) , <em><strong>(A1)</strong></em> for \({x^2}\) as denominator or \({{x^{ - 2}}}\) as numerator. Award a maximum of <em><strong>(A2)</strong></em> if an extra term is seen.</span></p>
<p><span><em><strong>[3 marks]</strong></em><br></span></p>
<div class="question_part_label">ii.a.</div>
</div>
<div class="question" style="padding-left: 20px;">
<p><span>For studying signs of the derivative at either side of \(x = 10\)     <em><strong>(M1)</strong></em></span></p>
<p><span>For saying there is a change of sign of the derivative     <em><strong>(M1)(AG)</strong></em></span></p>
<p><span><strong>OR</strong></span></p>
<p><span>For putting \(x = 10\) into \(C'\) and getting zero     <em><strong>(M1)</strong></em> </span></p>
<p><span>For clear sketch of the function or for mentioning that the function changes from decreasing to increasing at \(x = 10\)     <em><strong>(M1)(AG)</strong></em></span></p>
<p><span><strong>OR</strong></span></p>
<p><span>For solving \(C'(x) = 0\) and getting \(10\)     <em><strong>(M1)</strong></em></span></p>
<p><span>For clear sketch of the function or for mentioning that the function changes from decreasing to increasing at \(x = 10\)     <em><strong>(M1)(AG)</strong></em></span></p>
<p><span><span><strong>Note: </strong>For a sketch with a clear indication of the minimum or for a table with values of \(x\) at either side of \(x = 10\) award <em><strong>(M1)(M0)</strong></em>.</span></span></p>
<p><span><span><em><strong>[2 marks]</strong></em><br></span></span></p>
<div class="question_part_label">ii.b.</div>
</div>
<div class="question" style="padding-left: 20px;">
<p><span>\(C(10) = 10 + \frac{{100}}{{10}}\)     <em><strong>(M1)</strong></em></span></p>
<p><span>\(C(10) = 20\)     <em><strong>(A1)(G2)</strong></em></span></p>
<p><span><em><strong>[2 marks]<br></strong></em></span></p>
<div class="question_part_label">ii.c.</div>
</div>
<h2 style="margin-top: 1em">Examiners report</h2>
<div class="question" style="padding-left: 20px;">
<p><span style="font-family: times new roman,times; font-size: medium;">There were many well drawn graphs using correctly scaled and labelled axes with a good curve drawn. A number of students did not label the maximum point. Although many students showed in their graph the asymptotic behaviour of the curve, they did not know how to describe the asymptote. It was noticed that some students were tracing the curve to find the coordinates of the maximum instead of finding the maximum directly. The intersection between the line \(y = 1\) and the curve was not always read from their graph but from their GDC's graph.</span></p>
<div class="question_part_label">i.a.</div>
</div>
<div class="question" style="padding-left: 20px;">
<p><span style="font-family: times new roman,times; font-size: medium;">There were many well drawn graphs using correctly scaled and labelled axes with a good curve drawn. A number of students did not label the maximum point. Although many students showed in their graph the asymptotic behaviour of the curve, they did not know how to describe the asymptote. It was noticed that some students were tracing the curve to find the coordinates of the maximum instead of finding the maximum directly. The intersection between the line \(y = 1\) and the curve was not always read from their graph but from their GDC's graph.</span></p>
<div class="question_part_label">i.b.</div>
</div>
<div class="question" style="padding-left: 20px;">
<p><span style="font-family: times new roman,times; font-size: medium;">There were many well drawn graphs using correctly scaled and labelled axes with a good curve drawn. A number of students did not label the maximum point. Although many students showed in their graph the asymptotic behaviour of the curve, they did not know how to describe the asymptote. It was noticed that some students were tracing the curve to find the coordinates of the maximum instead of finding the maximum directly. The intersection between the line \(y = 1\) and the curve was not always read from their graph but from their GDC's graph.</span></p>
<div class="question_part_label">i.c.</div>
</div>
<div class="question" style="padding-left: 20px;">
<p><span style="font-family: times new roman,times; font-size: medium;">There were many well drawn graphs using correctly scaled and labelled axes with a good curve drawn. A number of students did not label the maximum point. Although many students showed in their graph the asymptotic behaviour of the curve, they did not know how to describe the asymptote. It was noticed that some students were tracing the curve to find the coordinates of the maximum instead of finding the maximum directly. The intersection between the line \(y = 1\) and the curve was not always read from their graph but from their GDC's graph.</span></p>
<div class="question_part_label">i.d.</div>
</div>
<div class="question" style="padding-left: 20px;">
<p><span style="font-family: times new roman,times; font-size: medium;">There were many well drawn graphs using correctly scaled and labelled axes with a good curve drawn. A number of students did not label the maximum point. Although many students showed in their graph the asymptotic behaviour of the curve, they did not know how to describe the asymptote. It was noticed that some students were tracing the curve to find the coordinates of the maximum instead of finding the maximum directly. The intersection between the line \(y = 1\) and the curve was not always read from their graph but from their GDC's graph.</span></p>
<div class="question_part_label">i.e.</div>
</div>
<div class="question" style="padding-left: 20px;">
<p><span style="font-family: times new roman,times; font-size: medium;">There were many well drawn graphs using correctly scaled and labelled axes with a good curve drawn. A number of students did not label the maximum point. Although many students showed in their graph the asymptotic behaviour of the curve, they did not know how to describe the asymptote. It was noticed that some students were tracing the curve to find the coordinates of the maximum instead of finding the maximum directly. The intersection between the line \(y = 1\) and the curve was not always read from their graph but from their GDC's graph.</span></p>
<div class="question_part_label">i.f.</div>
</div>
<div class="question" style="padding-left: 20px;">
<p><span style="font-family: times new roman,times; font-size: medium;">Finding the derivative was done at least partially correctly by most of the candidates. However, using it to find the minimum and to justify why it is a minimum was troublesome for the majority of the candidates. Even those who used a graph in their reasoning neglected to mention the change from decreasing to increasing or to supply a sign diagram. Many candidates recovered in the last part of the question when finding the minimum cost.</span></p>
<div class="question_part_label">ii.a.</div>
</div>
<div class="question" style="padding-left: 20px;">
<p><span style="font-family: times new roman,times; font-size: medium;">Finding the derivative was done at least partially correctly by most of the candidates. However, using it to find the minimum and to justify why it is a minimum was troublesome for the majority of the candidates. Even those who used a graph in their reasoning neglected to mention the change from decreasing to increasing or to supply a sign diagram. Many candidates recovered in the last part of the question when finding the minimum cost.</span></p>
<div class="question_part_label">ii.b.</div>
</div>
<div class="question" style="padding-left: 20px;">
<p><span style="font-family: times new roman,times; font-size: medium;">Finding the derivative was done at least partially correctly by most of the candidates. However, using it to find the minimum and to justify why it is a minimum was troublesome for the majority of the candidates. Even those who used a graph in their reasoning neglected to mention the change from decreasing to increasing or to supply a sign diagram. Many candidates recovered in the last part of the question when finding the minimum cost.</span></p>
<div class="question_part_label">ii.c.</div>
</div>
<br><hr><br><div class="specification">
<p>Consider the curve <em>y</em> = 2<em>x</em><sup>3</sup>&nbsp;&minus; 9<em>x</em><sup>2</sup> + 12<em>x</em> + 2, for&nbsp;&minus;1 &lt; <em>x</em> &lt; 3</p>
</div>

<div class="question" style="padding-left: 20px; padding-right: 20px;">
<p>Sketch the curve for −1 &lt; <em>x</em> &lt; 3 and −2 &lt; <em>y</em> &lt; 12.</p>
<div class="marks">[4]</div>
<div class="question_part_label">a.</div>
</div>
<div class="question" style="padding-left: 20px; padding-right: 20px;">
<p>A teacher asks her students to make some observations about the curve.</p>
<p>Three students responded.<br><strong>Nadia</strong> said <em>“The x-intercept of the curve is between −1 and zero”.</em><br><strong>Rick</strong> said <em>“The curve is decreasing when x &lt; 1 ”.</em><br><strong>Paula</strong> said <em>“The gradient of the curve is less than zero between x = 1 and x = 2 ”.</em></p>
<p>State the name of the student who made an <strong>incorrect</strong> observation.</p>
<div class="marks">[1]</div>
<div class="question_part_label">b.</div>
</div>
<div class="question" style="padding-left: 20px; padding-right: 20px;">
<p>Find the value of <em>y</em> when <em>x</em> = 1 .</p>
<div class="marks">[2]</div>
<div class="question_part_label">c.</div>
</div>
<div class="question" style="padding-left: 20px; padding-right: 20px;">
<p>Find \(\frac{{{\text{dy}}}}{{{\text{dx}}}}\).</p>
<div class="marks">[3]</div>
<div class="question_part_label">d.</div>
</div>
<div class="question" style="padding-left: 20px; padding-right: 20px;">
<p>Show that the stationary points of the curve are at <em>x</em> = 1 and <em>x</em> = 2.</p>
<div class="marks">[2]</div>
<div class="question_part_label">e.</div>
</div>
<div class="question" style="padding-left: 20px; padding-right: 20px;">
<p>Given that <em>y</em> = 2<em>x</em><sup>3</sup> − 9<em>x</em><sup>2</sup> + 12<em>x</em> + 2 = <em>k</em> has <strong>three</strong> solutions, find the possible values of <em>k</em>.</p>
<div class="marks">[3]</div>
<div class="question_part_label">f.</div>
</div>
<h2 style="margin-top: 1em">Markscheme</h2>
<div class="question" style="padding-left: 20px;">
<p><img src="data:image/png;base64,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"><em><strong>(A1)(A1)(A1)(A1)</strong></em></p>
<p><strong>Note:</strong> Award <em><strong>(A1)</strong></em> for correct window (condone a window which is slightly off) and axes labels. An indication of window is necessary. −1 to 3 on the <em>x</em>-axis and −2 to 12 on the <em>y</em>-axis and a graph in that window.<br><em><strong>(A1)</strong></em> for correct shape (curve having cubic shape and must be smooth).<br><em><strong>(A1)</strong></em> for both stationary points in the 1<sup>st</sup> quadrant with approximate correct position,<br><em><strong>(A1)</strong></em> for intercepts (negative <em>x</em>-intercept and positive <em>y</em> intercept) with approximate correct position.</p>
<p><em><strong>[4 marks]</strong></em></p>
<div class="question_part_label">a.</div>
</div>
<div class="question" style="padding-left: 20px;">
<p>Rick     <em><strong>(A1)</strong></em></p>
<p><strong>Note:</strong> Award <em><strong>(A0)</strong></em> if extra names stated.</p>
<p><em><strong>[1 mark]</strong></em></p>
<div class="question_part_label">b.</div>
</div>
<div class="question" style="padding-left: 20px;">
<p>2(1)<sup>3</sup> − 9(1)<sup>2</sup> + 12(1) + 2     <strong><em>(M1)</em></strong></p>
<p><strong>Note:</strong> Award <em><strong>(M1)</strong></em> for correct substitution into equation.</p>
<p>= 7     <em><strong>(A1)(G2)</strong></em></p>
<p><em><strong>[2 marks]</strong></em></p>
<p> </p>
<p> </p>
<p> </p>
<p> </p>
<p> </p>
<div class="question_part_label">c.</div>
</div>
<div class="question" style="padding-left: 20px;">
<p>6<em>x</em><sup>2</sup> − 18<em>x</em> + 12     <strong><em>(A1)(A1)(A1)</em></strong></p>
<p><strong>Note:</strong> Award <strong><em>(A1)</em></strong> for each correct term. Award at most <strong><em>(A1)(A1)(A0)</em></strong> if extra terms seen.</p>
<p><em><strong>[3 marks]</strong></em></p>
<div class="question_part_label">d.</div>
</div>
<div class="question" style="padding-left: 20px;">
<p>6<em>x</em><sup>2</sup> − 18<em>x</em> + 12 = 0     <strong><em>(M1)</em></strong></p>
<p><strong>Note:</strong> Award <em><strong>(M1)</strong></em> for equating their derivative to 0. If the derivative is not explicitly equated to 0, but a subsequent solving of their correct equation is seen, award <em><strong>(M1)</strong></em>.</p>
<p>6(<em>x </em> − 1)(<em>x</em> − 2) = 0  (or equivalent)      <em><strong>(M1)</strong></em></p>
<p><strong>Note:</strong> Award (M1) for correct factorization. The final <em><strong>(M1)</strong></em> is awarded only if answers are clearly stated.</p>
<p>Award <em><strong>(M0)(M0)</strong></em> for substitution of 1 and of 2 in their derivative.</p>
<p><em>x =</em> 1, <em>x =</em> 2 <em><strong>(AG)</strong></em></p>
<p><em><strong>[2 marks]</strong></em></p>
<div class="question_part_label">e.</div>
</div>
<div class="question" style="padding-left: 20px;">
<p>6 &lt; <em>k</em> &lt; 7     <em><strong>(A1)(A1)</strong></em><strong>(ft)</strong><em><strong>(A1)</strong></em></p>
<p><strong>Note:</strong> Award <em><strong>(A1)</strong></em> for an inequality with 6, award <em><strong>(A1)</strong></em><strong>(ft)</strong> for an inequality with 7 from their part (c) provided it is greater than 6, <em><strong>(A1)</strong></em> for their correct strict inequalities. Accept ]6, 7[ or (6, 7).</p>
<p><em><strong>[3 marks]</strong></em></p>
<div class="question_part_label">f.</div>
</div>
<h2 style="margin-top: 1em">Examiners report</h2>
<div class="question" style="padding-left: 20px;">
[N/A]
<div class="question_part_label">a.</div>
</div>
<div class="question" style="padding-left: 20px;">
[N/A]
<div class="question_part_label">b.</div>
</div>
<div class="question" style="padding-left: 20px;">
[N/A]
<div class="question_part_label">c.</div>
</div>
<div class="question" style="padding-left: 20px;">
[N/A]
<div class="question_part_label">d.</div>
</div>
<div class="question" style="padding-left: 20px;">
[N/A]
<div class="question_part_label">e.</div>
</div>
<div class="question" style="padding-left: 20px;">
[N/A]
<div class="question_part_label">f.</div>
</div>
<br><hr><br><div class="specification">
<p><span style="font-size: medium; font-family: times new roman,times;">On Monday Paco goes to a running track to train. He runs the first lap of the track</span> <span style="font-size: medium; font-family: times new roman,times;">in 120 seconds. Each lap Paco runs takes him 10 seconds longer than his previous lap.</span></p>
</div>

<div class="question" style="padding-left: 20px; padding-right: 20px;">
<p><span>Find the time, in seconds, Paco takes to run his fifth lap.</span></p>
<div class="marks">[3]</div>
<div class="question_part_label">a.</div>
</div>
<div class="question" style="padding-left: 20px; padding-right: 20px;">
<p><span>Paco runs his last lap in 260 seconds.</span></p>
<p><span>Find how many laps he has run on Monday.</span></p>
<div class="marks">[3]</div>
<div class="question_part_label">b.</div>
</div>
<div class="question" style="padding-left: 20px; padding-right: 20px;">
<p><span>Find the <strong>total</strong> time, in <strong>minutes,</strong> run by Paco on Monday.</span></p>
<div class="marks">[4]</div>
<div class="question_part_label">c.</div>
</div>
<div class="question" style="padding-left: 20px; padding-right: 20px;">
<p><span>On Wednesday Paco takes Lola to train. They both run the first lap of the track in 120 seconds. Each lap Lola runs takes 1.06 times as long as her previous lap.</span></p>
<p><span>Find the time, in seconds, Lola takes to run her third lap.</span></p>
<div class="marks">[3]</div>
<div class="question_part_label">d.</div>
</div>
<div class="question" style="padding-left: 20px; padding-right: 20px;">
<p><span>Find the <strong>total</strong> time, in seconds, Lola takes to run her first four laps.</span></p>
<div class="marks">[3]</div>
<div class="question_part_label">e.</div>
</div>
<div class="question" style="padding-left: 20px; padding-right: 20px;">
<p><span>Each lap Paco runs again takes him 10 seconds longer than his previous lap. After a certain number of laps Paco takes less time per lap than Lola.</span></p>
<p><span>Find the number of the lap when this happens.</span></p>
<div class="marks">[3]</div>
<div class="question_part_label">f.</div>
</div>
<h2 style="margin-top: 1em">Markscheme</h2>
<div class="question" style="padding-left: 20px;">
<p><span>\(120 + 10 \times 4\)     <em><strong>(M1)(A1)</strong></em></span></p>
<p><br><span><strong>Notes:</strong> Award <em><strong>(M1)</strong></em> for substituted AP formula, <em><strong>(A1)</strong></em> for correct substitutions. Accept a list of 4 correct terms.</span></p>
<p><br><span>= 160     <em><strong>(A1)(G3)</strong></em></span></p>
<div class="question_part_label">a.</div>
</div>
<div class="question" style="padding-left: 20px;">
<p><span>\(120 + (n - 1) \times 10 = 260\)     <em><strong>(M1)(M1)</strong></em></span></p>
<p><br><span><strong>Notes:</strong> Award <em><strong>(M1)</strong></em> for correctly substituted AP formula, <em><strong>(M1)</strong></em> for equating to 260. Accept a list of correct terms showing at least the 14<sup>th</sup> and 15<sup>th</sup> terms.</span></p>
<p><br><span>= 15     <em><strong>(A1)(G2)</strong></em></span></p>
<div class="question_part_label">b.</div>
</div>
<div class="question" style="padding-left: 20px;">
<p><span>\(\frac{{15}}{2}(120 + 260)\) <strong>or</strong> \(\frac{{15}}{2}(2 \times 120 + (15 - 1) \times 10)\)     <em><strong>(M1)(A1)</strong></em><strong>(ft)</strong></span></p>
<p><br><span><strong>Notes:</strong> Award <em><strong>(M1)</strong></em> for substituted AP sum formula, <em><strong>(A1)</strong></em><strong>(ft)</strong> for correct substitutions. Accept a sum of a list of 15 correct terms. Follow through from their answer to part (b).</span></p>
<p><br><span>2850 seconds     <em><strong>(A1)</strong></em><strong>(ft)</strong><em><strong>(G2)</strong></em></span></p>
<p><br><span><strong>Note:</strong> Award <em><strong>(G2)</strong></em> for 2850 seen with no working shown.</span></p>
<p><br><span>47.5 minutes     <em><strong>(A1)</strong></em><strong>(ft)</strong><em><strong>(G3)</strong></em></span><br><br></p>
<p><span><strong>Notes:</strong> A final <em><strong>(A1)</strong></em><strong>(ft)</strong> can be awarded for correct conversion from seconds</span> <span>into minutes of their incorrect answer.</span> <span>Follow through from their answer to part (b).</span></p>
<div class="question_part_label">c.</div>
</div>
<div class="question" style="padding-left: 20px;">
<p><span>\(120 \times {1.06^{3 - 1}}\)     <em><strong>(M1)(A1)</strong></em></span></p>
<p><br><span><strong>Notes:</strong> Award <em><strong>(M1)</strong></em> for substituted GP formula, <em><strong>(A1)</strong></em> for correct substitutions. Accept a list of 3 correct terms.</span></p>
<p><br><span>= 135 (134.832)     <em><strong>(A1)(G2)</strong></em></span></p>
<div class="question_part_label">d.</div>
</div>
<div class="question" style="padding-left: 20px;">
<p><span>\({S_4} = \frac{{120({{1.06}^4} - 1)}}{{(1.06 - 1)}}\)     <em><strong>(M1)(A1)</strong></em></span></p>
<p><br><span><strong>Notes:</strong> Award <em><strong>(M1)</strong></em> for substituted GP sum formula, <em><strong>(A1)</strong></em> for correct substitutions. Accept a sum of a list of 4 correct terms.</span></p>
<p><br><span>= 525 (524.953...)     <em><strong>(A1)(G2)</strong></em></span></p>
<div class="question_part_label">e.</div>
</div>
<div class="question" style="padding-left: 20px;">
<p><span>\(120 + (n - 1) \times 10 &lt; 120 \times {1.06^{n - 1}}\)     <em><strong>(M1)(M1)</strong></em></span></p>
<p><br><span><strong>Notes:</strong> Award <em><strong>(M1)</strong></em> for correct left hand side, <strong><em>(M1)</em></strong> for correct right hand side. Accept an equation. Follow through from their expressions given in parts (a) and (d).</span></p>
<p><br><strong><span>OR</span></strong></p>
<p><span>List of at least 2 terms for both sequences (120, 130, … and 120, 127.2, …)     <em><strong>(M1)</strong></em></span></p>
<p><span>List of correct 12<sup>th</sup> and 13<sup>th</sup> terms for both sequences (..., 230, 240 and …, 227.8, 241.5)     <em><strong>(M1)</strong></em></span></p>
<p><strong><span>OR</span></strong></p>
<p><span>A sketch with a line and an exponential curve,     <em><strong>(M1)</strong></em></span></p>
<p><span>An indication of the correct intersection point     <em><strong>(M1)</strong></em></span></p>
<p><span>13<sup>th</sup> lap     <em><strong>(A1)</strong></em><strong>(ft)</strong><em><strong>(G2)</strong></em></span></p>
<p><br><span><strong>Note:</strong> Do not award the final <em><strong>(A1)</strong></em><strong>(ft)</strong> if final answer is not a positive integer.</span></p>
<div class="question_part_label">f.</div>
</div>
<h2 style="margin-top: 1em">Examiners report</h2>
<div class="question" style="padding-left: 20px;">
[N/A]
<div class="question_part_label">a.</div>
</div>
<div class="question" style="padding-left: 20px;">
[N/A]
<div class="question_part_label">b.</div>
</div>
<div class="question" style="padding-left: 20px;">
[N/A]
<div class="question_part_label">c.</div>
</div>
<div class="question" style="padding-left: 20px;">
[N/A]
<div class="question_part_label">d.</div>
</div>
<div class="question" style="padding-left: 20px;">
[N/A]
<div class="question_part_label">e.</div>
</div>
<div class="question" style="padding-left: 20px;">
[N/A]
<div class="question_part_label">f.</div>
</div>
<br><hr><br><div class="specification">
<p>Consider the function \(f(x) =&nbsp; - {x^4} + a{x^2} + 5\), where \(a\) is a constant. Part of the graph of \(y = f(x)\) is shown below.</p>
<p style="text-align: center;"><img src="images/Schermafbeelding_2017-08-16_om_17.47.40.png" alt="M17/5/MATSD/SP2/ENG/TZ2/06"></p>
</div>

<div class="specification">
<p>It is known that at the point where \(x = 2\) the tangent to the graph of \(y = f(x)\) is horizontal.</p>
</div>

<div class="specification">
<p>There are two other points on the graph of \(y = f(x)\) at which the tangent is horizontal.</p>
</div>

<div class="question" style="padding-left: 20px; padding-right: 20px;">
<p>Write down the \(y\)-intercept of the graph.</p>
<div class="marks">[1]</div>
<div class="question_part_label">a.</div>
</div>
<div class="question" style="padding-left: 20px; padding-right: 20px;">
<p>Find \(f'(x)\).</p>
<div class="marks">[2]</div>
<div class="question_part_label">b.</div>
</div>
<div class="question" style="padding-left: 20px; padding-right: 20px;">
<p>Show that \(a = 8\).</p>
<div class="marks">[2]</div>
<div class="question_part_label">c.i.</div>
</div>
<div class="question" style="padding-left: 20px; padding-right: 20px;">
<p>Find \(f(2)\).</p>
<div class="marks">[2]</div>
<div class="question_part_label">c.ii.</div>
</div>
<div class="question" style="padding-left: 20px; padding-right: 20px;">
<p>Write down the \(x\)-coordinates of these two points;</p>
<div class="marks">[2]</div>
<div class="question_part_label">d.i.</div>
</div>
<div class="question" style="padding-left: 20px; padding-right: 20px;">
<p>Write down the intervals where the gradient of the graph of \(y = f(x)\) is positive.</p>
<div class="marks">[2]</div>
<div class="question_part_label">d.ii.</div>
</div>
<div class="question" style="padding-left: 20px; padding-right: 20px;">
<p>Write down the range of \(f(x)\).</p>
<div class="marks">[2]</div>
<div class="question_part_label">e.</div>
</div>
<div class="question" style="padding-left: 20px; padding-right: 20px;">
<p>Write down the number of possible solutions to the equation \(f(x) = 5\).</p>
<div class="marks">[1]</div>
<div class="question_part_label">f.</div>
</div>
<div class="question" style="padding-left: 20px; padding-right: 20px;">
<p>The equation \(f(x) = m\), where \(m \in \mathbb{R}\), has four solutions. Find the possible values of \(m\).</p>
<div class="marks">[2]</div>
<div class="question_part_label">g.</div>
</div>
<h2 style="margin-top: 1em">Markscheme</h2>
<div class="question" style="padding-left: 20px;">
<p>5     <strong><em>(A1)</em></strong></p>
<p> </p>
<p><strong>Note:</strong>     Accept an answer of \((0,{\text{ }}5)\).</p>
<p> </p>
<p><strong><em>[1 mark]</em></strong></p>
<div class="question_part_label">a.</div>
</div>
<div class="question" style="padding-left: 20px;">
<p>\(\left( {f'(x) = } \right) - 4{x^3} + 2ax\)     <strong><em>(A1)(A1)</em></strong></p>
<p> </p>
<p><strong>Note:</strong>     Award <strong><em>(A1) </em></strong>for \( - 4{x^3}\) and <strong><em>(A1) </em></strong>for \( + 2ax\). Award at most <strong><em>(A1)(A0) </em></strong>if extra terms are seen.</p>
<p> </p>
<p><strong><em>[2 marks]</em></strong></p>
<div class="question_part_label">b.</div>
</div>
<div class="question" style="padding-left: 20px;">
<p>\( - 4 \times {2^3} + 2a \times 2 = 0\)     <strong><em>(M1)(M1)</em></strong></p>
<p> </p>
<p><strong>Note:</strong>     Award <strong><em>(M1) </em></strong>for substitution of \(x = 2\) into their derivative, <strong><em>(M1) </em></strong>for equating their derivative, written in terms of \(a\), to 0 leading to a correct answer (note, the 8 does not need to be seen).</p>
<p> </p>
<p>\(a = 8\)     <strong><em>(AG)</em></strong></p>
<p><strong><em>[2 marks]</em></strong></p>
<div class="question_part_label">c.i.</div>
</div>
<div class="question" style="padding-left: 20px;">
<p>\(\left( {f(2) = } \right) - {2^4} + 8 \times {2^2} + 5\)     <strong><em>(M1)</em></strong></p>
<p> </p>
<p><strong>Note:</strong>     Award <strong><em>(M1) </em></strong>for correct substitution of \(x = 2\) and  \(a = 8\) into the formula of the function.</p>
<p> </p>
<p>21     <strong><em>(A1)(G2)</em></strong></p>
<p><strong><em>[2 marks]</em></strong></p>
<div class="question_part_label">c.ii.</div>
</div>
<div class="question" style="padding-left: 20px;">
<p>\((x = ){\text{ }} - 2,{\text{ }}(x = ){\text{ 0}}\)     <strong><em>(A1)(A1)</em></strong></p>
<p> </p>
<p><strong>Note:</strong>     Award <strong><em>(A1) </em></strong>for each correct solution. Award at most <strong><em>(A0)(A1)</em>(ft) </strong>if answers are given as \(( - 2{\text{ }},21)\) and \((0,{\text{ }}5)\) or \(( - 2,{\text{ }}0)\) and \((0,{\text{ }}0)\).</p>
<p> </p>
<p><strong><em>[2 marks]</em></strong></p>
<div class="question_part_label">d.i.</div>
</div>
<div class="question" style="padding-left: 20px;">
<p>\(x &lt;  - 2,{\text{ }}0 &lt; x &lt; 2\)     <strong><em>(A1)</em>(ft)<em>(A1)</em>(ft)</strong></p>
<p> </p>
<p><strong>Note:</strong>     Award <strong><em>(A1)</em>(ft) </strong>for \(x &lt;  - 2\), follow through from part (d)(i) provided their value is negative.</p>
<p>Award <strong><em>(A1)</em>(ft) </strong>for \(0 &lt; x &lt; 2\), follow through only from their 0 from part (d)(i); 2 must be the upper limit.</p>
<p>Accept interval notation.</p>
<p> </p>
<p><strong><em>[2 marks]</em></strong></p>
<div class="question_part_label">d.ii.</div>
</div>
<div class="question" style="padding-left: 20px;">
<p>\(y \leqslant 21\)     <strong><em>(A1)</em>(ft)<em>(A1)</em></strong></p>
<p> </p>
<p><strong>Notes:</strong>     Award <strong><em>(A1)</em>(ft) </strong>for 21 seen in an interval or an inequality, <strong><em>(A1) </em></strong>for “\(y \leqslant \)”.</p>
<p>Accept interval notation.</p>
<p>Accept \( - \infty  &lt; y \leqslant 21\) or \(f(x) \leqslant 21\).</p>
<p>Follow through from their answer to part (c)(ii). Award at most <strong><em>(A1)</em>(ft)<em>(A0) </em></strong>if \(x\) is seen instead of \(y\). Do not award the second <strong><em>(A1) </em></strong>if a (finite) lower limit is seen.</p>
<p> </p>
<p><strong><em>[2 marks]</em></strong></p>
<div class="question_part_label">e.</div>
</div>
<div class="question" style="padding-left: 20px;">
<p>3 (solutions)     <strong><em>(A1)</em></strong></p>
<p><strong><em>[1 mark]</em></strong></p>
<div class="question_part_label">f.</div>
</div>
<div class="question" style="padding-left: 20px;">
<p>\(5 &lt; m &lt; 21\) or equivalent     <strong><em>(A1)</em>(ft)<em>(A1)</em></strong></p>
<p> </p>
<p><strong>Note:</strong>     Award <strong><em>(A1)</em>(ft) </strong>for 5 and 21 seen in an interval or an inequality, <strong><em>(A1) </em></strong>for correct strict inequalities. Follow through from their answers to parts (a) and (c)(ii).</p>
<p>Accept interval notation.</p>
<p> </p>
<p><strong><em>[2 marks]</em></strong></p>
<div class="question_part_label">g.</div>
</div>
<h2 style="margin-top: 1em">Examiners report</h2>
<div class="question" style="padding-left: 20px;">
[N/A]
<div class="question_part_label">a.</div>
</div>
<div class="question" style="padding-left: 20px;">
[N/A]
<div class="question_part_label">b.</div>
</div>
<div class="question" style="padding-left: 20px;">
[N/A]
<div class="question_part_label">c.i.</div>
</div>
<div class="question" style="padding-left: 20px;">
[N/A]
<div class="question_part_label">c.ii.</div>
</div>
<div class="question" style="padding-left: 20px;">
[N/A]
<div class="question_part_label">d.i.</div>
</div>
<div class="question" style="padding-left: 20px;">
[N/A]
<div class="question_part_label">d.ii.</div>
</div>
<div class="question" style="padding-left: 20px;">
[N/A]
<div class="question_part_label">e.</div>
</div>
<div class="question" style="padding-left: 20px;">
[N/A]
<div class="question_part_label">f.</div>
</div>
<div class="question" style="padding-left: 20px;">
[N/A]
<div class="question_part_label">g.</div>
</div>
<br><hr><br><div class="specification">
<p class="p1">A water container is made in the shape of a cylinder with internal height \(h\) cm and internal base radius \(r\) cm.</p>
<p class="p2" style="text-align: center;"><img src="images/Schermafbeelding_2017-03-07_om_08.31.01.png" alt="N16/5/MATSD/SP2/ENG/TZ0/06"></p>
<p class="p1">The water container has no top. The inner surfaces of the container are to be coated with a water-resistant material.</p>
</div>

<div class="specification">
<p class="p1">The volume of the water container is \(0.5{\text{ }}{{\text{m}}^3}\).</p>
</div>

<div class="specification">
<p class="p1">The water container is designed so that the area to be coated is minimized.</p>
</div>

<div class="specification">
<p class="p1">One can of water-resistant material coats a surface area of \(2000{\text{ c}}{{\text{m}}^2}\).</p>
</div>

<div class="question" style="padding-left: 20px; padding-right: 20px;">
<p class="p1">Write down a formula for \(A\), <span class="s1">the surface area to be coated.</span></p>
<div class="marks">[2]</div>
<div class="question_part_label">a.</div>
</div>
<div class="question" style="padding-left: 20px; padding-right: 20px;">
<p class="p1">Express this volume in \({\text{c}}{{\text{m}}^3}\).</p>
<div class="marks">[1]</div>
<div class="question_part_label">b.</div>
</div>
<div class="question" style="padding-left: 20px; padding-right: 20px;">
<p class="p1"><span class="s1">Write down, in terms of \(r\) </span>and \(h\), an equation for the volume of this water container.</p>
<div class="marks">[1]</div>
<div class="question_part_label">c.</div>
</div>
<div class="question" style="padding-left: 20px; padding-right: 20px;">
<p class="p1">Show that \(A = \pi {r^2}\frac{{1\,000\,000}}{r}\).</p>
<div class="marks">[2]</div>
<div class="question_part_label">d.</div>
</div>
<div class="question" style="padding-left: 20px; padding-right: 20px;">
<p class="p1">Show that \(A = \pi {r^2} + \frac{{1\,000\,000}}{r}\).</p>
<div class="marks">[2]</div>
<div class="question_part_label">d.</div>
</div>
<div class="question" style="padding-left: 20px; padding-right: 20px;">
<p>Find \(\frac{{{\text{d}}A}}{{{\text{d}}r}}\).</p>
<div class="marks">[3]</div>
<div class="question_part_label">e.</div>
</div>
<div class="question" style="padding-left: 20px; padding-right: 20px;">
<p class="p1">Using your answer to part (e), find the value of \(r\) <span class="s1">which minimizes \(A\).</span></p>
<div class="marks">[3]</div>
<div class="question_part_label">f.</div>
</div>
<div class="question" style="padding-left: 20px; padding-right: 20px;">
<p class="p1">Find the value of this minimum area.</p>
<div class="marks">[2]</div>
<div class="question_part_label">g.</div>
</div>
<div class="question" style="padding-left: 20px; padding-right: 20px;">
<p class="p1">Find the least number of cans of water-resistant material that will coat the area in <span class="s1">part (g).</span></p>
<div class="marks">[3]</div>
<div class="question_part_label">h.</div>
</div>
<h2 style="margin-top: 1em">Markscheme</h2>
<div class="question" style="padding-left: 20px;">
<p class="p1"><span class="Apple-converted-space">\((A = ){\text{ }}\pi {r^2} + 2\pi rh\)    </span><strong><em>(A1)(A1)</em></strong></p>
<p class="p2"> </p>
<p class="p1"><strong>Note: <span class="Apple-converted-space">    </span></strong>Award <strong><em>(A1) </em></strong>for either \(\pi {r^2}\) <strong>OR</strong> \(2\pi rh\) seen. Award <strong><em>(A1) </em></strong>for two correct terms added together.</p>
<p class="p2"> </p>
<p class="p1"><strong><em>[2 marks]</em></strong></p>
<div class="question_part_label">a.</div>
</div>
<div class="question" style="padding-left: 20px;">
<p class="p1"><span class="Apple-converted-space">\(500\,000\)    </span><strong><em>(A1)</em></strong></p>
<p class="p2"> </p>
<p class="p1"><strong>Notes: <span class="Apple-converted-space">    </span></strong>Units <strong>not </strong>required.</p>
<p class="p2"> </p>
<p class="p1"><strong><em>[1 mark]</em></strong></p>
<div class="question_part_label">b.</div>
</div>
<div class="question" style="padding-left: 20px;">
<p class="p1"><span class="Apple-converted-space">\(500\,000 = \pi {r^2}h\)    </span><strong><em>(A1)</em>(ft)</strong></p>
<p class="p2"> </p>
<p class="p1"><strong>Notes: <span class="Apple-converted-space">    </span></strong>Award <strong><em>(A1)</em>(ft) </strong>for \(\pi {r^2}h\) equating to their part (b).</p>
<p class="p1"><span class="s1">Do not accept </span>unless \(V = \pi {r^2}h\) is explicitly defined as their part (b).</p>
<p class="p2"> </p>
<p class="p1"><strong><em>[1 mark]</em></strong></p>
<div class="question_part_label">c.</div>
</div>
<div class="question" style="padding-left: 20px;">
<p class="p1"><span class="Apple-converted-space">\(A = \pi {r^2} + 2\pi r\left( {\frac{{500\,000}}{{\pi {r^2}}}} \right)\)    </span><strong><em>(A1)</em>(ft)<em>(M1)</em></strong></p>
<p class="p2"> </p>
<p class="p1"><strong>Note: <span class="Apple-converted-space">    </span></strong>Award <strong><em>(A1)</em>(ft) </strong>for their \({\frac{{500\,000}}{{\pi {r^2}}}}\) seen.</p>
<p class="p1">Award <strong><em>(M1) </em></strong>for correctly substituting <strong>only</strong> \({\frac{{500\,000}}{{\pi {r^2}}}}\) into a <strong>correct </strong>part (a).</p>
<p class="p1">Award <strong><em>(A1)</em>(ft)<em>(M1) </em></strong>for rearranging part (c) to \(\pi rh = \frac{{500\,000}}{r}\) and substituting for \(\pi rh\) <span class="s1">in expression for \(A\).</span></p>
<p class="p3"> </p>
<p class="p4"><span class="Apple-converted-space">\(A = \pi {r^2} + \frac{{1\,000\,000}}{r}\)    </span><span class="s2"><strong><em>(AG)</em></strong></span></p>
<p class="p2"> </p>
<p class="p1"><strong>Notes: <span class="Apple-converted-space">    </span></strong>The conclusion, \(A = \pi {r^2} + \frac{{1\,000\,000}}{r}\), must be consistent with their working seen for the <strong><em>(A1) </em></strong>to be awarded.</p>
<p class="p4">Accept \({10^6}\) as equivalent to \({1\,000\,000}\).</p>
<p class="p3"> </p>
<p class="p1"><strong><em>[2 marks]</em></strong></p>
<div class="question_part_label">d.</div>
</div>
<div class="question" style="padding-left: 20px;">
<p class="p1"><span class="Apple-converted-space">\(A = \pi {r^2} + 2\pi r\left( {\frac{{500\,000}}{{\pi {r^2}}}} \right)\)    </span><strong><em>(A1)</em>(ft)<em>(M1)</em></strong></p>
<p class="p2"> </p>
<p class="p1"><strong>Note: <span class="Apple-converted-space">    </span></strong>Award <strong><em>(A1)</em>(ft) </strong>for their \({\frac{{500\,000}}{{\pi {r^2}}}}\) seen.</p>
<p class="p1">Award <strong><em>(M1) </em></strong>for correctly substituting <strong>only</strong> \({\frac{{500\,000}}{{\pi {r^2}}}}\) into a <strong>correct </strong>part (a).</p>
<p class="p1">Award <strong><em>(A1)</em>(ft)<em>(M1) </em></strong>for rearranging part (c) to \(\pi rh = \frac{{500\,000}}{r}\) and substituting for \(\pi rh\) <span class="s1">in expression for \(A\).</span></p>
<p class="p3"> </p>
<p class="p4"><span class="Apple-converted-space">\(A = \pi {r^2} + \frac{{1\,000\,000}}{r}\)    </span><span class="s2"><strong><em>(AG)</em></strong></span></p>
<p class="p2"> </p>
<p class="p1"><strong>Notes: <span class="Apple-converted-space">    </span></strong>The conclusion, \(A = \pi {r^2} + \frac{{1\,000\,000}}{r}\), must be consistent with their working seen for the <strong><em>(A1) </em></strong>to be awarded.</p>
<p class="p4">Accept \({10^6}\) as equivalent to \({1\,000\,000}\).</p>
<p class="p3"> </p>
<p class="p1"><strong><em>[2 marks]</em></strong></p>
<div class="question_part_label">d.</div>
</div>
<div class="question" style="padding-left: 20px;">
<p class="p1"><span class="Apple-converted-space">\(2\pi r - \frac{{{\text{1}}\,{\text{000}}\,{\text{000}}}}{{{r^2}}}\)    </span><strong><em>(A1)(A1)(A1)</em></strong></p>
<p class="p2"> </p>
<p class="p1"><strong>Note: <span class="Apple-converted-space">    </span></strong>Award <strong><em>(A1) </em></strong>for \(2\pi r\), <strong><em>(A1) </em></strong>for \(\frac{1}{{{r^2}}}\) or \({r^{ - 2}}\), <strong><em>(A1) </em></strong>for \( - {\text{1}}\,{\text{000}}\,{\text{000}}\).</p>
<p class="p2"> </p>
<p class="p1"><strong><em>[3 marks]</em></strong></p>
<div class="question_part_label">e.</div>
</div>
<div class="question" style="padding-left: 20px;">
<p class="p1"><span class="Apple-converted-space">\(2\pi r - \frac{{1\,000\,000}}{{{r^2}}} = 0\)    </span><strong><em>(M1)</em></strong></p>
<p class="p2"> </p>
<p class="p1"><strong>Note: <span class="Apple-converted-space">    </span></strong>Award <strong><em>(M1) </em></strong>for equating their part (e) to zero.</p>
<p class="p2"> </p>
<p class="p1"><span class="Apple-converted-space">\({r^3} = \frac{{1\,000\,000}}{{2\pi }}\) <strong>OR</strong> \(r = \sqrt[3]{{\frac{{1\,000\,000}}{{2\pi }}}}\)     </span><strong><em>(M1)</em></strong></p>
<p class="p3"> </p>
<p class="p1"><strong>Note: <span class="Apple-converted-space">    </span></strong>Award <strong><em>(M1) </em></strong>for isolating \(r\).</p>
<p class="p2"> </p>
<p class="p1"><strong>OR</strong></p>
<p class="p1">sketch of derivative function <span class="Apple-converted-space">    </span><strong><em>(M1)</em></strong></p>
<p class="p1">with its zero indicated <span class="Apple-converted-space">    </span><strong><em>(M1)</em></strong></p>
<p class="p1"><span class="Apple-converted-space">\((r = ){\text{ }}54.2{\text{ }}({\text{cm}}){\text{ }}(54.1926 \ldots )\)    </span><strong><em>(A1)</em>(ft)<em>(G2)</em></strong></p>
<p class="p1"><strong><em>[3 marks]</em></strong></p>
<div class="question_part_label">f.</div>
</div>
<div class="question" style="padding-left: 20px;">
<p class="p1"><span class="Apple-converted-space">\(\pi {(54.1926 \ldots )^2} + \frac{{1\,000\,000}}{{(54.1926 \ldots )}}\)    </span><strong><em>(M1)</em></strong></p>
<p class="p2"> </p>
<p class="p1"><strong>Note: <span class="Apple-converted-space">    </span></strong>Award <strong><em>(M1) </em></strong>for correct substitution of their part (f) into the given equation.</p>
<p class="p2"> </p>
<p class="p1"><span class="Apple-converted-space">\( = 27\,700{\text{ }}({\text{c}}{{\text{m}}^2}){\text{ }}(27\,679.0 \ldots )\)    </span><strong><em>(A1)</em>(ft)<em>(G2)</em></strong></p>
<p class="p1"><strong><em>[2 marks]</em></strong></p>
<div class="question_part_label">g.</div>
</div>
<div class="question" style="padding-left: 20px;">
<p class="p1"><span class="Apple-converted-space">\(\frac{{27\,679.0 \ldots }}{{2000}}\)    </span><strong><em>(M1)</em></strong></p>
<p class="p2"> </p>
<p class="p1"><strong>Note: <span class="Apple-converted-space">    </span></strong>Award <strong><em>(M1) </em></strong>for dividing their part (g) by <span class="s1">2000</span>.</p>
<p class="p2"> </p>
<p class="p1"><span class="Apple-converted-space">\( = 13.8395 \ldots \)    </span><strong><em>(A1)</em>(ft)</strong></p>
<p class="p2"> </p>
<p class="p1"><strong>Notes: <span class="Apple-converted-space">    </span></strong>Follow through from part (g).</p>
<p class="p2"> </p>
<p class="p1"><span class="s1">14 </span>(cans) <span class="Apple-converted-space">    </span><strong><em>(A1)</em>(ft)<em>(G3)</em></strong></p>
<p class="p2"> </p>
<p class="p1"><strong>Notes: <span class="Apple-converted-space">    </span></strong>Final <strong><em>(A1) </em></strong>awarded for rounding up their \(13.8395 \ldots \) to the next integer.</p>
<p class="p2"> </p>
<p class="p1"><strong><em>[3 marks]</em></strong></p>
<div class="question_part_label">h.</div>
</div>
<h2 style="margin-top: 1em">Examiners report</h2>
<div class="question" style="padding-left: 20px;">
[N/A]
<div class="question_part_label">a.</div>
</div>
<div class="question" style="padding-left: 20px;">
[N/A]
<div class="question_part_label">b.</div>
</div>
<div class="question" style="padding-left: 20px;">
[N/A]
<div class="question_part_label">c.</div>
</div>
<div class="question" style="padding-left: 20px;">
[N/A]
<div class="question_part_label">d.</div>
</div>
<div class="question" style="padding-left: 20px;">
[N/A]
<div class="question_part_label">d.</div>
</div>
<div class="question" style="padding-left: 20px;">
[N/A]
<div class="question_part_label">e.</div>
</div>
<div class="question" style="padding-left: 20px;">
[N/A]
<div class="question_part_label">f.</div>
</div>
<div class="question" style="padding-left: 20px;">
[N/A]
<div class="question_part_label">g.</div>
</div>
<div class="question" style="padding-left: 20px;">
[N/A]
<div class="question_part_label">h.</div>
</div>
<br><hr><br><div class="specification">
<p style="margin: 0.0px 0.0px 0.0px 0.0px; font: 21.0px 'Times New Roman';"><span style="font-family: 'times new roman', times; font-size: medium;">Consider the sequence \({u_1},{\text{ }}{u_2},{\text{ }}{u_3},{\text{ }} \ldots ,{\text{ }}{{\text{u}}_n},{\text{ }} \ldots \) where</span></p>
<p style="margin: 0.0px 0.0px 0.0px 0.0px; font: 21.0px 'Times New Roman';"><span style="font-family: 'times new roman', times; font-size: medium;">\[{u_1} = 600,{\text{ }}{u_2} = 617,{\text{ }}{u_3} = 634,{\text{ }}{u_4} = 651.\]</span></p>
<p style="margin: 0.0px 0.0px 0.0px 0.0px; font: 21.0px 'Times New Roman';"><span style="font-family: 'times new roman', times; font-size: medium;">The sequence continues in the same manner.</span></p>
</div>

<div class="question" style="padding-left: 20px; padding-right: 20px;">
<p><span>Find the value of \({u_{20}}\).</span></p>
<div class="marks">[3]</div>
<div class="question_part_label">a.</div>
</div>
<div class="question" style="padding-left: 20px; padding-right: 20px;">
<p><span>Find the sum of the first 10 terms of the sequence.</span></p>
<div class="marks">[3]</div>
<div class="question_part_label">b.</div>
</div>
<div class="question" style="padding-left: 20px; padding-right: 20px;">
<p><span>Now consider the sequence \({v_1},{\text{ }}{v_2},{\text{ }}{v_3},{\text{ }} \ldots ,{\text{ }}{v_n},{\text{ }} \ldots \) where</span></p>
<p><span>\[{v_1} = 3,{\text{ }}{v_2} = 6,{\text{ }}{v_3} = 12,{\text{ }}{v_4} = 24\]</span></p>
<p><span>This sequence continues in the same manner.</span></p>
<p><span>Find the exact value of \({v_{10}}\).</span></p>
<div class="marks">[3]</div>
<div class="question_part_label">c.</div>
</div>
<div class="question" style="padding-left: 20px; padding-right: 20px;">
<p><span>Now consider the sequence \({v_1},{\text{ }}{v_2},{\text{ }}{v_3},{\text{ }} \ldots ,{\text{ }}{v_n},{\text{ }} \ldots \) where</span></p>
<p><span>\[{v_1} = 3,{\text{ }}{v_2} = 6,{\text{ }}{v_3} = 12,{\text{ }}{v_4} = 24\]</span></p>
<p><span>This sequence continues in the same manner.</span></p>
<p><span>Find the sum of the first 8 terms of this sequence.</span></p>
<div class="marks">[3]</div>
<div class="question_part_label">d.</div>
</div>
<div class="question" style="padding-left: 20px; padding-right: 20px;">
<p><span>\(k\) is the smallest value of \(n\) for which \({v_n}\) is greater than \({u_n}\).</span></p>
<p><span>Calculate the value of \(k\).</span></p>
<div class="marks">[3]</div>
<div class="question_part_label">e.</div>
</div>
<h2 style="margin-top: 1em">Markscheme</h2>
<div class="question" style="padding-left: 20px;">
<p><span>\(600 + (20 - 1) \times 17\)     <strong><em>(M1)(A1)</em></strong></span></p>
<p> </p>
<p><span><strong>Note: </strong>Award <strong><em>(M1) </em></strong>for substituted arithmetic sequence formula, <strong><em>(A1) </em></strong>for correct substitutions. If a list is used, award <strong><em>(M1) </em></strong>for at least 6 correct terms seen, award <strong><em>(A1) </em></strong>for at least 20 correct terms seen.</span></p>
<p> </p>
<p><span>\( = 923\)     <strong><em>(A1)(G3)</em></strong></span></p>
<p><span><strong><em>[3 marks]</em></strong></span></p>
<div class="question_part_label">a.</div>
</div>
<div class="question" style="padding-left: 20px;">
<p><span>\(\frac{{10}}{2}\left[ {2 \times 600 + (10 - 1) \times 17} \right]\)     <strong><em>(M1)(A1)</em></strong></span></p>
<p> </p>
<p><span><strong>Note: </strong>Award <strong><em>(M1) </em></strong>for substituted arithmetic series formula, <strong><em>(A1) </em></strong>for their correct substitutions. Follow through from part (a). For consistent use of geometric series formula in part (b) with the geometric sequence formula in part (a) award a maximum of <strong><em>(M1)(A1)(A0) </em></strong>since their final answer cannot be an integer.</span></p>
<p> </p>
<p><span><strong>OR</strong></span></p>
<p><span>\({u_{10}} = 600 + (10 - 1)17 = 753\)     <strong><em>(M1)</em></strong></span></p>
<p><span>\({S_{10}} = \frac{{10}}{2}\left( {600 + {\text{their }}{u_{10}}} \right)\)     <strong><em>(M1)</em></strong></span></p>
<p> </p>
<p><span><strong>Note: </strong>Award <strong><em>(M1) </em></strong>for their correctly substituted arithmetic sequence formula, <strong><em>(M1) </em></strong>for their correctly substituted arithmetic series formula. Follow through from part (a) and <strong>within </strong>part (b).</span></p>
<p> </p>
<p><span><strong>Note: </strong>If a list is used, award <strong><em>(M1) </em></strong>for at least 10 correct terms seen, award <strong><em>(A1) </em></strong>for these terms being added.</span></p>
<p> </p>
<p><span>\( = 6765\)   (accept \(6770\))     <strong><em>(A1)</em>(ft)<em>(G2)</em></strong></span></p>
<p><span><strong><em>[3 marks]</em></strong></span></p>
<div class="question_part_label">b.</div>
</div>
<div class="question" style="padding-left: 20px;">
<p><span>\(3 \times {2^9}\)     <strong><em>(M1)(A1)</em></strong></span></p>
<p> </p>
<p><span><strong>Note: </strong>Award <strong><em>(M1) </em></strong>for substituted geometric sequence formula, <strong><em>(A1) </em></strong>for correct substitutions. If a list is used, award <strong><em>(M1) </em></strong>for at least 6 correct terms seen, award <strong><em>(A1) </em></strong>for at least 8 correct terms seen.</span></p>
<p> </p>
<p><span>\( = 1536\)     <strong><em>(A1)(G3)</em></strong></span></p>
<p> </p>
<p><span><strong>Note:</strong> Exact answer only. If both exact and rounded answer seen, award the final <strong><em>(A1)</em></strong>.</span></p>
<p> </p>
<p><span><strong><em>[3 marks]</em></strong></span></p>
<div class="question_part_label">c.</div>
</div>
<div class="question" style="padding-left: 20px;">
<p><span>\(\frac{{3 \times \left( {{2^8} - 1} \right)}}{{2 - 1}}\)     <strong><em>(M1)(A1)</em>(ft)</strong></span></p>
<p> </p>
<p><span><strong>Note: </strong>Award <strong><em>(M1) </em></strong>for substituted geometric series formula, <strong><em>(A1) </em></strong>for their correct substitutions. Follow through from part (c). If a list is used, award <strong><em>(M1) </em></strong>for at least 8 correct terms seen, award <strong><em>(A1) </em></strong>for these 8 correct terms being added. For consistent use of arithmetic series formula in part (d) with the arithmetic sequence formula in part (c) award a maximum of <strong><em>(M1)(A1)(A1)</em></strong>.</span></p>
<p> </p>
<p><span>\( = 765\)     <strong><em>(A1)</em>(ft)<em>(G2)</em></strong></span></p>
<p><span><strong><em>[3 marks]</em></strong></span></p>
<div class="question_part_label">d.</div>
</div>
<div class="question" style="padding-left: 20px;">
<p><span>\(3 \times {2^{k - 1}} &gt; 600 + (k - 1)(17)\)     <strong><em>(M1)</em></strong></span></p>
<p> </p>
<p><span><strong>Note: </strong>Award <strong><em>(M1) </em></strong>for their correct inequality; allow equation. </span></p>
<p><span>Follow through from parts (a) and (c). Accept sketches of the two functions as a valid method.</span></p>
<p> </p>
<p><span>\(k &gt; 8.93648 \ldots \)   (may be implied)     <strong><em>(A1)</em>(ft)</strong></span></p>
<p> </p>
<p><span><strong>Note: </strong>Award <strong><em>(A1) </em></strong>for \(8.93648…\) seen. The GDC gives answers of \(-34.3\) and \(8.936\) to the inequality; award <strong><em>(M1)(A1) </em></strong>if these are seen with working shown.</span></p>
<p> </p>
<p><span><strong>OR</strong></span></p>
<p><span>\({v_8} = 384\)     \({u_8} = 719\)     <strong><em>(M1)</em></strong></span></p>
<p><span>\({v_9} = 768\)     \({u_9} = 736\)     <strong><em>(M1)</em></strong></span></p>
<p> </p>
<p><span><strong>Note: </strong>Award <strong><em>(M1) </em></strong>for \({v_8}\) and \({u_8}\) both seen, <strong><em>(M1) </em></strong>for \({v_9}\) and \({u_9}\) both seen.</span></p>
<p> </p>
<p><span>\(k = 9\)     <strong><em>(A1)</em>(ft)<em>(G2)</em></strong></span></p>
<p> </p>
<p><span><strong>Note: </strong>Award <strong><em>(G1) </em></strong>for \(8.93648…\) and \(-34.3\) seen as final answer without working. Accept use of \(n\).</span></p>
<p> </p>
<p><span><strong><em>[3 marks]</em></strong></span></p>
<div class="question_part_label">e.</div>
</div>
<h2 style="margin-top: 1em">Examiners report</h2>
<div class="question" style="padding-left: 20px;">
[N/A]
<div class="question_part_label">a.</div>
</div>
<div class="question" style="padding-left: 20px;">
[N/A]
<div class="question_part_label">b.</div>
</div>
<div class="question" style="padding-left: 20px;">
[N/A]
<div class="question_part_label">c.</div>
</div>
<div class="question" style="padding-left: 20px;">
[N/A]
<div class="question_part_label">d.</div>
</div>
<div class="question" style="padding-left: 20px;">
[N/A]
<div class="question_part_label">e.</div>
</div>
<br><hr><br><div class="specification">
<p class="p1">In the month before their IB Diploma examinations, eight male students recorded the number of hours they spent on social media.</p>
<p class="p2">For each student, the number of hours spent on social media <span class="s1">(\(x\)) </span>and the number of IB Diploma points obtained <span class="s1">(\(y\)) </span>are shown in the following table.</p>
<p class="p2" style="text-align: center;"><img src="images/Schermafbeelding_2017-03-07_om_07.43.52.png" alt="N16/5/MATSD/SP2/ENG/TZ0/01"></p>
</div>

<div class="specification">
<p class="p1">Use your graphic display calculator to find</p>
</div>

<div class="specification">
<p class="p1">Ten female students also recorded the number of hours they spent on social media in the month before their IB Diploma examinations. Each of these female students spent between <span class="s1">3 </span>and <span class="s1">30 </span>hours on social media.</p>
<p class="p1">The equation of the regression line <span class="s1"><em>y </em></span>on <span class="s1"><em>x </em></span>for these ten female students is</p>
<p class="p1">\[y = &nbsp;- \frac{2}{3}x + \frac{{125}}{3}.\]</p>
<p class="p1">An eleventh girl spent <span class="s1">34 </span>hours on social media in the month before her IB Diploma examinations.</p>
</div>

<div class="question" style="padding-left: 20px; padding-right: 20px;">
<p class="p1"><span class="s1">On graph paper, draw a scatter diagram for these data. Use a scale of </span><span class="s2">2 cm </span>to represent <span class="s2">5 </span>hours on the \(x\)-axis and <span class="s2">2 cm </span>to represent <span class="s2">10 </span>points on the \(y\)-axis.</p>
<div class="marks">[4]</div>
<div class="question_part_label">a.</div>
</div>
<div class="question" style="padding-left: 20px; padding-right: 20px;">
<p class="p1"><span class="s1">(i) <span class="Apple-converted-space">    \({\bar x}\)</span>, </span>the mean number of hours spent on social media;</p>
<p class="p2">(ii) <span class="Apple-converted-space">    \({\bar y}\)</span>, the mean number of IB Diploma points.</p>
<div class="marks">[2]</div>
<div class="question_part_label">b.</div>
</div>
<div class="question" style="padding-left: 20px; padding-right: 20px;">
<p class="p1"><span class="s1">Plot the point \((\bar x,{\text{ }}\bar y)\) </span>on your scatter diagram and label this point <span class="s2">M</span>.</p>
<div class="marks">[2]</div>
<div class="question_part_label">c.</div>
</div>
<div class="question" style="padding-left: 20px; padding-right: 20px;">
<p class="p1"><span class="s1">Write down the value of \(r\), </span>the Pearson’s product–moment correlation coefficient, for these data.</p>
<div class="marks">[2]</div>
<div class="question_part_label">d.</div>
</div>
<div class="question" style="padding-left: 20px; padding-right: 20px;">
<p class="p1">Write down the equation of the regression line \(y\) <span class="s1">on \(x\) for these eight male students.</span></p>
<div class="marks">[2]</div>
<div class="question_part_label">e.</div>
</div>
<div class="question" style="padding-left: 20px; padding-right: 20px;">
<p class="p1">Draw the regression line, from part (e), on your scatter diagram.</p>
<div class="marks">[2]</div>
<div class="question_part_label">f.</div>
</div>
<div class="question" style="padding-left: 20px; padding-right: 20px;">
<p class="p1">Use the given equation of the regression line to estimate the number of IB Diploma <span class="s1">points that this girl obtained.</span></p>
<div class="marks">[2]</div>
<div class="question_part_label">g.</div>
</div>
<div class="question" style="padding-left: 20px; padding-right: 20px;">
<p class="p1">Write down a reason why this estimate is not reliable.</p>
<div class="marks">[1]</div>
<div class="question_part_label">h.</div>
</div>
<h2 style="margin-top: 1em">Markscheme</h2>
<div class="question" style="padding-left: 20px;">
<p class="p1"><img src="images/Schermafbeelding_2017-03-07_om_08.41.53.png" alt="N16/5/MATSD/SP2/ENG/TZ0/01.a/M">     <span class="s1"><strong><em>(A4)</em></strong></span></p>
<p class="p2"> </p>
<p class="p3"><strong>Notes</strong>: <span class="Apple-converted-space">    </span>Award <strong><em>(A1) </em></strong>for correct scale and labelled axes.</p>
<p class="p3">Award <strong><em>(A3) </em></strong>for <span class="s2">7 </span>or <span class="s2">8 </span>points correctly plotted,</p>
<p class="p3"><strong><em>(A2) </em></strong>for <span class="s2">5 </span>or <span class="s2">6 </span>points correctly plotted,</p>
<p class="p3"><strong><em>(A1) </em></strong>for <span class="s2">3 </span>or <span class="s2">4 </span>points correctly plotted.</p>
<p class="p3">Award at most <strong><em>(A0)(A3) </em></strong>if axes reversed.</p>
<p class="p3">Accept \(x\) and \(y\) sufficient for labelling.</p>
<p class="p3">If graph paper is not used, award <strong><em>(A0)</em></strong>.</p>
<p class="p3">If an inconsistent scale is used, award <strong><em>(A0)</em></strong><em>. </em>Candidates’ points should be read from this scale <strong>where possible </strong>and awarded accordingly.</p>
<p class="p3">A scale which is too small to be meaningful (ie mm instead of cm) earns <strong><em>(A0) </em></strong>for plotted points.</p>
<p class="p2"> </p>
<p class="p3"><strong><em>[4 marks]</em></strong></p>
<div class="question_part_label">a.</div>
</div>
<div class="question" style="padding-left: 20px;">
<p class="p1">(i) <span class="Apple-converted-space">    \(\bar x = 21\)</span> <span class="Apple-converted-space">    </span><strong><em>(A1)</em></strong></p>
<p class="p1">(ii) <span class="Apple-converted-space">   \(\bar y = 31\)</span> <span class="Apple-converted-space">    </span><strong><em>(A1)</em></strong></p>
<p class="p1"><strong><em>[2 marks]</em></strong></p>
<div class="question_part_label">b.</div>
</div>
<div class="question" style="padding-left: 20px;">
<p class="p1">\((\bar x,{\text{ }}\bar y)\) correctly plotted on graph <span class="Apple-converted-space">    </span><strong><em>(A1)</em>(ft)</strong></p>
<p class="p1">this point labelled <span class="s1">M <span class="Apple-converted-space">    </span></span><strong><em>(A1)</em></strong></p>
<p class="p2"> </p>
<p class="p1"><strong>Note: <span class="Apple-converted-space">    </span></strong>Follow through from parts (b)(i) and (b)(ii).</p>
<p class="p1">Only accept <span class="s1">M </span>for labelling.</p>
<p class="p2"> </p>
<p class="p3"><strong><em>[2 marks]</em></strong></p>
<div class="question_part_label">c.</div>
</div>
<div class="question" style="padding-left: 20px;">
<p class="p1"><span class="Apple-converted-space">\( - 0.973{\text{ }}( - 0.973388 \ldots )\)    </span><strong><em>(G2)</em></strong></p>
<p class="p2"> </p>
<p class="p1"><strong>Note: <span class="Apple-converted-space">    </span></strong>Award <strong><em>(G1) </em></strong>for <span class="s1">0.973</span>, without minus sign.</p>
<p class="p2"> </p>
<p class="p1"><strong><em>[2 marks]</em></strong></p>
<div class="question_part_label">d.</div>
</div>
<div class="question" style="padding-left: 20px;">
<p class="p1"><span class="Apple-converted-space">\(y =  - 0.761x + 47.0{\text{ }}(y =  - 0.760638 \ldots x + 46.9734 \ldots )\)    </span><strong><em>(A1)(A1)(G2)</em></strong></p>
<p class="p2"> </p>
<p class="p1"><strong>Notes: <span class="Apple-converted-space">    </span></strong>Award <strong><em>(A1) </em></strong>for \( - 0.761x\) and <strong><em>(A1)</em></strong> \( + 47.0\). Award a maximum of <strong><em>(A1)(A0) </em></strong>if answer is not an equation.</p>
<p class="p2"> </p>
<p class="p1"><strong><em>[2 marks]</em></strong></p>
<div class="question_part_label">e.</div>
</div>
<div class="question" style="padding-left: 20px;">
<p class="p1">line on graph <span class="Apple-converted-space">    </span><strong><em>(A1)</em>(ft)<em>(A1)</em>(ft)</strong></p>
<p class="p2"> </p>
<p class="p1"><strong>Notes: <span class="Apple-converted-space">    </span></strong>Award <strong><em>(A1)</em>(ft) </strong>for <strong>straight line </strong>that passes through their <span class="s1">M</span>, <strong><em>(A1)</em>(ft) </strong>for line (extrapolated if necessary) that passes through \((0,{\text{ }}47.0)\).</p>
<p class="p1">If M is not plotted or labelled, follow through from part (e).</p>
<p class="p2"> </p>
<p class="p1"><strong><em>[2 marks]</em></strong></p>
<div class="question_part_label">f.</div>
</div>
<div class="question" style="padding-left: 20px;">
<p class="p1"><span class="Apple-converted-space">\(y =  - \frac{2}{3}(34) + \frac{{125}}{3}\)    </span><span class="s1"><strong><em>(M1)</em></strong></span></p>
<p class="p2"> </p>
<p class="p3"><strong>Note: <span class="Apple-converted-space">    </span></strong>Award <strong><em>(M1) </em></strong>for correct substitution.</p>
<p class="p2"> </p>
<p class="p3"><span class="s2">19 </span>(points) <span class="Apple-converted-space">    </span><strong><em>(A1)(G2)</em></strong></p>
<p class="p3"><strong><em>[2 marks]</em></strong></p>
<div class="question_part_label">g.</div>
</div>
<div class="question" style="padding-left: 20px;">
<p class="p1">extrapolation <span class="Apple-converted-space">    </span><strong><em>(R1)</em></strong></p>
<p class="p1"><strong>OR</strong></p>
<p class="p1"><span class="s1">34 </span>hours is outside the given range of data <span class="Apple-converted-space">    </span><strong><em>(R1)</em></strong></p>
<p class="p2"> </p>
<p class="p1"><strong>Note: <span class="Apple-converted-space">    </span></strong>Do not accept ‘outlier’.</p>
<p class="p2"> </p>
<p class="p1"><strong><em>[1 mark]</em></strong></p>
<div class="question_part_label">h.</div>
</div>
<h2 style="margin-top: 1em">Examiners report</h2>
<div class="question" style="padding-left: 20px;">
[N/A]
<div class="question_part_label">a.</div>
</div>
<div class="question" style="padding-left: 20px;">
[N/A]
<div class="question_part_label">b.</div>
</div>
<div class="question" style="padding-left: 20px;">
[N/A]
<div class="question_part_label">c.</div>
</div>
<div class="question" style="padding-left: 20px;">
[N/A]
<div class="question_part_label">d.</div>
</div>
<div class="question" style="padding-left: 20px;">
[N/A]
<div class="question_part_label">e.</div>
</div>
<div class="question" style="padding-left: 20px;">
[N/A]
<div class="question_part_label">f.</div>
</div>
<div class="question" style="padding-left: 20px;">
[N/A]
<div class="question_part_label">g.</div>
</div>
<div class="question" style="padding-left: 20px;">
[N/A]
<div class="question_part_label">h.</div>
</div>
<br><hr><br><div class="specification">
<p style="margin: 0.0px 0.0px 0.0px 0.0px; font: 21.0px 'Times New Roman';"><span style="font-family: 'times new roman', times; font-size: medium;">Consider the function \(f(x) = \frac{3}{4}{x^4} - {x^3} - 9{x^2} + 20\).</span></p>
</div>

<div class="question" style="padding-left: 20px; padding-right: 20px;">
<p><span>Find \(f( - 2)\).</span></p>
<div class="marks">[2]</div>
<div class="question_part_label">a.</div>
</div>
<div class="question" style="padding-left: 20px; padding-right: 20px;">
<p><span>Find \(f'(x)\).</span></p>
<div class="marks">[3]</div>
<div class="question_part_label">b.</div>
</div>
<div class="question" style="padding-left: 20px; padding-right: 20px;">
<p><span>The graph of the function \(f(x)\) has a local minimum at the point where \(x =  - 2\).</span></p>
<p><span>Using your answer to part (b), show that there is a second local minimum at \(x = 3\).</span></p>
<div class="marks">[5]</div>
<div class="question_part_label">c.</div>
</div>
<div class="question" style="padding-left: 20px; padding-right: 20px;">
<p><span>The graph of the function \(f(x)\) has a local minimum at the point where \(x =  - 2\).</span></p>
<p><span>Sketch the graph of the function \(f(x)\) for \( - 5 \leqslant x \leqslant 5\) and \( - 40 \leqslant y \leqslant 50\). Indicate on your</span></p>
<p><span>sketch the coordinates of the \(y\)-intercept.</span></p>
<div class="marks">[4]</div>
<div class="question_part_label">d.</div>
</div>
<div class="question" style="padding-left: 20px; padding-right: 20px;">
<p><span>The graph of the function \(f(x)\) has a local minimum at the point where \(x =  - 2\).</span></p>
<p><span>Write down the coordinates of the local maximum.</span></p>
<div class="marks">[2]</div>
<div class="question_part_label">e.</div>
</div>
<div class="question" style="padding-left: 20px; padding-right: 20px;">
<p><span>Let \(T\) be the tangent to the graph of the function \(f(x)\) at the point \((2, –12)\).</span></p>
<p><span>Find the gradient of \(T\).</span></p>
<div class="marks">[2]</div>
<div class="question_part_label">f.</div>
</div>
<div class="question" style="padding-left: 20px; padding-right: 20px;">
<p><span>The line \(L\) passes through the point \((2, −12)\) and is perpendicular to \(T\).</span></p>
<p><span>\(L\) has equation \(x + by + c = 0\), where \(b\) and \(c \in \mathbb{Z}\).</span></p>
<p><span>Find</span></p>
<p><span>(i)     the gradient of \(L\);</span></p>
<p><span>(ii)     the value of \(b\) and the value of \(c\).</span></p>
<div class="marks">[5]</div>
<div class="question_part_label">g.</div>
</div>
<h2 style="margin-top: 1em">Markscheme</h2>
<div class="question" style="padding-left: 20px;">
<p><span>\(\frac{3}{4}{( - 2)^4} - {( - 2)^3} - 9{( - 2)^2} + 20\)     <strong><em>(M1)</em></strong></span></p>
<p> </p>
<p><span><strong>Note: </strong>Award <strong><em>(M1) </em></strong>for substituting \(x =  - 2\) in the function.</span></p>
<p> </p>
<p><span>\(= 4\)     <strong><em>(A1)(G2)</em></strong></span></p>
<p> </p>
<p><span><strong>Note: </strong>If the coordinates \(( - 2,{\text{ }}4)\) are given as the answer award, at most, <strong><em>(M1)(A0)</em></strong><em>. </em>If no working shown award <strong><em>(G1)</em></strong><em>.</em></span></p>
<p><span><em>     </em>If \(x =  - 2,{\text{ }}y = 4\) seen then award full marks.</span></p>
<p> </p>
<p><span><strong><em>[2 marks]</em></strong></span></p>
<div class="question_part_label">a.</div>
</div>
<div class="question" style="padding-left: 20px;">
<p><span>\(3{x^3} - 3{x^2} - 18x\)     <strong><em>(A1)(A1)(A1)</em></strong></span></p>
<p> </p>
<p><span><strong>Note: </strong>Award <strong><em>(A1) </em></strong>for each correct term, award at most <strong><em>(A1)(A1)(A0) </em></strong>if extra terms seen.</span></p>
<p> </p>
<p><span><strong><em>[3 marks]</em></strong></span></p>
<div class="question_part_label">b.</div>
</div>
<div class="question" style="padding-left: 20px;">
<p><span>\(f'(3) = 3 \times {(3)^3} - 3 \times {(3)^2} - 18 \times 3\)     <strong><em>(M1)</em></strong></span></p>
<p> </p>
<p><span><strong>Note: </strong>Award <strong><em>(M1) </em></strong>for substitution in their \(f'(x)\) of \(x = 3\).</span></p>
<p> </p>
<p><span>\( = 0\)     <em><strong>(A1)</strong></em></span></p>
<p><span><strong>OR</strong></span></p>
<p><span>\(3{x^3} - 3{x^2} - 18x = 0\)     <strong><em>(M1)</em></strong></span></p>
<p> </p>
<p><span><strong>Note: </strong>Award <strong><em>(M1) </em></strong>for equating their \(f'(x)\) to zero.</span></p>
<p> </p>
<p><span>\(x = 3\)     <strong><em>(A1)</em></strong></span></p>
<p><span>\(f'({x_1}) = 3 \times {({x_1})^3} - 3 \times {({x_1})^2} - 18 \times {x_1} &lt; 0\) where \(0 &lt; {x_1} &lt; 3\)     <strong><em>(M1)</em></strong></span></p>
<p> </p>
<p><span><strong>Note: </strong>Award <strong><em>(M1) </em></strong>for substituting a value of \({x_1}\) in the range \(0 &lt; {x_1} &lt; 3\) into their \(f'\) and showing it is negative (decreasing).</span></p>
<p> </p>
<p><span>\(f'({x_2}) = 3 \times {({x_2})^3} - 3 \times {({x_2})^2} - 18 \times {x_2} &gt; 0\) where \({x_2} &gt; 3\)     <strong><em>(M1)</em></strong></span></p>
<p> </p>
<p><span><strong>Note: </strong>Award <strong><em>(M1) </em></strong>for substituting a value of \({x_2}\) in the range \({x_2} &gt; 3\) into their \(f'\) and showing it is positive (increasing).</span></p>
<p><span><strong>OR</strong></span></p>
<p><span><em>With or without a sketch:</em></span></p>
<p><span>Showing \(f({x_1}) &gt; f(3)\) where \({x_1} &lt; 3\) and \({x_1}\) is close to 3.     <strong><em>(M1)</em></strong></span></p>
<p><span>Showing \(f({x_2}) &gt; f(3)\) where \({x_2} &gt; 3\) and \({x_2}\) is close to 3.     <strong><em>(M1)</em></strong></span></p>
<p> </p>
<p><span><strong>Note: </strong>If a sketch of \(f(x)\) is drawn <strong>in this part of the question and</strong> \(x = 3\) is identified as a stationary point on the curve, then</span></p>
<p><span>     (i) award, at most, <strong><em>(M1)(A1)(M1)(M0) </em></strong>if the stationary point has been found;</span></p>
<p><span>     (ii) award, at most, <strong><em>(M0)(A0)(M1)(M0) </em></strong>if the stationary point has not been previously found.</span></p>
<p> </p>
<p><span>Since the gradients go from negative (decreasing) through zero to positive (increasing) it is a local minimum     <strong><em>(R1)(AG)</em></strong></span></p>
<p> </p>
<p><span><strong>Note: </strong>Only award <strong><em>(R1) </em></strong>if the first two marks have been awarded <em>ie</em> \(f'(3)\) has been shown to be equal to \(0\).</span></p>
<p> </p>
<p><span><strong><em>[5 marks]</em></strong></span></p>
<div class="question_part_label">c.</div>
</div>
<div class="question" style="padding-left: 20px;">
<p><span><img src="images/Schermafbeelding_2014-09-03_om_16.12.18.png" alt><span>     <strong><em>(A1)(A1)(A1)(A1)</em></strong></span></span></p>
<p> </p>
<p><span><strong>Notes: </strong>Award <strong><em>(A1) </em></strong>for labelled axes and indication of scale on both axes.</span></p>
<p><span>     Award <strong><em>(A1) </em></strong>for smooth curve with correct shape.</span></p>
<p><span>     Award <strong><em>(A1) </em></strong>for local minima in \({2^{{\text{nd}}}}\) and \({4^{{\text{th}}}}\) quadrants.</span></p>
<p><span>     Award <strong><em>(A1) </em></strong>for <em>y </em>intercept \((0, 20)\) seen and labelled. Accept \(20\) on \(y\)<em>-</em>axis.</span></p>
<p><span>     Do <strong>not </strong>award the third <strong><em>(A1) </em></strong>mark if there is a turning point on the \(x\)-axis.</span></p>
<p><span>     If the derivative function is sketched then award, at most, <strong><em>(A1)(A0)(A0)(A0)</em></strong>.</span></p>
<p><span>     For a smooth curve (with correct shape) there should be <strong>ONE </strong>continuous thin line, no part of which is straight and no (one to many) mappings of \(x\).</span></p>
<p> </p>
<p><span><strong><em>[4 marks]</em></strong></span></p>
<div class="question_part_label">d.</div>
</div>
<div class="question" style="padding-left: 20px;">
<p><span>\((0, 20)\)     <strong><em>(G1)(G1)</em></strong></span></p>
<p> </p>
<p><span><strong>Note: </strong>If parentheses are omitted award <strong><em>(G0)(G1)</em></strong>.</span></p>
<p><span><strong>OR</strong></span></p>
<p><span>\(x = 0,{\text{ }}y = 20\)     <strong><em>(G1)(G1)</em></strong></span></p>
<p> </p>
<p><span><strong>Note: </strong>If the derivative function is sketched in part (d), award <strong><em>(G1)</em>(ft)<em>(G1)</em>(ft) </strong>for \((–1.12, 12.2)\).</span></p>
<p> </p>
<p><span><strong><em>[2 marks]</em></strong></span></p>
<div class="question_part_label">e.</div>
</div>
<div class="question" style="padding-left: 20px;">
<p><span>\(f'(2) = 3{(2)^3} - 3{(2)^2} - 18(2)\)     <strong><em>(M1)</em></strong></span></p>
<p> </p>
<p><span><strong>Notes: </strong>Award <strong><em>(M1) </em></strong>for substituting \(x = 2\) into their \(f'(x)\).</span></p>
<p> </p>
<p><span>\( =  - 24\)     <strong><em>(A1)</em>(ft)<em>(G2)</em></strong></span></p>
<p><span><strong><em>[2 marks]</em></strong></span></p>
<div class="question_part_label">f.</div>
</div>
<div class="question" style="padding-left: 20px;">
<p><span>(i)     Gradient of perpendicular \( = \frac{1}{{24}}\)   \((0.0417, 0.041666…)\)     <strong><em>(A1)</em>(ft)<em>(G1)</em></strong></span></p>
<p> </p>
<p><span><strong>Note: </strong>Follow through from part (f).</span></p>
<p> </p>
<p><span>(ii)     \(y + 12 = \frac{1}{{24}}(x - 2)\)     <strong><em>(M1)(M1)</em></strong></span></p>
<p> </p>
<p><span><strong>Note: </strong>Award <strong><em>(M1) </em></strong>for correct substitution of \((2, –12)\), <strong><em>(M1) </em></strong>for correct substitution of their perpendicular gradient into equation of line.</span></p>
<p><span><strong>OR</strong></span></p>
<p><span>\( - 12 = \frac{1}{{24}} \times 2 + d\)     <strong><em>(M1)</em></strong></span></p>
<p><span>\(d =  - \frac{{145}}{{12}}\)</span></p>
<p><span>\(y = \frac{1}{{24}}x - \frac{{145}}{{12}}\)     <strong><em>(M1)</em></strong></span></p>
<p> </p>
<p><span><strong>Note: </strong>Award <strong><em>(M1) </em></strong>for correct substitution of \((2, –12)\) and gradient into equation of a straight line, <strong><em>(M1) </em></strong>for correct substitution of the perpendicular gradient and correct substitution of \(d\)into equation of line.</span></p>
<p> </p>
<p><span>\(b =  - 24,{\text{ }}c =  - 290\)     <strong><em>(A1)</em>(ft)<em>(A1)</em>(ft)<em>(G3)</em></strong></span></p>
<p> </p>
<p><span><strong>Note: </strong>Follow through from parts (f) and g(i).</span></p>
<p><span>     To award <strong>(ft) </strong>marks, \(b\) and \(c\) must be integers.</span></p>
<p><span>     Where candidate has used \(0.042\) from g(i), award <strong><em>(A1)</em>(ft) </strong>for \(–288\).</span></p>
<p> </p>
<p><span><strong><em>[5 marks]</em></strong></span></p>
<div class="question_part_label">g.</div>
</div>
<h2 style="margin-top: 1em">Examiners report</h2>
<div class="question" style="padding-left: 20px;">
<p style="margin: 0.0px 0.0px 0.0px 0.0px; font: 19.0px Helvetica;"><span style="font-family: 'times new roman', times; font-size: medium;">Surprisingly, a correct method for substituting the value of &ndash;2 into the given function led many candidates to a variety of incorrect answers. This suggests a poor handling of negative signs and/or poor use of the graphic display calculator. Many correct answers were seen in part (b) as candidates seemed to be well-drilled in the process of differentiation. Part (c), however, proved to be quite a discriminator. There were 5 marks for this part of the question and simply showing that \(x - 3\) is a turning point was not sufficient for all of these marks. Many simply scored only two marks by substituting \(x - 3\) into their answer to part (b). Once they had shown that there was a turning point at \(x - 3\), candidates were not expected to use the second derivative but to show that the function decreases for \(x &lt; 3\) and increases for \(x &gt; 3\). Part (d) required a sketch which could have either been done on lined paper or on graph paper. The majority of candidates obtained at least two marks here with the most common errors seen being incomplete labelled axes and curves which were far from being smooth. In part (e), many candidates identified the correct coordinates for the two marks available. But for many candidates, this is where responses stopped as, in part (f), connecting the gradient function found in part (b) to the given coordinates proved problematic and only a significant minority of candidates were able to arrive at the required answer of &ndash;24. Indeed, there were many NR (no responses) to this part and the final part of the question. As many candidates found part (f) difficult, even more candidates found getting beyond the gradient of <em>L</em> very difficult indeed. A minority of candidates wrote down the gradient of their perpendicular but then did not seem to know where to proceed from there. Substituting their gradient for <em>b</em> and the coordinates (2,&nbsp;&ndash;12) into the equation \(x + by + c = 0\) was a popular, but erroneous, method. It was a rare event indeed to see a script with a correct answer for this part of the question.</span></p>
<div class="question_part_label">a.</div>
</div>
<div class="question" style="padding-left: 20px;">
<p style="margin: 0.0px 0.0px 0.0px 0.0px; font: 19.0px Helvetica;"><span style="font-family: 'times new roman', times; font-size: medium;">Surprisingly, a correct method for substituting the value of &ndash;2 into the given function led many candidates to a variety of incorrect answers. This suggests a poor handling of negative signs and/or poor use of the graphic display calculator. Many correct answers were seen in part (b) as candidates seemed to be well-drilled in the process of differentiation. Part (c), however, proved to be quite a discriminator. There were 5 marks for this part of the question and simply showing that \(x - 3\) is a turning point was not sufficient for all of these marks. Many simply scored only two marks by substituting \(x - 3\) into their answer to part (b). Once they had shown that there was a turning point at \(x - 3\), candidates were not expected to use the second derivative but to show that the function decreases for \(x &lt; 3\) and increases for \(x &gt; 3\). Part (d) required a sketch which could have either been done on lined paper or on graph paper. The majority of candidates obtained at least two marks here with the most common errors seen being incomplete labelled axes and curves which were far from being smooth. In part (e), many candidates identified the correct coordinates for the two marks available. But for many candidates, this is where responses stopped as, in part (f), connecting the gradient function found in part (b) to the given coordinates proved problematic and only a significant minority of candidates were able to arrive at the required answer of &ndash;24. Indeed, there were many NR (no responses) to this part and the final part of the question. As many candidates found part (f) difficult, even more candidates found getting beyond the gradient of <em>L</em> very difficult indeed. A minority of candidates wrote down the gradient of their perpendicular but then did not seem to know where to proceed from there. Substituting their gradient for <em>b</em> and the coordinates (2,&nbsp;&ndash;12) into the equation \(x + by + c = 0\) was a popular, but erroneous, method. It was a rare event indeed to see a script with a correct answer for this part of the question.</span></p>
<div class="question_part_label">b.</div>
</div>
<div class="question" style="padding-left: 20px;">
<p style="margin: 0.0px 0.0px 0.0px 0.0px; font: 19.0px Helvetica;"><span style="font-family: 'times new roman', times; font-size: medium;">Surprisingly, a correct method for substituting the value of &ndash;2 into the given function led many candidates to a variety of incorrect answers. This suggests a poor handling of negative signs and/or poor use of the graphic display calculator. Many correct answers were seen in part (b) as candidates seemed to be well-drilled in the process of differentiation. Part (c), however, proved to be quite a discriminator. There were 5 marks for this part of the question and simply showing that \(x - 3\) is a turning point was not sufficient for all of these marks. Many simply scored only two marks by substituting \(x - 3\) into their answer to part (b). Once they had shown that there was a turning point at \(x - 3\), candidates were not expected to use the second derivative but to show that the function decreases for \(x &lt; 3\) and increases for \(x &gt; 3\). Part (d) required a sketch which could have either been done on lined paper or on graph paper. The majority of candidates obtained at least two marks here with the most common errors seen being incomplete labelled axes and curves which were far from being smooth. In part (e), many candidates identified the correct coordinates for the two marks available. But for many candidates, this is where responses stopped as, in part (f), connecting the gradient function found in part (b) to the given coordinates proved problematic and only a significant minority of candidates were able to arrive at the required answer of &ndash;24. Indeed, there were many NR (no responses) to this part and the final part of the question. As many candidates found part (f) difficult, even more candidates found getting beyond the gradient of <em>L</em> very difficult indeed. A minority of candidates wrote down the gradient of their perpendicular but then did not seem to know where to proceed from there. Substituting their gradient for <em>b</em> and the coordinates (2,&nbsp;&ndash;12) into the equation \(x + by + c = 0\) was a popular, but erroneous, method. It was a rare event indeed to see a script with a correct answer for this part of the question.</span></p>
<div class="question_part_label">c.</div>
</div>
<div class="question" style="padding-left: 20px;">
<p style="margin: 0.0px 0.0px 0.0px 0.0px; font: 19.0px Helvetica;"><span style="font-family: 'times new roman', times; font-size: medium;">Surprisingly, a correct method for substituting the value of &ndash;2 into the given function led many candidates to a variety of incorrect answers. This suggests a poor handling of negative signs and/or poor use of the graphic display calculator. Many correct answers were seen in part (b) as candidates seemed to be well-drilled in the process of differentiation. Part (c), however, proved to be quite a discriminator. There were 5 marks for this part of the question and simply showing that \(x - 3\) is a turning point was not sufficient for all of these marks. Many simply scored only two marks by substituting \(x - 3\) into their answer to part (b). Once they had shown that there was a turning point at \(x - 3\), candidates were not expected to use the second derivative but to show that the function decreases for \(x &lt; 3\) and increases for \(x &gt; 3\). Part (d) required a sketch which could have either been done on lined paper or on graph paper. The majority of candidates obtained at least two marks here with the most common errors seen being incomplete labelled axes and curves which were far from being smooth. In part (e), many candidates identified the correct coordinates for the two marks available. But for many candidates, this is where responses stopped as, in part (f), connecting the gradient function found in part (b) to the given coordinates proved problematic and only a significant minority of candidates were able to arrive at the required answer of &ndash;24. Indeed, there were many NR (no responses) to this part and the final part of the question. As many candidates found part (f) difficult, even more candidates found getting beyond the gradient of <em>L</em> very difficult indeed. A minority of candidates wrote down the gradient of their perpendicular but then did not seem to know where to proceed from there. Substituting their gradient for <em>b</em> and the coordinates (2,&nbsp;&ndash;12) into the equation \(x + by + c = 0\) was a popular, but erroneous, method. It was a rare event indeed to see a script with a correct answer for this part of the question.</span></p>
<div class="question_part_label">d.</div>
</div>
<div class="question" style="padding-left: 20px;">
<p style="margin: 0.0px 0.0px 0.0px 0.0px; font: 19.0px Helvetica;"><span style="font-family: 'times new roman', times; font-size: medium;">Surprisingly, a correct method for substituting the value of &ndash;2 into the given function led many candidates to a variety of incorrect answers. This suggests a poor handling of negative signs and/or poor use of the graphic display calculator. Many correct answers were seen in part (b) as candidates seemed to be well-drilled in the process of differentiation. Part (c), however, proved to be quite a discriminator. There were 5 marks for this part of the question and simply showing that \(x - 3\) is a turning point was not sufficient for all of these marks. Many simply scored only two marks by substituting \(x - 3\) into their answer to part (b). Once they had shown that there was a turning point at \(x - 3\), candidates were not expected to use the second derivative but to show that the function decreases for \(x &lt; 3\) and increases for \(x &gt; 3\). Part (d) required a sketch which could have either been done on lined paper or on graph paper. The majority of candidates obtained at least two marks here with the most common errors seen being incomplete labelled axes and curves which were far from being smooth. In part (e), many candidates identified the correct coordinates for the two marks available. But for many candidates, this is where responses stopped as, in part (f), connecting the gradient function found in part (b) to the given coordinates proved problematic and only a significant minority of candidates were able to arrive at the required answer of &ndash;24. Indeed, there were many NR (no responses) to this part and the final part of the question. As many candidates found part (f) difficult, even more candidates found getting beyond the gradient of <em>L</em> very difficult indeed. A minority of candidates wrote down the gradient of their perpendicular but then did not seem to know where to proceed from there. Substituting their gradient for <em>b</em> and the coordinates (2,&nbsp;&ndash;12) into the equation \(x + by + c = 0\) was a popular, but erroneous, method. It was a rare event indeed to see a script with a correct answer for this part of the question.</span></p>
<div class="question_part_label">e.</div>
</div>
<div class="question" style="padding-left: 20px;">
<p style="margin: 0.0px 0.0px 0.0px 0.0px; font: 19.0px Helvetica;"><span style="font-family: 'times new roman', times; font-size: medium;">Surprisingly, a correct method for substituting the value of &ndash;2 into the given function led many candidates to a variety of incorrect answers. This suggests a poor handling of negative signs and/or poor use of the graphic display calculator. Many correct answers were seen in part (b) as candidates seemed to be well-drilled in the process of differentiation. Part (c), however, proved to be quite a discriminator. There were 5 marks for this part of the question and simply showing that \(x - 3\) is a turning point was not sufficient for all of these marks. Many simply scored only two marks by substituting \(x - 3\) into their answer to part (b). Once they had shown that there was a turning point at \(x - 3\), candidates were not expected to use the second derivative but to show that the function decreases for \(x &lt; 3\) and increases for \(x &gt; 3\). Part (d) required a sketch which could have either been done on lined paper or on graph paper. The majority of candidates obtained at least two marks here with the most common errors seen being incomplete labelled axes and curves which were far from being smooth. In part (e), many candidates identified the correct coordinates for the two marks available. But for many candidates, this is where responses stopped as, in part (f), connecting the gradient function found in part (b) to the given coordinates proved problematic and only a significant minority of candidates were able to arrive at the required answer of &ndash;24. Indeed, there were many NR (no responses) to this part and the final part of the question. As many candidates found part (f) difficult, even more candidates found getting beyond the gradient of <em>L</em> very difficult indeed. A minority of candidates wrote down the gradient of their perpendicular but then did not seem to know where to proceed from there. Substituting their gradient for <em>b</em> and the coordinates (2,&nbsp;&ndash;12) into the equation \(x + by + c = 0\) was a popular, but erroneous, method. It was a rare event indeed to see a script with a correct answer for this part of the question.</span></p>
<div class="question_part_label">f.</div>
</div>
<div class="question" style="padding-left: 20px;">
<p style="margin: 0.0px 0.0px 0.0px 0.0px; font: 19.0px Helvetica;"><span style="font-family: 'times new roman', times; font-size: medium;">Surprisingly, a correct method for substituting the value of &ndash;2 into the given function led many candidates to a variety of incorrect answers. This suggests a poor handling of negative signs and/or poor use of the graphic display calculator. Many correct answers were seen in part (b) as candidates seemed to be well-drilled in the process of differentiation. Part (c), however, proved to be quite a discriminator. There were 5 marks for this part of the question and simply showing that \(x - 3\) is a turning point was not sufficient for all of these marks. Many simply scored only two marks by substituting \(x - 3\) into their answer to part (b). Once they had shown that there was a turning point at \(x - 3\), candidates were not expected to use the second derivative but to show that the function decreases for \(x &lt; 3\) and increases for \(x &gt; 3\). Part (d) required a sketch which could have either been done on lined paper or on graph paper. The majority of candidates obtained at least two marks here with the most common errors seen being incomplete labelled axes and curves which were far from being smooth. In part (e), many candidates identified the correct coordinates for the two marks available. But for many candidates, this is where responses stopped as, in part (f), connecting the gradient function found in part (b) to the given coordinates proved problematic and only a significant minority of candidates were able to arrive at the required answer of &ndash;24. Indeed, there were many NR (no responses) to this part and the final part of the question. As many candidates found part (f) difficult, even more candidates found getting beyond the gradient of <em>L</em> very difficult indeed. A minority of candidates wrote down the gradient of their perpendicular but then did not seem to know where to proceed from there. Substituting their gradient for <em>b</em> and the coordinates (2,&nbsp;&ndash;12) into the equation \(x + by + c = 0\) was a popular, but erroneous, method. It was a rare event indeed to see a script with a correct answer for this part of the question.</span></p>
<div class="question_part_label">g.</div>
</div>
<br><hr><br><div class="specification">
<p class="p1">Consider the function \(f(x) = \frac{{96}}{{{x^2}}} + kx\), where \(k\) is a constant and \(x \ne 0\).</p>
</div>

<div class="question" style="padding-left: 20px; padding-right: 20px;">
<p class="p1">Write down \(f'(x)\).</p>
<div class="marks">[3]</div>
<div class="question_part_label">a.</div>
</div>
<div class="question" style="padding-left: 20px; padding-right: 20px;">
<p class="p1">The graph of \(y = f(x)\) has a local minimum point at \(x = 4\).</p>
<p class="p1">Show that \(k = 3\).</p>
<div class="marks">[2]</div>
<div class="question_part_label">b.</div>
</div>
<div class="question" style="padding-left: 20px; padding-right: 20px;">
<p class="p1">The graph of \(y = f(x)\) has a local minimum point at \(x = 4\).</p>
<p class="p1">Find \(f(2)\).</p>
<div class="marks">[2]</div>
<div class="question_part_label">c.</div>
</div>
<div class="question" style="padding-left: 20px; padding-right: 20px;">
<p class="p1">The graph of \(y = f(x)\) has a local minimum point at \(x = 4\).</p>
<p class="p1">Find \(f'(2)\)</p>
<div class="marks">[2]</div>
<div class="question_part_label">d.</div>
</div>
<div class="question" style="padding-left: 20px; padding-right: 20px;">
<p class="p1">The graph of \(y = f(x)\) has a local minimum point at \(x = 4\).</p>
<p class="p1">Find the equation of the normal to the graph of \(y = f(x)\) at the point where \(x = 2\).</p>
<p class="p1">Give your answer in the form \(ax + by + d = 0\) where \(a,{\text{ }}b,{\text{ }}d \in \mathbb{Z}\).</p>
<div class="marks">[3]</div>
<div class="question_part_label">e.</div>
</div>
<div class="question" style="padding-left: 20px; padding-right: 20px;">
<p class="p1">The graph of \(y = f(x)\) has a local minimum point at \(x = 4\).</p>
<p class="p1"><span class="s1">Sketch the graph of \(y = f(x)\)</span>, for \( - 5 \leqslant x \leqslant 10\) and \( - 10 \leqslant y \leqslant 100\)<span class="s1">.</span></p>
<div class="marks">[4]</div>
<div class="question_part_label">f.</div>
</div>
<div class="question" style="padding-left: 20px; padding-right: 20px;">
<p class="p1">The graph of \(y = f(x)\) has a local minimum point at \(x = 4\).</p>
<p class="p1">Write down the coordinates of the point where the graph of \(y = f(x)\) intersects the \(x\)-axis.</p>
<div class="marks">[2]</div>
<div class="question_part_label">g.</div>
</div>
<div class="question" style="padding-left: 20px; padding-right: 20px;">
<p class="p1">The graph of \(y = f(x)\) has a local minimum point at \(x = 4\).</p>
<p class="p1">State the values of \(x\) for which \(f(x)\) is decreasing.</p>
<div class="marks">[2]</div>
<div class="question_part_label">h.</div>
</div>
<h2 style="margin-top: 1em">Markscheme</h2>
<div class="question" style="padding-left: 20px;">
<p class="p1">\(\frac{{ - 192}}{{{x^3}}} + k\) <span class="Apple-converted-space">    </span><strong><em>(A1)(A1)(A1)</em></strong></p>
<p class="p2"> </p>
<p class="p1"><strong>Note: </strong>Award <strong><em>(A1) </em></strong>for \(-192\), <strong><em>(A1) </em></strong>for \({x^{ - 3}}\), <strong><em>(A1) </em></strong>for \(k\) (only).</p>
<div class="question_part_label">a.</div>
</div>
<div class="question" style="padding-left: 20px;">
<p class="p1">at local minimum \(f'(x) = 0\) <span class="Apple-converted-space">    </span><strong><em>(M1)</em></strong></p>
<p class="p1"><strong>Note: </strong>Award <strong><em>(M1) </em></strong>for seeing \(f'(x) = 0\) (may be implicit in their working).</p>
<p class="p2"> </p>
<p class="p1">\(\frac{{ - 192}}{{{4^3}}} + k = 0\) <span class="Apple-converted-space">    </span><strong><em>(A1)</em></strong></p>
<p class="p1">\(k = 3\) <span class="Apple-converted-space">    </span><strong><em>(AG)</em></strong></p>
<p class="p1"><strong>Note: </strong>Award <strong><em>(A1) </em></strong>for substituting \(x = 4\) in their \(f'(x) = 0\), provided it leads to \(k = 3\)<em>. </em>The conclusion \(k = 3\) must be seen for the <strong><em>(A1) </em></strong>to be awarded.</p>
<div class="question_part_label">b.</div>
</div>
<div class="question" style="padding-left: 20px;">
<p class="p1">\(\frac{{96}}{{{2^2}}} + 3(2)\) <span class="Apple-converted-space">    </span><strong><em>(M1)</em></strong></p>
<p class="p1"><strong>Note:<span class="Apple-converted-space"> </span></strong>Award <strong><em>(M1) </em></strong>for substituting \(x = 2\) and \(k = 3\) in \(f(x)\).</p>
<p class="p2"> </p>
<p class="p1">\( = 30\) <span class="Apple-converted-space">    </span><strong><em>(A1)(G2)</em></strong></p>
<div class="question_part_label">c.</div>
</div>
<div class="question" style="padding-left: 20px;">
<p>\(\frac{{ - 192}}{{{2^3}}} + 3\)     <strong><em>(M1)</em></strong></p>
<p><strong>Note: </strong>Award <strong><em>(M1) </em></strong>for substituting \(x = 2\) and \(k = 3\) in their \(f'(x)\).</p>
<p> </p>
<p>\( =  - 21\)     <strong><em>(A1)</em>(ft)<em>(G2)</em></strong></p>
<p><strong>Note: </strong>Follow through from part (a).</p>
<div class="question_part_label">d.</div>
</div>
<div class="question" style="padding-left: 20px;">
<p class="p1">\(y - 30 = \frac{1}{{21}}(x - 2)\) <span class="Apple-converted-space">    </span><strong><em>(A1)</em>(ft)<em>(M1)</em></strong></p>
<p class="p1"><strong>Notes:<span class="Apple-converted-space"> </span></strong>Award <strong><em>(A1)</em>(ft) </strong>for their \(\frac{1}{{21}}\) seen, <strong><em>(M1) </em></strong>for the correct substitution of their point and their normal gradient in equation of a line.</p>
<p class="p1">Follow through from part (c) and part (d).</p>
<p class="p2"> </p>
<p class="p1"><strong>OR</strong></p>
<p class="p1">gradient of normal \( = \frac{1}{{21}}\) <span class="Apple-converted-space">    </span><strong><em>(A1)</em>(ft)</strong></p>
<p class="p1">\(30 = \frac{1}{{21}} \times 2 + c\) <span class="Apple-converted-space">    </span><strong><em>(M1)</em></strong></p>
<p class="p1">\(c = 29\frac{{19}}{{21}}\)</p>
<p class="p1">\(y = \frac{1}{{21}}x + 29\frac{{19}}{{21}}\;\;\;(y = 0.0476x + 29.904)\)</p>
<p class="p1">\(x - 21y + 628 = 0\) <span class="Apple-converted-space">    </span><strong><em>(A1)</em>(ft)<em>(G2)</em></strong></p>
<p class="p1"><strong>Notes: <span class="Apple-converted-space"> </span></strong>Accept equivalent answers.</p>
<div class="question_part_label">e.</div>
</div>
<div class="question" style="padding-left: 20px;">
<p class="p1"><span class="s1"><img src="images/Schermafbeelding_2015-12-21_om_09.26.22.png" alt> <span class="Apple-converted-space">    </span></span><strong><em>(A1)(A1)(A1)(A1)</em></strong></p>
<p class="p2"> </p>
<p class="p1"><strong>Notes: </strong>Award <strong><em>(A1) </em></strong>for correct window (at least one value, other than zero, labelled on each axis), the axes must also be labelled; <strong><em>(A1) </em></strong>for a smooth curve with the correct shape (graph should not touch \(y\)-axis and should not curve away from the \(y\)-axis), on the given domain; <strong><em>(A1) </em></strong>for axis intercept in approximately the correct position (nearer \(-5\)<span class="s2"> </span>than zero); <strong><em>(A1) </em></strong>for local minimum in approximately the correct position (first quadrant, nearer the \(y\)-axis than \(x = 10\)).</p>
<p class="p1">If there is no scale, award a maximum of <strong><em>(A0)(A1)(A0)(A1) </em></strong>– the final <strong><em>(A1) </em></strong>being awarded for the zero and local minimum in approximately correct positions relative to each other.</p>
<div class="question_part_label">f.</div>
</div>
<div class="question" style="padding-left: 20px;">
<p>\(( - 3.17,{\text{ }}0)\;\;\;\left( {( - 3.17480 \ldots ,{\text{ 0)}}} \right)\)     <strong><em>(G1)(G1)</em></strong></p>
<p> </p>
<p><strong>Notes: </strong>If parentheses are omitted award <strong><em>(G0)(G1)</em>(ft)</strong>.</p>
<p>Accept \(x =  - 3.17,{\text{ }}y = 0\). Award <strong><em>(G1) </em></strong>for \(-3.17\) seen.</p>
<div class="question_part_label">g.</div>
</div>
<div class="question" style="padding-left: 20px;">
<p class="p1">\(0 &lt; x \leqslant 4{\text{ or }}0 &lt; x &lt; 4\) <span class="Apple-converted-space">    </span><strong><em>(A1)(A1)</em></strong></p>
<p class="p1"><strong>Notes: </strong>Award <strong><em>(A1) </em></strong>for correct end points of interval, <strong><em>(A1) </em></strong>for correct notation (note: lower inequality must be strict).</p>
<p class="p1">Award a maximum of <strong><em>(A1)(A0) </em></strong>if \(y\) or \(f(x)\) used in place of \(x\).</p>
<div class="question_part_label">h.</div>
</div>
<h2 style="margin-top: 1em">Examiners report</h2>
<div class="question" style="padding-left: 20px;">
<p>Differentiation of terms including negative indices remains a testing process; it will continue to be tested. There was, however, a noticeable improvement in responses compared to previous years. The parameter k was problematic for a number of candidates.</p>
<div class="question_part_label">a.</div>
</div>
<div class="question" style="padding-left: 20px;">
<p>In part (b), the manipulation of the derivative to find the local minimum point caused difficulties for all but the most able; note that a GDC approach is not accepted in such questions and that candidates are expected to be able to apply the theory of the calculus as appropriate. Further, once a parameter is given, candidates are expected to use this value in subsequent parts.</p>
<div class="question_part_label">b.</div>
</div>
<div class="question" style="padding-left: 20px;">
<p>Parts (c) and (d) were accessible and all but the weakest candidates scored well.</p>
<div class="question_part_label">c.</div>
</div>
<div class="question" style="padding-left: 20px;">
<p>Parts (c) and (d) were accessible and all but the weakest candidates scored well.</p>
<div class="question_part_label">d.</div>
</div>
<div class="question" style="padding-left: 20px;">
<p>Part (e) discriminated at the highest level; the gradient of the normal often was not used, the form of the answer not given correctly.</p>
<div class="question_part_label">e.</div>
</div>
<div class="question" style="padding-left: 20px;">
<p>Curve sketching is a skill that most candidates find very difficult; axes must be labelled and some indication of the window must be present; care must be taken with the domain and the range; any asymptotic behaviour must be indicated. It was very rare to see sketches that attained full marks, yet this should be a skill that all can attain. There were many no attempts seen, yet some of these had correct answers to part (g).</p>
<div class="question_part_label">f.</div>
</div>
<div class="question" style="padding-left: 20px;">
<p>Curve sketching is a skill that most candidates find very difficult; axes must be labelled and some indication of the window must be present; care must be taken with the domain and the range; any asymptotic behaviour must be indicated. It was very rare to see sketches that attained full marks, yet this should be a skill that all can attain. There were many no attempts seen, yet some of these had correct answers to part (g).</p>
<div class="question_part_label">g.</div>
</div>
<div class="question" style="padding-left: 20px;">
<p>Part (h) was not well attempted in the main; decreasing (and increasing) functions is a testing concept for the majority.</p>
<div class="question_part_label">h.</div>
</div>
<br><hr><br><div class="specification">
<p>Violeta plans to grow flowers in a rectangular plot. She places a fence to mark out the perimeter of the plot and uses 200 metres of fence. The length of the plot is \(x\) metres.</p>
<p style="text-align: center;"><img src="images/Schermafbeelding_2017-08-16_om_17.40.47.png" alt="M17/5/MATSD/SP2/ENG/TZ2/05"></p>
</div>

<div class="specification">
<p>Violeta places the fence so that the area of the plot is maximized.</p>
</div>

<div class="specification">
<p>By selling her flowers, Violeta earns 2 Bulgarian Levs (BGN) per square metre of the plot.</p>
</div>

<div class="specification">
<p>Violeta wants to invest her 5000 BGN.</p>
<p>A bank offers a nominal annual interest rate of 4%, compounded <strong>half-yearly</strong>.</p>
</div>

<div class="specification">
<p>Another bank offers an interest rate of \(r\)% compounded <strong>annually</strong>, that would allow her to double her money in 12 years.</p>
</div>

<div class="question" style="padding-left: 20px; padding-right: 20px;">
<p>Show that the width of the plot, in metres, is given by \(100 - x\).</p>
<div class="marks">[1]</div>
<div class="question_part_label">a.</div>
</div>
<div class="question" style="padding-left: 20px; padding-right: 20px;">
<p>Write down the area of the plot in terms of \(x\).</p>
<div class="marks">[1]</div>
<div class="question_part_label">b.</div>
</div>
<div class="question" style="padding-left: 20px; padding-right: 20px;">
<p>Find the value of \(x\) that maximizes the area of the plot.</p>
<div class="marks">[2]</div>
<div class="question_part_label">c.</div>
</div>
<div class="question" style="padding-left: 20px; padding-right: 20px;">
<p>Show that Violeta earns 5000 BGN from selling the flowers grown on the plot.</p>
<div class="marks">[2]</div>
<div class="question_part_label">d.</div>
</div>
<div class="question" style="padding-left: 20px; padding-right: 20px;">
<p>Find the amount of money that Violeta would have after 6 years. Give your answer correct to two decimal places.</p>
<div class="marks">[3]</div>
<div class="question_part_label">e.i.</div>
</div>
<div class="question" style="padding-left: 20px; padding-right: 20px;">
<p>Find how long it would take for the interest earned to be 2000 BGN.</p>
<div class="marks">[3]</div>
<div class="question_part_label">e.ii.</div>
</div>
<div class="question" style="padding-left: 20px; padding-right: 20px;">
<p>Find the lowest possible value for \(r\).</p>
<div class="marks">[2]</div>
<div class="question_part_label">f.</div>
</div>
<h2 style="margin-top: 1em">Markscheme</h2>
<div class="question" style="padding-left: 20px;">
<p>\(\frac{{200 - 2x}}{2}\) (or equivalent)     <strong><em>(M1)</em></strong></p>
<p><strong>OR</strong></p>
<p>\(2x + 2y = 200\) (or equivalent)     <strong><em>(M1)</em></strong></p>
<p> </p>
<p><strong>Note:</strong>     Award <strong><em>(M1) </em></strong>for a correct expression leading to \(100 - x\) (the \(100 - x\) does not need to be seen). The 200 must be seen for the <strong><em>(M1) </em></strong>to be awarded. Do not accept \(100 - x\) substituted in the perimeter of the rectangle formula.</p>
<p> </p>
<p>\(100 - x\)     <strong><em>(AG)</em></strong></p>
<p><strong><em>[1 mark]</em></strong></p>
<div class="question_part_label">a.</div>
</div>
<div class="question" style="padding-left: 20px;">
<p>\(({\text{area}} = ){\text{ }}x(100 - x)\)\(\,\,\,\)<strong>OR</strong>\(\,\,\,\)\( - {x^2} + 100x\) (or equivalent)     <strong><em>(A1)</em></strong></p>
<p><strong><em>[1 mark]</em></strong></p>
<div class="question_part_label">b.</div>
</div>
<div class="question" style="padding-left: 20px;">
<p>\(x = \frac{{ - 100}}{{ - 2}}\)\(\,\,\,\)<strong>OR</strong>\(\,\,\,\)\( - 2x + 100 = 0\)\(\,\,\,\)<strong>OR</strong>\(\,\,\,\)graphical method     <strong><em>(M1)</em></strong></p>
<p> </p>
<p><strong>Note:</strong>     Award <strong><em>(M1) </em></strong>for use of axis of symmetry formula or first derivative equated to zero or a sketch graph.</p>
<p> </p>
<p>\(x = 50\)     <strong><em>(A1)</em>(ft)<em>(G2)</em></strong></p>
<p> </p>
<p><strong>Note:</strong>     Follow through from part (b), provided <em>x </em>is positive and less than 100.</p>
<p> </p>
<p><strong><em>[2 marks]</em></strong></p>
<div class="question_part_label">c.</div>
</div>
<div class="question" style="padding-left: 20px;">
<p>\(50(100 - 50) \times 2\)     <strong><em>(M1)(M1)</em></strong></p>
<p> </p>
<p><strong>Note:</strong>     Award <strong><em>(M1) </em></strong>for substituting their \(x\) into their formula for area (accept “\(50 \times 50\)” for the substituted formula), and <strong><em>(M1) </em></strong>for multiplying by 2. Award at most <strong><em>(M0)(M1) </em></strong>if their calculation does not lead to 5000 (BGN), although the 5000 (BGN) does not need to be seen explicitly.</p>
<p>Substitution of 50 into area formula may be seen in part (c).</p>
<p> </p>
<p>5000 (BGN)     <strong><em>(AG)</em></strong></p>
<p><strong><em>[2 marks]</em></strong></p>
<div class="question_part_label">d.</div>
</div>
<div class="question" style="padding-left: 20px;">
<p>\(5000{\left( {1 + \frac{4}{{2 \times 100}}} \right)^{2 \times 6}}\)     <strong><em>(M1)(A1)</em></strong></p>
<p> </p>
<p><strong>Note:</strong>     Award <strong><em>(M1) </em></strong>for substitution into compound interest formula, <strong><em>(A1) </em></strong>for correct substitution.</p>
<p><strong>OR</strong></p>
<p>\({\text{N}} = 6\)</p>
<p>\({\text{I}}\%  = 4\)</p>
<p>\({\text{PV}} =  - 5000\)</p>
<p>\({\text{P/Y}} = 1\)</p>
<p>\({\text{C/Y}} = 2\)     <strong><em>(M1)(A1)</em></strong></p>
<p> </p>
<p><strong>Note:</strong>     Award <strong><em>(A1) </em></strong>for \({\text{C/Y}} = 2\)  seen, <strong><em>(M1) </em></strong>for other correct entries.</p>
<p> </p>
<p><strong>OR</strong></p>
<p>\({\text{N}} = 12\)</p>
<p>\({\text{I}}\%  = 4\)</p>
<p>\({\text{PV}} =  - 5000\)</p>
<p>\({\text{P/Y}} = 2\)</p>
<p>\({\text{C/Y}} = 2\)     <strong><em>(M1)(A1)</em></strong></p>
<p> </p>
<p><strong>Note:</strong>     Award <strong><em>(A1) </em></strong>for \({\text{C/Y}} = 2\) seen, <strong><em>(M1) </em></strong>for other correct entries.</p>
<p> </p>
<p>6341.21 (BGN)     <strong><em>(A1)(G3)</em></strong></p>
<p> </p>
<p><strong>Note:</strong>     Award <strong><em>(A1) </em></strong>for correct answer, to two decimal places only.</p>
<p>Award <strong><em>(G2) </em></strong>for 6341.20 or a correct, unrounded final answer if no working is seen (6341.2089…).</p>
<p> </p>
<p><strong><em>[3 marks]</em></strong></p>
<div class="question_part_label">e.i.</div>
</div>
<div class="question" style="padding-left: 20px;">
<p>\(5000{\left( {1 + \frac{4}{{2 \times 100}}} \right)^{2 \times t}} = 7000\)     <strong><em>(M1)(A1)</em>(ft)</strong></p>
<p> </p>
<p><strong>Note:</strong>     Award <strong><em>(M1) </em></strong>for using the compound interest formula with a variable for time, <strong><em>(A1)</em>(ft) </strong>for substituting the correct values and equating to 7000. Follow through for their “2” from part (e)(i).</p>
<p> </p>
<p><strong>OR</strong></p>
<p>\({\text{I% }} = 4\)</p>
<p>\({\text{PV}} = ( \pm )5000\)</p>
<p>\({\text{FV}} =  \mp 7000\)</p>
<p>\({\text{P/Y}} = 1\)</p>
<p>\({\text{C/Y}} = 2\)     <strong><em>(M1)(A1)</em></strong></p>
<p> </p>
<p><strong>Note:</strong>     Award <strong><em>(A1) </em></strong>for 7000 seen, <strong><em>(M1) </em></strong>for the other correct entries.</p>
<p>Award <strong><em>(M1) </em></strong>for their C/Y from part (e)(i).</p>
<p> </p>
<p><strong>OR</strong></p>
<p>\({\text{I% }} = 4\)</p>
<p>\({\text{PV}} = ( \pm )5000\)</p>
<p>\({\text{FV}} =  \mp 7000\)</p>
<p>\({\text{P/Y}} = 2\)</p>
<p>\({\text{C/Y}} = 2\)     <strong><em>(M1)(A1)</em></strong></p>
<p> </p>
<p><strong>Note:</strong>     Award <strong><em>(A1) </em></strong>for 7000 seen, <strong><em>(M1) </em></strong>for the other correct entries.</p>
<p>Award <strong><em>(M1) </em></strong>for their C/Y from part (e)(i).</p>
<p> </p>
<p><strong>OR</strong></p>
<p><img src="images/Schermafbeelding_2017-08-17_om_09.56.01.png" alt="M17/5/MATSD/SP2/ENG/TZ2/05.e.ii/M">     <strong><em>(M1)(A1)</em>(ft)</strong></p>
<p> </p>
<p><strong>Note:</strong>     Award <strong><em>(M1) </em></strong>for a sketch with a straight line intercepted by appropriate curve, <strong><em>(A1)</em>(ft) </strong>for numerical answer in the range of 8.4 and 8.5.</p>
<p>Follow through from their part (e)(i).</p>
<p>\(t = 8.50{\text{ (years) }}(8.49564 \ldots )\)     <strong><em>(A1)</em>(ft)<em>(G3)</em></strong></p>
<p> </p>
<p><strong>Note:</strong>     Award only <strong><em>(A1) </em></strong>if 16.9912… is seen without working. If working is seen, award at most <strong><em>(M1)(A1)(A0)</em></strong>.</p>
<p> </p>
<p><strong><em>[3 marks]</em></strong></p>
<div class="question_part_label">e.ii.</div>
</div>
<div class="question" style="padding-left: 20px;">
<p>\(5000{\left( {1 + \frac{r}{{100}}} \right)^{12}} = 10000\)     <strong><em>(M1)</em></strong></p>
<p> </p>
<p><strong>Note: </strong>Award <strong><em>(M1) </em></strong>for correct substitution into compound interest formula with 10 000 seen. </p>
<p><strong>OR</strong></p>
<p>\(2 = {\left( {1 + \frac{r}{{100}}} \right)^{12}}\)     <strong><em>(M1)</em></strong></p>
<p> </p>
<p><strong>Note:</strong>     Award <strong><em>(M1) </em></strong>for correct substitution and simplification of compound interest formula, equating to 2.</p>
<p> </p>
<p>\(r = 5.95{\text{ (% ) }}(5.94630 \ldots )\)     <strong><em>(A1)(G2)</em></strong></p>
<p><strong><em>[2 marks]</em></strong></p>
<div class="question_part_label">f.</div>
</div>
<h2 style="margin-top: 1em">Examiners report</h2>
<div class="question" style="padding-left: 20px;">
[N/A]
<div class="question_part_label">a.</div>
</div>
<div class="question" style="padding-left: 20px;">
[N/A]
<div class="question_part_label">b.</div>
</div>
<div class="question" style="padding-left: 20px;">
[N/A]
<div class="question_part_label">c.</div>
</div>
<div class="question" style="padding-left: 20px;">
[N/A]
<div class="question_part_label">d.</div>
</div>
<div class="question" style="padding-left: 20px;">
[N/A]
<div class="question_part_label">e.i.</div>
</div>
<div class="question" style="padding-left: 20px;">
[N/A]
<div class="question_part_label">e.ii.</div>
</div>
<div class="question" style="padding-left: 20px;">
[N/A]
<div class="question_part_label">f.</div>
</div>
<br><hr><br><div class="specification">
<p>Consider the function \(f(x) = 0.3{x^3} + \frac{{10}}{x} + {2^{ - x}}\).</p>
</div>

<div class="specification">
<p>Consider a second function, \(g(x) = 2x - 3\).</p>
</div>

<div class="question" style="padding-left: 20px; padding-right: 20px;">
<p>Calculate \(f(1)\).</p>
<div class="marks">[2]</div>
<div class="question_part_label">a.</div>
</div>
<div class="question" style="padding-left: 20px; padding-right: 20px;">
<p>Sketch the graph of \(y = f(x)\) for \( - 7 \leqslant x \leqslant 4\) and \( - 30 \leqslant y \leqslant 30\).</p>
<div class="marks">[4]</div>
<div class="question_part_label">b.</div>
</div>
<div class="question" style="padding-left: 20px; padding-right: 20px;">
<p>Write down the equation of the vertical asymptote.</p>
<div class="marks">[2]</div>
<div class="question_part_label">c.</div>
</div>
<div class="question" style="padding-left: 20px; padding-right: 20px;">
<p>Write down the coordinates of the \(x\)-intercept.</p>
<div class="marks">[2]</div>
<div class="question_part_label">d.</div>
</div>
<div class="question" style="padding-left: 20px; padding-right: 20px;">
<p>Write down the possible values of \(x\) for which \(x &lt; 0\) and \(f’(x) &gt; 0\).</p>
<div class="marks">[2]</div>
<div class="question_part_label">e.</div>
</div>
<div class="question" style="padding-left: 20px; padding-right: 20px;">
<p>Find the solution of \(f(x) = g(x)\).</p>
<div class="marks">[2]</div>
<div class="question_part_label">f.</div>
</div>
<h2 style="margin-top: 1em">Markscheme</h2>
<div class="question" style="padding-left: 20px;">
<p>\(0.3{(1)^3} + \frac{{10}}{1} + {2^{ - 1}}\)     <strong><em>(M1)</em></strong></p>
<p> </p>
<p><strong>Note:</strong>     Award <strong><em>(M1) </em></strong>for correct substitution into function.</p>
<p> </p>
<p>\( = 10.8\)     <strong><em>(A1)(G2)</em></strong></p>
<p><strong><em>[2 marks]</em></strong></p>
<div class="question_part_label">a.</div>
</div>
<div class="question" style="padding-left: 20px;">
<p><img src="images/Schermafbeelding_2017-08-16_om_14.01.31.png" alt="M17/5/MATSD/SP2/ENG/TZ1/03.b/M">     <strong><em>(A1)(A1)(A1)(A1)</em></strong></p>
<p> </p>
<p><strong>Note:</strong>     Award <strong><em>(A1) </em></strong>for indication of correct window and labelled axes.</p>
<p>Award <strong><em>(A1) </em></strong>for correct shape and position for \(x &lt; 0\) (with the local maximum, local minimum and \(x\)-intercept in relative approximate location in \({{\text{3}}^{{\text{rd}}}}\) quadrant).</p>
<p>Award <strong><em>(A1) </em></strong>for correct shape and position for \(x &gt; 0\) (with the local minimum in relative approximate location in \({{\text{1}}^{{\text{st}}}}\) quadrant).</p>
<p>Award <strong><em>(A1) </em></strong>for smooth curve with indication of asymptote (graph should not touch \(y\)-axis and should not curve away from the \(y\)-axis). The asymptote is only assessed in this mark.</p>
<p> </p>
<p><strong><em>[4 marks]</em></strong></p>
<div class="question_part_label">b.</div>
</div>
<div class="question" style="padding-left: 20px;">
<p>\(x = 0\)     <strong><em>(A2)</em></strong></p>
<p> </p>
<p><strong>Note:</strong>     Award <strong><em>(A1) </em></strong>for “\(x = {\text{(a constant)}}\)” and <strong><em>(A1) </em></strong>for “\({\text{(a constant)}} = 0\)”.</p>
<p>The answer must be an equation.</p>
<p> </p>
<p><strong><em>[2 marks]</em></strong></p>
<div class="question_part_label">c.</div>
</div>
<div class="question" style="padding-left: 20px;">
<p>\(( - 6.18,{\text{ }}0){\text{ }}( - 6.17516 \ldots ,{\text{ }}0)\)     <strong><em>(A1)(A1)</em></strong></p>
<p> </p>
<p><strong>Note:</strong>     Award <strong><em>(A1) </em></strong>for each correct coordinate. Award <strong><em>(A0)(A1) </em></strong>if parentheses are missing.</p>
<p> </p>
<p><strong><em>[2 marks]</em></strong></p>
<div class="question_part_label">d.</div>
</div>
<div class="question" style="padding-left: 20px;">
<p>\( - 4.99 &lt; x &lt;  - 2.47{\text{ }}( - 4.98688 \ldots  &lt; x &lt;  - 2.46635 \ldots )\)     <strong><em>(A1)(A1)</em></strong></p>
<p> </p>
<p><strong>Note:</strong>     Award <strong><em>(A1) </em></strong>for both correct end points, <strong><em>(A1) </em></strong>for strict inequalities used with 2 endpoints.</p>
<p> </p>
<p><strong><em>[2 marks]</em></strong></p>
<div class="question_part_label">e.</div>
</div>
<div class="question" style="padding-left: 20px;">
<p>\(0.3{x^3} + \frac{{10}}{x} + {2^{ - x}} = 2x - 3\)     <strong><em>(M1)</em></strong></p>
<p> </p>
<p><strong>Note:</strong>     Award <strong><em>(M1) </em></strong>for equating the expressions for \(f\) and \(g\) or for the line \(y = 2x - 3\) sketched (positive gradient, negative \(y\)-intercept) on their graph from part (a).</p>
<p> </p>
<p>\((x = ){\text{ }} - 1.34{\text{ }}( - 1.33650 \ldots )\)     <strong><em>(A1)(G2)</em></strong></p>
<p> </p>
<p><strong>Note:</strong>     Award a maximum of <strong><em>(M1)(A0) </em></strong>or <strong><em>(G1) </em></strong>for coordinate pair seen as final answer.</p>
<p> </p>
<p><strong><em>[2 marks]</em></strong></p>
<div class="question_part_label">f.</div>
</div>
<h2 style="margin-top: 1em">Examiners report</h2>
<div class="question" style="padding-left: 20px;">
[N/A]
<div class="question_part_label">a.</div>
</div>
<div class="question" style="padding-left: 20px;">
[N/A]
<div class="question_part_label">b.</div>
</div>
<div class="question" style="padding-left: 20px;">
[N/A]
<div class="question_part_label">c.</div>
</div>
<div class="question" style="padding-left: 20px;">
[N/A]
<div class="question_part_label">d.</div>
</div>
<div class="question" style="padding-left: 20px;">
[N/A]
<div class="question_part_label">e.</div>
</div>
<div class="question" style="padding-left: 20px;">
[N/A]
<div class="question_part_label">f.</div>
</div>
<br><hr><br><div class="specification">
<p><span style="font-family: times new roman,times; font-size: medium;">Consider the functions \(f(x) = \frac{{2x + 3}}{{x + 4}}\) and \(g(x) = x + 0.5\) .</span></p>
</div>

<div class="question" style="padding-left: 20px; padding-right: 20px;">
<p><span>Sketch the graph of the function \(f(x)\), for \( - 10 \leqslant x \leqslant 10\) . Indicating clearly the axis intercepts and any asymptotes.</span></p>
<div class="marks">[6]</div>
<div class="question_part_label">a.</div>
</div>
<div class="question" style="padding-left: 20px; padding-right: 20px;">
<p><span>Write down the equation of the vertical asymptote.</span></p>
<div class="marks">[2]</div>
<div class="question_part_label">b.</div>
</div>
<div class="question" style="padding-left: 20px; padding-right: 20px;">
<p><span>On the same diagram as part (a) sketch the graph of \(g(x) = x + 0.5\) .</span></p>
<div class="marks">[2]</div>
<div class="question_part_label">c.</div>
</div>
<div class="question" style="padding-left: 20px; padding-right: 20px;">
<p><span>Using your graphical display calculator write down the coordinates of <strong>one</strong> of the points of intersection on the graphs of \(f\) and \(g\), <strong>giving your answer correct to five decimal places</strong>.</span></p>
<div class="marks">[3]</div>
<div class="question_part_label">d.</div>
</div>
<div class="question" style="padding-left: 20px; padding-right: 20px;">
<p><span>Write down the gradient of the line \(g(x) = x + 0.5\) .</span></p>
<div class="marks">[1]</div>
<div class="question_part_label">e.</div>
</div>
<div class="question" style="padding-left: 20px; padding-right: 20px;">
<p><span>The line \(L\) passes through the point with coordinates \(( - 2{\text{, }} - 3)\) and is perpendicular to the line \(g(x)\) . Find the equation of \(L\).</span></p>
<div class="marks">[3]</div>
<div class="question_part_label">f.</div>
</div>
<h2 style="margin-top: 1em">Markscheme</h2>
<div class="question" style="padding-left: 20px;">
<p><span><img src="data:image/png;base64,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" alt><span>     <em><strong>(A6)</strong></em></span></span></p>
<p><span><strong>Notes:</strong> <em><strong>(A1)</strong></em> for labels and some idea of scale.</span><br><span><em><strong>(A1)</strong></em> for \(x\)-intercept seen, <em><strong>(A1)</strong></em> for \(y\)-intercept seen in roughly the correct places (coordinates not required).</span><br><span><em><strong>(A1)</strong></em> for vertical asymptote seen, <em><strong>(A1)</strong></em> for horizontal asymptote seen in roughly the correct places (equations of the lines not required).</span><br><span><em><strong>(A1)</strong></em> for correct general shape.</span></p>
<p><span><em><strong>[6 marks]</strong></em><br></span></p>
<div class="question_part_label">a.</div>
</div>
<div class="question" style="padding-left: 20px;">
<p><span>\(x = - 4\)     <em><strong>(A1)(A1)</strong></em><strong>(ft)</strong></span></p>
<p><span><strong>Note: <em>(A1)</em></strong> for \(x =\), <strong><em>(A1)</em>(ft)</strong> for \( - 4\).</span></p>
<p><span><em><strong>[2 marks]</strong></em><br></span></p>
<div class="question_part_label">b.</div>
</div>
<div class="question" style="padding-left: 20px;">
<p><span><img src="data:image/png;base64,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" alt><span>     <em><strong>(A1)(A1)</strong></em></span></span></p>
<p><span><span><strong>Note:</strong> <em><strong>(A1)</strong></em> for correct axis intercepts, <em><strong>(A1)</strong></em> for straight line</span><br></span></p>
<p><span><span><em><strong>[2 marks]</strong></em><br></span></span></p>
<div class="question_part_label">c.</div>
</div>
<div class="question" style="padding-left: 20px;">
<p><span>\(( - 2.85078{\text{, }} - 2.35078)\) OR \((0.35078{\text{, }}0.85078)\)     <em><strong>(G1)(G1)(A1)</strong></em><strong>(ft)</strong><br></span></p>
<p><br><span><strong>Notes: <em>(A1)</em></strong> for \(x\)-coordinate, <em><strong>(A1)</strong></em> for \(y\)-coordinate, <strong><em>(A1)</em>(ft)</strong> for correct accuracy. Brackets required. If brackets not used award <strong><em>(G1)(G0)(A1)</em>(ft)</strong>.<br>Accept \(x = - 2.85078\), \(y = - 2.35078\) or \(x = 0.35078\), \(y = 0.85078\).</span></p>
<p><span><em><strong>[3 marks]</strong></em><br></span></p>
<div class="question_part_label">d.</div>
</div>
<div class="question" style="padding-left: 20px;">
<p><span>\({\text{gradient}} = 1\)     <em><strong>(A1)</strong></em></span></p>
<p><span><em><strong>[1 mark]<br></strong></em></span></p>
<div class="question_part_label">e.</div>
</div>
<div class="question" style="padding-left: 20px;">
<p><span>\({\text{gradient of perpendicular}} = - 1\)     <em><strong>(A1)</strong></em><strong>(ft)</strong></span></p>
<p><span><em>(can be implied in the next step)</em></span></p>
<p><span>\(y = mx + c\)</span></p>
<p><span>\( - 3 = - 1 \times - 2 + c\)     <em><strong>(M1)</strong></em></span></p>
<p><span>\(c = - 5\)</span></p>
<p><span>\(y = - x - 5\)     <em><strong>(A1)</strong></em><strong>(ft)<em>(G2)</em></strong></span></p>
<p><span><strong>OR</strong></span></p>
<p><span>\(y + 3 = - (x + 2)\)     <em><strong>(M1)</strong></em><em><strong>(A1)</strong></em><strong>(ft)<em>(G2)</em></strong></span></p>
<p><br><span><strong>Note: </strong>Award <em><strong>(G2)</strong></em> for correct answer with no working at all but <em><strong>(A1)(G1)</strong></em> if the gradient is mentioned as \( - 1\) then correct answer with no further working.</span></p>
<p><span><em><strong>[3 marks]</strong></em><br></span></p>
<div class="question_part_label">f.</div>
</div>
<h2 style="margin-top: 1em">Examiners report</h2>
<div class="question" style="padding-left: 20px;">
<p><span style="font-family: times new roman,times; font-size: medium;">This was not very well done. The graph was often correct but was so small that it was difficult to check if axes intercepts were correct or not. Often the vertical asymptote looked as if it were joined to the rest of the graph. Very few of the candidates put a scale and/or labels on their axes.</span></p>
<div class="question_part_label">a.</div>
</div>
<div class="question" style="padding-left: 20px;">
<p><span style="font-family: times new roman,times; font-size: medium;">Reasonably well done. Some put \(y = - 4\) while others omitted the minus sign.</span></p>
<div class="question_part_label">b.</div>
</div>
<div class="question" style="padding-left: 20px;">
<p><span style="font-family: times new roman,times; font-size: medium;">Fairly well done &ndash; but once again too small to check the axes intercepts properly. Also, many candidates did not appear to have a ruler to draw the straight line.</span></p>
<div class="question_part_label">c.</div>
</div>
<div class="question" style="padding-left: 20px;">
<p><span style="font-family: times new roman,times; font-size: medium;">Well done.</span></p>
<div class="question_part_label">d.</div>
</div>
<div class="question" style="padding-left: 20px;">
<p><span style="font-family: times new roman,times; font-size: medium;">Most could find the gradient of the line.</span></p>
<div class="question_part_label">e.</div>
</div>
<div class="question" style="padding-left: 20px;">
<p><span style="font-family: times new roman,times; font-size: medium;">Many forgot to find the gradient of the perpendicular line. Others had problems with the equation of a line in general.</span></p>
<div class="question_part_label">f.</div>
</div>
<br><hr><br><div class="specification">
<p>Consider the function \(g(x) = {x^3} + k{x^2} - 15x + 5\).</p>
</div>

<div class="specification">
<p>The tangent to the graph of \(y = g(x)\) at \(x = 2\) is parallel to the line \(y = 21x + 7\).</p>
</div>

<div class="question" style="padding-left: 20px; padding-right: 20px;">
<p>Find \(g'(x)\).</p>
<div class="marks">[3]</div>
<div class="question_part_label">a.</div>
</div>
<div class="question" style="padding-left: 20px; padding-right: 20px;">
<p>Show that \(k = 6\).</p>
<div class="marks">[2]</div>
<div class="question_part_label">b.i.</div>
</div>
<div class="question" style="padding-left: 20px; padding-right: 20px;">
<p>Find the equation of the tangent to the graph of \(y = g(x)\) at \(x = 2\). Give your answer in the form \(y = mx + c\).</p>
<div class="marks">[3]</div>
<div class="question_part_label">b.ii.</div>
</div>
<div class="question" style="padding-left: 20px; padding-right: 20px;">
<p>Use your answer to part (a) and the value of \(k\), to find the \(x\)-coordinates of the stationary points of the graph of \(y = g(x)\).</p>
<div class="marks">[3]</div>
<div class="question_part_label">c.</div>
</div>
<div class="question" style="padding-left: 20px; padding-right: 20px;">
<p>Find \(g’( - 1)\).</p>
<div class="marks">[2]</div>
<div class="question_part_label">d.i.</div>
</div>
<div class="question" style="padding-left: 20px; padding-right: 20px;">
<p>Hence justify that \(g\) is decreasing at \(x =  - 1\).</p>
<div class="marks">[1]</div>
<div class="question_part_label">d.ii.</div>
</div>
<div class="question" style="padding-left: 20px; padding-right: 20px;">
<p>Find the \(y\)-coordinate of the local minimum.</p>
<div class="marks">[2]</div>
<div class="question_part_label">e.</div>
</div>
<h2 style="margin-top: 1em">Markscheme</h2>
<div class="question" style="padding-left: 20px;">
<p>\(3{x^2} + 2kx - 15\)     <strong><em>(A1)(A1)(A1)</em></strong></p>
<p> </p>
<p><strong>Note:</strong>     Award <strong><em>(A1) </em></strong>for \(3{x^2}\), <strong><em>(A1) </em></strong>for \(2kx\) and <strong><em>(A1) </em></strong>for \( - 15\). Award at most <strong><em>(A1)(A1)(A0) </em></strong>if additional terms are seen.</p>
<p> </p>
<p><strong><em>[3 marks]</em></strong></p>
<div class="question_part_label">a.</div>
</div>
<div class="question" style="padding-left: 20px;">
<p>\(21 = 3{(2)^2} + 2k(2) - 15\)     <strong><em>(M1)(M1)</em></strong></p>
<p> </p>
<p><strong>Note:</strong>     Award <strong><em>(M1) </em></strong>for equating their derivative to 21. Award <strong><em>(M1) </em></strong>for substituting 2 into their derivative. The second <strong><em>(M1) </em></strong>should only be awarded if correct working leads to the final answer of \(k = 6\).</p>
<p>Substituting in the known value, \(k = 6\), invalidates the process; award <strong><em>(M0)(M0)</em></strong>.</p>
<p> </p>
<p>\(k = 6\)     <strong><em>(AG)</em></strong></p>
<p><strong><em>[2 marks]</em></strong></p>
<div class="question_part_label">b.i.</div>
</div>
<div class="question" style="padding-left: 20px;">
<p>\(g(2) = {(2)^3} + (6){(2)^2} - 15(2) + 5{\text{ }}( = 7)\)     <strong><em>(M1)</em></strong></p>
<p> </p>
<p><strong>Note:</strong>     Award <strong><em>(M1) </em></strong>for substituting 2 into \(g\).</p>
<p> </p>
<p>\(7 = 21(2) + c\)     <strong><em>(M1)</em></strong></p>
<p> </p>
<p><strong>Note:</strong>     Award <strong><em>(M1) </em></strong>for correct substitution of 21, 2 and their 7 into gradient intercept form.</p>
<p> </p>
<p><strong>OR</strong></p>
<p> </p>
<p>\(y - 7 = 21(x - 2)\)     <strong><em>(M1)</em></strong></p>
<p> </p>
<p><strong>Note:</strong>     Award <strong><em>(M1) </em></strong>for correct substitution of 21, 2 and their 7 into gradient point form.</p>
<p> </p>
<p>\(y = 21x - 35\)     <strong><em>(A1)</em></strong>     <strong><em>(G2)</em></strong></p>
<p><strong><em>[3 marks]</em></strong></p>
<div class="question_part_label">b.ii.</div>
</div>
<div class="question" style="padding-left: 20px;">
<p>\(3{x^2} + 12x - 15 = 0\) (or equivalent)     <strong><em>(M1)</em></strong></p>
<p> </p>
<p><strong>Note:</strong>     Award <strong><em>(M1) </em></strong>for equating their part (a) (with \(k = 6\) substituted) to zero.</p>
<p> </p>
<p>\(x =  - 5,{\text{ }}x = 1\)     <strong><em>(A1)</em>(ft)<em>(A1)</em>(ft)</strong></p>
<p> </p>
<p><strong>Note:</strong>     Follow through from part (a).</p>
<p> </p>
<p><strong><em>[3 marks]</em></strong></p>
<div class="question_part_label">c.</div>
</div>
<div class="question" style="padding-left: 20px;">
<p>\(3{( - 1)^2} + 12( - 1) - 15\)     <strong><em>(M1)</em></strong></p>
<p> </p>
<p><strong>Note:</strong>     Award <strong><em>(M1) </em></strong>for substituting \( - 1\) into their derivative, with \(k = 6\) substituted. Follow through from part (a).</p>
<p> </p>
<p>\( =  - 24\)     <strong><em>(A1)</em>(ft)</strong>     <strong><em>(G2)</em></strong></p>
<p><strong><em>[2 marks]</em></strong></p>
<div class="question_part_label">d.i.</div>
</div>
<div class="question" style="padding-left: 20px;">
<p>\(g’( - 1) &lt; 0\) (therefore \(g\) is decreasing when \(x =  - 1\))     <strong><em>(R1)</em></strong></p>
<p><strong><em>[1 marks]</em></strong></p>
<div class="question_part_label">d.ii.</div>
</div>
<div class="question" style="padding-left: 20px;">
<p>\(g(1) = {(1)^3} + (6){(1)^2} - 15(1) + 5\)     <strong><em>(M1)</em></strong></p>
<p> </p>
<p><strong>Note:</strong>     Award <strong><em>(M1) </em></strong>for correctly substituting 6 and their 1 into \(g\).</p>
<p> </p>
<p>\( =  - 3\)     <strong><em>(A1)</em>(ft)</strong>     <strong><em>(G2)</em></strong></p>
<p> </p>
<p><strong>Note:</strong>     Award, at most, (<strong><em>M1)(A0) </em></strong>or <strong><em>(G1) </em></strong>if answer is given as a coordinate pair. Follow through from part (c).</p>
<p> </p>
<p><strong><em>[2 marks]</em></strong></p>
<div class="question_part_label">e.</div>
</div>
<h2 style="margin-top: 1em">Examiners report</h2>
<div class="question" style="padding-left: 20px;">
[N/A]
<div class="question_part_label">a.</div>
</div>
<div class="question" style="padding-left: 20px;">
[N/A]
<div class="question_part_label">b.i.</div>
</div>
<div class="question" style="padding-left: 20px;">
[N/A]
<div class="question_part_label">b.ii.</div>
</div>
<div class="question" style="padding-left: 20px;">
[N/A]
<div class="question_part_label">c.</div>
</div>
<div class="question" style="padding-left: 20px;">
[N/A]
<div class="question_part_label">d.i.</div>
</div>
<div class="question" style="padding-left: 20px;">
[N/A]
<div class="question_part_label">d.ii.</div>
</div>
<div class="question" style="padding-left: 20px;">
[N/A]
<div class="question_part_label">e.</div>
</div>
<br><hr><br><div class="specification">
<p class="p1">The line \({L_1}\) has equation \(2y - x - 7 = 0\) and is shown on the diagram.</p>
<p class="p1" style="text-align: center;"><img src="images/Schermafbeelding_2017-03-07_om_07.59.34.png" alt="N16/5/MATSD/SP2/ENG/TZ0/03"></p>
<p class="p1" style="text-align: left;">The point <span class="s1">A </span>has coordinates \((1,{\text{ }}4)\).</p>
</div>

<div class="specification">
<p class="p1">The point <span class="s1">C </span>has coordinates \((5,{\text{ }}12)\). <span class="s1">M </span>is the midpoint of <span class="s1">AC</span>.</p>
</div>

<div class="specification">
<p class="p1">The straight line, \({L_2}\), is perpendicular to <span class="s1">AC </span>and passes through <span class="s1">M</span>.</p>
</div>

<div class="specification">
<p class="p1">The point <span class="s1">D </span>is the intersection of \({L_1}\) and \({L_2}\).</p>
</div>

<div class="specification">
<p class="p1">The length of <span class="s1">MD </span>is \(\frac{{\sqrt {45} }}{2}\).</p>
</div>

<div class="specification">
<p class="p1">The point <span class="s1">B </span>is such that <span class="s1">ABCD </span>is a rhombus.</p>
</div>

<div class="question" style="padding-left: 20px; padding-right: 20px;">
<p class="p1"><span class="s1">Show that </span><span class="s2">A </span>lies on \({L_1}\).</p>
<div class="marks">[2]</div>
<div class="question_part_label">a.</div>
</div>
<div class="question" style="padding-left: 20px; padding-right: 20px;">
<p class="p1">Find the coordinates of <span class="s1">M</span><span class="s2">.</span></p>
<div class="marks">[2]</div>
<div class="question_part_label">b.</div>
</div>
<div class="question" style="padding-left: 20px; padding-right: 20px;">
<p class="p1">Find the length of <span class="s1">AC</span><span class="s2">.</span></p>
<div class="marks">[2]</div>
<div class="question_part_label">c.</div>
</div>
<div class="question" style="padding-left: 20px; padding-right: 20px;">
<p class="p1">Show that the equation of \({L_2}\) <span class="s1">is \(2y + x - 19 = 0\).</span></p>
<div class="marks">[5]</div>
<div class="question_part_label">d.</div>
</div>
<div class="question" style="padding-left: 20px; padding-right: 20px;">
<p class="p1">Find the coordinates of <span class="s1">D</span><span class="s2">.</span></p>
<div class="marks">[2]</div>
<div class="question_part_label">e.</div>
</div>
<div class="question" style="padding-left: 20px; padding-right: 20px;">
<p class="p1"><span class="s1">Write down the length of </span><span class="s2">MD </span>correct to five significant figures.</p>
<div class="marks">[1]</div>
<div class="question_part_label">f.</div>
</div>
<div class="question" style="padding-left: 20px; padding-right: 20px;">
<p class="p1">Find the area of <span class="s1">ABCD</span><span class="s2">.</span></p>
<div class="marks">[3]</div>
<div class="question_part_label">g.</div>
</div>
<h2 style="margin-top: 1em">Markscheme</h2>
<div class="question" style="padding-left: 20px;">
<p class="p1">\(2 \times 4 - 1 - 7 = 0\) (or equivalent) <span class="Apple-converted-space">    </span><strong><em>(R1)</em></strong></p>
<p class="p2"> </p>
<p class="p1"><strong>Note: <span class="Apple-converted-space">    </span></strong>For <strong><em>(R1) </em></strong>accept substitution of \(x = 1\) or \(y = 4\) into the equation followed by a confirmation that \(y = 4\) or \(x = 1\).</p>
<p class="p2"> </p>
<p class="p1">(since the point satisfies the equation of the line,) <span class="s1">A </span>lies on \({L_1}\) <span class="Apple-converted-space">    </span><strong><em>(A1)</em></strong></p>
<p class="p2"> </p>
<p class="p1"><strong>Note: <span class="Apple-converted-space">    </span></strong>Do not award <strong><em>(A1)(R0)</em></strong>.</p>
<p class="p2"> </p>
<p class="p1"><strong><em>[2 marks]</em></strong></p>
<div class="question_part_label">a.</div>
</div>
<div class="question" style="padding-left: 20px;">
<p class="p1">\(\frac{{1 + 5}}{2}\) <strong>OR</strong> \(\frac{{4 + 12}}{2}\) seen <span class="Apple-converted-space">    </span><strong><em>(M1)</em></strong></p>
<p class="p2"> </p>
<p class="p1"><strong>Note: <span class="Apple-converted-space">    </span></strong>Award <strong><em>(M1) </em></strong>for at least one correct substitution into the midpoint formula.</p>
<p class="p2"> </p>
<p class="p1"><span class="Apple-converted-space">\((3,{\text{ }}8)\)    </span><strong><em>(A1)(G2)</em></strong></p>
<p class="p2"> </p>
<p class="p1"><strong>Notes: <span class="Apple-converted-space">    </span></strong>Accept \(x = 3,{\text{ }}y = 8\).</p>
<p class="p1">Award <strong><em>(M1)(A0) </em></strong>for \(\left( {\frac{{1 + 5}}{2},{\text{ }}\frac{{4 + 12}}{2}} \right)\).</p>
<p class="p1">Award <strong><em>(G1) </em></strong>for each correct coordinate seen without working.</p>
<p class="p2"> </p>
<p class="p1"><strong><em>[2 marks]</em></strong></p>
<div class="question_part_label">b.</div>
</div>
<div class="question" style="padding-left: 20px;">
<p class="p1"><span class="Apple-converted-space">\(\sqrt {{{(5 - 1)}^2} + {{(12 - 4)}^2}} \)    </span><strong><em>(M1)</em></strong></p>
<p class="p2"> </p>
<p class="p1"><strong>Note: <span class="Apple-converted-space">    </span></strong>Award <strong><em>(M1) </em></strong>for a correct substitution into distance between two points formula.</p>
<p class="p2"> </p>
<p class="p1"><span class="Apple-converted-space">\( = 8.94{\text{ }}\left( {4\sqrt 5 ,{\text{ }}\sqrt {80} ,{\text{ }}8.94427 \ldots } \right)\)    </span><strong><em>(A1)(G2)</em></strong></p>
<p class="p1"><strong><em>[2 marks]</em></strong></p>
<div class="question_part_label">c.</div>
</div>
<div class="question" style="padding-left: 20px;">
<p class="p1">gradient of \({\text{AC}} = \frac{{12 - 4}}{{5 - 1}}\) <span class="Apple-converted-space">    </span><strong><em>(M1)</em></strong></p>
<p class="p2"> </p>
<p class="p1"><strong>Note: <span class="Apple-converted-space">    </span></strong>Award <strong><em>(M1) </em></strong>for correct substitution into gradient formula.</p>
<p class="p2"> </p>
<p class="p1"><span class="Apple-converted-space">\( = 2\)    </span><strong><em>(A1)</em></strong></p>
<p class="p2"> </p>
<p class="p1"><strong>Note: <span class="Apple-converted-space">    </span></strong>Award <strong><em>(M1)(A1) </em></strong>for gradient of \({\text{AC}} = 2\) with or without working</p>
<p class="p2"> </p>
<p class="p1">gradient of the normal \( =  - \frac{1}{2}\) <span class="Apple-converted-space">    </span><strong><em>(M1)</em></strong></p>
<p class="p2"> </p>
<p class="p1"><strong>Note: <span class="Apple-converted-space">    </span></strong>Award <strong><em>(M1) </em></strong>for the negative reciprocal of their gradient of <span class="s1">AC</span>.</p>
<p class="p2"> </p>
<p class="p1">\(y - 8 =  - \frac{1}{2}(x - 3)\) <strong>OR</strong> \(8 =  - \frac{1}{2}(3) + c\) <span class="Apple-converted-space">    </span><strong><em>(M1)</em></strong></p>
<p class="p2"> </p>
<p class="p1"><strong>Note: <span class="Apple-converted-space">    </span></strong>Award <strong><em>(M1) </em></strong>for substitution of their point and gradient into straight line formula. This <strong><em>(M1) </em></strong>can <strong>only </strong>be awarded where \( - \frac{1}{2}\) (gradient) is correctly determined as the gradient of the normal to <span class="s1">AC</span>.</p>
<p class="p2"> </p>
<p class="p1">\(2y - 16 =  - (x - 3)\) <strong>OR</strong> \( - 2y + 16 = x - 3\) <strong>OR</strong> \(2y =  - x + 19\) <span class="Apple-converted-space">    </span><strong><em>(A1)</em></strong></p>
<p class="p2"> </p>
<p class="p1"><strong>Note: <span class="Apple-converted-space">    </span></strong>Award <strong><em>(A1) </em></strong>for correctly removing fractions, <strong>but only </strong>if their equation is equivalent to the given equation.</p>
<p class="p2"> </p>
<p class="p1"><span class="Apple-converted-space">\(2y + x - 19 = 0\)    </span><strong><em>(AG)</em></strong></p>
<p class="p2"> </p>
<p class="p1"><strong>Note: <span class="Apple-converted-space">    </span></strong>The conclusion \(2y + x - 19 = 0\) must be seen for the <strong><em>(A1) </em></strong>to be awarded.</p>
<p class="p1">Where the candidate has <strong>shown </strong>the gradient of the normal to \({\text{AC}} =  - 0.5\), award <strong><em>(M1) </em></strong>for \(2(8) + 3 - 19 = 0\) and <strong><em>(A1) </em></strong>for (therefore) \(2y + x - 19 = 0\).</p>
<p class="p1">Simply substituting \((3,{\text{ }}8)\) into the equation of \({L_2}\) with no other prior working, earns no marks.</p>
<p class="p2"> </p>
<p class="p1"><strong><em>[5 marks]</em></strong></p>
<div class="question_part_label">d.</div>
</div>
<div class="question" style="padding-left: 20px;">
<p class="p1"><span class="Apple-converted-space">\((6,{\text{ }}6.5)\)    </span><strong><em>(A1)(A1)(G2)</em></strong></p>
<p class="p2"> </p>
<p class="p1"><strong>Note: <span class="Apple-converted-space">    </span></strong>Award <strong><em>(A1) </em></strong>for <span class="s1">6</span>, <strong><em>(A1) </em></strong>for <span class="s1">6.5</span>. Award a maximum of <strong><em>(A1)(A0) </em></strong>if answers are not given as a coordinate pair. Accept \(x = 6,{\text{ }}y = 6.5\).</p>
<p class="p1">Award <strong><em>(M1)(A0) </em></strong>for an attempt to solve the two simultaneous equations \(2y - x - 7 = 0\) and \(2y + x - 19 = 0\) algebraically, leading to at least one incorrect or missing coordinate.</p>
<p class="p2"> </p>
<p class="p1"><strong><em>[2 marks]</em></strong></p>
<div class="question_part_label">e.</div>
</div>
<div class="question" style="padding-left: 20px;">
<p class="p1">3.3541 <span class="Apple-converted-space">    </span><span class="s1"><strong><em>(A1)</em></strong></span></p>
<p class="p2"> </p>
<p class="p3"><strong>Note: <span class="Apple-converted-space">    </span></strong>Answer <span>must</span> be to <span class="s2">5 </span>significant figures.</p>
<p class="p2"> </p>
<p class="p3"><strong><em>[1 mark]</em></strong></p>
<div class="question_part_label">f.</div>
</div>
<div class="question" style="padding-left: 20px;">
<p class="p1"><span class="s1">\(2 \times \frac{1}{2} \times \sqrt {80}  \times \frac{{\sqrt {45} }}{2}\) <span class="Apple-converted-space">    </span></span><strong><em>(M1)(M1)</em></strong></p>
<p class="p2"> </p>
<p class="p1"><strong>Notes: <span class="Apple-converted-space">    </span></strong>Award <strong><em>(M1) </em></strong>for correct substitution into area of triangle formula.</p>
<p class="p1">If their triangle is a quarter of the rhombus then award <strong><em>(M1) </em></strong>for multiplying their triangle by <span class="s2">4</span>.</p>
<p class="p1">If their triangle is a half of the rhombus then award <strong><em>(M1) </em></strong>for multiplying their triangle by <span class="s2">2</span>.</p>
<p class="p2"> </p>
<p class="p1"><strong>OR</strong></p>
<p class="p1"><span class="Apple-converted-space">\(\frac{1}{2} \times \sqrt {80}  \times \sqrt {45} \)    </span><strong><em>(M1)(M1)</em></strong></p>
<p class="p2"> </p>
<p class="p1"><strong>Notes: <span class="Apple-converted-space">    </span></strong>Award <strong><em>(M1) </em></strong>for doubling <span class="s2">MD </span>to get the diagonal <span class="s2">BD</span>, <strong><em>(M1) </em></strong>for correct substitution into the area of a rhombus formula.</p>
<p class="p1">Award <strong><em>(M1)(M1) </em></strong>for \(\sqrt {80}  \times \) their (f).</p>
<p class="p2"> </p>
<p class="p1"><span class="Apple-converted-space">\( = 30\)    </span><strong><em>(A1)</em>(ft)<em>(G3)</em></strong></p>
<p class="p2"> </p>
<p class="p1"><strong>Notes: <span class="Apple-converted-space">    </span></strong>Follow through from parts (c) and (f).</p>
<p class="p1">\(8.94 \times 3.3541 = 29.9856 \ldots \)</p>
<p class="p2"> </p>
<p class="p1"><strong><em>[3 marks]</em></strong></p>
<div class="question_part_label">g.</div>
</div>
<h2 style="margin-top: 1em">Examiners report</h2>
<div class="question" style="padding-left: 20px;">
[N/A]
<div class="question_part_label">a.</div>
</div>
<div class="question" style="padding-left: 20px;">
[N/A]
<div class="question_part_label">b.</div>
</div>
<div class="question" style="padding-left: 20px;">
[N/A]
<div class="question_part_label">c.</div>
</div>
<div class="question" style="padding-left: 20px;">
[N/A]
<div class="question_part_label">d.</div>
</div>
<div class="question" style="padding-left: 20px;">
[N/A]
<div class="question_part_label">e.</div>
</div>
<div class="question" style="padding-left: 20px;">
[N/A]
<div class="question_part_label">f.</div>
</div>
<div class="question" style="padding-left: 20px;">
[N/A]
<div class="question_part_label">g.</div>
</div>
<br><hr><br><div class="specification">
<p><span style="font-family: times new roman,times; font-size: medium;">The function \(f(x)\) is defined by \(f(x) = 1.5x + 4 + \frac{6}{x}{\text{, }}x \ne 0\) .</span></p>
</div>

<div class="question" style="padding-left: 20px; padding-right: 20px;">
<p><span>Write down the equation of the vertical asymptote.</span></p>
<div class="marks">[2]</div>
<div class="question_part_label">a.</div>
</div>
<div class="question" style="padding-left: 20px; padding-right: 20px;">
<p><span>Find \(f'(x)\) .</span></p>
<div class="marks">[3]</div>
<div class="question_part_label">b.</div>
</div>
<div class="question" style="padding-left: 20px; padding-right: 20px;">
<p><span>Find the gradient of the graph of the function at \(x = - 1\).</span></p>
<div class="marks">[2]</div>
<div class="question_part_label">c.</div>
</div>
<div class="question" style="padding-left: 20px; padding-right: 20px;">
<p><span>Using your answer to part (c), decide whether the function \(f(x)\) is increasing or decreasing at \(x = - 1\). Justify your answer.</span></p>
<div class="marks">[2]</div>
<div class="question_part_label">d.</div>
</div>
<div class="question" style="padding-left: 20px; padding-right: 20px;">
<p><span>Sketch the graph of \(f(x)\) for \( - 10 \leqslant x \leqslant 10\) and \( - 20 \leqslant y \leqslant 20\) .</span></p>
<div class="marks">[4]</div>
<div class="question_part_label">e.</div>
</div>
<div class="question" style="padding-left: 20px; padding-right: 20px;">
<p><span>\({{\text{P}}_1}\) is the local maximum point and \({{\text{P}}_2}\) is the local minimum point on the graph of \(f(x)\) .</span></p>
<p><span>Using your graphic display calculator, write down the coordinates of</span></p>
<p><span>(i)     \({{\text{P}}_1}\) ;</span></p>
<p><span>(ii)    \({{\text{P}}_2}\) .</span></p>
<div class="marks">[4]</div>
<div class="question_part_label">f.</div>
</div>
<div class="question" style="padding-left: 20px; padding-right: 20px;">
<p><span>Using your sketch from (e), determine the range of the function \(f(x)\) for \( - 10 \leqslant x \leqslant 10\) .</span></p>
<div class="marks">[3]</div>
<div class="question_part_label">g.</div>
</div>
<h2 style="margin-top: 1em">Markscheme</h2>
<div class="question" style="padding-left: 20px;">
<p><span>\(x = 0\)     <em><strong>(A1)(A1)</strong></em></span></p>
<p><span><strong>Note:</strong> Award <em><strong>(A1)</strong></em> for \(x = {\text{constant}}\), <em><strong>(A1)</strong></em> for \(0\).</span></p>
<p><em><strong><span>[2 marks]</span></strong></em></p>
<div class="question_part_label">a.</div>
</div>
<div class="question" style="padding-left: 20px;">
<p><span>\(f'(x) = 1.5 - \frac{6}{{{x^2}}}\)     <em><strong>(A1)(A1)(A1)</strong></em></span></p>
<p><span><strong>Notes:</strong> Award <em><strong>(A1)</strong></em> for \(1.5\), <em><strong>(A1)</strong></em> for \( - 6\), <em><strong>(A1)</strong></em> for \({x^{ - 2}}\) . Award <em><strong>(A1)(A1)(A0)</strong></em> at most if any other term present.</span></p>
<p><em><strong><span>[3 marks]</span></strong></em></p>
<div class="question_part_label">b.</div>
</div>
<div class="question" style="padding-left: 20px;">
<p><span>\(1.5 - \frac{6}{{( - 1)}}\)     <em><strong>(M1)</strong></em></span></p>
<p><span>\( =  - 4.5\)     <strong><em>(A1)</em>(ft)<em>(G2)</em></strong></span></p>
<p><span><strong>Note:</strong> Follow through from their derivative function.</span></p>
<p><em><strong><span>[2 marks]</span></strong></em></p>
<div class="question_part_label">c.</div>
</div>
<div class="question" style="padding-left: 20px;">
<p><span>Decreasing, the derivative (gradient or slope) is negative (at \(x = - 1\))     <strong><em>(A1)(R1)</em>(ft)</strong></span></p>
<p><span><strong>Notes:</strong> Do not award <em><strong>(A1)(R0)</strong></em>. Follow through from their answer to part (c).</span></p>
<p><em><strong><span>[2 marks]</span></strong></em></p>
<div class="question_part_label">d.</div>
</div>
<div class="question" style="padding-left: 20px;">
<p><span><img src="data:image/png;base64,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" alt><span>     <em><strong>(A4)</strong></em></span></span><br><span><strong>Notes:</strong> Award <em><strong>(A1)</strong></em> for labels and some indication of scales and an appropriate window.</span></p>
<p><span>Award <em><strong>(A1)</strong></em> for correct shape of the two unconnected, and smooth branches.</span></p>
<p><span>Award <em><strong>(A1)</strong></em> for the maximum and minimum points in the approximately correct positions.</span></p>
<p><span>Award <em><strong>(A1)</strong></em> for correct asymptotic behaviour at \(x = 0\) .</span></p>
<p> </p>
<p><span><strong>Notes:</strong> Please be rigorous.</span></p>
<p><span>The axes need not be drawn with a ruler.</span></p>
<p><span>The branches must be smooth and single continuous lines that do not deviate from their proper direction.</span></p>
<p><span>The max and min points must be symmetrical about point \((0{\text{, }}4)\) .</span></p>
<p><span>The \(y\)-axis must be an asymptote for <strong>both</strong> branches.</span></p>
<p><em><strong><span>[4 marks]</span></strong></em></p>
<div class="question_part_label">e.</div>
</div>
<div class="question" style="padding-left: 20px;">
<p><span>(i)     \(( - 2{\text{, }} - 2)\) or \(x = - 2\), \(y = - 2\)     <em><strong>(G1)(G1)</strong></em></span></p>
<p> </p>
<p><span>(ii)    \((2{\text{, }}10)\) or \(x = 2\), \(y = 10\)     <em><strong>(G1)(G1)</strong></em></span></p>
<p> </p>
<p><span><em><strong>[4 marks]</strong></em></span></p>
<div class="question_part_label">f.</div>
</div>
<div class="question" style="padding-left: 20px;">
<p><span>\(\{  - 2 \geqslant y\} \) or \(\{ y \geqslant 10\} \)     <em><strong>(A1)(A1)</strong></em><strong>(ft)</strong><em><strong>(A1)</strong></em></span></p>
<p><span><strong>Notes:</strong> <strong><em>(A1)</em>(ft)</strong> for \(y &gt; 10\) or \(y \geqslant 10\) . <strong><em>(A1)</em>(ft)</strong> for \(y &lt; - 2\) or \(y \leqslant - 2\) . <em><strong>(A1)</strong></em> for weak (non-strict) inequalities used in <strong>both</strong> of the above. Follow through from their (e) and (f).</span></p>
<p><em><strong><span>[3 marks]</span></strong></em></p>
<div class="question_part_label">g.</div>
</div>
<h2 style="margin-top: 1em">Examiners report</h2>
<div class="question" style="padding-left: 20px;">
<p><span style="font-family: times new roman,times; font-size: medium;">Part a) was either answered well or poorly. </span></p>
<div class="question_part_label">a.</div>
</div>
<div class="question" style="padding-left: 20px;">
<p><span style="font-family: times new roman,times; font-size: medium;">Most candidates found the first term of the derivative in part b) correctly, but the rest of the terms were incorrect. </span></p>
<div class="question_part_label">b.</div>
</div>
<div class="question" style="padding-left: 20px;">
<p><span style="font-family: times new roman,times; font-size: medium;">The gradient in c) was for the most part correctly calculated, although some candidates substituted incorrectly in</span> <span style="font-family: times new roman,times; font-size: medium;">\(f(x)\) instead of in \(f'(x)\) .</span></p>
<div class="question_part_label">c.</div>
</div>
<div class="question" style="padding-left: 20px;">
<p><span style="font-family: times new roman,times; font-size: medium;">Part d) had mixed responses. </span></p>
<div class="question_part_label">d.</div>
</div>
<div class="question" style="padding-left: 20px;">
<p><span style="font-family: times new roman,times; font-size: medium;">Lack of labels of the axes, appropriate scale, window, incorrect maximum and minimum and incorrect asymptotic behaviour were the main problems with the sketches in e). </span></p>
<div class="question_part_label">e.</div>
</div>
<div class="question" style="padding-left: 20px;">
<p><span style="font-family: times new roman,times; font-size: medium;">Part f) was also either answered correctly or entirely incorrectly. Some candidates used the trace function on the GDC instead of the min and max functions, and thus acquired coordinates with unacceptable accuracy. Some were unclear that a point of local maximum may be positioned on the coordinate system &ldquo;below&rdquo; the point of local minimum, and exchanged the pairs of coordinates of those points in f(i) and f(ii). </span></p>
<div class="question_part_label">f.</div>
</div>
<div class="question" style="padding-left: 20px;">
<p><span style="font-family: times new roman,times; font-size: medium;">Very few candidates were able to identify the range of the function in (g) irrespective of whether or not they had the sketches drawn correctly.</span></p>
<div class="question_part_label">g.</div>
</div>
<br><hr><br><div class="specification">
<p>A pan, in which to cook a pizza, is in the shape of a cylinder. The pan has a diameter of 35 cm and a height of 0.5 cm.</p>
<p style="text-align: center;"><img src="images/Schermafbeelding_2017-08-16_om_11.14.51.png" alt="M17/5/MATSD/SP2/ENG/TZ1/04"></p>
</div>

<div class="specification">
<p>A chef had enough pizza dough to exactly fill the pan. The dough was in the shape of a sphere.</p>
</div>

<div class="specification">
<p>The pizza was cooked in a hot oven. Once taken out of the oven, the pizza was placed in a dining room.</p>
<p>The temperature, \(P\), of the pizza, in degrees Celsius, &deg;C, can be modelled by</p>
<p>\[P(t) = a{(2.06)^{ - t}} + 19,{\text{ }}t \geqslant 0\]</p>
<p>where \(a\) is a constant and \(t\) is the time, in minutes, since the pizza was taken out of the oven.</p>
<p>When the pizza was taken out of the oven its temperature was 230 &deg;C.</p>
</div>

<div class="specification">
<p>The pizza can be eaten once its temperature drops to 45 &deg;C.</p>
</div>

<div class="question" style="padding-left: 20px; padding-right: 20px;">
<p>Calculate the volume of this pan.</p>
<div class="marks">[3]</div>
<div class="question_part_label">a.</div>
</div>
<div class="question" style="padding-left: 20px; padding-right: 20px;">
<p>Find the radius of the sphere in cm, correct to one decimal place.</p>
<div class="marks">[4]</div>
<div class="question_part_label">b.</div>
</div>
<div class="question" style="padding-left: 20px; padding-right: 20px;">
<p>Find the value of \(a\).</p>
<div class="marks">[2]</div>
<div class="question_part_label">c.</div>
</div>
<div class="question" style="padding-left: 20px; padding-right: 20px;">
<p>Find the temperature that the pizza will be 5 minutes after it is taken out of the oven.</p>
<div class="marks">[2]</div>
<div class="question_part_label">d.</div>
</div>
<div class="question" style="padding-left: 20px; padding-right: 20px;">
<p>Calculate, to the nearest second, the time since the pizza was taken out of the oven until it can be eaten.</p>
<div class="marks">[3]</div>
<div class="question_part_label">e.</div>
</div>
<div class="question" style="padding-left: 20px; padding-right: 20px;">
<p>In the context of this model, state what the value of 19 represents.</p>
<div class="marks">[1]</div>
<div class="question_part_label">f.</div>
</div>
<h2 style="margin-top: 1em">Markscheme</h2>
<div class="question" style="padding-left: 20px;">
<p>\((V = ){\text{ }}\pi  \times {{\text{(17.5)}}^2} \times 0.5\)     <strong><em>(A1)(M1)</em></strong></p>
<p> </p>
<p><strong>Notes:</strong>     Award <strong><em>(A1) </em></strong>for 17.5 (or equivalent) seen.</p>
<p>Award <strong><em>(M1) </em></strong>for correct substitutions into volume of a cylinder formula.</p>
<p> </p>
<p>\( = 481{\text{ c}}{{\text{m}}^3}{\text{ }}(481.056 \ldots {\text{ c}}{{\text{m}}^3},{\text{ }}153.125\pi {\text{ c}}{{\text{m}}^3})\)     <strong><em>(A1)(G2)</em></strong></p>
<p><strong><em>[3 marks]</em></strong></p>
<div class="question_part_label">a.</div>
</div>
<div class="question" style="padding-left: 20px;">
<p>\(\frac{4}{3} \times \pi  \times {r^3} = 481.056 \ldots \)     <strong><em>(M1)</em></strong></p>
<p> </p>
<p><strong>Note:</strong>     Award <strong><em>(M1) </em></strong>for equating <strong>their </strong>answer to part (a) to the volume of sphere.</p>
<p> </p>
<p>\({r^3} = \frac{{3 \times 481.056 \ldots }}{{4\pi }}{\text{ }}( = 114.843 \ldots )\)     <strong><em>(M1)</em></strong></p>
<p> </p>
<p><strong>Note:</strong>     Award <strong><em>(M1) </em></strong>for correctly rearranging so \({r^3}\) is the subject.</p>
<p> </p>
<p>\(r = 4.86074 \ldots {\text{ (cm)}}\)     <strong><em>(A1)</em>(ft)<em>(G2)</em></strong></p>
<p> </p>
<p><strong>Note:</strong>     Award <strong><em>(A1) </em></strong>for correct unrounded answer seen. Follow through from part (a).</p>
<p> </p>
<p>\( = 4.9{\text{ (cm)}}\)     <strong><em>(A1)</em>(ft)<em>(G3)</em></strong></p>
<p> </p>
<p><strong>Note:</strong>     The final <strong><em>(A1)</em>(ft) </strong>is awarded for rounding their unrounded answer to one decimal place.</p>
<p> </p>
<p><strong><em>[4 marks]</em></strong></p>
<div class="question_part_label">b.</div>
</div>
<div class="question" style="padding-left: 20px;">
<p>\(230 = a{(2.06)^0} + 19\)     <strong><em>(M1)</em></strong></p>
<p> </p>
<p><strong>Note:</strong>     Award <strong><em>(M1) </em></strong>for correct substitution.</p>
<p> </p>
<p>\(a = 211\)     <strong><em>(A1)(G2)</em></strong></p>
<p><strong><em>[2 marks]</em></strong></p>
<div class="question_part_label">c.</div>
</div>
<div class="question" style="padding-left: 20px;">
<p>\((P = ){\text{ }}211 \times {(2.06)^{ - 5}} + 19\)      <strong><em>(M1)</em></strong></p>
<p> </p>
<p><strong>Note:</strong>     Award <strong><em>(M1) </em></strong>for correct substitution into the function, \(P(t)\). Follow through from part (c). The negative sign in the exponent is required for correct substitution.</p>
<p> </p>
<p>\( = 24.7\) (°C) \((24.6878 \ldots \) (°C))     <strong><em>(A1)</em>(ft)<em>(G2)</em></strong></p>
<p><strong><em>[2 marks]</em></strong></p>
<div class="question_part_label">d.</div>
</div>
<div class="question" style="padding-left: 20px;">
<p>\(45 = 211 \times {(2.06)^{ - t}} + 19\)     <strong><em>(M1)</em></strong></p>
<p> </p>
<p><strong>Note:</strong>     Award <strong><em>(M1) </em></strong>for equating 45 to the exponential equation and for correct substitution (follow through for their \(a\) in part (c)).</p>
<p> </p>
<p>\((t = ){\text{ }}2.89711 \ldots \)     <strong><em>(A1)</em>(ft)<em>(G1)</em></strong></p>
<p>\(174{\text{ (seconds) }}\left( {173.826 \ldots {\text{ (seconds)}}} \right)\)     <strong><em>(A1)</em>(ft)<em>(G2)</em></strong></p>
<p> </p>
<p><strong>Note:</strong>     Award final <strong><em>(A1)</em>(ft) </strong>for converting their \({\text{2.89711}} \ldots \) minutes into seconds.</p>
<p> </p>
<p><strong><em>[3 marks]</em></strong></p>
<div class="question_part_label">e.</div>
</div>
<div class="question" style="padding-left: 20px;">
<p>the temperature of the (dining) room     <strong><em>(A1)</em></strong></p>
<p><strong>OR</strong></p>
<p>the lowest final temperature to which the pizza will cool     <strong><em>(A1)</em></strong></p>
<p><strong><em>[1 mark]</em></strong></p>
<div class="question_part_label">f.</div>
</div>
<h2 style="margin-top: 1em">Examiners report</h2>
<div class="question" style="padding-left: 20px;">
[N/A]
<div class="question_part_label">a.</div>
</div>
<div class="question" style="padding-left: 20px;">
[N/A]
<div class="question_part_label">b.</div>
</div>
<div class="question" style="padding-left: 20px;">
[N/A]
<div class="question_part_label">c.</div>
</div>
<div class="question" style="padding-left: 20px;">
[N/A]
<div class="question_part_label">d.</div>
</div>
<div class="question" style="padding-left: 20px;">
[N/A]
<div class="question_part_label">e.</div>
</div>
<div class="question" style="padding-left: 20px;">
[N/A]
<div class="question_part_label">f.</div>
</div>
<br><hr><br><div class="specification">
<p class="p1">The following table shows the number of bicycles, \(x\), produced daily by a factory and their total production cost, \(y\)<span class="s1">, in US dollars (USD)</span>. The table shows data recorded over seven days.</p>
<p class="p1" style="text-align: center;"><img src="images/Schermafbeelding_2015-12-22_om_10.06.31.png" alt></p>
</div>

<div class="question" style="padding-left: 20px; padding-right: 20px;">
<p class="p1">(i)     Write down the Pearson’s product–moment correlation coefficient, \(r\), for these data.</p>
<p class="p1">(ii)     Hence comment on the result.</p>
<div class="marks">[4]</div>
<div class="question_part_label">a.</div>
</div>
<div class="question" style="padding-left: 20px; padding-right: 20px;">
<p class="p1">Write down the equation of the regression line \(y\) on \(x\) for these data, in the form \(y = ax + b\).</p>
<div class="marks">[2]</div>
<div class="question_part_label">b.</div>
</div>
<div class="question" style="padding-left: 20px; padding-right: 20px;">
<p class="p1">Estimate the total cost, <strong>to the nearest </strong><span class="s1"><strong>USD</strong></span>, of producing \(13\)<span class="s1"> </span>bicycles on a particular day.</p>
<div class="marks">[3]</div>
<div class="question_part_label">c.</div>
</div>
<div class="question" style="padding-left: 20px; padding-right: 20px;">
<p class="p1">All the bicycles that are produced are sold. The bicycles are sold for <span class="s1">304 USD </span><strong>each</strong>.</p>
<p class="p1">Explain why the factory does <strong>not </strong>make a profit when producing \(13\)<span class="s1"> </span>bicycles on a particular day.</p>
<div class="marks">[2]</div>
<div class="question_part_label">d.</div>
</div>
<div class="question" style="padding-left: 20px; padding-right: 20px;">
<p class="p1">All the bicycles that are produced are sold. The bicycles are sold for <span class="s1">304 USD </span><strong>each</strong>.</p>
<p class="p1">(i)     Write down an expression for the total selling price of \(x\) bicycles.</p>
<p class="p1">(ii)     Write down an expression for the <strong>profit </strong>the factory makes when producing \(x\) bicycles on a particular day.</p>
<p class="p1">(iii)     Find the least number of bicycles that the factory should produce, on a particular day, in order to make a profit.</p>
<div class="marks">[5]</div>
<div class="question_part_label">e.</div>
</div>
<h2 style="margin-top: 1em">Markscheme</h2>
<div class="question" style="padding-left: 20px;">
<p class="p1">(i) <span class="Apple-converted-space">    </span>\(r = 0.985\;\;\;(0.984905 \ldots )\) <span class="Apple-converted-space">    </span><strong><em>(G2)</em></strong></p>
<p class="p1"><strong>Notes: </strong>If unrounded answer is not seen, award <strong><em>(G1)(G0) </em></strong>for \(0.99\)<span class="s1"> </span>or \(0.984\). Award <strong><em>(G2) </em></strong>for \(0.98\).</p>
<p class="p2"> </p>
<p class="p1">(ii) <span class="Apple-converted-space">    </span>strong, positive <span class="Apple-converted-space">    </span><strong><em>(A1)(A1)</em></strong></p>
<div class="question_part_label">a.</div>
</div>
<div class="question" style="padding-left: 20px;">
<p class="p1">\(y = 259.909 \ldots x + 698.648 \ldots \;\;\;(y = 260x + 699)\) <span class="Apple-converted-space">    </span><strong><em>(G1)(G1)</em></strong></p>
<p class="p1"><strong>Notes: </strong>Award <strong><em>(G1) </em></strong>for \(260x\) and <strong><em>(G1) </em></strong>for \(699\). If the answer is not an equation award a maximum of <strong><em>(G1)(G0)</em></strong>.</p>
<div class="question_part_label">b.</div>
</div>
<div class="question" style="padding-left: 20px;">
<p>\(y = 259.909 \ldots  \times 13 + 698.648 \ldots \)     <strong><em>(M1)</em></strong></p>
<p><strong>Note: </strong>Award <strong><em>(M1) </em></strong>for substitution of \(13\) into their regression line equation from part (b).</p>
<p> </p>
<p>\(y = 4077.47 \ldots \)     <strong><em>(A1)</em>(ft)<em>(G2)</em></strong></p>
<p>\(y = 4077{\text{ (USD)}}\)     <strong><em>(A1)</em>(ft)</strong></p>
<p><strong>Notes: </strong>Follow through from their answer to part (b). If rounded values from part (b) used, answer is \(4079\). Award the final <strong><em>(A1)</em>(ft) </strong>for a correct rounding to the nearest USD of their answer. The unrounded answer may not be seen.</p>
<p>If answer is \(4077\) and no working is seen, award <strong><em>(G2)</em></strong>.</p>
<div class="question_part_label">c.</div>
</div>
<div class="question" style="padding-left: 20px;">
<p>\(13 \times 304 - (4077.47) =  - 125.477 \ldots \;\;\;( - 125)\;\;\;\)<strong>OR</strong></p>
<p>\(4077.47 - (13 \times 304) = 125.477 \ldots \;\;\;(125)\)     <strong><em>(M1)</em></strong></p>
<p><strong>Notes: </strong>Award <strong><em>(M1) </em></strong>for calculating the difference between \(13 \times 304\) and their answer to part (c).</p>
<p>If rounded values are used in equation, answer is \( - 127\).</p>
<p> </p>
<p>profit is negative\(\;\;\;\)<strong>OR</strong>\(\;\;\;{\text{cost}} &gt; {\text{sales}}\)     <strong><em>(A1)</em></strong></p>
<p> </p>
<p><strong>OR</strong></p>
<p>\(13 \times 304 = 3952\)     <strong><em>(M1)</em></strong></p>
<p><strong>Note: </strong>Award <strong><em>(M1) </em></strong>for calculating the price of \(13\) bikes.</p>
<p> </p>
<p>\(3952 &lt; 4077.47\)     <strong><em>(A1)</em>(ft)</strong></p>
<p><strong>Note: </strong>Award <strong><em>(A1) </em></strong>for showing \(3952\) is less than their part (c). This may be communicated in words. Follow through from part (c), but only if value is greater than \(3952\).</p>
<p> </p>
<p><strong>OR</strong></p>
<p>\(\frac{{4077}}{{13}} = 313.62\)     <strong><em>(M1)</em></strong></p>
<p><strong>Note: </strong>Award <strong><em>(M1) </em></strong>for calculating the cost of \(1\) bicycle.</p>
<p> </p>
<p>\(313.62 &gt; 304\)     <strong><em>(A1)</em>(ft)</strong></p>
<p><strong>Note: </strong>Award <strong><em>(A1) </em></strong>for showing \(313.62\) is greater than \(304\). This may be communicated in words. Follow through from part (c), but only if value is greater than \(304\).</p>
<p> </p>
<p><strong>OR</strong></p>
<p>\(\frac{{4077}}{{304}} = 13.41\)     <strong><em>(M1)</em></strong></p>
<p><strong>Note: </strong>Award <strong><em>(M1) </em></strong>for calculating the number of bicycles that should have been be sold to cover total cost.</p>
<p> </p>
<p>\(13.41 &gt; 13\)     <strong><em>(A1)</em>(ft)</strong></p>
<p><strong>Note: </strong>Award <strong><em>(A1) </em></strong>for showing \(13.41\) is greater than \(13\). This may be communicated in words. Follow through from part (c), but only if value is greater than \(13\).</p>
<div class="question_part_label">d.</div>
</div>
<div class="question" style="padding-left: 20px;">
<p class="p1">(i) <span class="Apple-converted-space">    </span>\(304x\) <span class="Apple-converted-space">    </span><strong><em>(A1)</em></strong></p>
<p class="p1"> </p>
<p class="p1">(ii) <span class="Apple-converted-space">    </span>\(304x - (259.909 \ldots x + 698.648 \ldots )\) <span class="Apple-converted-space">    </span><strong><em>(A1)</em>(ft)<em>(A1)</em>(ft)</strong></p>
<p class="p1"><strong>Note: </strong>Award <strong><em>(A1)</em>(ft) </strong>for difference between their answers to parts (b) and (e)(i), <strong><em>(A1)</em>(ft) </strong>for correct expression.</p>
<p class="p2"> </p>
<p class="p1">(iii) <span class="Apple-converted-space">    </span>\(304x - (259.909 \ldots x + 698.648 \ldots ) &gt; 0\) <span class="Apple-converted-space">    </span><strong><em>(M1)</em></strong></p>
<p class="p1"><strong>Notes: </strong>Award <strong><em>(M1) </em></strong>for comparing their expression in part (e)(ii) to \(0\). Accept an equation. Accept \(3040x - y &gt; 0\) or equivalent.</p>
<p class="p2"> </p>
<p class="p1">\(x = 16{\text{ bicycles}}\) <span class="Apple-converted-space">    </span><strong><em>(A1)</em>(ft)<em>(G2)</em></strong></p>
<p class="p1"><strong>Notes:<span class="Apple-converted-space"> </span></strong>Follow through from their answer to part (b). Answer must be a positive integer greater than \(13\)<span class="s1"> </span>for the <strong><em>(A1)</em>(ft) </strong>to be awarded.</p>
<p class="p1">Award <strong><em>(G1) </em></strong>for an answer of \(15.84\).</p>
<div class="question_part_label">e.</div>
</div>
<h2 style="margin-top: 1em">Examiners report</h2>
<div class="question" style="padding-left: 20px;">
[N/A]
<div class="question_part_label">a.</div>
</div>
<div class="question" style="padding-left: 20px;">
[N/A]
<div class="question_part_label">b.</div>
</div>
<div class="question" style="padding-left: 20px;">
[N/A]
<div class="question_part_label">c.</div>
</div>
<div class="question" style="padding-left: 20px;">
[N/A]
<div class="question_part_label">d.</div>
</div>
<div class="question" style="padding-left: 20px;">
[N/A]
<div class="question_part_label">e.</div>
</div>
<br><hr><br><div class="specification">
<p><span style="font-size: medium; font-family: times new roman,times;">The diagram below shows the graph of a line \(L\) passing through (1, 1) and (2 , 3) and the graph \(P\) of the function \(f (x) = x^2 &minus; 3x &minus; 4\)</span></p>
<p><img src="data:image/png;base64,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" alt></p>
</div>

<div class="question" style="padding-left: 20px; padding-right: 20px;">
<p><span>Find the gradient of the line <em>L</em>.</span></p>
<div class="marks">[2]</div>
<div class="question_part_label">a.</div>
</div>
<div class="question" style="padding-left: 20px; padding-right: 20px;">
<p><span>Differentiate \(f (x)\) .</span></p>
<div class="marks">[2]</div>
<div class="question_part_label">b.</div>
</div>
<div class="question" style="padding-left: 20px; padding-right: 20px;">
<p><span>Find the coordinates of the point where the tangent to <em>P</em> is parallel to the line <em>L</em>.</span></p>
<div class="marks">[3]</div>
<div class="question_part_label">c.</div>
</div>
<div class="question" style="padding-left: 20px; padding-right: 20px;">
<p><span>Find the coordinates of the point where the tangent to <em>P</em> is perpendicular to the line<em> L</em>.</span></p>
<div class="marks">[4]</div>
<div class="question_part_label">d.</div>
</div>
<div class="question" style="padding-left: 20px; padding-right: 20px;">
<p><span>Find</span></p>
<p><span>(i) the gradient of the tangent to <em>P</em> at the point with coordinates (2, − 6).</span></p>
<p><span>(ii) the equation of the tangent to <em>P</em> at this point.</span></p>
<div class="marks">[3]</div>
<div class="question_part_label">e.</div>
</div>
<div class="question" style="padding-left: 20px; padding-right: 20px;">
<p><span>State the equation of the axis of symmetry of <em>P</em>.</span></p>
<div class="marks">[1]</div>
<div class="question_part_label">f.</div>
</div>
<div class="question" style="padding-left: 20px; padding-right: 20px;">
<p><span>Find the coordinates of the vertex of <em>P</em> and state the gradient of the curve at this point.</span></p>
<div class="marks">[3]</div>
<div class="question_part_label">g.</div>
</div>
<h2 style="margin-top: 1em">Markscheme</h2>
<div class="question" style="padding-left: 20px;">
<p><span><em>for attempt at substituted</em> \(\frac{{ydistance}}{{xdistance}}\)     <em><strong>(M1)</strong></em></span></p>
<p><span>gradient = 2     <em><strong>(A1)(G2)</strong></em></span></p>
<p><span><em><strong>[2 marks]<br></strong></em></span></p>
<div class="question_part_label">a.</div>
</div>
<div class="question" style="padding-left: 20px;">
<p><span>\(2x - 3\)     <em><strong>(A1)(A1)</strong></em></span></p>
<p><span><em><strong>(A1)</strong> for</em> \(2x\) , <em><strong>(A1)</strong> for</em> \(-3\)</span></p>
<p><span><em><strong>[2 marks]</strong></em><br></span></p>
<div class="question_part_label">b.</div>
</div>
<div class="question" style="padding-left: 20px;">
<p><span><em>for their</em> \(2x - 3 =\) <em>their gradient and attempt to solve     <strong>(M1)</strong></em></span></p>
<p><span>\(x = 2.5\)     <em><strong>(A1)</strong></em><strong>(ft)</strong></span></p>
<p><span>\(y = -5.25\) <em>(</em><strong>(ft)</strong><em> from their x value)     <strong>(A1)</strong></em><strong>(ft)</strong><em><strong>(G2)</strong></em></span></p>
<p><span><em><strong>[3 marks]<br></strong></em></span></p>
<div class="question_part_label">c.</div>
</div>
<div class="question" style="padding-left: 20px;">
<p><em><span>for seeing </span></em><span>\(\frac{{ - 1}}{{their(a)}}\)     <em><strong>(M1)</strong></em></span></p>
<p><span><em>solving</em> \(2x – 3 = - \frac{1}{2}\) <em>(or their value)     <strong>(M1)</strong></em></span></p>
<p><span><em>x</em> = 1.25     <em><strong>(A1)</strong></em><strong>(ft)</strong><em><strong>(G1)</strong></em><br></span></p>
<p><span><em>y</em> = – 6.1875     <em><strong>(A1)</strong></em><strong>(ft)</strong><em><strong>(G1)</strong><strong><br></strong></em></span></p>
<p><span><em><strong>[4 marks]<br></strong></em></span></p>
<div class="question_part_label">d.</div>
</div>
<div class="question" style="padding-left: 20px;">
<p><span>(i) \(2 \times 2 - 3 = 1\) <em>(</em><strong>(ft)</strong> <em>from (b))     <strong>(A1)</strong></em><strong>(ft)</strong><em><strong>(G1)</strong></em></span></p>
<p><span><em><strong> </strong></em></span></p>
<p><span>(ii) \(y = mx + c\) <em>or equivalent method to find</em> \(c \Rightarrow -6 = 2 + c\)     <em><strong>(M1)</strong></em></span></p>
<p><span>\(y = x - 8\)     <em><strong>(A1)</strong></em><strong>(ft)</strong><em><strong>(G2)</strong></em></span></p>
<p><span><em><strong> </strong></em></span></p>
<p><span><em><strong>[3 marks]<br></strong></em></span></p>
<div class="question_part_label">e.</div>
</div>
<div class="question" style="padding-left: 20px;">
<p><span>\(x = 1.5\)     <em><strong>(A1)</strong></em></span></p>
<p><span><em><strong>[1 mark]<br></strong></em></span></p>
<div class="question_part_label">f.</div>
</div>
<div class="question" style="padding-left: 20px;">
<p><span><em>for substituting their answer to part (f) into the equation of the parabola</em> (1.5, −6.25) <em>accept</em> <em>x</em> = 1.5, <em>y</em> = −6.25     <em><strong>(M1)(A1)</strong></em><strong>(ft)</strong><em><strong>(G2)</strong></em></span></p>
<p><span>gradient is zero <em>(accept</em> \(\frac{{{\text{d}}y}}{{{\text{d}}x}} = 0\)<em>)     <strong>(A1)</strong></em></span></p>
<p><span><em><strong>[3 marks]<br></strong></em></span></p>
<div class="question_part_label">g.</div>
</div>
<h2 style="margin-top: 1em">Examiners report</h2>
<div class="question" style="padding-left: 20px;">
<p><span style="font-size: medium; font-family: times new roman,times;">Parts (a) and (b) were very well done. After that, only the stronger candidates were able to cope. The equation of the tangent at the point with coordinates (2, 6) was badly done but some candidates managed to find the equation of the tangent line from their GDC. The equation of the axis of symmetry was reasonably well done although many just wrote down 1.5 instead of <em>x</em> = 1.5.</span></p>
<p><span style="font-size: medium; font-family: times new roman,times;">Some forgot to write down that the gradient at the vertex was 0.</span></p>
<div class="question_part_label">a.</div>
</div>
<div class="question" style="padding-left: 20px;">
<p><span style="font-size: medium; font-family: times new roman,times;">Parts (a) and (b) were very well done. After that, only the stronger candidates were able to cope. The equation of the tangent at the point with coordinates (2, 6) was badly done but some candidates managed to find the equation of the tangent line from their GDC. The equation of the axis of symmetry was reasonably well done although many just wrote down 1.5 instead of <em>x</em> = 1.5.</span></p>
<p><span style="font-size: medium; font-family: times new roman,times;">Some forgot to write down that the gradient at the vertex was 0.</span></p>
<div class="question_part_label">b.</div>
</div>
<div class="question" style="padding-left: 20px;">
<p><span style="font-size: medium; font-family: times new roman,times;">Parts (a) and (b) were very well done. After that, only the stronger candidates were able to cope. The equation of the tangent at the point with coordinates (2, 6) was badly done but some candidates managed to find the equation of the tangent line from their GDC. The equation of the axis of symmetry was reasonably well done although many just wrote down 1.5 instead of <em>x</em> = 1.5.</span></p>
<p><span style="font-size: medium; font-family: times new roman,times;">Some forgot to write down that the gradient at the vertex was 0.</span></p>
<div class="question_part_label">c.</div>
</div>
<div class="question" style="padding-left: 20px;">
<p><span style="font-size: medium; font-family: times new roman,times;">Parts (a) and (b) were very well done. After that, only the stronger candidates were able to cope. The equation of the tangent at the point with coordinates (2, 6) was badly done but some candidates managed to find the equation of the tangent line from their GDC. The equation of the axis of symmetry was reasonably well done although many just wrote down 1.5 instead of <em>x</em> = 1.5.</span></p>
<p><span style="font-size: medium; font-family: times new roman,times;">Some forgot to write down that the gradient at the vertex was 0.</span></p>
<div class="question_part_label">d.</div>
</div>
<div class="question" style="padding-left: 20px;">
<p><span style="font-size: medium; font-family: times new roman,times;">Parts (a) and (b) were very well done. After that, only the stronger candidates were able to cope. The equation of the tangent at the point with coordinates (2, 6) was badly done but some candidates managed to find the equation of the tangent line from their GDC. The equation of the axis of symmetry was reasonably well done although many just wrote down 1.5 instead of <em>x</em> = 1.5.</span></p>
<p><span style="font-size: medium; font-family: times new roman,times;">Some forgot to write down that the gradient at the vertex was 0.</span></p>
<div class="question_part_label">e.</div>
</div>
<div class="question" style="padding-left: 20px;">
<p><span style="font-size: medium; font-family: times new roman,times;">Parts (a) and (b) were very well done. After that, only the stronger candidates were able to cope. The equation of the tangent at the point with coordinates (2, 6) was badly done but some candidates managed to find the equation of the tangent line from their GDC. The equation of the axis of symmetry was reasonably well done although many just wrote down 1.5 instead of <em>x</em> = 1.5.</span></p>
<p><span style="font-size: medium; font-family: times new roman,times;">Some forgot to write down that the gradient at the vertex was 0.</span></p>
<div class="question_part_label">f.</div>
</div>
<div class="question" style="padding-left: 20px;">
<p><span style="font-size: medium; font-family: times new roman,times;">Parts (a) and (b) were very well done. After that, only the stronger candidates were able to cope. The equation of the tangent at the point with coordinates (2, 6) was badly done but some candidates managed to find the equation of the tangent line from their GDC. The equation of the axis of symmetry was reasonably well done although many just wrote down 1.5 instead of <em>x</em> = 1.5.</span></p>
<p><span style="font-size: medium; font-family: times new roman,times;">Some forgot to write down that the gradient at the vertex was 0.</span></p>
<div class="question_part_label">g.</div>
</div>
<br><hr><br><div class="specification">
<p><span style="font-size: medium; font-family: times new roman,times;">The diagram shows an <strong>aerial</strong> view of a bicycle track. The track can be modelled by the quadratic function</span></p>
<p><span style="font-size: medium; font-family: times new roman,times;">\(y = \frac{{ - {x^2}}}{{10}} + \frac{{27}}{2}x\), where \(x \geqslant 0,{\text{ }}y \geqslant 0\)</span></p>
<p><span style="font-size: medium; font-family: times new roman,times;">(<em>x</em> , <em>y</em>) are the coordinates of a point <em>x</em> metres east and <em>y</em> metres north of O , where O is the origin (0, 0) . B is a point on the bicycle track with coordinates (100, 350) .<br></span></p>
<p><span style="font-size: medium; font-family: times new roman,times;"><img src="data:image/png;base64,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" alt></span></p>
</div>

<div class="question" style="padding-left: 20px; padding-right: 20px;">
<p><span>The coordinates of point A are (75, 450). Determine whether point A is on the bicycle track. Give a reason for your answer.</span></p>
<div class="marks">[3]</div>
<div class="question_part_label">a.</div>
</div>
<div class="question" style="padding-left: 20px; padding-right: 20px;">
<p><span>Find the derivative of \(y = \frac{{ - {x^2}}}{{10}} + \frac{{27}}{2}x\).</span></p>
<div class="marks">[2]</div>
<div class="question_part_label">b.</div>
</div>
<div class="question" style="padding-left: 20px; padding-right: 20px;">
<p><span>Use the answer in part (b) to determine if A (75, 450) is the point furthest north on the track between O and B. Give a reason for your answer.</span></p>
<div class="marks">[4]</div>
<div class="question_part_label">c.</div>
</div>
<div class="question" style="padding-left: 20px; padding-right: 20px;">
<p><span>(i) Write down the midpoint of the line segment OB.</span></p>
<p><span>(ii) Find the gradient of the line segment OB.</span></p>
<div class="marks">[3]</div>
<div class="question_part_label">d.</div>
</div>
<div class="question" style="padding-left: 20px; padding-right: 20px;">
<p><span>Scott starts from a point C(0,150) . He hikes along a straight road towards the bicycle track, parallel to the line segment OB.</span></p>
<p><span>Find the equation of Scott’s road. Express your answer in the form \(ax + by = c\), where \(a, b {\text{ and }} c \in \mathbb{R}\).</span></p>
<div class="marks">[3]</div>
<div class="question_part_label">e.</div>
</div>
<div class="question" style="padding-left: 20px; padding-right: 20px;">
<p><span>Use your graphic display calculator to find the coordinates of the point where Scott first crosses the bicycle track.</span></p>
<div class="marks">[2]</div>
<div class="question_part_label">f.</div>
</div>
<h2 style="margin-top: 1em">Markscheme</h2>
<div class="question" style="padding-left: 20px;">
<p><span>\(y =  - \frac{{{{75}^2}}}{{10}} + \frac{{27}}{2} \times 75\)     <em><strong>(M1)</strong></em></span></p>
<p><span><strong> </strong></span></p>
<p><span><strong>Note:</strong> Award <strong><em>(M1)</em></strong> for substitution of 75 in the formula of the function.</span></p>
<p><br><span>= 450     <em><strong>(A1)</strong></em></span></p>
<p><span>Yes, point A is on the bike track. <em><strong>(A1)</strong></em></span></p>
<p><br><span><strong>Note:</strong> Do not award the final <em><strong>(A1)</strong></em> if correct working is not seen.</span></p>
<div class="question_part_label">a.</div>
</div>
<div class="question" style="padding-left: 20px;">
<p><span>\(\frac{{{\text{d}}y}}{{{\text{d}}x}} =  - \frac{{2x}}{{10}} + \frac{{27}}{2}\left( {\frac{{{\text{d}}y}}{{{\text{d}}x}} =  - 0.2x + 13.5} \right)\)     <em><strong>(A1)(A1)</strong></em></span></p>
<p><br><span><strong>Notes:</strong> Award <em><strong>(A1)</strong></em> for each correct term. If extra terms are seen award at most <em><strong>(A1)(A0)</strong></em>. Accept equivalent forms.</span></p>
<div class="question_part_label">b.</div>
</div>
<div class="question" style="padding-left: 20px;">
<p><span>\( - \frac{{2x}}{{10}} + \frac{{27}}{2} = 0\)     <em><strong>(M1)</strong></em></span></p>
<p><br><span><strong>Note:</strong> Award <em><strong>(M1)</strong></em> for equating their derivative from part (b) to zero.</span></p>
<p><br><span>\(x = 67.5\)     <em><strong>(A1)</strong></em><strong>(ft)</strong></span></p>
<p><br><span><strong>Note:</strong> Follow through from their derivative from part (b).</span></p>
<p><br><span>\( {\text{(Their) }} 67.5 \ne 75\)      <em><strong>(R1)</strong></em></span></p>
<p><span><strong> </strong></span></p>
<p><span><strong>Note:</strong> Award <em><strong>(R1)</strong></em> for a comparison of their 67.5 with 75. Comparison may be implied</span> <span>(<em>eg</em> 67.5 is the <em>x</em>-coordinate of the furthest north point).</span></p>
<p><span><br><strong>OR</strong><br></span></p>
<p><span>\(\frac{{{\text{d}}y}}{{{\text{d}}x}} =  - \frac{{2 \times (75)}}{{10}} + \frac{{27}}{2}\)     <em><strong>(M1)</strong></em></span></p>
<p><span><br><strong>Note:</strong> Award <em><strong>(M1)</strong></em> for substitution of 75 into their derivative from part (b).</span></p>
<p><span><br>\(= -1.5\)     <em><strong>(A1)</strong></em><strong>(ft)</strong><br></span></p>
<p><span><strong><br>Note: </strong>Follow through from their derivative from part (b).<strong><br></strong></span></p>
<p><span><strong> </strong></span></p>
<p><span>\({\text{(Their)}} -1.5 \ne 0\)     <em><strong>(R1)</strong></em><br></span></p>
<p><span><strong><br>Note:</strong> Award <em><strong>(R1)</strong></em> for a comparison of their –1.5 with 0. Comparison may be implied (<em>eg</em> The gradient of the parabola at the furthest north point (vertex) is 0).<strong><br></strong></span></p>
<p><span><strong> </strong></span></p>
<p><span>Hence A is not the furthest north point.     <em><strong>(A1)</strong></em><strong>(ft)</strong>    <br></span></p>
<p><span><br><strong>Note:</strong> Do not award <em><strong>(R0)(A1)</strong></em><strong>(ft)</strong>. Follow through from their derivative from part (b).<br></span></p>
<div class="question_part_label">c.</div>
</div>
<div class="question" style="padding-left: 20px;">
<p><span>(i) M(50,175)     <em><strong>(A1)</strong></em></span></p>
<p><br><span><strong>Note:</strong> If parentheses are omitted award <em><strong>(A0)</strong></em>. </span><span>Accept <em>x</em> = 50, <em>y</em> = 175.</span></p>
<p><br><span>(ii) \(\frac{{350 - 0}}{{100 - 0}}\)</span><span>     </span><em><strong><span>(M1)</span></strong></em></p>
<p><br><span><strong>Note:</strong> Award <em><strong>(M1)</strong></em> for correct substitution in gradient formula.</span></p>
<p><br><span>\( = 3.5\left( {\frac{{350}}{{100}},\frac{7}{2}} \right)\)     <em><strong>(A1)</strong></em><strong>(ft)</strong><em><strong>(G2)</strong></em></span></p>
<p><br><span><strong>Note:</strong> Follow through from (d)(i) if midpoint is used to calculate gradient. Award <em><strong>(G1)(G0)</strong></em> for answer 3.5<em>x</em> without working.</span></p>
<div class="question_part_label">d.</div>
</div>
<div class="question" style="padding-left: 20px;">
<p><span>\(y = 3.5x + 150\)     <em><strong>(A1)</strong></em><strong>(ft)</strong><em><strong>(A1)</strong></em><strong>(ft)</strong></span></p>
<p><br><span><strong>Note:</strong> Award <em><strong>(A1)</strong></em><strong>(ft)</strong> for using their gradient from part (d), <em><strong>(A1)</strong></em><strong>(ft)</strong> for correct equation of line.</span><br><br></p>
<p><span>\(3.5x - y = -150\) <strong>or</strong> \(7x - 2y = -300\) (or equivalent)     <em><strong>(A1)</strong></em><strong>(ft)</strong></span></p>
<p><br><span><strong>Note:</strong> Award <em><strong>(A1)</strong></em><strong>(ft)</strong> for expressing their equation in the form \(ax + by = c\).</span></p>
<div class="question_part_label">e.</div>
</div>
<div class="question" style="padding-left: 20px;">
<p><span>(18.4, 214) (18.3772..., 214.320...)     <em><strong>(A1)</strong></em><strong>(ft)</strong><em><strong>(A1)</strong></em><strong>(ft)</strong><em><strong>(G2)</strong></em><strong>(ft)</strong></span></p>
<p><br><span><strong>Notes:</strong> Follow through from their equation in (e). Coordinates must be positive for follow through marks to be awarded. If parentheses are omitted and not already penalized in (d)(i) award at most <em><strong>(A0)(A1)</strong></em><strong>(ft)</strong>. If coordinates of the two intersection points are given award <em><strong>(A0)(A1)</strong></em><strong>(ft)</strong>. Accept <em>x</em> = 18.4, <em>y</em> = 214.</span></p>
<div class="question_part_label">f.</div>
</div>
<h2 style="margin-top: 1em">Examiners report</h2>
<div class="question" style="padding-left: 20px;">
[N/A]
<div class="question_part_label">a.</div>
</div>
<div class="question" style="padding-left: 20px;">
[N/A]
<div class="question_part_label">b.</div>
</div>
<div class="question" style="padding-left: 20px;">
[N/A]
<div class="question_part_label">c.</div>
</div>
<div class="question" style="padding-left: 20px;">
[N/A]
<div class="question_part_label">d.</div>
</div>
<div class="question" style="padding-left: 20px;">
[N/A]
<div class="question_part_label">e.</div>
</div>
<div class="question" style="padding-left: 20px;">
[N/A]
<div class="question_part_label">f.</div>
</div>
<br><hr><br><div class="specification">
<p><strong><span style="font-size: medium; font-family: times new roman,times;">Give all answers in this question to the nearest whole currency unit.</span></strong></p>
<p><span style="font-size: medium; font-family: times new roman,times;">Ying and Ruby each have 5000 USD to invest.</span></p>
<p><span style="font-size: medium; font-family: times new roman,times;">Ying invests his 5000 USD in a bank account that pays a nominal annual interest rate of 4.2 % <strong>compounded yearly</strong>. Ruby invests her 5000 USD in an account that offers a fixed interest of 230 USD each year.</span></p>
</div>

<div class="question" style="padding-left: 20px; padding-right: 20px;">
<p><span>Find the amount of money that Ruby will have in the bank after 3 years.</span></p>
<div class="marks">[2]</div>
<div class="question_part_label">a.</div>
</div>
<div class="question" style="padding-left: 20px; padding-right: 20px;">
<p><span>Show that Ying will have 7545 USD in the bank at the end of 10 years.</span></p>
<div class="marks">[3]</div>
<div class="question_part_label">b.</div>
</div>
<div class="question" style="padding-left: 20px; padding-right: 20px;">
<p><span>Find the number of complete years it will take for Ying’s investment to first exceed 6500 USD.</span></p>
<div class="marks">[3]</div>
<div class="question_part_label">c.</div>
</div>
<div class="question" style="padding-left: 20px; padding-right: 20px;">
<p><span>Find the number of complete years it will take for Ying’s investment to exceed Ruby’s investment.</span></p>
<div class="marks">[3]</div>
<div class="question_part_label">d.</div>
</div>
<div class="question" style="padding-left: 20px; padding-right: 20px;">
<p><span>Ruby moves from the USA to Italy. She transfers 6610 USD into an Italian bank which has an exchange rate of 1 USD = 0.735 Euros. The bank charges 1.8 % commission.</span></p>
<p><span>Calculate the amount of money Ruby will invest in the Italian bank after commission.</span></p>
<div class="marks">[4]</div>
<div class="question_part_label">e.</div>
</div>
<div class="question" style="padding-left: 20px; padding-right: 20px;">
<p><span>Ruby returns to the USA for a short holiday. She converts 800 Euros at a bank in Chicago and receives 1006.20 USD. The bank advertises an exchange rate of 1 Euro = 1.29 USD.</span></p>
<p><span>Calculate the percentage commission Ruby is charged by the bank.</span></p>
<div class="marks">[5]</div>
<div class="question_part_label">f.</div>
</div>
<h2 style="margin-top: 1em">Markscheme</h2>
<div class="question" style="padding-left: 20px;">
<p><span>5000 + 3 × 230 = 5690     <em><strong>(M1)(A1)(G2)</strong></em></span></p>
<p> </p>
<p><span><strong>Note:</strong> Accept alternative method.</span></p>
<p><span> </span></p>
<p><em><strong><span>[2 marks]</span></strong></em></p>
<div class="question_part_label">a.</div>
</div>
<div class="question" style="padding-left: 20px;">
<p><span>\({\text{A}} = 5000{\left( {1 + \frac{{4.2}}{{100}}} \right)^{10}}\) <em>or equivalent</em>     <em><strong>(M1)(A1)</strong></em></span></p>
<p><span>\( = 7544.79 \ldots \)     <em><strong>(A1)</strong></em></span></p>
<p><span>\( = 7545{\text{ USD}}\)     <em><strong>(AG)</strong></em></span></p>
<p><span><em><strong> </strong></em></span></p>
<p><span><strong>Note:</strong> Award <em><strong>(M1)</strong></em> for correct substituted compound interest formula, <em><strong>(A1)</strong></em> for correct substitutions, <em><strong>(A1)</strong></em> for unrounded answer seen.</span></p>
<p><span>If final line not seen award at most <em><strong>(M1)(A1)(A0)</strong></em>.</span></p>
<p><span> </span></p>
<p><em><strong><span>[3 marks]</span></strong></em></p>
<div class="question_part_label">b.</div>
</div>
<div class="question" style="padding-left: 20px;">
<p><span>5000(1.042)<sup><em>n</em> </sup>&gt; 6500     <em><strong>(M1)(A1)</strong></em></span></p>
<p> </p>
<p><span><strong>Notes:</strong> Award <em><strong>(M1)</strong></em> for setting up correct equation/inequality, <em><strong>(A1)</strong></em> for correct values.</span></p>
<p><span>Follow through from their formula in part (b).</span></p>
<p> </p>
<p><strong><span>OR</span></strong></p>
<p><span>List of values seen with at least 2 terms     <em><strong>(M1)</strong></em></span></p>
<p><span>Lists of values including at least the terms with <em>n</em> = 6 and <em>n</em> = 7     <em><strong>(A1)</strong></em></span></p>
<p> </p>
<p><span><strong>Note:</strong> Follow through from their formula in part (b).</span></p>
<p> </p>
<p><strong><span>OR</span></strong></p>
<p><span>Sketch showing 2 graphs, one exponential, the other a horizontal line     <em><strong>(M1)</strong></em></span></p>
<p><span>Point of intersection identified or vertical line     <em><strong>(M1)</strong></em></span></p>
<p><br><span><strong>Note:</strong> Follow through from their formula in part (b).</span></p>
<p><br><span><em>n</em> = 7     <em><strong>(A1)</strong></em><strong>(ft)</strong><em><strong>(G2)</strong></em></span></p>
<p><span> </span></p>
<p><span><em><strong>[3 marks]</strong></em></span></p>
<div class="question_part_label">c.</div>
</div>
<div class="question" style="padding-left: 20px;">
<p><span>5000(1.042)<sup><em>n</em></sup> &gt; 5000 + 230<em>n</em>     <em><strong>(M1)(A1)</strong></em></span></p>
<p> </p>
<p><span><strong>Note:</strong> Award <em><strong>(M1)</strong></em> for setting up correct equation/inequality, <em><strong>(A1)</strong></em> for correct values.</span></p>
<p> </p>
<p><strong><span>OR</span></strong></p>
<p><span>2 lists of values seen (at least 2 terms per list)     <em><strong>(M1)</strong></em></span></p>
<p><span>Lists of values including at least the terms with <em>n</em> = 5 and <em>n</em> = 6     <em><strong>(A1)</strong></em></span></p>
<p><br><span><strong>Note:</strong> One of the lists may be written under (c).</span></p>
<p><br><strong><span>OR</span></strong></p>
<p><span>Sketch showing 2 graphs of correct shape     <em><strong>(M1)</strong></em></span></p>
<p><span>Point of intersection identified or vertical line     <em><strong>(M1)</strong></em></span></p>
<p><span><em>n</em> = 6     <em><strong>(A1)</strong></em><strong>(ft)</strong><em><strong>(G2)</strong></em></span></p>
<p><br><span><strong>Note:</strong> Follow through from their formulae used in parts (a) and (b).</span></p>
<p><span> </span></p>
<p><em><strong><span>[3 marks]</span></strong></em></p>
<div class="question_part_label">d.</div>
</div>
<div class="question" style="padding-left: 20px;">
<p><span>6610 × 0.735     <em><strong>(M1)</strong></em></span></p>
<p><span>= 4858.35     <em><strong>(A1)</strong></em></span></p>
<p><span>4858.35 × 0.982(= 4770.8997...)     <em><strong>(M1)</strong></em></span></p>
<p><span>= 4771 Euros     <em><strong>(A1)</strong></em><strong>(ft)</strong><em><strong>(G3)</strong></em></span></p>
<p> </p>
<p><span><strong>Note:</strong> Accept alternative method.</span></p>
<p><span> </span></p>
<p><em><strong><span>[4 marks]</span></strong></em></p>
<div class="question_part_label">e.</div>
</div>
<div class="question" style="padding-left: 20px;">
<p><span>800 × 1.29 (= 1032 USD)     <em><strong>(M1)(A1)</strong></em></span></p>
<p><br><span><strong>Note:</strong> Award <em><strong>(M1)</strong></em> for multiplying by 1.29, <em><strong>(A1)</strong></em> for 1032. Award <em><strong>(G2)</strong></em> for 1032 if product not seen.</span></p>
<p><br><span>(1032 – 1006.20 = 25.8)</span></p>
<p><span>\(25.8 \times \frac{100}{1032} \%\)     <em><strong>(A1)(M1)</strong></em></span></p>
<p> </p>
<p><span><strong>Note:</strong> Award <em><strong>(A1)</strong></em> for 25.8 seen, <em><strong>(M1)</strong></em> for multiplying by \(\frac{100}{1032}\).</span></p>
<p> </p>
<p><strong><span>OR</span></strong></p>
<p><span>\(\frac{1006.20}{1032} = 0.975\)     <em><strong>(M1)(A1)</strong></em></span></p>
<p><strong><span>OR</span></strong></p>
<p><span>\(\frac{1006.20}{1032} \times 100 = 97.5\)     <em><strong>(M1)(A1)</strong></em></span></p>
<p><span>\( = 2.5{\text{ }}\% \)     <em><strong>(A1)(G3)</strong></em></span></p>
<p> </p>
<p><span><strong>Notes:</strong> If working not shown award <em><strong>(G3)</strong></em> for 2.5.</span></p>
<p><span>Accept alternative method.</span></p>
<p><span> </span></p>
<p><em><strong><span>[5 marks]</span></strong></em></p>
<div class="question_part_label">f.</div>
</div>
<h2 style="margin-top: 1em">Examiners report</h2>
<div class="question" style="padding-left: 20px;">
<p><span style="font-size: medium; font-family: times new roman,times;">Most of the students read carefully the instruction written in the heading of the question and therefore gave their answers with the accuracy stated but some did not. </span></p>
<p><span style="font-size: medium; font-family: times new roman,times;">Simple interest was well done as well as compound interest with only a small minority of candidates making no progress. A number of students lost the answer mark in (b) for not showing the unrounded answer before writing the answer given. It is also important to mention that calculator commands are not accepted as correct working and therefore full marks are not awarded. Also, some candidates wrote their answers without showing any working leading to a number of marks being lost.</span></p>
<div class="question_part_label">a.</div>
</div>
<div class="question" style="padding-left: 20px;">
<p><span style="font-size: medium; font-family: times new roman,times;">Most of the students read carefully the instruction written in the heading of the question and therefore gave their answers with the accuracy stated but some did not. </span></p>
<p><span style="font-size: medium; font-family: times new roman,times;">Simple interest was well done as well as compound interest with only a small minority of candidates making no progress. A number of students lost the answer mark in (b) for not showing the unrounded answer before writing the answer given. It is also important to mention that calculator commands are not accepted as correct working and therefore full marks are not awarded. Also, some candidates wrote their answers without showing any working leading to a number of marks being lost.</span></p>
<div class="question_part_label">b.</div>
</div>
<div class="question" style="padding-left: 20px;">
<p><span style="font-size: medium; font-family: times new roman,times;">Most of the students read carefully the instruction written in the heading of the question and therefore gave their answers with the accuracy stated but some did not. </span></p>
<p><span style="font-size: medium; font-family: times new roman,times;">Simple interest was well done as well as compound interest with only a small minority of candidates making no progress. A number of students lost the answer mark in (b) for not showing the unrounded answer before writing the answer given. It is also important to mention that calculator commands are not accepted as correct working and therefore full marks are not awarded. Also, some candidates wrote their answers without showing any working leading to a number of marks being lost.</span></p>
<div class="question_part_label">c.</div>
</div>
<div class="question" style="padding-left: 20px;">
<p><span style="font-size: medium; font-family: times new roman,times;">Most of the students read carefully the instruction written in the heading of the question and therefore gave their answers with the accuracy stated but some did not. </span></p>
<p><span style="font-size: medium; font-family: times new roman,times;">Simple interest was well done as well as compound interest with only a small minority of candidates making no progress. A number of students lost the answer mark in (b) for not showing the unrounded answer before writing the answer given. It is also important to mention that calculator commands are not accepted as correct working and therefore full marks are not awarded. Also, some candidates wrote their answers without showing any working leading to a number of marks being lost.</span></p>
<p><span style="font-size: medium; font-family: times new roman,times;">It was nice to see many students recovering after part (d) and to gain full marks in the last two parts of the question.</span></p>
<div class="question_part_label">d.</div>
</div>
<div class="question" style="padding-left: 20px;">
<p><span style="font-size: medium; font-family: times new roman,times;">Most of the students read carefully the instruction written in the heading of the question and therefore gave their answers with the accuracy stated but some did not. </span></p>
<p><span style="font-size: medium; font-family: times new roman,times;">Simple interest was well done as well as compound interest with only a small minority of candidates making no progress. A number of students lost the answer mark in (b) for not showing the unrounded answer before writing the answer given. It is also important to mention that calculator commands are not accepted as correct working and therefore full marks are not awarded. Also, some candidates wrote their answers without showing any working leading to a number of marks being lost.</span></p>
<p><span style="font-size: medium; font-family: times new roman,times;">It was nice to see many students recovering after part (d) and to gain full marks in the last two parts of the question.</span></p>
<div class="question_part_label">e.</div>
</div>
<div class="question" style="padding-left: 20px;">
<p><span style="font-size: medium; font-family: times new roman,times;">Most of the students read carefully the instruction written in the heading of the question and therefore gave their answers with the accuracy stated but some did not. </span></p>
<p><span style="font-size: medium; font-family: times new roman,times;">Simple interest was well done as well as compound interest with only a small minority of candidates making no progress. A number of students lost the answer mark in (b) for not showing the unrounded answer before writing the answer given. It is also important to mention that calculator commands are not accepted as correct working and therefore full marks are not awarded. Also, some candidates wrote their answers without showing any working leading to a number of marks being lost.</span></p>
<p><span style="font-size: medium; font-family: times new roman,times;">It was nice to see many students recovering after part (d) and to gain full marks in the last two parts of the question.</span></p>
<div class="question_part_label">f.</div>
</div>
<br><hr><br><div class="specification">
<p><span style="font-family: times new roman,times; font-size: medium;">Daniel wants to invest \(\$ 25\,000\) for a total of three years. There are two investment options.</span></p>
<p><span style="font-family: times new roman,times; font-size: medium;"><strong>Option One</strong>&nbsp; &nbsp; &nbsp;pays compound interest at a nominal annual rate of interest of 5 %, compounded <strong>annually</strong>.</span></p>
<p><span style="font-family: times new roman,times; font-size: medium;"><strong>Option Two</strong> &nbsp; &nbsp; pays compound interest at a nominal annual rate of interest of 4.8 %, compounded <strong>monthly</strong>.</span></p>
</div>

<div class="specification">
<p><span style="font-family: times new roman,times; font-size: medium;">An arithmetic sequence is defined as</span></p>
<p style="text-align: center;"><span style="font-family: times new roman,times; font-size: medium;"><em>u<sub>n</sub></em> = 135 + 7<em>n</em>,&nbsp;&nbsp;&nbsp;&nbsp; <em>n</em> = 1, 2, 3, &hellip;</span></p>
</div>

<div class="question" style="padding-left: 20px; padding-right: 20px;">
<p><span>Calculate the value of his investment at the end of the third year for each investment option, <strong>correct to two decimal places</strong>.</span></p>
<div class="marks">[8]</div>
<div class="question_part_label">A.a.</div>
</div>
<div class="question" style="padding-left: 20px; padding-right: 20px;">
<p><span>Determine Daniel’s best investment option.</span></p>
<div class="marks">[1]</div>
<div class="question_part_label">A.b.</div>
</div>
<div class="question" style="padding-left: 20px; padding-right: 20px;">
<p><span>Calculate <em>u</em><sub>1</sub>, the first term in the sequence.</span></p>
<div class="marks">[2]</div>
<div class="question_part_label">B.a.</div>
</div>
<div class="question" style="padding-left: 20px; padding-right: 20px;">
<p><span>Show that the common difference is 7.</span></p>
<div class="marks">[2]</div>
<div class="question_part_label">B.b.</div>
</div>
<div class="question" style="padding-left: 20px; padding-right: 20px;">
<p><span><em>S<sub>n</sub></em> is the sum of the first <em>n</em> terms of the sequence.</span></p>
<p><span><span>Find an expression for <em>S<sub>n</sub></em>. Give your answer in the form <em>S<sub>n</sub></em></span><span> = <em>An</em><sup>2</sup> + <em>Bn</em>, where <em>A</em> and<em> B</em> are constants.</span></span></p>
<div class="marks">[3]</div>
<div class="question_part_label">B.c.</div>
</div>
<div class="question" style="padding-left: 20px; padding-right: 20px;">
<p><span>The first term, <em>v</em><sub>1</sub>, of a geometric sequence is 20 and its fourth term <em>v</em><sub>4</sub> is 67.5.</span></p>
<p><span>Show that the common ratio, <em>r</em>, of the geometric sequence is 1.5.</span></p>
<div class="marks">[2]</div>
<div class="question_part_label">B.d.</div>
</div>
<div class="question" style="padding-left: 20px; padding-right: 20px;">
<p><span><em>T<sub>n</sub></em> is the sum of the first <em>n</em> terms of the geometric sequence.</span></p>
<p><span>Calculate <em>T</em><sub>7</sub>, the sum of the first seven terms of the geometric sequence.</span></p>
<div class="marks">[2]</div>
<div class="question_part_label">B.e.</div>
</div>
<div class="question" style="padding-left: 20px; padding-right: 20px;">
<p><span><span><em>T</em><sub><span><em>n</em></span></sub> is the sum of the first <em>n</em> terms of the geometric sequence.</span></span></p>
<p><span>Use your graphic display calculator to find the smallest value of <em>n</em> for which <em>T<sub>n</sub></em> &gt; <em>S<sub>n</sub></em>.</span></p>
<div class="marks">[2]</div>
<div class="question_part_label">B.f.</div>
</div>
<h2 style="margin-top: 1em">Markscheme</h2>
<div class="question" style="padding-left: 20px;">
<p><span><strong>Option 1: </strong>    Amount    \( = 25\,000{\left( {1 + \frac{5}{{100}}} \right)^3}\)     <em><strong>(M1)(A1)</strong></em></span></p>
<p><span>= \(28\,940.63\)     <em><strong>(A1)(G2)</strong></em></span></p>
<p><br><span><strong>Note:</strong> Award <em><strong>(M1)</strong></em> for substitution in compound interest formula, <em><strong>(A1)</strong></em> for correct substitution. Give full credit for use of lists.</span></p>
<p><br><span><strong>Option 2:</strong>     Amount     \( = 25\,000{\left( {1 + \frac{{4.8}}{{12(100)}}} \right)^{3 \times 12}}\)     <em><strong>(M1)</strong></em></span></p>
<p><span>= \(28\,863.81\)     <em><strong>(A1)(G2)</strong></em></span></p>
<p><br><span><strong>Note:</strong> Award<em><strong> (M1)</strong></em> for correct substitution in the compound interest formula. Give full credit for use of lists.</span></p>
<p><span> </span></p>
<p><em><strong><span>[8 marks]</span></strong></em></p>
<div class="question_part_label">A.a.</div>
</div>
<div class="question" style="padding-left: 20px;">
<p><span>Option 1 is the best investment option.     <strong><em>(A1)</em>(ft)</strong></span></p>
<p><span><br><em><strong>[1 mark]</strong></em></span></p>
<div class="question_part_label">A.b.</div>
</div>
<div class="question" style="padding-left: 20px;">
<p><span><em>u</em><sub>1</sub> = 135 + 7(1)    <em><strong> (M1)</strong></em></span></p>
<p><span>= 142    <em><strong> (A1)(G2)</strong></em></span></p>
<p><span><em><strong>[2 marks]</strong></em></span></p>
<div class="question_part_label">B.a.</div>
</div>
<div class="question" style="padding-left: 20px;">
<p><span><em>u</em><sub>2</sub> = 135 + 7(2) = 149   <em><strong>  (M1)</strong></em></span></p>
<p><span><em>d</em> = 149 – 142     <strong>OR</strong> <em>alternatives</em>    <strong><em> (M1)</em>(ft)</strong></span></p>
<p><span><em>d</em> = 7   <em><strong>  (AG)</strong></em></span></p>
<p><span><em><strong>[2 marks]</strong></em></span></p>
<div class="question_part_label">B.b.</div>
</div>
<div class="question" style="padding-left: 20px;">
<p><span>\({S_n} = \frac{{n[2(142) + 7(n - 1)]}}{2}\)    <em><strong> (M1)</strong></em><strong>(ft)</strong></span></p>
<p><span><strong> </strong></span></p>
<p><span><strong>Note:</strong> Award <em><strong>(M1)</strong></em> for correct substitution in correct formula.</span></p>
<p><span> </span></p>
<p><span>\( = \frac{{n[277 + 7n]}}{2}\)     <strong>OR</strong> <em>equivalent</em>    <em><strong> (A1)</strong></em></span></p>
<p><span>\( = \frac{{7{n^2}}}{2} + \frac{{277n}}{2}\)     (= 3.5<em>n</em><sup>2 </sup>+ 138.5<em>n</em>)     <em><strong>(A1)(G3)</strong></em></span></p>
<p><span><em><strong>[3 marks]</strong></em></span></p>
<div class="question_part_label">B.c.</div>
</div>
<div class="question" style="padding-left: 20px;">
<p><span>20<em>r</em><sup>3 </sup>= 67.5     <em><strong>(M1)</strong></em></span></p>
<p><span><em>r</em><sup>3</sup> = 3.375     <strong>OR</strong> \(r = \sqrt[3]{{3.375}}\)     <em><strong>(A1)</strong></em></span></p>
<p><span><em>r</em> = 1.5   <em><strong>  (AG)</strong></em></span></p>
<p><span><em><strong>[2 marks]</strong></em></span></p>
<div class="question_part_label">B.d.</div>
</div>
<div class="question" style="padding-left: 20px;">
<p><span>\({T_7} = \frac{{20({{1.5}^7} - 1)}}{{(1.5 - 1)}}\)     <strong><em>(M1)</em></strong></span></p>
<p><br><span><strong>Note:</strong> Award <strong><em>(M1)</em></strong> for correct substitution in correct formula.</span></p>
<p><br><span>= 643<em> (accept 643.4375)  </em>   <strong><em>(A1)(G2)</em></strong></span></p>
<p><span><strong><em>[2 marks]</em></strong></span></p>
<div class="question_part_label">B.e.</div>
</div>
<div class="question" style="padding-left: 20px;">
<p><span>\(\frac{{20({{1.5}^n} - 1)}}{{(1.5 - 1)}} &gt; \frac{{7{n^2}}}{2} + \frac{{277n}}{2}\)     <em><strong>(M1)</strong></em></span></p>
<p><br><span><strong>Note:</strong> Award <em><strong>(M1)</strong></em> for an attempt using lists or for relevant graph.</span></p>
<p><br><span><em>n</em> = 10     <em><strong>(A1)</strong></em><strong>(ft)</strong><em><strong>(G2) </strong></em></span></p>
<p> </p>
<p><span><strong>Note:</strong> Follow through from their (c).</span></p>
<p><span> </span></p>
<p><em><strong><span>[2 marks]</span></strong></em></p>
<div class="question_part_label">B.f.</div>
</div>
<h2 style="margin-top: 1em">Examiners report</h2>
<div class="question" style="padding-left: 20px;">
<p><span style="font-family: times new roman,times; font-size: medium;">For many, this question came as a welcome relief following the previous two questions. For those with a sound grasp of the topic, there were many very successful attempts.</span></p>
<p><span style="font-family: times new roman,times; font-size: medium;">A common error was to make all the comparisons using interest alone; though much credit was given for doing this, candidates should be aware of what is being asked for in the question.</span></p>
<p><span style="font-family: times new roman,times; font-size: medium;">Many did not understand the notion of monthly compounding periods.</span></p>
<div class="question_part_label">A.a.</div>
</div>
<div class="question" style="padding-left: 20px;">
<p><span style="font-family: times new roman,times; font-size: medium;">For many, this question came as a welcome relief following the previous two questions. For those with a sound grasp of the topic, there were many very successful attempts.</span></p>
<p><span style="font-family: times new roman,times; font-size: medium;">A common error was to make all the comparisons using interest alone; though much credit was given for doing this, candidates should be aware of what is being asked for in the question.</span></p>
<p><span style="font-family: times new roman,times; font-size: medium;">Many did not understand the notion of monthly compounding periods.</span></p>
<div class="question_part_label">A.b.</div>
</div>
<div class="question" style="padding-left: 20px;">
<p><span style="font-family: 'times new roman', times; font-size: medium;">For many, this question came as a welcome relief following the previous two questions. For those with a sound grasp of the topic, there were many very successful attempts.</span></p>
<p><span style="font-family: 'times new roman', times; font-size: medium;">A common weakness was seen in the &ldquo;show that&rdquo; parts of the question where, despite a lenient approach to method, many were unable to communicate their thoughts on paper.</span></p>
<p><span style="font-family: 'times new roman', times; font-size: medium;">For many, finding an expression for <em>S<sub>n</sub></em> in (c) was problematical.<br></span></p>
<p><span style="font-family: 'times new roman', times; font-size: medium;">The final part was challenging to the great majority, with a large number not attempting it at all; only the highly competent reached the correct answer.</span></p>
<div class="question_part_label">B.a.</div>
</div>
<div class="question" style="padding-left: 20px;">
<p><span style="font-family: times new roman,times; font-size: medium;">For many, this question came as a welcome relief following the previous two questions. For those with a sound grasp of the topic, there were many very successful attempts.</span></p>
<p><span style="font-family: times new roman,times; font-size: medium;">A common weakness was seen in the &ldquo;show that&rdquo; parts of the question where, despite a lenient approach to method, many were unable to communicate their thoughts on paper.</span></p>
<p><span style="font-family: times new roman,times; font-size: medium;">For many, finding an expression for <em>S<sub>n</sub></em> in (c) was problematical.<br></span></p>
<p><span style="font-family: times new roman,times; font-size: medium;">The final part was challenging to the great majority, with a large number not attempting it at all; only the highly competent reached the correct answer.</span></p>
<div class="question_part_label">B.b.</div>
</div>
<div class="question" style="padding-left: 20px;">
<p><span style="font-family: times new roman,times; font-size: medium;">For many, this question came as a welcome relief following the previous two questions. For those with a sound grasp of the topic, there were many very successful attempts.</span></p>
<p><span style="font-family: times new roman,times; font-size: medium;">A common weakness was seen in the &ldquo;show that&rdquo; parts of the question where, despite a lenient approach to method, many were unable to communicate their thoughts on paper.</span></p>
<p><span style="font-family: times new roman,times; font-size: medium;">For many, finding an expression for <em>S<sub>n</sub></em> in (c) was problematical.<br></span></p>
<p><span style="font-family: times new roman,times; font-size: medium;">The final part was challenging to the great majority, with a large number not attempting it at all; only the highly competent reached the correct answer.</span></p>
<div class="question_part_label">B.c.</div>
</div>
<div class="question" style="padding-left: 20px;">
<p><span style="font-family: times new roman,times; font-size: medium;">For many, this question came as a welcome relief following the previous two questions. For those with a sound grasp of the topic, there were many very successful attempts.</span></p>
<p><span style="font-family: times new roman,times; font-size: medium;">A common weakness was seen in the &ldquo;show that&rdquo; parts of the question where, despite a lenient approach to method, many were unable to communicate their thoughts on paper.</span></p>
<p><span style="font-family: times new roman,times; font-size: medium;">For many, finding an expression for <em>S<sub>n</sub></em> in (c) was problematical.<br></span></p>
<p><span style="font-family: times new roman,times; font-size: medium;">The final part was challenging to the great majority, with a large number not attempting it at all; only the highly competent reached the correct answer.</span></p>
<div class="question_part_label">B.d.</div>
</div>
<div class="question" style="padding-left: 20px;">
<p><span style="font-family: times new roman,times; font-size: medium;">For many, this question came as a welcome relief following the previous two questions. For those with a sound grasp of the topic, there were many very successful attempts.</span></p>
<p><span style="font-family: times new roman,times; font-size: medium;">A common weakness was seen in the &ldquo;show that&rdquo; parts of the question where, despite a lenient approach to method, many were unable to communicate their thoughts on paper.</span></p>
<p><span style="font-family: times new roman,times; font-size: medium;">For many, finding an expression for <em>S<sub>n</sub></em> in (c) was problematical.<br></span></p>
<p><span style="font-family: times new roman,times; font-size: medium;">The final part was challenging to the great majority, with a large number not attempting it at all; only the highly competent reached the correct answer.</span></p>
<div class="question_part_label">B.e.</div>
</div>
<div class="question" style="padding-left: 20px;">
<p><span style="font-family: times new roman,times; font-size: medium;">For many, this question came as a welcome relief following the previous two questions. For those with a sound grasp of the topic, there were many very successful attempts.</span></p>
<p><span style="font-family: times new roman,times; font-size: medium;">A common weakness was seen in the &ldquo;show that&rdquo; parts of the question where, despite a lenient approach to method, many were unable to communicate their thoughts on paper.</span></p>
<p><span style="font-family: times new roman,times; font-size: medium;">For many, finding an expression for <em>S<sub>n</sub></em> in (c) was problematical.<br></span></p>
<p><span style="font-family: times new roman,times; font-size: medium;">The final part was challenging to the great majority, with a large number not attempting it at all; only the highly competent reached the correct answer.</span></p>
<div class="question_part_label">B.f.</div>
</div>
<br><hr><br><div class="specification">
<p class="p1"><span class="s1">A cup of boiling water is placed in a room to cool. The temperature of the room is 20&deg;C </span>. This situation can be modelled by the exponential function \(T = a + b({k^{ - m}})\), where \(T\) is the temperature of the water, in &deg;<span class="s1">C </span>, and \(m\) is the number of minutes for which the cup has been placed in the room. A sketch of the situation is given as follows.</p>
<p class="p1" style="text-align: center;"><img src="images/Schermafbeelding_2015-12-21_om_09.45.14.png" alt></p>
</div>

<div class="question" style="padding-left: 20px; padding-right: 20px;">
<p class="p1">Explain why \(a = 20\).</p>
<div class="marks">[2]</div>
<div class="question_part_label">a.</div>
</div>
<div class="question" style="padding-left: 20px; padding-right: 20px;">
<p class="p1"><span class="s1">Initially, at \(m = 0\)</span>, the temperature of the water is 100°C<span class="s1">.</span></p>
<p class="p2">Find the value of \(b\).</p>
<div class="marks">[2]</div>
<div class="question_part_label">b.</div>
</div>
<div class="question" style="padding-left: 20px; padding-right: 20px;">
<p class="p1">After being placed in the room for one minute, the temperature of the water is 84°C<span class="s1">.</span></p>
<p class="p2">Show that \(k = 1.25\).</p>
<div class="marks">[2]</div>
<div class="question_part_label">c.</div>
</div>
<div class="question" style="padding-left: 20px; padding-right: 20px;">
<p class="p1">After being placed in the room for one minute, the temperature of the water is 84°C<span class="s1">.</span></p>
<p class="p1">Find the temperature of the water three minutes after it has been placed in the room.</p>
<div class="marks">[2]</div>
<div class="question_part_label">d.</div>
</div>
<div class="question" style="padding-left: 20px; padding-right: 20px;">
<p class="p1">After being placed in the room for one minute, the temperature of the water is 84°C<span class="s1">.</span></p>
<p class="p1">Find the total time needed for the water to reach a temperature of <span class="s1">35°</span><span class="s1">C</span>. Give your answer in minutes and seconds, correct to the nearest second.</p>
<div class="marks">[2]</div>
<div class="question_part_label">e.</div>
</div>
<h2 style="margin-top: 1em">Markscheme</h2>
<div class="question" style="padding-left: 20px;">
<p class="p1">the temperature of the water cannot fall below room temperature <span class="Apple-converted-space">    </span><strong><em>(R1)</em></strong></p>
<p class="p1">an (informal) explanation that as \(m \to \infty ,{\text{ }}{k^{ - m}} \to 0\) <span class="Apple-converted-space">    </span><strong><em>R1)</em></strong></p>
<p class="p1"> </p>
<p class="p1"><strong>OR</strong></p>
<p class="p1">recognition that there is a horizontal asymptote at \(y = a\) <span class="Apple-converted-space">    </span><strong><em>(R1)</em></strong></p>
<p class="p1"><strong>Note: </strong>Award <strong><em>(R1) </em></strong>for a contextual reason involving room temperature.</p>
<p class="p1">Award <strong><em>(R1) </em></strong>for a mathematical reason similar to one of the two alternatives.</p>
<div class="question_part_label">a.</div>
</div>
<div class="question" style="padding-left: 20px;">
<p class="p1">\(100 = 20 + b({k^0})\) <span class="Apple-converted-space">    </span><strong><em>(M1)</em></strong></p>
<p class="p1"><strong>Note: </strong>Award <strong><em>(M1) </em></strong>for substituting \(100\), \(20\)<span class="s1"> </span>and \(0\).</p>
<p class="p2"> </p>
<p class="p1">\(b = 80\) <span class="Apple-converted-space">    </span><strong><em>(A1)(G2)</em></strong></p>
<p class="p1"><strong>Note: </strong>The <strong><em>(A1) </em></strong>is awarded only if all working seen is consistent with the final answer of \(80\).</p>
<div class="question_part_label">b.</div>
</div>
<div class="question" style="padding-left: 20px;">
<p class="p1">\(84 = 20 + 80{k^{ - 1}}\) <span class="Apple-converted-space">    </span><strong><em>(M1)</em></strong></p>
<p class="p1"><strong>Note: </strong>Substituting \(k = 1.25\) at any stage is an invalid method and is awarded <strong><em>(M0)(M0)</em></strong>. Award <strong><em>(M1) </em></strong>for correctly substituting \(84\), \(20\)<span class="s1"> </span>and their \(80\)<span class="s1">.</span></p>
<p class="p3"> </p>
<p class="p1">\(\frac{{64}}{{80}} = {k^{ - 1}}\) <span class="Apple-converted-space">    </span><strong><em>(M1)</em></strong></p>
<p class="p1">\(k = 1.25\) <span class="Apple-converted-space">    </span><strong><em>(AG)</em></strong></p>
<p class="p1"><strong>Note:<span class="Apple-converted-space"> </span></strong>Award <strong><em>(M1) </em></strong>for correct rearrangement that isolates \(k\); \(k = 1.25\) must be consistent with their working <strong>and </strong>the conclusion \(k = 1.25\) must be seen.</p>
<div class="question_part_label">c.</div>
</div>
<div class="question" style="padding-left: 20px;">
<p class="p1">\(T = 20 + 80({1.25^{ - 3}})\) <span class="Apple-converted-space">    </span><strong><em>(M1)</em></strong></p>
<p class="p1"><strong>Note: </strong>Award <strong><em>(M1) </em></strong>for their correct substitutions into \(T\). Follow through from part (b) and \(k = 1.25\).</p>
<p class="p2"> </p>
<p class="p1">\(T = 61.0\;\;\;(60.96)\) <span class="Apple-converted-space">    </span><strong><em>(A1)</em>(ft)<em>(G2)</em></strong></p>
<div class="question_part_label">d.</div>
</div>
<div class="question" style="padding-left: 20px;">
<p class="p1">\(35 = 20 + 80({1.25^{ - m}})\) <span class="Apple-converted-space">    </span><strong><em>(M1)</em></strong></p>
<p class="p1"><strong>Note: </strong>Award <strong><em>(M1) </em></strong>for their correct substitutions into \(T\). Follow through from part (b). Accept graphical solutions. Award <strong><em>(M1) </em></strong>for sketch of function.</p>
<p class="p2"> </p>
<p class="p1">\((m = ){\text{ }}7.50{\text{ (minutes)}}\;\;\;{\text{(7.50179}} \ldots {\text{)}}\) <span class="Apple-converted-space">    </span><strong><em>(A1)</em>(ft)<em>(G2)</em></strong></p>
<p class="p1"><span class="s1">7 </span>minutes and <span class="s1">30 </span>seconds <span class="Apple-converted-space">    </span><strong><em>(A1)</em></strong></p>
<p class="p1"><strong>Note:<span class="Apple-converted-space"> </span></strong>Award the final <strong><em>(A1) </em></strong>for correct conversion of <strong>their</strong> \(m\) in minutes to minutes and seconds, but only if answer in minutes is explicitly shown.</p>
<div class="question_part_label">e.</div>
</div>
<h2 style="margin-top: 1em">Examiners report</h2>
<div class="question" style="padding-left: 20px;">
<p>Comments on some of the G2 forms indicated that teachers felt the presence of three parameters in the formula was inconsistent with the aims of Mathematical Studies; however, one parameter was given and a justification required, a second required knowledge only of k<sup>0</sup>=1 and the third was also given. Candidates should have been exposed to graphs of this type in their classwork.</p>
<p>The response of the candidature indicated that most were able to make some progress with the question, though for many the &ldquo;show that&rdquo; part was not attempted. As in previous sessions, comments were made about the use (or not) of logarithms in the final part &ndash; logarithms are not part of the Mathematical Studies SL syllabus (however, their correct use is never penalized), but efficient use of the GDC is very much part of a candidate&rsquo;s &ldquo;toolbox&rdquo;. Questions of this nature &ndash; essentially requiring the use of the GDC as part of a problem solving exercise &ndash; will continue to be set.</p>
<div class="question_part_label">a.</div>
</div>
<div class="question" style="padding-left: 20px;">
<p>Comments on some of the G2 forms indicated that teachers felt the presence of three parameters in the formula was inconsistent with the aims of Mathematical Studies; however, one parameter was given and a justification required, a second required knowledge only of k<sup>0</sup>=1 and the third was also given. Candidates should have been exposed to graphs of this type in their classwork.</p>
<p>The response of the candidature indicated that most were able to make some progress with the question, though for many the &ldquo;show that&rdquo; part was not attempted. As in previous sessions, comments were made about the use (or not) of logarithms in the final part &ndash; logarithms are not part of the Mathematical Studies SL syllabus (however, their correct use is never penalized), but efficient use of the GDC is very much part of a candidate&rsquo;s &ldquo;toolbox&rdquo;. Questions of this nature &ndash; essentially requiring the use of the GDC as part of a problem solving exercise &ndash; will continue to be set.</p>
<div class="question_part_label">b.</div>
</div>
<div class="question" style="padding-left: 20px;">
<p>Comments on some of the G2 forms indicated that teachers felt the presence of three parameters in the formula was inconsistent with the aims of Mathematical Studies; however, one parameter was given and a justification required, a second required knowledge only of k<sup>0</sup>=1 and the third was also given. Candidates should have been exposed to graphs of this type in their classwork.</p>
<p>The response of the candidature indicated that most were able to make some progress with the question, though for many the &ldquo;show that&rdquo; part was not attempted. As in previous sessions, comments were made about the use (or not) of logarithms in the final part &ndash; logarithms are not part of the Mathematical Studies SL syllabus (however, their correct use is never penalized), but efficient use of the GDC is very much part of a candidate&rsquo;s &ldquo;toolbox&rdquo;. Questions of this nature &ndash; essentially requiring the use of the GDC as part of a problem solving exercise &ndash; will continue to be set.</p>
<div class="question_part_label">c.</div>
</div>
<div class="question" style="padding-left: 20px;">
<p>Comments on some of the G2 forms indicated that teachers felt the presence of three parameters in the formula was inconsistent with the aims of Mathematical Studies; however, one parameter was given and a justification required, a second required knowledge only of k<sup>0</sup>=1 and the third was also given. Candidates should have been exposed to graphs of this type in their classwork.</p>
<p>The response of the candidature indicated that most were able to make some progress with the question, though for many the &ldquo;show that&rdquo; part was not attempted. As in previous sessions, comments were made about the use (or not) of logarithms in the final part &ndash; logarithms are not part of the Mathematical Studies SL syllabus (however, their correct use is never penalized), but efficient use of the GDC is very much part of a candidate&rsquo;s &ldquo;toolbox&rdquo;. Questions of this nature &ndash; essentially requiring the use of the GDC as part of a problem solving exercise &ndash; will continue to be set.</p>
<div class="question_part_label">d.</div>
</div>
<div class="question" style="padding-left: 20px;">
<p>Comments on some of the G2 forms indicated that teachers felt the presence of three parameters in the formula was inconsistent with the aims of Mathematical Studies; however, one parameter was given and a justification required, a second required knowledge only of k<sup>0</sup>=1 and the third was also given. Candidates should have been exposed to graphs of this type in their classwork.</p>
<p>The response of the candidature indicated that most were able to make some progress with the question, though for many the &ldquo;show that&rdquo; part was not attempted. As in previous sessions, comments were made about the use (or not) of logarithms in the final part &ndash; logarithms are not part of the Mathematical Studies SL syllabus (however, their correct use is never penalized), but efficient use of the GDC is very much part of a candidate&rsquo;s &ldquo;toolbox&rdquo;. Questions of this nature &ndash; essentially requiring the use of the GDC as part of a problem solving exercise &ndash; will continue to be set.</p>
<div class="question_part_label">e.</div>
</div>
<br><hr><br><div class="question" style="padding-left: 20px; padding-right: 20px;">
<p>Antonio and Barbara start work at the same company on the same day. They each earn an annual salary of \(8000\) euros during the first year of employment. The company gives them a salary increase following the completion of each year of employment. Antonio is paid using plan A and Barbara is paid using plan B.</p>
<p>Plan A: The annual salary increases by \(450\) euros each year.</p>
<p>Plan B: The annual salary increases by \(5\,\% \) each year.</p>
<p>Calculate</p>
<p>i)     Antonio’s annual salary during his second year of employment;</p>
<p>ii)    Barbara’s annual salary during her second year of employment.</p>
<div class="marks">[3]</div>
<div class="question_part_label">a.</div>
</div>
<div class="question" style="padding-left: 20px; padding-right: 20px;">
<p>Write down an expression for</p>
<p>i)     Antonio’s annual salary during his \(n\) th year of employment;</p>
<p>ii)    Barbara’s annual salary during her \(n\) th year of employment.</p>
<div class="marks">[4]</div>
<div class="question_part_label">b.</div>
</div>
<div class="question" style="padding-left: 20px; padding-right: 20px;">
<p>Determine the number of years for which Antonio’s annual salary is greater than or equal to Barbara’s annual salary.</p>
<div class="marks">[2]</div>
<div class="question_part_label">c.</div>
</div>
<div class="question" style="padding-left: 20px; padding-right: 20px;">
<p>Both Antonio and Barbara plan to work at the company for a total of \(15\) years.</p>
<p>i)     Calculate the <strong>total amount</strong> that <strong>Barbara</strong> will be paid during these \(15\) years.</p>
<p>ii)    Determine whether Antonio earns more than Barbara during these \(15\) years.</p>
<div class="marks">[7]</div>
<div class="question_part_label">d.</div>
</div>
<h2 style="margin-top: 1em">Markscheme</h2>
<div class="question" style="padding-left: 20px;">
<p>i)     \(8450\,({\text{euro}})\)       <em><strong>(A1)</strong></em></p>
<p> </p>
<p>ii)    \(8000 \times 1.05\)       <em><strong>(M1)</strong></em></p>
<p><strong>Note:</strong> Award <em><strong>(M1)</strong></em> for \(8000 \times 1.05\)  <strong>OR</strong> \(\left( {8000 \times 0.05} \right) + 8000.\)</p>
<p>\( = 8400\,({\text{euro}})\)       <em><strong>(A1)(G3)</strong></em></p>
<div class="question_part_label">a.</div>
</div>
<div class="question" style="padding-left: 20px;">
<p>i)     \(8000 + 450\,\left( {n - 1} \right)\,\,\,\left( {{\text{accept}}\,\,450\,n + 7550} \right)\)        <em><strong>(M1)(A1)</strong></em></p>
<p><strong>Note:</strong> Award <em><strong>(M1)</strong></em> for substitution in arithmetic sequence formula; <em><strong>(A1)</strong></em> for correct substitutions.</p>
<p> </p>
<p>ii)    \(8000 \times {1.05^{\left( {n - 1} \right)\,}}\)        <em><strong>(M1)(A1)</strong></em></p>
<p><strong>Note:</strong> Award <em><strong>(M1)</strong></em> for substitution in arithmetic sequence formula; <em><strong>(A1)</strong></em> for correct substitutions.</p>
<p> </p>
<div class="question_part_label">b.</div>
</div>
<div class="question" style="padding-left: 20px;">
<p>\(8000 + 450\,(n - 1) \geqslant 8000 \times {1.05^{n - 1}}\)        <em><strong>(M1)</strong></em></p>
<p><strong>Note:</strong> Award <em><strong>(M1)</strong></em> for setting a correct inequality using their expressions for (b)(i) and (b)(ii). Accept an equation.</p>
<p><strong>OR</strong></p>
<p>list of at least 4 correct terms of each sequence        <em><strong>(M1)</strong></em></p>
<p><strong>Note:</strong> Award <em><strong>(M1)</strong></em> for correct lists corresponding to their answers for parts (b)(i) and (b)(ii).</p>
<p>\(6\)        <em><strong>(A1)</strong></em><strong>(ft)</strong><em><strong>(G2)</strong></em></p>
<p><strong>Note:</strong> Value must be an integer for the final <em><strong>(A1)</strong></em> to be awarded. Follow through from parts (b)(i) and (b)(ii). Award <em><strong>(G1)</strong></em> for a final answer of \(6.70018...\) seen without working.</p>
<div class="question_part_label">c.</div>
</div>
<div class="question" style="padding-left: 20px;">
<p>i)     \({S_{15}} = \frac{{8000 \times \left( {{{1.05}^{15}} - 1} \right)}}{{1.05 - 1}}\)         <em><strong>(M1)(A1)</strong></em><strong>(ft)</strong></p>
<p><strong>Note:</strong> Award <em><strong>(M1)</strong></em> for substitution into geometric series formula and <em><strong>(A1)</strong></em> for correct substitution of \({u_1}\) and their \(r\) from part (b)(ii). Follow through from part (b)(ii).</p>
<p><strong>OR</strong></p>
<p>\(8000 + 8400 + 8820... + 15839.45\)        <em><strong>(M1)(A1)</strong></em><strong>(ft)</strong></p>
<p><strong>Note:</strong> Follow through from part (b)(ii).</p>
<p>\( = 173\,000\,({\text{euro}})\,\,\,(172629...)\)         <em><strong>(A1)</strong></em><strong>(ft)<em>(G2)</em></strong></p>
<p> </p>
<p>ii)    \({S_{15}} = \frac{{15}}{2}\left( {2 \times 8000 + 450 \times 14} \right)\)         <em><strong>(M1)(A1)</strong></em><strong>(ft)</strong></p>
<p><strong>Note:</strong> Award <em><strong>(M1)</strong></em> for substitution into arithmetic series formula and <em><strong>(A1</strong></em>) for correct substitution, using their first term and their last term from part (b)(i), or their \({u_1}\) and \(d\). Follow through from part (b)(i).</p>
<p><strong>OR</strong></p>
<p>\(8000 + 8450 + 8900... + 14300\)          <em><strong>(M1)(A1)</strong></em><strong>(ft)</strong></p>
<p><strong>Note:</strong> Follow through from part (b)(i).</p>
<p>\( = 167\,000\,({\text{euro}})\,\,\,(167\,250)\)         <em><strong>(A1)</strong></em><strong>(ft)<em>(G2)</em></strong></p>
<p>Antonio does not earn more than Barbara</p>
<p>(his total salary will be less than Barbara’s)         <em><strong>(A1)</strong></em><strong>(ft)</strong></p>
<p><strong>Note:</strong> Award <em><strong>(A1)</strong></em><strong>(ft)</strong> for a final answer that is consistent with their part (d)(i) and (d)(ii). Accept “Barbara earns more”. The final <em><strong>(A1)</strong></em> can only be awarded if two total salaries are seen.</p>
<p> </p>
<div class="question_part_label">d.</div>
</div>
<h2 style="margin-top: 1em">Examiners report</h2>
<div class="question" style="padding-left: 20px;">
<p>Question 5: Arithmetic and Geometric progression<br>Most candidates calculated the salaries in the second year correctly. The most common error was to calculate the salaries for the third instead of the second year. In part (b) the use of \(n\) instead of \(n - 1\) &nbsp;was very common. For the geometric sequence often a ratio of 0.05 instead of 1.05 was used. Also many of the expressions given did not represent a geometric sequence. Candidates who used a list for part (c) did usually better than the ones that tried to solve an equation. In part (d) the sum of the arithmetic progression was done better than the geometric series. Many candidates calculated the 15th term of the progression and not the series. In general this question part was not answered well.</p>
<div class="question_part_label">a.</div>
</div>
<div class="question" style="padding-left: 20px;">
<p>Question 5: Arithmetic and Geometric progression</p>
<p>Most candidates calculated the salaries in the second year correctly. The most common error was to calculate the salaries for the third instead of the second year. In part (b) the use of&nbsp;\(n\)&nbsp;instead of&nbsp;\(n - 1\) &nbsp;was very common. For the geometric sequence often a ratio of 0.05 instead of 1.05 was used. Also many of the expressions given did not represent a geometric sequence. Candidates who used a list for part (c) did usually better than the ones that tried to solve an equation. In part (d) the sum of the arithmetic progression was done better than the geometric series. Many candidates calculated the 15th term of the progression and not the series. In general this question part was not answered well.</p>
<div class="question_part_label">b.</div>
</div>
<div class="question" style="padding-left: 20px;">
<p>Question 5: Arithmetic and Geometric progression</p>
<p>Most candidates calculated the salaries in the second year correctly. The most common error was to calculate the salaries for the third instead of the second year. In part (b) the use of&nbsp;\(n\)&nbsp;instead of&nbsp;\(n - 1\) &nbsp;was very common. For the geometric sequence often a ratio of 0.05 instead of 1.05 was used. Also many of the expressions given did not represent a geometric sequence. Candidates who used a list for part (c) did usually better than the ones that tried to solve an equation. In part (d) the sum of the arithmetic progression was done better than the geometric series. Many candidates calculated the 15th term of the progression and not the series. In general this question part was not answered well.</p>
<div class="question_part_label">c.</div>
</div>
<div class="question" style="padding-left: 20px;">
<p>Question 5: Arithmetic and Geometric progression</p>
<p>Most candidates calculated the salaries in the second year correctly. The most common error was to calculate the salaries for the third instead of the second year. In part (b) the use of&nbsp;\(n\)&nbsp;instead of&nbsp;\(n - 1\) &nbsp;was very common. For the geometric sequence often a ratio of 0.05 instead of 1.05 was used. Also many of the expressions given did not represent a geometric sequence. Candidates who used a list for part (c) did usually better than the ones that tried to solve an equation. In part (d) the sum of the arithmetic progression was done better than the geometric series. Many candidates calculated the 15th term of the progression and not the series. In general this question part was not answered well.</p>
<div class="question_part_label">d.</div>
</div>
<br><hr><br><div class="specification">
<p><span style="font-size: medium; font-family: times new roman,times;">Given \(f (x) = x^2 &minus; 3x^{&minus;1}, x \in {\mathbb{R}}, - 5 \leqslant x \leqslant 5, x \ne 0\),</span></p>
</div>

<div class="specification">
<p><span style="font-size: medium; font-family: times new roman,times;">A football is kicked from a point A (a, 0), 0 &lt; a &lt; 10 on the ground towards a goal to the right of A.</span></p>
<p><span style="font-size: medium; font-family: times new roman,times;">The ball follows a path that can be modelled by <strong>part</strong> of the graph</span></p>
<p style="margin-left: 30px;"><span style="font-size: medium; font-family: times new roman,times;">\(y = &minus; 0.021x^2 + 1.245x &minus; 6.01, x \in {\mathbb{R}}, y \geqslant&nbsp; 0\).</span></p>
<p><span style="font-size: medium; font-family: times new roman,times;"><em>x</em> is the horizontal distance of the ball from the origin</span></p>
<p><span style="font-size: medium; font-family: times new roman,times;"><em>y</em> is the height above the ground</span></p>
<p><span style="font-size: medium; font-family: times new roman,times;">Both <em>x</em> and <em>y</em> are measured in metres.</span></p>
</div>

<div class="question" style="padding-left: 20px; padding-right: 20px;">
<p><span>Write down the equation of the vertical asymptote.</span></p>
<div class="marks">[1]</div>
<div class="question_part_label">i.a.</div>
</div>
<div class="question" style="padding-left: 20px; padding-right: 20px;">
<p><span>Find \(f ′(x)\).</span></p>
<div class="marks">[2]</div>
<div class="question_part_label">i.b.</div>
</div>
<div class="question" style="padding-left: 20px; padding-right: 20px;">
<p><span>Using your graphic display calculator or otherwise, write down the coordinates of any point where the graph of \(y = f (x)\) has zero gradient.</span></p>
<div class="marks">[2]</div>
<div class="question_part_label">i.c.</div>
</div>
<div class="question" style="padding-left: 20px; padding-right: 20px;">
<p><span>Write down all intervals in the given domain for which \(f (x)\) is increasing.</span></p>
<div class="marks">[3]</div>
<div class="question_part_label">i.d.</div>
</div>
<div class="question" style="padding-left: 20px; padding-right: 20px;">
<p><span>Using your graphic display calculator or otherwise, find the value of <em>a</em>.</span></p>
<div class="marks">[1]</div>
<div class="question_part_label">ii.a.</div>
</div>
<div class="question" style="padding-left: 20px; padding-right: 20px;">
<p><span>Find \(\frac{{dy}}{{dx}}\).</span></p>
<div class="marks">[2]</div>
<div class="question_part_label">ii.b.</div>
</div>
<div class="question" style="padding-left: 20px; padding-right: 20px;">
<p><span>(i) Use your answer to part (b) to calculate the horizontal distance the ball has travelled from A when its height is a maximum.</span></p>
<p><span>(ii) Find the maximum vertical height reached by the football.</span></p>
<div class="marks">[4]</div>
<div class="question_part_label">ii.c.</div>
</div>
<div class="question" style="padding-left: 20px; padding-right: 20px;">
<p><span>Draw a graph showing the path of the football from the point where it is kicked to the point where it hits the ground again. Use 1 cm to represent 5 m on the horizontal axis and 1 cm to represent 2 m on the vertical scale.</span></p>
<div class="marks">[4]</div>
<div class="question_part_label">ii.d.</div>
</div>
<div class="question" style="padding-left: 20px; padding-right: 20px;">
<p><span>The goal posts are 35 m from <strong>the point where the ball is kicked</strong>.</span></p>
<p><span>At what height does the ball pass over the goal posts?</span></p>
<div class="marks">[2]</div>
<div class="question_part_label">ii.e.</div>
</div>
<h2 style="margin-top: 1em">Markscheme</h2>
<div class="question" style="padding-left: 20px;">
<p><span>equation of asymptote is <em>x</em> = 0     <em><strong>(A1)</strong></em></span></p>
<p><em><span>(Must be an equation.)</span></em></p>
<p><strong><em><span>[1 mark]</span></em></strong></p>
<p> </p>
<div class="question_part_label">i.a.</div>
</div>
<div class="question" style="padding-left: 20px;">
<p><span>\(f '(x) = 2x + 3x^{-2}\)     <em>(or equivalent)     <strong>(A1)</strong> for each term     <strong>(A1)(A1)</strong></em></span></p>
<p><span><em><strong>[2 marks]</strong></em></span></p>
<div class="question_part_label">i.b.</div>
</div>
<div class="question" style="padding-left: 20px;">
<p><span>stationary point (–1.14, 3.93)     <em><strong>(G1)(G1)</strong></em><strong>(ft)</strong></span></p>
<p><span><em>(-1,4) or similar error is awarded <strong>(G0)(G1)</strong></em><strong>(ft)</strong>. <em>Here and also as follow through in part (d) accept exact values</em> \( - {\left( {\frac{3}{2}} \right)^{\frac{1}{3}}}\)<em>for the x coordinate and </em>\(3{\left( {\frac{3}{2}} \right)^{\frac{2}{3}}}\) <em>for the y</em> <em>coordinate.</em></span></p>
<p><span><strong>OR</strong> \(2x + \frac{3}{{{x^2}}} = 0\) </span><span><em>or equivalent</em></span></p>
<p><span>Correct coordinates as above</span>    <span> <em><strong>(M1)</strong></em></span></p>
<p><span><em>Follow through from candidate’s</em> \(f ′(x)\).     <em><strong>(A1)</strong></em><strong>(ft)</strong></span></p>
<p><em><span><strong>[2 marks]</strong></span></em></p>
<div class="question_part_label">i.c.</div>
</div>
<div class="question" style="padding-left: 20px;">
<p><em><span>In all alternative answers for (d), follow through from</span> <span>candidate’s x coordinate in part (c).</span></em></p>
<p><span>Alternative answers include:</span></p>
<p><span>–1.14 ≤ <em>x</em> &lt; 0,     0 &lt; <em>x</em> &lt; 5     <em><strong>(A1)(A1)</strong></em><strong>(ft)</strong><em><strong>(A1)</strong></em></span></p>
<p><span><strong>OR</strong> [</span><span><span>–</span>1.14,0), (0,5)</span></p>
<p><span><em>Accept alternative bracket notation for open interval ] [. (Union of these sets is not correct, award <strong>(A2)</strong> if all else is right in this case.)</em></span></p>
<p><span><strong>OR</strong> \( - 1.14 \leqslant x &lt; 5,x \ne 0\)</span></p>
<p><span><em>In all versions 0 <strong>must</strong> be excluded <strong>(A1)</strong>. -1.14 must be the left bound </em><em>. 5 must be the right bound <strong>(A1)</strong>. For </em>\(x \geqslant - 1.14\)</span><em><span> or </span></em><span>\(x &gt; - 1.14\)</span><em><span> alone, award <strong>(A1)</strong>. For </span></em><span>\( - 1.4 \leqslant x &lt; 0\) </span><em><span>together with</span></em><span> \(x &gt; 0\)</span><em><span> award <strong>(A2)</strong>.</span></em></p>
<p><strong><em><span>[3 marks]</span></em></strong></p>
<p> </p>
<div class="question_part_label">i.d.</div>
</div>
<div class="question" style="padding-left: 20px;">
<p><span><em>a</em> = 5.30 <em>(3sf)</em>   <em>(Allow (5.30, 0) but 5.3 receives an</em> <strong><em>(AP)</em></strong><em>.)</em></span>     <em><strong><span>(A1)</span></strong></em></p>
<p><em><strong><span>[1 mark]</span></strong></em></p>
<div class="question_part_label">ii.a.</div>
</div>
<div class="question" style="padding-left: 20px;">
<p><span>\(\frac{{{\text{d}}y}}{{{\text{d}}x}} = - 0.042x + 1.245\)   <em>  <strong>(A1)</strong> for each term.     <strong>(A1)(A1)</strong></em></span></p>
<p><span><em><strong>[2 marks]</strong></em></span></p>
<div class="question_part_label">ii.b.</div>
</div>
<div class="question" style="padding-left: 20px;">
<p><em><span>Unit penalty <strong>(UP)</strong> is applicable where indicated in the left hand column.</span></em></p>
<p> </p>
<p><span>(i) Maximum value when \(f ' (x) = 0\), \( - 0.042x + 1.245 = 0\),     <em><strong>(M1)</strong></em></span></p>
<p><em><span><strong>(M1)</strong> is for either of the above but at least one must be</span> <span>seen.</span></em></p>
<p><span>(<em>x</em> = 29.6.)</span></p>
<p><span>Football has travelled 29.6 – 5.30 = 24.3 m (3sf)</span> <span>horizontally.     <em><strong>(A1)</strong></em><strong>(ft)</strong></span></p>
<p><em><span>For answer of 24.3 m with no working or for correct</span> <span>subtraction of 5.3 from candidate’s x-coordinate at</span> <span>the maximum (if not 29.6), award <strong>(A1)</strong>(d).</span></em></p>
<p><br><span><strong><em>(UP)</em></strong> (ii) Maximum vertical height, <em>f</em> (29.6) = 12.4 m     <strong><em>(M1)(A1)</em>(ft)<em>(G2)</em></strong></span></p>
<p><em><span><strong>(M1)</strong> is for substitution into f of a value seen in part (c)(i).</span> <span>f(24.3) with or without evaluation is awarded <strong>(M1)(A0)</strong>.</span> <span>For any other value without working, award <strong>(G0)</strong>. </span><span>If lines are seen on the graph in part (d) award <strong>(M1)</strong></span> </em><span><em>and then <strong>(A1)</strong> for candidate’s value</em> \( \pm 0.5\)<em> (3sf not</em></span><em> <span>required.)</span></em></p>
<p> </p>
<p><strong><em><span>[4 marks]</span></em></strong></p>
<div class="question_part_label">ii.c.</div>
</div>
<div class="question" style="padding-left: 20px;">
<p><span>(not to scale)</span></p>
<p><span><img src="data:image/png;base64,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" alt>     <strong><em>(A1)(A1)(A1)</em>(ft)<em>(A1)</em>(ft)</strong></span></p>
<p><em><span>Award <strong>(A1)</strong> for labels (units not required) and scale,</span></em><span> </span><span><strong><em>(A1)</em>(ft)</strong></span><em><span> </span></em><em><span>for max</span></em><span>(29.6,12.4), </span><span> <strong><em>(A1)</em>(ft) </strong></span><span>for</span><em><span> x-intercepts at 5.30 and 53.9, (all coordinates can be within 0.5), <strong>(A1)</strong> for well-drawn parabola ending at the x-intercepts.</span></em></p>
<p><strong><em><span>[4 marks]</span></em></strong></p>
<div class="question_part_label">ii.d.</div>
</div>
<div class="question" style="padding-left: 20px;">
<p><span><span><em></em></span></span></p>
<p><em><span>Unit penalty <strong>(UP)</strong> is applicable where indicated in the left hand column.</span></em></p>
<p><span><em><strong>(UP)</strong> f</em> (40.3) = 10.1 m (3sf).</span></p>
<p><em><span>Follow through from (a).</span> <span>If graph used, award <strong>(M1)</strong> for lines drawn and <strong>(A1)</strong> for</span> <span>candidate’s value </span></em><span>\( \pm 0.5\)</span><em><span>. (3sf not required).    </span><strong><span> (M1)(A1)</span></strong></em><span><strong>(ft)<em>(G2)</em></strong></span></p>
<p><span><strong><em>[2 marks]</em></strong></span></p>
<div class="question_part_label">ii.e.</div>
</div>
<h2 style="margin-top: 1em">Examiners report</h2>
<div class="question" style="padding-left: 20px;">
<p><span style="font-size: medium; font-family: times new roman,times;">(i) An attempt at part (a) was seen only rarely. If there was an attempt, it was often not a meaningful equation. If an equation was seen, sometimes it was for <em>y</em>, not <em>x</em>.</span></p>
<p>&nbsp;</p>
<div class="question_part_label">i.a.</div>
</div>
<div class="question" style="padding-left: 20px;">
<p><span style="font-size: medium; font-family: times new roman,times;">The derivative seemed manageable for many, though with the expected mis-handling of the negative power quite often. Parts (c) and (d) proved problematical. Marking of (d) was lenient and it was reaffirmed that testing of the concept in (d) will be done in a more straightforward context in future, when done at all.</span></p>
<div class="question_part_label">i.b.</div>
</div>
<div class="question" style="padding-left: 20px;">
<p><span style="font-size: medium; font-family: times new roman,times;">The derivative seemed manageable for many, though with the expected mis-handling of the negative power quite often. Parts (c) and (d) proved problematical. Marking of (d) was lenient and it was reaffirmed that testing of the concept in (d) will be done in a more straightforward context in future, when done at all.</span></p>
<div class="question_part_label">i.c.</div>
</div>
<div class="question" style="padding-left: 20px;">
<p><span style="font-size: medium; font-family: times new roman,times;">The derivative seemed manageable for many, though with the expected mis-handling of the negative power quite often. Parts (c) and (d) proved problematical. Marking of (d) was lenient and it was reaffirmed that testing of the concept in (d) will be done in a more straightforward context in future, when done at all.</span></p>
<div class="question_part_label">i.d.</div>
</div>
<div class="question" style="padding-left: 20px;">
<p><span style="font-size: medium; font-family: times new roman,times;">(ii) Many candidates failed to recognise that extensive use of the GDC was intended for this question. An indicator of this was the choice of awkward coefficients. It is recognised that the context confused some candidates and that the horizontal shift was a bit disturbing for some.</span></p>
<p><span style="font-size: medium; font-family: times new roman,times;">Nevertheless, a lot of candidates could have earned more marks here if they had persevered. Many gave up on the graph, and elementary marks for scale and labels were lost unnecessarily.</span></p>
<p><span style="font-size: medium; font-family: times new roman,times;">As this was the first time for the unit penalty, we were lenient about the units left off the labels but this is likely to change in the future.</span></p>
<div class="question_part_label">ii.a.</div>
</div>
<div class="question" style="padding-left: 20px;">
<p><span style="font-size: medium; font-family: times new roman,times;">(ii) Many candidates failed to recognise that extensive use of the GDC was intended for this question. An indicator of this was the choice of awkward coefficients. It is recognised that the context confused some candidates and that the horizontal shift was a bit disturbing for some.</span></p>
<p><span style="font-size: medium; font-family: times new roman,times;">Nevertheless, a lot of candidates could have earned more marks here if they had persevered. Many gave up on the graph, and elementary marks for scale and labels were lost unnecessarily.</span></p>
<p><span style="font-size: medium; font-family: times new roman,times;">As this was the first time for the unit penalty, we were lenient about the units left off the labels but this is likely to change in the future.</span></p>
<div class="question_part_label">ii.b.</div>
</div>
<div class="question" style="padding-left: 20px;">
<p><span style="font-size: medium; font-family: times new roman,times;">(ii) Many candidates failed to recognise that extensive use of the GDC was intended for this question. An indicator of this was the choice of awkward coefficients. It is recognised that the context confused some candidates and that the horizontal shift was a bit disturbing for some.</span></p>
<p><span style="font-size: medium; font-family: times new roman,times;">Nevertheless, a lot of candidates could have earned more marks here if they had persevered. Many gave up on the graph, and elementary marks for scale and labels were lost unnecessarily.</span></p>
<p><span style="font-size: medium; font-family: times new roman,times;">As this was the first time for the unit penalty, we were lenient about the units left off the labels but this is likely to change in the future.</span></p>
<div class="question_part_label">ii.c.</div>
</div>
<div class="question" style="padding-left: 20px;">
<p><span style="font-size: medium; font-family: times new roman,times;">(ii) Many candidates failed to recognise that extensive use of the GDC was intended for this question. An indicator of this was the choice of awkward coefficients. It is recognised that the context confused some candidates and that the horizontal shift was a bit disturbing for some.</span></p>
<p><span style="font-size: medium; font-family: times new roman,times;">Nevertheless, a lot of candidates could have earned more marks here if they had persevered. Many gave up on the graph, and elementary marks for scale and labels were lost unnecessarily.</span></p>
<p><span style="font-size: medium; font-family: times new roman,times;">As this was the first time for the unit penalty, we were lenient about the units left off the labels but this is likely to change in the future.</span></p>
<div class="question_part_label">ii.d.</div>
</div>
<div class="question" style="padding-left: 20px;">
<p><span style="font-size: medium; font-family: times new roman,times;">(ii) Many candidates failed to recognise that extensive use of the GDC was intended for this question. An indicator of this was the choice of awkward coefficients. It is recognised that the context confused some candidates and that the horizontal shift was a bit disturbing for some.</span></p>
<p><span style="font-size: medium; font-family: times new roman,times;">Nevertheless, a lot of candidates could have earned more marks here if they had persevered. Many gave up on the graph, and elementary marks for scale and labels were lost unnecessarily.</span></p>
<p><span style="font-size: medium; font-family: times new roman,times;">As this was the first time for the unit penalty, we were lenient about the units left off the labels but this is likely to change in the future.</span></p>
<div class="question_part_label">ii.e.</div>
</div>
<br><hr><br><div class="specification">
<p><span style="font-size: medium; font-family: times new roman,times;">The graph of the function \(f(x) = \frac{{14}}{x} + x - 6\), for 1 &le; <em>x</em> &le; 7 is given below.</span></p>
<p><span style="font-size: medium; font-family: times new roman,times;"><img src="data:image/png;base64,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" alt></span></p>
</div>

<div class="question" style="padding-left: 20px; padding-right: 20px;">
<p><span>Calculate \(f (1)\). </span></p>
<div class="marks">[2]</div>
<div class="question_part_label">a.</div>
</div>
<div class="question" style="padding-left: 20px; padding-right: 20px;">
<p><span>Find \(f ′(x)\).</span></p>
<div class="marks">[3]</div>
<div class="question_part_label">b.</div>
</div>
<div class="question" style="padding-left: 20px; padding-right: 20px;">
<p><span><strong>Use your answer to part (b)</strong> to show that the <em>x</em>-coordinate of the local minimum point of the graph of \(f\) is 3.7 correct to 2 significant figures.</span></p>
<div class="marks">[3]</div>
<div class="question_part_label">c.</div>
</div>
<div class="question" style="padding-left: 20px; padding-right: 20px;">
<p><span>Find the range of \(f\).</span></p>
<div class="marks">[3]</div>
<div class="question_part_label">d.</div>
</div>
<div class="question" style="padding-left: 20px; padding-right: 20px;">
<p><span>Points A and B lie on the graph of \(f\). The <em>x</em>-coordinates of A and B are 1 and 7 respectively.</span></p>
<p><span>Write down the <em>y</em>-coordinate of B.</span></p>
<div class="marks">[1]</div>
<div class="question_part_label">e.</div>
</div>
<div class="question" style="padding-left: 20px; padding-right: 20px;">
<p><span>Points A and B lie on the graph of f . The <em>x</em>-coordinates of A and B are 1 and 7 respectively.<br></span></p>
<p><span>Find the gradient of the straight line passing through A and B.</span></p>
<div class="marks">[2]</div>
<div class="question_part_label">f.</div>
</div>
<div class="question" style="padding-left: 20px; padding-right: 20px;">
<p><span>M is the midpoint of the line segment AB.</span></p>
<p><span>Write down the coordinates of M.</span></p>
<div class="marks">[2]</div>
<div class="question_part_label">g.</div>
</div>
<div class="question" style="padding-left: 20px; padding-right: 20px;">
<p><span><em>L</em> is the tangent to the graph of the function \(y = f (x)\), at the point on the graph with the same <em>x</em>-coordinate as M.</span></p>
<p><span>Find the gradient of <em>L</em>.</span></p>
<div class="marks">[2]</div>
<div class="question_part_label">h.</div>
</div>
<div class="question" style="padding-left: 20px; padding-right: 20px;">
<p><span>Find the equation of <em>L</em>. Give your answer in the form \(y = mx + c\).</span></p>
<div class="marks">[3]</div>
<div class="question_part_label">i.</div>
</div>
<h2 style="margin-top: 1em">Markscheme</h2>
<div class="question" style="padding-left: 20px;">
<p><span>\(\frac{{14}}{{(1)}} + (1) - 6\)     <em><strong>(M1)</strong></em></span><br><br></p>
<p><span><strong>Note:</strong> Award <em><strong>(M1)</strong></em> for substituting \(x = 1\) into \(f\).</span><br><br></p>
<p><span>\(= 9\)     <em><strong>(A1)(G2)</strong></em></span></p>
<div class="question_part_label">a.</div>
</div>
<div class="question" style="padding-left: 20px;">
<p><span>\( - \frac{{14}}{{{x^2}}} + 1\)     <em><strong>(A3)</strong></em></span></p>
<p><br><span><strong>Note:</strong> Award <em><strong>(A1)</strong></em> for \(-14\), <em><strong>(A1)</strong></em> for \(\frac{{14}}{{{x^2}}}\) or for \(x^{-2}\), <em><strong>(A1)</strong></em> for \(1\). </span></p>
<p><span>   Award at most <em><strong>(A2)</strong></em> if any extra terms are present.</span></p>
<div class="question_part_label">b.</div>
</div>
<div class="question" style="padding-left: 20px;">
<p><span>\( - \frac{{14}}{{{x^2}}} + 1 = 0\) or \(f ' (x) = 0 \)     <em><strong>(M1)</strong></em><br><br></span></p>
<p><span><strong>Note:</strong> Award <em><strong>(M1)</strong></em> for equating <strong>their</strong> derivative in part (b) to 0.<br><br></span></p>
<p><span>\(\frac{{14}}{{{x^2}}} = 1\)</span><span><span> or \({x^2} = 14\)</span> or equivalent     <em><strong>(M1)</strong></em></span></p>
<p><span><strong>Note:</strong> Award <em><strong>(M1)</strong></em> for correct rearrangement of their equation.</span></p>
<p><span><br>\(x = 3.74165...(\sqrt {14})\)     <em><strong>(A1)</strong></em><br></span></p>
<p><span>\(x = 3.7\)     <em><strong>(AG)</strong></em></span></p>
<p><span><strong>Notes:</strong> Both the unrounded and rounded answers must be seen to award the <em><strong>(A1)</strong></em>. This is a “show that” question; appeals to their GDC are not accepted –award a maximum of <em><strong>(M1)(M0)(A0)</strong></em>. </span></p>
<p><span>Specifically, </span><span><span>\( - \frac{{14}}{{{x^2}}} + 1 = 0\)</span> followed by \(x = 3.74165..., x = 3.7\) is awarded <em><strong>(M1)(M0)(A0)</strong></em>.</span></p>
<div class="question_part_label">c.</div>
</div>
<div class="question" style="padding-left: 20px;">
<p><span>\(1.48 \leqslant y \leqslant 9\)     <em><strong>(A1)(A1)</strong></em><strong>(ft)</strong><em><strong>(A1)</strong></em></span></p>
<p><span><em><strong><br><br></strong></em><strong>Note:</strong> Accept alternative notations, for example [1.48,9]. (\(x = \sqrt{14}\) leads to answer 1.48331...)</span></p>
<p><span><br><strong>Note:</strong> Award <em><strong>(A1)</strong></em> for 1.48331…seen, accept 1.48378… from using the given answer \(x = 3.7\), <em><strong>(A1)</strong></em><strong>(ft)</strong> for their 9 from part (a) seen, <em><strong>(A1)</strong></em> for the correct notation for their interval (accept \( \leqslant y \leqslant \)</span><span> or \( \leqslant f \leqslant \)</span><span> ).<br></span></p>
<div class="question_part_label">d.</div>
</div>
<div class="question" style="padding-left: 20px;">
<p><span>3     <em><strong>(A1)</strong></em> </span></p>
<p><br><span><strong>Note:</strong> Do not accept a coordinate pair.</span></p>
<div class="question_part_label">e.</div>
</div>
<div class="question" style="padding-left: 20px;">
<p><span>\(\frac{{3 - 9}}{{7 - 1}}\)     <em><strong>(M1)</strong></em></span></p>
<p><br><span><strong>Note:</strong> Award <em><strong>(M1)</strong></em> for their correct substitution into the gradient formula.</span></p>
<p><br><span>\(= -1\)     <em><strong>(A1)</strong></em><strong>(ft)</strong><em><strong>(G2)</strong></em> </span></p>
<p><br><span><strong>Note:</strong> Follow through from their answers to parts (a) and (e).</span></p>
<div class="question_part_label">f.</div>
</div>
<div class="question" style="padding-left: 20px;">
<p><span>(4, 6)     <em><strong>(A1)</strong></em><strong>(ft)</strong><em><strong>(A1)</strong></em></span></p>
<p><br><span><strong>Note:</strong> Accept \(x = 4\), \(y = 6\). Award at most <em><strong>(A1)(A0)</strong></em> if parentheses not seen. </span></p>
<p><span>If coordinates reversed award <em><strong>(A0)(A1)</strong></em><strong>(ft)</strong>. </span></p>
<p><span>Follow through from their answers to parts (a) and (e).</span></p>
<div class="question_part_label">g.</div>
</div>
<div class="question" style="padding-left: 20px;">
<p><span>\( - \frac{{14}}{{{4^2}}} + 1\)     <em><strong>(M1)</strong></em></span></p>
<p><span><br><strong>Note:</strong> Award <em><strong>(M1)</strong></em> for substitution into their gradient function. </span></p>
<p><span>Follow through from their answers to parts (b) and (g).<br></span></p>
<p><span><br>\( = \frac{1}{8}(0.125)\)     <em><strong>(A1)</strong></em><strong>(ft)</strong><em><strong>(G2)</strong></em><br></span></p>
<div class="question_part_label">h.</div>
</div>
<div class="question" style="padding-left: 20px;">
<p><span>\(y - 1.5 = \frac{1}{8}(x - 4)\)     <em><strong>(M1)</strong></em><strong>(ft)</strong><em><strong>(M1)</strong></em></span></p>
<p><br><span><strong>Note:</strong> Award <em><strong>(M1)</strong></em> for substituting their (4, 1.5) in any straight line formula, </span></p>
<p><span><em><strong>(</strong></em></span><em><strong>M1)</strong></em><span> for substituting their gradient in any straight line formula.</span></p>
<p><br><span>\(y = \frac{x}{8} + 4\)     <em><strong>(A1)</strong></em><strong>(ft)</strong><em><strong>(G2)</strong></em></span></p>
<p><br><span><strong>Note:</strong> The form of the line has been specified in the question.</span></p>
<div class="question_part_label">i.</div>
</div>
<h2 style="margin-top: 1em">Examiners report</h2>
<div class="question" style="padding-left: 20px;">
<p><span style="font-family: times new roman,times; font-size: medium;">Most candidates were able to evaluate the function and find the derivative for \(x + 6\) but the term with the negative index was problematic. The few candidates who equated their derivative to zero at the local minimum point progressed well and showed a thorough understanding of the differential calculus. Many did not attain full marks for the range of the function, either confusing this with the statistical concept of range or using the <em>y</em>-coordinate at B. Most were able to find the gradient and midpoint of the straight line passing through A and B. The final parts were also challenging for the majority: many had difficulty finding the gradient of the tangent L, instead using the slope formula for a straight line; the most common error in part (i) was to substitute in the coordinates of midpoint M rather than the point on the curve. Greater insight into the problem would have come from using the given sketch of the curve and annotating it; it seems that many candidates do not link the algebraic nature of the differential calculus with the curve in question.</span></p>
<div class="question_part_label">a.</div>
</div>
<div class="question" style="padding-left: 20px;">
<p><span style="font-family: times new roman,times; font-size: medium;">Most candidates were able to evaluate the function and find the derivative for \(x + 6\) but the term with the negative index was problematic. The few candidates who equated their derivative to zero at the local minimum point progressed well and showed a thorough understanding of the differential calculus. Many did not attain full marks for the range of the function, either confusing this with the statistical concept of range or using the <em>y</em>-coordinate at B. Most were able to find the gradient and midpoint of the straight line passing through A and B. The final parts were also challenging for the majority: many had difficulty finding the gradient of the tangent L, instead using the slope formula for a straight line; the most common error in part (i) was to substitute in the coordinates of midpoint M rather than the point on the curve. Greater insight into the problem would have come from using the given sketch of the curve and annotating it; it seems that many candidates do not link the algebraic nature of the differential calculus with the curve in question.</span></p>
<div class="question_part_label">b.</div>
</div>
<div class="question" style="padding-left: 20px;">
<p><span style="font-family: times new roman,times; font-size: medium;">Most candidates were able to evaluate the function and find the derivative for \(x + 6\) but the term with the negative index was problematic. The few candidates who equated their derivative to zero at the local minimum point progressed well and showed a thorough understanding of the differential calculus. Many did not attain full marks for the range of the function, either confusing this with the statistical concept of range or using the <em>y</em>-coordinate at B. Most were able to find the gradient and midpoint of the straight line passing through A and B. The final parts were also challenging for the majority: many had difficulty finding the gradient of the tangent L, instead using the slope formula for a straight line; the most common error in part (i) was to substitute in the coordinates of midpoint M rather than the point on the curve. Greater insight into the problem would have come from using the given sketch of the curve and annotating it; it seems that many candidates do not link the algebraic nature of the differential calculus with the curve in question.</span></p>
<div class="question_part_label">c.</div>
</div>
<div class="question" style="padding-left: 20px;">
<p><span style="font-family: times new roman,times; font-size: medium;">Most candidates were able to evaluate the function and find the derivative for \(x + 6\) but the term with the negative index was problematic. The few candidates who equated their derivative to zero at the local minimum point progressed well and showed a thorough understanding of the differential calculus. Many did not attain full marks for the range of the function, either confusing this with the statistical concept of range or using the <em>y</em>-coordinate at B. Most were able to find the gradient and midpoint of the straight line passing through A and B. The final parts were also challenging for the majority: many had difficulty finding the gradient of the tangent L, instead using the slope formula for a straight line; the most common error in part (i) was to substitute in the coordinates of midpoint M rather than the point on the curve. Greater insight into the problem would have come from using the given sketch of the curve and annotating it; it seems that many candidates do not link the algebraic nature of the differential calculus with the curve in question.</span></p>
<div class="question_part_label">d.</div>
</div>
<div class="question" style="padding-left: 20px;">
<p><span style="font-family: times new roman,times; font-size: medium;">Most candidates were able to evaluate the function and find the derivative for \(x + 6\) but the term with the negative index was problematic. The few candidates who equated their derivative to zero at the local minimum point progressed well and showed a thorough understanding of the differential calculus. Many did not attain full marks for the range of the function, either confusing this with the statistical concept of range or using the <em>y</em>-coordinate at B. Most were able to find the gradient and midpoint of the straight line passing through A and B. The final parts were also challenging for the majority: many had difficulty finding the gradient of the tangent L, instead using the slope formula for a straight line; the most common error in part (i) was to substitute in the coordinates of midpoint M rather than the point on the curve. Greater insight into the problem would have come from using the given sketch of the curve and annotating it; it seems that many candidates do not link the algebraic nature of the differential calculus with the curve in question.</span></p>
<div class="question_part_label">e.</div>
</div>
<div class="question" style="padding-left: 20px;">
<p><span style="font-family: times new roman,times; font-size: medium;">Most candidates were able to evaluate the function and find the derivative for \(x + 6\) but the term with the negative index was problematic. The few candidates who equated their derivative to zero at the local minimum point progressed well and showed a thorough understanding of the differential calculus. Many did not attain full marks for the range of the function, either confusing this with the statistical concept of range or using the <em>y</em>-coordinate at B. Most were able to find the gradient and midpoint of the straight line passing through A and B. The final parts were also challenging for the majority: many had difficulty finding the gradient of the tangent L, instead using the slope formula for a straight line; the most common error in part (i) was to substitute in the coordinates of midpoint M rather than the point on the curve. Greater insight into the problem would have come from using the given sketch of the curve and annotating it; it seems that many candidates do not link the algebraic nature of the differential calculus with the curve in question.</span></p>
<div class="question_part_label">f.</div>
</div>
<div class="question" style="padding-left: 20px;">
<p><span style="font-family: times new roman,times; font-size: medium;">Most candidates were able to evaluate the function and find the derivative for \(x + 6\) but the term with the negative index was problematic. The few candidates who equated their derivative to zero at the local minimum point progressed well and showed a thorough understanding of the differential calculus. Many did not attain full marks for the range of the function, either confusing this with the statistical concept of range or using the <em>y</em>-coordinate at B. Most were able to find the gradient and midpoint of the straight line passing through A and B. The final parts were also challenging for the majority: many had difficulty finding the gradient of the tangent L, instead using the slope formula for a straight line; the most common error in part (i) was to substitute in the coordinates of midpoint M rather than the point on the curve. Greater insight into the problem would have come from using the given sketch of the curve and annotating it; it seems that many candidates do not link the algebraic nature of the differential calculus with the curve in question.</span></p>
<div class="question_part_label">g.</div>
</div>
<div class="question" style="padding-left: 20px;">
<p><span style="font-family: times new roman,times; font-size: medium;">Most candidates were able to evaluate the function and find the derivative for \(x + 6\) but the term with the negative index was problematic. The few candidates who equated their derivative to zero at the local minimum point progressed well and showed a thorough understanding of the differential calculus. Many did not attain full marks for the range of the function, either confusing this with the statistical concept of range or using the <em>y</em>-coordinate at B. Most were able to find the gradient and midpoint of the straight line passing through A and B. The final parts were also challenging for the majority: many had difficulty finding the gradient of the tangent L, instead using the slope formula for a straight line; the most common error in part (i) was to substitute in the coordinates of midpoint M rather than the point on the curve. Greater insight into the problem would have come from using the given sketch of the curve and annotating it; it seems that many candidates do not link the algebraic nature of the differential calculus with the curve in question.</span></p>
<div class="question_part_label">h.</div>
</div>
<div class="question" style="padding-left: 20px;">
<p><span style="font-family: times new roman,times; font-size: medium;">Most candidates were able to evaluate the function and find the derivative for \(x + 6\) but the term with the negative index was problematic. The few candidates who equated their derivative to zero at the local minimum point progressed well and showed a thorough understanding of the differential calculus. Many did not attain full marks for the range of the function, either confusing this with the statistical concept of range or using the <em>y</em>-coordinate at B. Most were able to find the gradient and midpoint of the straight line passing through A and B. The final parts were also challenging for the majority: many had difficulty finding the gradient of the tangent L, instead using the slope formula for a straight line; the most common error in part (i) was to substitute in the coordinates of midpoint M rather than the point on the curve. Greater insight into the problem would have come from using the given sketch of the curve and annotating it; it seems that many candidates do not link the algebraic nature of the differential calculus with the curve in question.</span></p>
<div class="question_part_label">i.</div>
</div>
<br><hr><br><div class="specification">
<p class="p1">Consider the function \(f(x) = 0.5{x^2} - \frac{8}{x},{\text{ }}x \ne 0\).</p>
</div>

<div class="question" style="padding-left: 20px; padding-right: 20px;">
<p class="p1">Find \(f( - 2)\).</p>
<div class="marks">[2]</div>
<div class="question_part_label">a.</div>
</div>
<div class="question" style="padding-left: 20px; padding-right: 20px;">
<p class="p1">Find \(f'(x)\).</p>
<div class="marks">[3]</div>
<div class="question_part_label">b.</div>
</div>
<div class="question" style="padding-left: 20px; padding-right: 20px;">
<p>Find the gradient of the graph of \(f\) at \(x =  - 2\).</p>
<div class="marks">[2]</div>
<div class="question_part_label">c.</div>
</div>
<div class="question" style="padding-left: 20px; padding-right: 20px;">
<p>Let \(T\) be the tangent to the graph of \(f\) at \(x =  - 2\).</p>
<p>Write down the equation of \(T\).</p>
<div class="marks">[2]</div>
<div class="question_part_label">d.</div>
</div>
<div class="question" style="padding-left: 20px; padding-right: 20px;">
<p>Let \(T\) be the tangent to the graph of \(f\) at \(x =  - 2\).</p>
<p>Sketch the graph of \(f\) for \( - 5 \leqslant x \leqslant 5\) and \( - 20 \leqslant y \leqslant 20\).</p>
<div class="marks">[4]</div>
<div class="question_part_label">e.</div>
</div>
<div class="question" style="padding-left: 20px; padding-right: 20px;">
<p>Let \(T\) be the tangent to the graph of \(f\) at \(x =  - 2\).</p>
<p>Draw \(T\) on your sketch.</p>
<div class="marks">[2]</div>
<div class="question_part_label">f.</div>
</div>
<div class="question" style="padding-left: 20px; padding-right: 20px;">
<p class="p1">The tangent, \(T\), intersects the graph of \(f\) <span class="s1">at a second point, P.</span></p>
<p class="p2">Use your graphic display calculator to find the coordinates of P.</p>
<div class="marks">[2]</div>
<div class="question_part_label">g.</div>
</div>
<h2 style="margin-top: 1em">Markscheme</h2>
<div class="question" style="padding-left: 20px;">
<p>\(0.5 \times {( - 2)^2} - \frac{8}{{ - 2}}\)     <strong><em>(M1)</em></strong></p>
<p><strong>Note: </strong>Award <strong><em>(M1) </em></strong>for substitution of \(x =  - 2\) into the formula of the function.</p>
<p> </p>
<p>\(6\)     <strong><em>(A1)(G2)</em></strong></p>
<div class="question_part_label">a.</div>
</div>
<div class="question" style="padding-left: 20px;">
<p class="p1">\(f'(x) = x + 8{x^{ - 2}}\) <span class="Apple-converted-space">    </span><strong><em>(A1)(A1)(A1)</em></strong></p>
<p class="p1"> </p>
<p class="p1"><strong>Notes: </strong>Award <strong><em>(A1) </em></strong>for \(x\), <strong><em>(A1) </em></strong>for \(8\), <strong><em>(A1) </em></strong>for \({x^{ - 2}}\) or \(\frac{1}{{{x^2}}}\) (each term must have correct sign). Award at most <strong><em>(A1)(A1)(A0) </em></strong>if there are additional terms present or further incorrect simplifications are seen.</p>
<div class="question_part_label">b.</div>
</div>
<div class="question" style="padding-left: 20px;">
<p>\(f'( - 2) =  - 2 + 8{( - 2)^{ - 2}}\)     <strong><em>(M1)</em></strong></p>
<p><strong>Note: </strong>Award <strong><em>(M1) </em></strong>for \(x =  - 2\) substituted into their \(f'(x)\) from part (b).</p>
<p> </p>
<p>\( = 0\)     <strong><em>(A1)</em>(ft)<em>(G2)</em></strong></p>
<p><strong>Note: </strong>Follow through from their derivative function.</p>
<div class="question_part_label">c.</div>
</div>
<div class="question" style="padding-left: 20px;">
<p class="p1">\(y = 6\;\;\;\)<strong>OR</strong>\(\;\;\;y = 0x + 6\;\;\;\)<strong>OR</strong>\(\;\;\;y - 6 = 0(x + 2)\) <span class="Apple-converted-space">    </span><strong><em>(A1)</em>(ft)<em>(A1)</em>(ft)<em>(G2)</em></strong></p>
<p class="p2"> </p>
<p class="p1"><strong>Notes:<span class="Apple-converted-space"> </span></strong>Award <strong><em>(A1)</em>(ft) </strong>for their gradient from part (c), <strong><em>(A1)</em>(ft) </strong>for their answer from part (a). Answer must be an equation.</p>
<p class="p1">Award <strong><em>(A0)(A0) </em></strong>for \(x = 6\).</p>
<div class="question_part_label">d.</div>
</div>
<div class="question" style="padding-left: 20px;">
<p class="p1"><img src="data:image/png;base64,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" alt> <span class="Apple-converted-space">    </span><strong><em>(A1)(A1)(A1)(A1)</em></strong></p>
<p class="p1"> </p>
<p class="p1"><strong>Notes: </strong>Award <strong><em>(A1) </em></strong>for labels and some indication of scales in the stated window. The point \((-2,{\text{ }}6)\) correctly labelled, or an \(x\)-value and a \(y\)-value on their axes in approximately the correct position, are acceptable indication of scales.</p>
<p class="p1">Award <strong><em>(A1) </em></strong>for correct general shape (curve must be smooth and must not cross the \(y\)-axis).</p>
<p class="p1">Award <strong><em>(A1) </em></strong>for \(x\)-intercept in approximately the correct position.</p>
<p class="p1">Award <strong><em>(A1) </em></strong>for local minimum in the second quadrant.</p>
<div class="question_part_label">e.</div>
</div>
<div class="question" style="padding-left: 20px;">
<p>Tangent to graph drawn approximately at \(x =  - 2\)     <strong><em>(A1)</em>(ft)<em>(A1)</em>(ft)</strong></p>
<p> </p>
<p><strong>Notes: </strong>Award <strong><em>(A1)</em>(ft) </strong>for straight line tangent to curve at approximately \(x =  - 2\), with approximately correct gradient. Tangent must be straight for the <strong><em>(A1)</em>(ft) </strong>to be awarded.</p>
<p>Award <strong><em>(A1)</em>(ft) </strong>for (extended) line passing through approximately their \(y\)-intercept from (d). Follow through from their gradient in part (c) and their equation in part (d).</p>
<div class="question_part_label">f.</div>
</div>
<div class="question" style="padding-left: 20px;">
<p class="p1">\((4,{\text{ }}6)\;\;\;\)<strong>OR</strong>\(\;\;\;x = 4,{\text{ }}y = 6\) <span class="Apple-converted-space">    </span><strong><em>(G1)</em>(ft)<em>(G1)</em>(ft)</strong></p>
<p class="p1"><strong>Notes: </strong>Follow through from their tangent from part (d). If brackets are missing then award <strong><em>(G0)(G1)</em>(ft).</strong></p>
<p class="p1">If line intersects their graph at more than one point (apart from \(( - 2,{\text{ }}6)\)<span class="s1">)</span>, follow through from the first point of intersection (to the right of \( - 2\)).</p>
<p class="p1">Award <strong><em>(G0)(G0) </em></strong>for \(( - 2,{\text{ }}6)\).</p>
<div class="question_part_label">g.</div>
</div>
<h2 style="margin-top: 1em">Examiners report</h2>
<div class="question" style="padding-left: 20px;">
[N/A]
<div class="question_part_label">a.</div>
</div>
<div class="question" style="padding-left: 20px;">
[N/A]
<div class="question_part_label">b.</div>
</div>
<div class="question" style="padding-left: 20px;">
[N/A]
<div class="question_part_label">c.</div>
</div>
<div class="question" style="padding-left: 20px;">
[N/A]
<div class="question_part_label">d.</div>
</div>
<div class="question" style="padding-left: 20px;">
[N/A]
<div class="question_part_label">e.</div>
</div>
<div class="question" style="padding-left: 20px;">
[N/A]
<div class="question_part_label">f.</div>
</div>
<div class="question" style="padding-left: 20px;">
[N/A]
<div class="question_part_label">g.</div>
</div>
<br><hr><br><div class="specification">
<p><span style="font-family: times new roman,times; font-size: medium;">A closed rectangular box has a height \(y{\text{ cm}}\) and width \(x{\text{ cm}}\). Its length is twice its width. It has a fixed outer surface area of \(300{\text{ c}}{{\text{m}}^2}\) .</span></p>
<p><span style="font-family: times new roman,times; font-size: medium;"><img style="display: block; margin-left: auto; margin-right: auto;" src="data:image/png;base64,iVBORw0KGgoAAAANSUhEUgAAAOEAAADlCAIAAAAqQAc7AAAMm0lEQVR4nO3dwWsbdxrG8f4nMzAniQpafKhaiN3eHLUIYxo3aS4FW7VdSinNYROPqrIlkKYLMlaXGvYiUzuX0HREN0lL7TIE3GFxOtRtzSJ+sIeQiDkY4QqBjAnDbw8jy7I8SuxX8jsz8vNhLmrtVJa/yI8mGvcFCRBuLwR9BwCeA41C2KFRCDs0CmGHRiHsBqRRIYSmqDj6eKyurgb9XW2KfKPValWfmwv8Ozp4R8kwgv7eNkW40UajsbK87D2gFycmhBBB36NoE0Jcu3oNjfbN6urqGyMjmqK+MTISnp9KEVWtVr+8edNLs1D46ts7d9BoT4QQFycmvAd0ZXm50WgEfY8irNFolAzDezCvfHJlff2XP/7cunvvPholap+e+txctVoN+h5Fm2VZr58b1hT13UuX7t67/8efW96BRikwPftLCDE7M6Mp6quvJFdWbrXqRKNEJcPA9OyXjun58Fe7I1A0ejKbm5uYnv3iOz19DzR6LE+ePMH07KNu0xONUjQajcWvFzE9+0UIMTPddXqi0RNrn56WZQV9d6KtWq1+cePGs6cnGj2B9ulZMgxMz14cf3qi0WNpn55f3ryJ6dkjy7JGzp075vREo8/RPj0/mJ3F9OwRYXqi0a68H0aYnv1Cnp5o1J9lWZie/dLj9ESjnYQQmJ591Pv0RKMHqtUqpmcftabnm+dTvUxPNCrl4el5cWIC07NHren52ivJ3qcnGj2Ynm+MjGB69qj9zV+5T3N9mZ5nutH26bn49SKmZ49WV1e96Tn9/nQfp+cZbbR9eupzc5iePRJCTFy4cErT88w1iunZX9Vq1bsI7vSm59lq1LKs1jl5TM8esU3Ps9KoEOKD2VlMz37hnJ6D32j7hQeYnr3jn56D3Gj7X8FhevYuqOk5sI12TM++36czJdjpOYCNdkxPvDDqUeDTc6Aa7ZieT548OdW7NfBCMj0HpNGO6bm5uclwtwZYa3q+GoLpOSCN/vjjjy/GYi/GYhffeUefm8PR4xGq6TkgjbaeRHH065h+fzrwEAewUZz77AvvwUSjJ4JGWaFRAjTKCo0SoFFWaJQAjbJCowRolBUaJUCjrNAoARplhUYJ0CgrNEqARlmhUQI0ygqNEqBRVmiUAI2yQqMEaJQVGiVAo6zQKAEaZYVGCdAoKzRKgEZZoVECNMoKjRKgUVZolACNskKjBGiUFRolQKOs0CgBGmWFRgnQKCs0SoBGWaFRAjTKCo0SoFFWaJQAjbJCowRolBUaJUCjrNAoARplhUYJ0CgrNEqARlmhUQI0ygqNEqBRVmiUAI2yQqMEaJQVGiVAo6zQKAEaZYVGCdAoKzRKgEZZoVECNMoKjRKgUVZolACNskKjBGiUFRolQKOsItmoY2S8/6v5aMGuu1JKWTOzseSs8cg9+hXWxc/5qYSiakoys2A5rpTSrYsH3+WnhnTzL8cqTCY1JZ7KrTnunmPfyo7GNSWZKW7Vuz9oaJRVJBuVUspdUbysKZeXxK6UUroVMzc+Vix3NlrfmB8d/8ysuFK6FWM2Fh8rlt2amY2pmqImJvMl23GlW7cLKSWZyd82RU3KPce8nmr9yX7QKKvINiqf2vkh5fy87T3f7YqiPm/vHP7i3JqZG9LNWvOW83BhZiy/UZdSuuWldDzR+lc1M+vl632gKI4pw1lzu9uDhkZZRbdRVxTHlJczxmMppVsxPvrQqHQ+i26b+vBBiIc+GY1GR3QblY6RaTa6Yy/8o1TZO/LFbZv68MFmbYdGIyTCjdbMbEwdyluPzfxnvq+Wmps1ntIN0czUrf+2btdcNBolEW70qT3/kpp4b+bq39ccv0Kl9F4zxTXvJICiako8hT0aORFu1Otscrl89Ed5+0c5G8t6WlNUTUlni6aou64ojrWqTRfL5YObCX3tsZlLHNz027JolFm0G535wnSOztBTh0ZZRbbRHXvh+lLZ92nu1KFRVlFrdMfOjyf+Vvxp4fNC59lQPmiUVdQa9U4n6ct215dJDNAoq6g1GgpolBUaJUCjrNAoARplhUYJ0CgrNEqARlmhUQI0ygqNEqBRVmiUAI2yQqMEaJQVGiVAo6zQKAEaZYVGCdAoKzRKgEZZoVECNMoKjRKgUVZolACNskKjBGiUFRolQKOs0CgBGmWFRgnQKCs0SoBGWaFRAjTKCo0SoFFWaJQAjbJCowRolBUaJUCjrNAoARplhUYJ0CgrNEqARlmhUQI0ygqNEqBRVmiUAI2yQqMEaJQVGiVAo6zQKAEaZYVGCdAoKzRKgEZZoVECNMoKjRKgUVZolACNskKjBGiUFRolQKOs0CgBGmWFRgnQKCs0SoBGWaFRAjTKCo0SoFFWaJQAjbJCowRolBUaJUCjrNAoARplhUYJ0CgrNEqARlmhUQI0ygqNEqBRVmiUAI2yQqMEaJQVGiVAo6zQKAEaZYVGCdAoKzRKgEZZoVECNMoKjRKgUVZolACNskKjBGiUFRolQKOs0CgBGmWFRgnQKCs0SoBGWaFRAjTKKhKNfnvnDho9u0Le6MNf7ULhq1dfSWqKurq6GvSj1YRGWYW50ZWVW2+eT2mKOnHhQqi+3WiUVTgbvXvv/vT705qijpw7F56nzxY0yipsja6v/5L7NKcpqqaoK8vLjUYj6EfIBxplFZ5G26fntavXqtVq0I9NV2iUVUgaDe309IVGWQXeaMinpy80yirARiMxPX2hUVaBNOpNz9eiMD19oVFW/I1Ga3r6QqOsOBuN4vT0hUZZ8TQa3enpC42yOu1G26fnFzduRG56+kKjrE610db0nJmeGaTvFxpldUqNtk9Py7KC/ir7DI2y6nuj6+u/XPnkijc9S4YR9enpC42y6mOj3vT06hyY6ekLjbLqV6MrK7e8t4MM2PT0hUZZ9d7o3Xv33710yX96OkZGUTVFTcwYFVe6jlWYTGrK+Xm7fuSO1MRaIRNTNUVNTC4+dPaa/9C8PT+ZzpoVZ2MxE1M1Jf2ZWXFdx/5GTymqFptZKtdYHqdD0CirXho93vTcqxgfJ5ThrPF9Qb9Vrrt+H7Nj5y+mcmuOK6X7qDST1NJF4bo1M5dQVE1JZvKG7exJuWPnx7XY1PydB6LuSrdi5tJauih8/8jThEZZ0Ro92fSsmdmYqsU+LlX2un7ASzmz5rW252wsZtIFu+5KKV1RHFOGs+a2lFJKr9rLS2JXSinlrihe1mKtT+SDRlkRGj359Nw29eHuT3huzcwluqSGRuFkjbam5+vnhk9y1nPb1Ie1g7Y6eOWNz9s7Pv8OjcIxG+3hrOeeY+az+sepg9Q6uaI4pqjaaK4k9l8A1bdMuyrRKMhjNNo+Pb+8efOkZz3divHRh0blaXkpHU/oa5Xy7cL3j440tWPnx73/RPMYxR6Ffc9utDU9Z2dOfNbTFcUxJZ7SDVF39ytMZ43y0dNOUkrZOp2kxFN60RS1ZoLNauNjxd/KBzeHs+Z/TX247ab/M/QpQaOsujVKnZ5nAhpldbTRs/AX7j1Co6zaG+1xep4daJRVq9HW9Lx29Roe22dDo6y8B9M7Lrz9NqbncRy3URx9PF5LJjE9j+/5jVqWpc/N4ejXkX7rra2tLYZv7cB4fqMAwUKjEHZoFMIu8EbduvhpfjKpKaqmpLPfbDjsb6GFkAu4UbfyQ2HhJ1F3m2+2jcVT+Q3/v2KGsyrYRnf/9+A/lYMnzsDeWQNhFvjP+nbPeos4nFmha3Roxqi4UsrHpcmXNWX/uhzXebgwlVDUobz99MhnHSza2FShOWjdunjwXX5qSDf/al4eGU/l1hx3z7FvZUfjmpLMFLcwKiIhVI1um/pU+zUMbsWYjakJ/fbPC9e7XDXr1u1CavS66eztXxJ5eUnsNq87U9TEZL5kO673YUoyk79tipqUe455PdX1agoIl/A06tbtf2Y6XzB5l+YkZ42jbyZvfcB46y23rmMVJt9rVu5670U3m2nXzGwsPlYse3/O4TecQ6iFptH6ht/14B2XKxxRM7OxLqmh0UERjkbdR/9euO33Cwu8Rg/a6lQzs91OV6HRQRGCRt2KufAv09n/hQX1reWi5YXlOmuf6/rV0bbUOj+3vJSOH75wZ2fT/L0m0ejgCLrRermkpw+/dW2/JPdR6UO9VKk1T5pWfl9e+KHS+XTqvRhq//Rx7NEBE2ij3i8b6nx75eWl8h9L6fj+BeBeha0rHo9qnU5StVF9yRT11iXk3pEulssHNxP62uPmLzbybnZ5hobQCPp5FOB50CiEHRqFsEOjEHZoFMIOjULYoVEIOzQKYYdGIezQKIQdGoWwQ6MQdv8HxC1hdHkfi1QAAAAASUVORK5CYII=" alt></span></p>
</div>

<div class="question" style="padding-left: 20px; padding-right: 20px;">
<p><span>Factorise \(3{x^2} + 13x - 10\).</span></p>
<div class="marks">[2]</div>
<div class="question_part_label">i.a.</div>
</div>
<div class="question" style="padding-left: 20px; padding-right: 20px;">
<p><span>Solve the equation \(3{x^2} + 13x - 10 = 0\).</span></p>
<div class="marks">[2]</div>
<div class="question_part_label">i.b.</div>
</div>
<div class="question" style="padding-left: 20px; padding-right: 20px;">
<p><span>Consider a function \(f(x) = 3{x^2} + 13x - 10\) .</span></p>
<p><span>Find the equation of the axis of symmetry on the graph of this function.</span></p>
<div class="marks">[2]</div>
<div class="question_part_label">i.c.</div>
</div>
<div class="question" style="padding-left: 20px; padding-right: 20px;">
<p><span>Consider a function \(f(x) = 3{x^2} + 13x - 10\) .</span></p>
<p><span>Calculate the minimum value of this function.</span></p>
<div class="marks">[2]</div>
<div class="question_part_label">i.d.</div>
</div>
<div class="question" style="padding-left: 20px; padding-right: 20px;">
<p><span>Show that \(4{x^2} + 6xy = 300\).</span></p>
<div class="marks">[2]</div>
<div class="question_part_label">ii.a.</div>
</div>
<div class="question" style="padding-left: 20px; padding-right: 20px;">
<p><span>Find an expression for \(y\) in terms of \(x\).</span></p>
<div class="marks">[2]</div>
<div class="question_part_label">ii.b.</div>
</div>
<div class="question" style="padding-left: 20px; padding-right: 20px;">
<p><span>Hence show that the volume \(V\) of the box is given by \(V = 100x - \frac{4}{3}{x^3}\).</span></p>
<div class="marks">[2]</div>
<div class="question_part_label">ii.c.</div>
</div>
<div class="question" style="padding-left: 20px; padding-right: 20px;">
<p><span>Find \(\frac{{{\text{d}}V}}{{{\text{d}}x}}\).</span></p>
<div class="marks">[2]</div>
<div class="question_part_label">ii.d.</div>
</div>
<div class="question" style="padding-left: 20px; padding-right: 20px;">
<p><span>(i)     Hence find the value of \(x\) and of \(y\) required to make the volume of the box a maximum.</span></p>
<p><span>(ii)    Calculate the maximum volume.</span></p>
<div class="marks">[5]</div>
<div class="question_part_label">ii.e.</div>
</div>
<h2 style="margin-top: 1em">Markscheme</h2>
<div class="question" style="padding-left: 20px;">
<p><span>\((3x - 2)(x + 5)\)     <em><strong>(A1)(A1)</strong></em></span></p>
<p><span><em><strong>[2 marks]<br></strong></em></span></p>
<div class="question_part_label">i.a.</div>
</div>
<div class="question" style="padding-left: 20px;">
<p><span>\((3x - 2)(x + 5) = 0\)</span></p>
<p><span>\(x = \frac{2}{3}\) or \(x = - 5\)     <em><strong>(A1)</strong></em><strong>(ft)</strong><em><strong>(A1)</strong></em><strong>(ft)</strong><em><strong>(G2)</strong></em></span></p>
<p><span><em><strong>[2 marks]<br></strong></em></span></p>
<div class="question_part_label">i.b.</div>
</div>
<div class="question" style="padding-left: 20px;">
<p><span>\(x = \frac{{ - 13}}{6}{\text{ }}( - 2.17)\)     <em><strong>(A1)(A1)</strong></em><strong>(ft)<em>(G2)</em></strong></span></p>
<p><br><span><strong>Note: <em>(A1)</em></strong> is for \(x = \), <em><strong>(A1)</strong></em> for value. <strong>(ft)</strong> if value is half way between roots in (b).</span></p>
<p><span><em><strong>[2 marks]</strong></em><br></span></p>
<div class="question_part_label">i.c.</div>
</div>
<div class="question" style="padding-left: 20px;">
<p><span>Minimum \(y = 3{\left( {\frac{{ - 13}}{6}} \right)^2} + 13\left( {\frac{{ - 13}}{6}} \right) - 10\)     <em><strong>(M1)</strong></em></span><br><br><span><strong>Note: <em>(M1)</em></strong> for substituting their value of \(x\) from (c) into \(f(x)\) .</span></p>
<p><br><span>\( = - 24.1\)     <em><strong>(A1)</strong></em><strong>(ft)<em>(G2)</em></strong></span></p>
<p><span><strong><em>[2 marks]<br></em></strong></span></p>
<div class="question_part_label">i.d.</div>
</div>
<div class="question" style="padding-left: 20px;">
<p><span>\({\text{Area}} = 2(2x)x + 2xy + 2(2x)y\)     <em><strong>(M1)(A1)</strong></em></span></p>
<p><br><span><strong>Note: <em>(M1)</em></strong> for using the correct surface area formula (which can</span> <span>be implied if numbers in the correct place). <em><strong>(A1)</strong></em> for using correct numbers.</span><br><br></p>
<p><span>\(300 = 4{x^2} + 6xy\)     <em><strong>(AG)</strong></em></span></p>
<p><span><span><strong><br>Note: </strong>Final line must be seen or previous</span> <span><em><strong>(A1)</strong></em> mark is lost.</span></span></p>
<p><span><span><em><strong>[2 marks]</strong></em><br></span></span></p>
<div class="question_part_label">ii.a.</div>
</div>
<div class="question" style="padding-left: 20px;">
<p><span>\(6xy = 300 - 4{x^2}\)     <em><strong>(M1)</strong></em></span></p>
<p><span>\(y = \frac{{300 - 4{x^2}}}{{6x}}\) <em>or</em> \(\frac{{150 - 2{x^2}}}{{3x}}\)     <em><strong>(A1)</strong></em></span></p>
<p><span><em><strong>[2 marks]<br></strong></em></span></p>
<div class="question_part_label">ii.b.</div>
</div>
<div class="question" style="padding-left: 20px;">
<p><span>\({\text{Volume}} = x(2x)y\)     <em><strong>(M1)</strong></em></span></p>
<p><span>\(V = 2{x^2}\left( {\frac{{300 - 4{x^2}}}{{6x}}} \right)\)     <em><strong>(A1)</strong></em><strong>(ft)</strong></span></p>
<p><span>\( = 100x - \frac{4}{3}{x^3}\)     <strong>(AG)</strong></span></p>
<p><br><span><strong>Note: </strong>Final line must be seen or previous <em><strong>(A1)</strong></em> mark is lost.</span></p>
<p><span><em><strong>[2 marks]</strong></em><br></span></p>
<div class="question_part_label">ii.c.</div>
</div>
<div class="question" style="padding-left: 20px;">
<p><span>\(\frac{{{\text{d}}V}}{{{\text{d}}x}} = 100 - \frac{{12{x^2}}}{3}\)  or  \(100 - 4{x^2}\)     <em><strong>(A1)(A1)</strong></em></span></p>
<p><span><em><strong> </strong></em><strong>Note: <em>(A1)</em></strong> for each term.</span></p>
<p><span><em><strong>[2 marks]</strong></em><br></span></p>
<div class="question_part_label">ii.d.</div>
</div>
<div class="question" style="padding-left: 20px;">
<p><span><em>Unit penalty <strong>(UP)</strong> is applicable where indicated in the left hand column</em></span></p>
<p><span>(i)     For maximum \(\frac{{{\text{d}}V}}{{{\text{d}}x}} = 0\)  or  \(100 - 4{x^2} = 0\)     <em><strong>(M1)</strong></em></span></p>
<p><span>\(x = 5\)     <em><strong>(A1)</strong></em><strong>(ft)</strong></span></p>
<p><span>\(y = \frac{{300 - 4{{(5)}^2}}}{{6(5)}}\)  or  \(\left( {\frac{{150 - 2{{(5)}^2}}}{{3(5)}}} \right)\)     <em><strong>(M1)</strong></em></span></p>
<p><span>\( = \frac{{20}}{3}\)     <em><strong>(A1)</strong></em><strong>(ft)</strong></span></p>
<p><span><em><strong>(UP)</strong></em>     (ii)    \(333\frac{1}{3}{\text{ c}}{{\text{m}}^3}{\text{ }}(333{\text{ c}}{{\text{m}}^3})\)</span></p>
<p><span><strong><br>Note: (ft)</strong> from their (e)(i) if working for volume is seen.</span></p>
<p><span><em><strong>[5 marks]</strong></em><br></span></p>
<div class="question_part_label">ii.e.</div>
</div>
<h2 style="margin-top: 1em">Examiners report</h2>
<div class="question" style="padding-left: 20px;">
<p><span style="font-family: times new roman,times; font-size: medium;">Most candidates made a good attempt to factorise the expression.</span></p>
<div class="question_part_label">i.a.</div>
</div>
<div class="question" style="padding-left: 20px;">
<p><span style="font-family: times new roman,times; font-size: medium;">Many gained both marks here from a correct answer or ft from the previous part.</span></p>
<div class="question_part_label">i.b.</div>
</div>
<div class="question" style="padding-left: 20px;">
<p><span style="font-family: times new roman,times; font-size: medium;">Many used the formula correctly. Some forgot to put \(x = \)</span> .</p>
<div class="question_part_label">i.c.</div>
</div>
<div class="question" style="padding-left: 20px;">
<p><span style="font-family: times new roman,times; font-size: medium;">Most candidates found this value from their GDC.</span></p>
<div class="question_part_label">i.d.</div>
</div>
<div class="question" style="padding-left: 20px;">
<p><span style="font-family: times new roman,times; font-size: medium;">A good attempt was made to show the correct surface area.</span></p>
<div class="question_part_label">ii.a.</div>
</div>
<div class="question" style="padding-left: 20px;">
<p><span style="font-family: times new roman,times; font-size: medium;">Many could rearrange the equation correctly.</span></p>
<div class="question_part_label">ii.b.</div>
</div>
<div class="question" style="padding-left: 20px;">
<p><span style="font-family: times new roman,times; font-size: medium;">Although this was not a difficult question it probably looked complicated for the candidates and it was often left out.</span></p>
<div class="question_part_label">ii.c.</div>
</div>
<div class="question" style="padding-left: 20px;">
<p><span style="font-family: times new roman,times; font-size: medium;">Those who reached this length could usually manage the differentiation.</span></p>
<div class="question_part_label">ii.d.</div>
</div>
<div class="question" style="padding-left: 20px;">
<p><span style="font-family: times new roman,times; font-size: medium;">(i)&nbsp;&nbsp;&nbsp;&nbsp; Many found the correct value of \(x\) but not of \(y\).</span></p>
<p><span style="font-family: times new roman,times; font-size: medium;">(ii)&nbsp;&nbsp;&nbsp; This was well done and again the units were included in most scripts.</span></p>
<div class="question_part_label">ii.e.</div>
</div>
<br><hr><br><div class="specification">
<p><span style="font-family: times new roman,times; font-size: medium;"><strong>Throughout this question <em>all</em> the numerical answers must be given correct to the nearest whole number.</strong></span></p>
</div>

<div class="question" style="padding-left: 20px; padding-right: 20px;">
<p><span>Park School started in January 2000 with \(100\) students. Every full year, there is an increase of \(6\% \) in the number of students.</span></p>
<p><span>Find the number of students attending Park School in</span></p>
<p><span>(i)     January 2001;</span></p>
<p><span>(ii)    January 2003.</span></p>
<div class="marks">[4]</div>
<div class="question_part_label">a.</div>
</div>
<div class="question" style="padding-left: 20px; padding-right: 20px;">
<p><span>Park School started in January 2000 with \(100\) students. Every full year, there is an increase of \(6\% \) in the number of students.</span></p>
<p><span>Show that the number of students attending Park School in January 2007 is \(150\).</span></p>
<div class="marks">[2]</div>
<div class="question_part_label">b.</div>
</div>
<div class="question" style="padding-left: 20px; padding-right: 20px;">
<p><span>Grove School had \(110\) students in January 2000. Every full year, the number of students is \(10\) more than in the previous year.</span></p>
<p><span>Find the number of students attending Grove School in January 2003.</span></p>
<div class="marks">[2]</div>
<div class="question_part_label">c.</div>
</div>
<div class="question" style="padding-left: 20px; padding-right: 20px;">
<p><span>Grove School had \(110\) students in January 2000. Every full year, the number of students is \(10\) more than in the previous year.</span></p>
<p><span>Find the year in which the number of students attending Grove School will be first \(60\% \) <strong>more than</strong> in January 2000.</span></p>
<div class="marks">[4]</div>
<div class="question_part_label">d.</div>
</div>
<div class="question" style="padding-left: 20px; padding-right: 20px;">
<p><span>Each January, one of these two schools, the one that has more students, is given extra money to spend on sports equipment.</span></p>
<p><span>(i)     Decide which school gets the money in 2007. Justify your answer.</span></p>
<p><span>(ii)    Find the first year in which Park School will be given this extra money.</span></p>
<div class="marks">[5]</div>
<div class="question_part_label">e.</div>
</div>
<h2 style="margin-top: 1em">Markscheme</h2>
<div class="question" style="padding-left: 20px;">
<p><span>(i)     \(100 \times 1.06 = 106\)     <em><strong>(M1)(A1)(G2)</strong></em></span></p>
<p><br><span><strong>Note: <em>(M1)</em></strong> for multiplying by \(1.06\) or equivalent. <em><strong>(A1)</strong></em> for correct answer.</span></p>
<p><br><span>(ii)    \(100 \times {1.06^3} = 119\)     <em><strong>(M1)(A1)(G2)</strong></em></span></p>
<p><br><span><strong>Note:</strong> <em><strong>(M1)</strong></em> for multiplying by \({1.06^3}\) or equivalent or for list of values. <em><strong>(A1)</strong></em> for correct answer.</span></p>
<p><span><em><strong>[4 marks]</strong></em><br></span></p>
<div class="question_part_label">a.</div>
</div>
<div class="question" style="padding-left: 20px;">
<p><span>\(100 \times {1.06^7} = 150.36 \ldots  = 150\) correct to the nearest whole     <em><strong>(M1)(A1)(AG)</strong></em></span></p>
<p><br><span><strong>Note: <em>(M1)</em></strong> for correct formula or for list of values. <em><strong>(A1)</strong></em> for correct substitution or for \(150\) in the correct position in the list. Unrounded answer must be seen for the <em><strong>(A1)</strong></em>.</span></p>
<p><span><em><strong>[2 marks]</strong></em><br></span></p>
<div class="question_part_label">b.</div>
</div>
<div class="question" style="padding-left: 20px;">
<p><span>\(110 + 3 \times 10 = 140\)     <em><strong>(M1)(A1)(G2)</strong></em></span></p>
<p><br><span><strong>Note: <em>(M1)</em></strong> for adding \(30\) or for list of values. <em><strong>(A1)</strong></em> for correct answer.</span></p>
<p><span><em><strong>[2 marks]</strong></em><br></span></p>
<div class="question_part_label">c.</div>
</div>
<div class="question" style="padding-left: 20px;">
<p><span><strong>In (d) and (e) follow through from (c) if consistent wrong use of correct AP formula.</strong></span></p>
<p><span>\(110 + (n - 1) \times 10 &gt; 176\)     <em><strong>(A1)(M1)</strong></em></span></p>
<p><span>\(n = 8\therefore {\text{year 2007}}\)     <em><strong>(A1)(A1)</strong></em><strong>(ft)</strong><em><strong>(G2)</strong></em></span></p>
<p><br><span><strong>Note: <em>(A1)</em></strong> for \(176\) or \(66\) seen. <em><strong>(M1)</strong></em> for showing list of values and comparing them to \(176\) or for equating formula to \(176\) or for writing the inequality. If \(n = 8\) not seen can still get <em><strong>(A2)</strong></em> for 2007. Answer \(n = 8\) with no working gets <em><strong>(G1)</strong></em>.</span></p>
<p><br><span><strong>OR</strong></span></p>
<p><span>\(110 + n \times 10 &gt; 176\)     <em><strong>(A1)(M1)</strong></em></span></p>
<p><span>\(n = 7\therefore {\text{year 2007}}\)     <em><strong>(A1)(A1)</strong></em><strong>(ft)<em>(G2)</em></strong></span></p>
<p><span><strong><em>[4 marks]<br></em></strong></span></p>
<div class="question_part_label">d.</div>
</div>
<div class="question" style="padding-left: 20px;">
<p><span><strong>In (d) and (e) follow through from (c) if consistent wrong use of correct AP formula.</strong></span></p>
<p><span>(i)     \(180\)     <em><strong>(A1)</strong></em><strong>(ft)</strong><br></span></p>
<p><span>Grove School gets the money.     <em><strong>(A1)</strong></em><strong>(ft)</strong></span></p>
<p><span><br><strong>Note: <em>(A1)</em></strong> for \(180\) seen. <em><strong>(A1)</strong></em> for correct answer.</span></p>
<p><span><br>(ii)    \({\text{100}} \times {\text{1}}{\text{.0}}{{\text{6}}^{n - 1}} &gt; 110 + (n - 1) \times 10\)     <em><strong>(M1)</strong></em>    </span></p>
<p><span>\(n = 20\therefore {\text{year 2019}}\)     <em><strong>(A1)(A1)</strong></em><strong>(ft)<em>(G2)</em></strong><br></span></p>
<p><span><br><strong>Note: <em>(M1)</em></strong> for showing lists of values for each school and comparing them or for equating both formulae or writing the correct inequality. If \(n = 20\) not seen can still get <em><strong>(A2)</strong></em> for 2019. Follow through with ratio used in (b) and/or formula used in (d).</span></p>
<p><span><br><strong>OR</strong></span></p>
<p><span>\(100 \times {1.06^n} &gt; 110 + n \times 10\)     <em><strong>(M1)</strong></em><br></span></p>
<p><span>\(n = 19\therefore {\text{year 2019}}\)     <em><strong>(A1)(A1)</strong></em><strong>(ft)</strong><em><strong>(G2)</strong></em><br></span></p>
<p><span><strong>OR</strong></span></p>
<p><span>graphically</span></p>
<p><span><br><strong>Note: <em>(M1)</em></strong> for sketch of both functions on the same graph, <em><strong>(A1)</strong></em> for the intersection point, <em><strong>(A1)</strong></em> for correct answer.</span></p>
<p><span><em><strong>[5 marks]</strong></em><br></span></p>
<div class="question_part_label">e.</div>
</div>
<h2 style="margin-top: 1em">Examiners report</h2>
<div class="question" style="padding-left: 20px;">
<p><span style="font-family: times new roman,times; font-size: medium;">This question was well answered by the majority of the candidates. Most of the candidates were able to distinguish between the arithmetic and the geometric progression. A number of candidates worked out term by term by hand for which they needed more time than those that used the formulae to find the requested terms. Some of the students that found the terms the long way also lost a mark for premature rounding. It was pleasing to see how the last part of the question was answered using different methods. Those candidates that worked throughout the question using AP and GP formulae used either the solver or a graph to find the solution of the inequality. Those candidates that worked throughout the question in the long way also managed to compare the terms and find the correct year.</span></p>
<div class="question_part_label">a.</div>
</div>
<div class="question" style="padding-left: 20px;">
<p><span style="font-family: times new roman,times; font-size: medium;">This question was well answered by the majority of the candidates. Most of the candidates were able to distinguish between the arithmetic and the geometric progression. A number of candidates worked out term by term by hand for which they needed more time than those that used the formulae to find the requested terms. Some of the students that found the terms the long way also lost a mark for premature rounding. It was pleasing to see how the last part of the question was answered using different methods. Those candidates that worked throughout the question using AP and GP formulae used either the solver or a graph to find the solution of the inequality. Those candidates that worked throughout the question in the long way also managed to compare the terms and find the correct year.</span></p>
<div class="question_part_label">b.</div>
</div>
<div class="question" style="padding-left: 20px;">
<p><span style="font-family: times new roman,times; font-size: medium;">This question was well answered by the majority of the candidates. Most of the candidates were able to distinguish between the arithmetic and the geometric progression. A number of candidates worked out term by term by hand for which they needed more time than those that used the formulae to find the requested terms. Some of the students that found the terms the long way also lost a mark for premature rounding. It was pleasing to see how the last part of the question was answered using different methods. Those candidates that worked throughout the question using AP and GP formulae used either the solver or a graph to find the solution of the inequality. Those candidates that worked throughout the question in the long way also managed to compare the terms and find the correct year.</span></p>
<div class="question_part_label">c.</div>
</div>
<div class="question" style="padding-left: 20px;">
<p><span style="font-family: times new roman,times; font-size: medium;">This question was well answered by the majority of the candidates. Most of the candidates were able to distinguish between the arithmetic and the geometric progression. A number of candidates worked out term by term by hand for which they needed more time than those that used the formulae to find the requested terms. Some of the students that found the terms the long way also lost a mark for premature rounding. It was pleasing to see how the last part of the question was answered using different methods. Those candidates that worked throughout the question using AP and GP formulae used either the solver or a graph to find the solution of the inequality. Those candidates that worked throughout the question in the long way also managed to compare the terms and find the correct year.</span></p>
<div class="question_part_label">d.</div>
</div>
<div class="question" style="padding-left: 20px;">
<p><span style="font-family: times new roman,times; font-size: medium;">This question was well answered by the majority of the candidates. Most of the candidates were able to distinguish between the arithmetic and the geometric progression. A number of candidates worked out term by term by hand for which they needed more time than those that used the formulae to find the requested terms. Some of the students that found the terms the long way also lost a mark for premature rounding. It was pleasing to see how the last part of the question was answered using different methods. Those candidates that worked throughout the question using AP and GP formulae used either the solver or a graph to find the solution of the inequality. Those candidates that worked throughout the question in the long way also managed to compare the terms and find the correct year.</span></p>
<div class="question_part_label">e.</div>
</div>
<br><hr><br><div class="specification">
<p style="margin: 0.0px 0.0px 0.0px 0.0px; font: 21.0px 'Times New Roman';"><span style="font-family: 'times new roman', times; font-size: medium;">A parcel is in the shape of a rectangular prism, as shown in the diagram. It has a length \(l\) cm, width \(w\) cm and height of \(20\) cm.</span></p>
<p style="margin: 0.0px 0.0px 0.0px 0.0px; font: 21.0px 'Times New Roman';"><span style="font-family: 'times new roman', times; font-size: medium;">The total volume of the parcel is \(3000{\text{ c}}{{\text{m}}^3}\).</span></p>
</div>

<div class="question" style="padding-left: 20px; padding-right: 20px;">
<p><span>Express the volume of the parcel in terms of \(l\) and \(w\).</span></p>
<div class="marks">[1]</div>
<div class="question_part_label">a.</div>
</div>
<div class="question" style="padding-left: 20px; padding-right: 20px;">
<p><span>Show that \(l = \frac{{150}}{w}\).</span></p>
<div class="marks">[2]</div>
<div class="question_part_label">b.</div>
</div>
<div class="question" style="padding-left: 20px; padding-right: 20px;">
<p><span>The parcel is tied up using a length of string that fits <strong>exactly </strong>around the parcel, as shown in the following diagram.</span></p>
<p><span><br><img src="images/Schermafbeelding_2014-09-02_om_11.55.29_5.png" alt><br></span></p>
<p><span>Show that the length of string, \(S\) cm, required to tie up the parcel can be written as</span></p>
<p><span>\[S = 40 + 4w + \frac{{300}}{w},{\text{ }}0 &lt; w \leqslant 20.\]</span></p>
<p><span> </span></p>
<div class="marks">[2]</div>
<div class="question_part_label">c.</div>
</div>
<div class="question" style="padding-left: 20px; padding-right: 20px;">
<p><span>The parcel is tied up using a length of string that fits <strong>exactly </strong>around the parcel, as shown in the following diagram.</span></p>
<p><span><br><img src="images/Schermafbeelding_2014-09-02_om_11.55.29_4.png" alt><br></span></p>
<p><span>Draw the graph of \(S\) for \(0 &lt; w \leqslant 20\) and \(0 &lt; S \leqslant 500\), clearly showing the local minimum point. Use a scale of \(2\) cm to represent \(5\) units on the horizontal axis \(w\)<em> </em>(cm), and a scale of \(2\) cm to represent \(100\) units on the vertical axis \(S\) (cm).</span></p>
<div class="marks">[2]</div>
<div class="question_part_label">d.</div>
</div>
<div class="question" style="padding-left: 20px; padding-right: 20px;">
<p><span>The parcel is tied up using a length of string that fits <strong>exactly </strong>around the parcel, as shown in the following diagram.</span></p>
<p><span><br><img src="images/Schermafbeelding_2014-09-02_om_11.55.29_3.png" alt><br></span></p>
<p><span>Find \(\frac{{{\text{d}}S}}{{{\text{d}}w}}\).</span></p>
<div class="marks">[3]</div>
<div class="question_part_label">e.</div>
</div>
<div class="question" style="padding-left: 20px; padding-right: 20px;">
<p><span>The parcel is tied up using a length of string that fits <strong>exactly </strong>around the parcel, as shown in the following diagram.</span></p>
<p><span><br><img src="images/Schermafbeelding_2014-09-02_om_11.55.29_2.png" alt><br></span></p>
<p><span>Find the value of \(w\) for which \(S\) is a minimum.</span></p>
<div class="marks">[2]</div>
<div class="question_part_label">f.</div>
</div>
<div class="question" style="padding-left: 20px; padding-right: 20px;">
<p><span>The parcel is tied up using a length of string that fits <strong>exactly </strong>around the parcel, as shown in the following diagram.</span></p>
<p><span><br><img src="images/Schermafbeelding_2014-09-02_om_11.55.29.png" alt><br></span></p>
<p><span>Write down the value, \(l\), of the parcel for which the length of string is a minimum.</span></p>
<div class="marks">[1]</div>
<div class="question_part_label">g.</div>
</div>
<div class="question" style="padding-left: 20px; padding-right: 20px;">
<p><span>The parcel is tied up using a length of string that fits <strong>exactly </strong>around the parcel, as shown in the following diagram.</span></p>
<p><span><br><img src="images/Schermafbeelding_2014-09-02_om_11.55.29_1.png" alt><br></span></p>
<p><span>Find the minimum length of string required to tie up the parcel.</span></p>
<div class="marks">[2]</div>
<div class="question_part_label">h.</div>
</div>
<h2 style="margin-top: 1em">Markscheme</h2>
<div class="question" style="padding-left: 20px;">
<p><span>\(20lw\)   <strong>OR</strong>   \(V = 20lw\)     <strong><em>(A1)</em></strong></span></p>
<p><span><strong><em>[1 mark]</em></strong></span></p>
<div class="question_part_label">a.</div>
</div>
<div class="question" style="padding-left: 20px;">
<p><span>\(3000 = 20lw\)     <strong><em>(M1)</em></strong></span></p>
<p> </p>
<p><span><strong>Note: </strong>Award <strong><em>(M1) </em></strong>for equating their answer to part (a) to \(3000\).</span></p>
<p> </p>
<p><span>\(l = \frac{{3000}}{{20w}}\)     <strong><em>(M1)</em></strong></span></p>
<p> </p>
<p><span><strong>Note: </strong>Award <strong><em>(M1) </em></strong>for rearranging equation to make \(l\) subject of the formula. The above equation must be seen to award <strong><em>(M1)</em></strong>.</span></p>
<p> </p>
<p><span><strong>OR</strong></span></p>
<p><span>\(150 = lw\)     <strong><em>(M1)</em></strong></span></p>
<p> </p>
<p><span><strong>Note: </strong>Award <strong><em>(M1) </em></strong>for division by \(20\) on both sides. The above equation must be seen to award <strong><em>(M1)</em></strong>.</span></p>
<p> </p>
<p><span>\(l = \frac{{150}}{w}\)     <strong><em>(AG)</em></strong></span></p>
<p><span><strong><em>[2 marks]</em></strong></span></p>
<div class="question_part_label">b.</div>
</div>
<div class="question" style="padding-left: 20px;">
<p><span>\(S = 2l + 4w + 2(20)\)     <strong><em>(M1)</em></strong></span></p>
<p> </p>
<p><span><strong>Note: </strong>Award <strong><em>(M1) </em></strong>for setting up a correct expression for \(S\).</span></p>
<p> </p>
<p><span>\(2\left( {\frac{{150}}{w}} \right) + 4w + 2(20)\)     <strong><em>(M1)</em></strong></span></p>
<p> </p>
<p><span><strong>Notes: </strong>Award <strong><em>(M1) </em></strong>for correct substitution into the expression for \(S\). The above expression must be seen to award <strong><em>(M1)</em></strong>.</span></p>
<p> </p>
<p><span>\( = 40 + 4w + \frac{{300}}{w}\)     <strong><em>(AG)</em></strong></span></p>
<p><span><strong><em>[2 marks]</em></strong></span></p>
<div class="question_part_label">c.</div>
</div>
<div class="question" style="padding-left: 20px;">
<p><span><span><span><strong><br><img src="images/curvy.jpg" alt>    </strong> <strong><em>(A1)(A1)(A1)(A1)</em></strong></span></span></span></p>
<p><span><strong><em> </em></strong></span></p>
<p><span><strong>Note: </strong>Award <strong><em>(A1) </em></strong>for correct scales, window and labels on axes, <strong><em>(A1) </em></strong>for approximately correct shape, <strong><em>(A1) </em></strong>for minimum point in approximately correct position, <strong><em>(A1) </em></strong>for asymptotic behaviour at \(w = 0\).</span></p>
<p><span>     Axes must be drawn with a ruler and labeled \(w\) and \(S\)<em>.</em></span></p>
<p><span>     For a smooth curve (with approximately correct shape) there should be <strong>one </strong>continuous thin line, no part of which is straight and no (one-to-many) mappings of \(w\).</span></p>
<p><span>     The \(S\)-axis must be an asymptote. The curve must not touch the \(S\)-axis nor must the curve approach the asymptote then deviate away later.</span></p>
<p> </p>
<p><span><strong><em>[4 marks]</em></strong></span></p>
<div class="question_part_label">d.</div>
</div>
<div class="question" style="padding-left: 20px;">
<p><span>\(4 - \frac{{300}}{{{w^2}}}\)     <strong><em>(A1)(A1)(A1)</em></strong></span></p>
<p> </p>
<p><span><strong>Notes: </strong>Award <strong><em>(A1) </em></strong>for \(4\), <strong><em>(A1) </em></strong>for \(-300\), <strong><em>(A1) </em></strong>for \(\frac{1}{{{w^2}}}\) or \({w^{ - 2}}\). If extra terms present, award at most <strong><em>(A1)(A1)(A0)</em></strong><em>.</em></span></p>
<p><span><em> </em></span></p>
<p><span><strong><em>[3 marks]</em></strong></span></p>
<div class="question_part_label">e.</div>
</div>
<div class="question" style="padding-left: 20px;">
<p><span>\(4 - \frac{{300}}{{{w^2}}} = 0\)   <strong>OR</strong>   \(\frac{{300}}{{{w^2}}} = 4\)   <strong>OR</strong>   \(\frac{{{\text{d}}S}}{{{\text{d}}w}} = 0\)     <strong><em>(M1)</em></strong></span></p>
<p> </p>
<p><span><strong>Note: </strong>Award <strong><em>(M1) </em></strong>for equating their derivative to zero.</span></p>
<p> </p>
<p><span>\(w = 8.66{\text{ }}\left( {\sqrt {75} ,{\text{ 8.66025}} \ldots } \right)\)     <strong><em>(A1)</em>(ft)<em>(G2)</em></strong></span></p>
<p> </p>
<p><span><strong>Note: </strong>Follow through from their answer to part (e).</span></p>
<p> </p>
<p><span><strong><em>[2 marks]</em></strong></span></p>
<div class="question_part_label">f.</div>
</div>
<div class="question" style="padding-left: 20px;">
<p><span>\(17.3 \left( {\frac{{150}}{{\sqrt {75} }},{\text{ 17.3205}} \ldots } \right)\)     <strong><em>(A1)</em>(ft)</strong></span></p>
<p> </p>
<p><span><strong>Note: </strong>Follow through from their answer to part (f).</span></p>
<p> </p>
<p><span><strong><em>[1 mark]</em></strong></span></p>
<div class="question_part_label">g.</div>
</div>
<div class="question" style="padding-left: 20px;">
<p><span>\(40 + 4\sqrt {75}  + \frac{{300}}{{\sqrt {75} }}\)     <strong><em>(M1)</em></strong></span></p>
<p> </p>
<p><span><strong>Note: </strong>Award <strong><em>(M1) </em></strong>for substitution of their answer to part (f) into the expression for \(S\).</span></p>
<p> </p>
<p><span>\( = 110{\text{ (cm) }}\left( {40 + 40\sqrt 3 ,{\text{ 109.282}} \ldots } \right)\)     <strong><em>(A1)</em>(ft)<em>(G2)</em></strong></span></p>
<p> </p>
<p><span><strong>Note: </strong>Do not accept \(109\).</span></p>
<p><span>     Follow through from their answers to parts (f) and (g).</span></p>
<p> </p>
<p><span><strong><em>[2 marks]</em></strong></span></p>
<div class="question_part_label">h.</div>
</div>
<h2 style="margin-top: 1em">Examiners report</h2>
<div class="question" style="padding-left: 20px;">
[N/A]
<div class="question_part_label">a.</div>
</div>
<div class="question" style="padding-left: 20px;">
[N/A]
<div class="question_part_label">b.</div>
</div>
<div class="question" style="padding-left: 20px;">
[N/A]
<div class="question_part_label">c.</div>
</div>
<div class="question" style="padding-left: 20px;">
[N/A]
<div class="question_part_label">d.</div>
</div>
<div class="question" style="padding-left: 20px;">
[N/A]
<div class="question_part_label">e.</div>
</div>
<div class="question" style="padding-left: 20px;">
[N/A]
<div class="question_part_label">f.</div>
</div>
<div class="question" style="padding-left: 20px;">
[N/A]
<div class="question_part_label">g.</div>
</div>
<div class="question" style="padding-left: 20px;">
[N/A]
<div class="question_part_label">h.</div>
</div>
<br><hr><br><div class="specification">
<p><span style="font-size: medium; font-family: times new roman,times;">Consider the function \(g(x) = bx - 3 + \frac{1}{{{x^2}}},{\text{ }}x \ne 0\).</span></p>
</div>

<div class="question" style="padding-left: 20px; padding-right: 20px;">
<p><span>Write down the equation of the vertical asymptote of the graph of <em>y</em> = <em>g</em>(<em>x</em>) .</span></p>
<div class="marks">[2]</div>
<div class="question_part_label">a.</div>
</div>
<div class="question" style="padding-left: 20px; padding-right: 20px;">
<p><span>Write down <em>g</em>′(<em>x</em>) .</span></p>
<div class="marks">[3]</div>
<div class="question_part_label">b.</div>
</div>
<div class="question" style="padding-left: 20px; padding-right: 20px;">
<p><span>The line <em>T</em> is the tangent to the graph of <em>y</em> = <em>g</em>(<em>x</em>) at the point where <em>x</em> = 1. The gradient of <em>T</em> is 3.</span></p>
<p><span>Show that <em>b</em> = 5.</span></p>
<div class="marks">[2]</div>
<div class="question_part_label">c.</div>
</div>
<div class="question" style="padding-left: 20px; padding-right: 20px;">
<p><span><span>The line <em>T</em> is the tangent to the graph of <em>y</em> = <em>g</em>(<em>x</em>) at the point where <em>x</em> = 1. The gradient of <em>T</em> is 3.</span></span></p>
<p><span>Find the equation of <em>T</em>.</span></p>
<div class="marks">[3]</div>
<div class="question_part_label">d.</div>
</div>
<div class="question" style="padding-left: 20px; padding-right: 20px;">
<p><span>Using your graphic display calculator find the coordinates of the point where the graph of <em>y</em> = <em>g</em>(<em>x</em>) intersects the <em>x</em>-axis.</span></p>
<div class="marks">[2]</div>
<div class="question_part_label">e.</div>
</div>
<div class="question" style="padding-left: 20px; padding-right: 20px;">
<p><span>(i) Sketch the graph of <em>y</em> = <em>g</em>(<em>x</em>) for −2 ≤ <em>x</em> ≤ 5 and −15 ≤ <em>y</em> ≤ 25, indicating clearly your answer to part (e).</span></p>
<p><span>(ii) Draw the line <em>T</em> on your sketch.</span></p>
<div class="marks">[6]</div>
<div class="question_part_label">f.</div>
</div>
<div class="question" style="padding-left: 20px; padding-right: 20px;">
<p><span>Using your graphic display calculator find the coordinates of the local minimum point of <em>y</em> = <em>g</em>(<em>x</em>) .</span></p>
<div class="marks">[2]</div>
<div class="question_part_label">g.</div>
</div>
<div class="question" style="padding-left: 20px; padding-right: 20px;">
<p><span>Write down the interval for which <em>g</em>(<em>x</em>) is increasing in the domain 0 &lt; <em>x</em> &lt; 5 .</span></p>
<div class="marks">[2]</div>
<div class="question_part_label">h.</div>
</div>
<h2 style="margin-top: 1em">Markscheme</h2>
<div class="question" style="padding-left: 20px;">
<p><span><em>x</em> = 0     <em><strong>(A1)(A1)</strong></em></span></p>
<p><span><strong>Notes:</strong> Award <em><strong>(A1)</strong></em> for <em>x</em>=constant, <em><strong>(A1)</strong></em> for 0. Award <em><strong>(A0)(A0)</strong></em> if answer is not an equation.</span></p>
<p><em><strong><span>[2 marks]</span></strong></em></p>
<div class="question_part_label">a.</div>
</div>
<div class="question" style="padding-left: 20px;">
<p><span>\(b - \frac{2}{{{x^3}}}\)     <em><strong>(A1)(A1)(A1)</strong></em></span></p>
<p><span><strong>Note:</strong> Award <em><strong>(A1)</strong></em> for <em>b</em>, <em><strong>(A1)</strong></em> for </span><span><span>−</span>2, <em><strong>(A1)</strong></em> for \(\frac{1}{{{x^3}}}\) (or <em>x</em><sup>−3</sup>). Award at most <em><strong>(A1)(A1)(A0)</strong></em> if extra terms seen.</span></p>
<p><em><strong><span>[3 marks]</span></strong></em></p>
<div class="question_part_label">b.</div>
</div>
<div class="question" style="padding-left: 20px;">
<p><span>\(3 = b - \frac{2}{{{{(1)}^3}}}\)     <em><strong>(M1)(M1)</strong></em></span></p>
<p><span><strong>Note:</strong> Award <em><strong>(M1)</strong></em> for substituting 1 into their gradient function, <em><strong>(M1)</strong></em> for equating their gradient function to 3.</span></p>
<p> </p>
<p><span><em>b</em> = 5     <em><strong>(AG)</strong></em></span></p>
<p><span><strong>Note:</strong> Award at most <em><strong>(M1)(A0)</strong></em> if final line is not seen or <em>b</em> does not equal 5.</span></p>
<p><em><strong><span>[2 marks]</span></strong></em></p>
<div class="question_part_label">c.</div>
</div>
<div class="question" style="padding-left: 20px;">
<p><span><em>g</em>(1) = 3 <strong>or</strong> (1, 3)<em> (seen or implied from the line below)</em>     <em><strong>(A1)</strong></em></span></p>
<p><span>3 = 3 </span><span><span>× </span>1 + <em>c</em>     <em><strong>(M1)</strong></em></span></p>
<p><span><strong>Note:</strong> Award <em><strong>(M1)</strong></em> for correct substitution of their point (1, 3) and gradient 3 into equation <em>y</em> = <em>mx</em> + <em>c</em>. Follow through from their point of tangency.</span></p>
<p> </p>
<p><span><em>y</em> = </span><span>3<em>x</em>     <em><strong>(A1)</strong></em><strong>(ft)</strong><em><strong>(G2)</strong></em></span></p>
<p><strong><span>OR</span></strong></p>
<p><em><span>y </span></em><span><span>− </span>3 = 3(<em>x </em></span><span><span>− </span>1)     <em><strong>(M1)(A1)</strong></em><strong>(ft)</strong><em><strong>(G2)</strong></em></span> </p>
<p><span><strong>Note:</strong> Award <em><strong>(M1)</strong></em> for substitution of gradient 3 and their point (1, 3) into <em>y </em></span><span><span>− </span><em>y</em><sub>1</sub> = <em>m</em>(<em>x </em></span><span><span>− </span><em>x</em><sub>1</sub>), <em><strong>(A1)</strong></em><strong>(ft)</strong> for correct substitutions. Follow through from their point of tangency. Award at most <em><strong>(A1)(M1)(A0)</strong></em><strong>(ft)</strong> if further incorrect working seen.</span></p>
<p><em><strong><span>[3 marks]</span></strong></em></p>
<div class="question_part_label">d.</div>
</div>
<div class="question" style="padding-left: 20px;">
<p><span>(−0.439, 0)   ((−0.438785..., 0))     <em><strong>(G1)(G1)</strong></em></span></p>
<p><span><strong>Notes:</strong> If no parentheses award at most <em><strong>(G1)(G0)</strong></em>. Accept <em>x</em> = 0.439, <em>y</em> = 0.</span></p>
<p><em><strong><span>[2 marks]</span></strong></em></p>
<div class="question_part_label">e.</div>
</div>
<div class="question" style="padding-left: 20px;">
<p><span>(i)</span></p>
<p><span><img src="data:image/png;base64,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" alt></span></p>
<p><span>Award <em><strong>(A1)</strong></em> for labels and some indication of scale in the stated window.</span></p>
<p><span>Award <em><strong>(A1)</strong></em> for correct general shape (curve must be smooth and must not cross the y-axis)</span></p>
<p><span>Award <em><strong>(A1)</strong></em><strong>(ft)</strong> for <em>x</em>-intercept consistent with their part (e).</span></p>
<p><span>Award <em><strong>(A1)</strong></em> for local minimum in the first quadrant.     <em><strong>(A1)(A1)(A1)</strong></em><strong>(ft)</strong><em><strong>(A1)</strong></em></span></p>
<p> </p>
<p><span>(ii) Tangent to curve drawn at approximately <em>x</em> = 1     <em><strong>(A1)(A1)</strong></em></span> </p>
<p><span><strong>Note:</strong> Award <em><strong>(A1)</strong></em> for a line tangent to curve approximately at <em>x</em> = 1. Must be a straight line for the mark to be awarded. Award <em><strong>(A1)</strong></em><strong>(ft)</strong> for line passing through the origin. Follow through from their answer to part (d).</span></p>
<p><em><strong><span>[6 marks]</span></strong></em></p>
<div class="question_part_label">f.</div>
</div>
<div class="question" style="padding-left: 20px;">
<p><span>(0.737, 2.53) ((0.736806..., 2.52604...))     <em><strong>(G1)(G1)</strong></em></span></p>
<p><span><strong>Notes:</strong> Do not penalize for lack of parentheses if already penalized in (e). Accept<em> x</em> = 0.737, <em>y</em> = 2.53.</span></p>
<p><em><strong><span>[2 marks]</span></strong></em></p>
<div class="question_part_label">g.</div>
</div>
<div class="question" style="padding-left: 20px;">
<p><span>0.737 &lt; </span><span><span><em>x</em> &lt; </span>5  <strong>OR</strong>  (0.737;5)     <em><strong>(A1)(A1)</strong></em><strong>(ft)</strong></span></p>
<p><span><strong>Notes:</strong> Award <em><strong>(A1)</strong></em> for correct strict or weak inequalities with <em>x</em> seen if the interval is given as inequalities, <em><strong>(A1)</strong></em><strong>(ft)</strong> for 0.737 and 5 or their value from part (g).</span></p>
<p><em><strong><span>[2 marks]</span></strong></em></p>
<div class="question_part_label">h.</div>
</div>
<h2 style="margin-top: 1em">Examiners report</h2>
<div class="question" style="padding-left: 20px;">
<p><span style="font-size: medium; font-family: times new roman,times;">This question was moderately well answered. The concept of vertical asymptote in part (a) seemed to be problematic for a great number of candidates. In many cases students showed partial understanding of the vertical asymptote but found it difficult to write a correct equation.</span></p>
<div class="question_part_label">a.</div>
</div>
<div class="question" style="padding-left: 20px;">
<p><span style="font-size: medium; font-family: times new roman,times;">This question was moderately well answered. The concept of vertical asymptote in part (a) seemed to be problematic for a great number of candidates. In many cases students showed partial understanding of the vertical asymptote but found it difficult to write a correct equation. Finding the derivative in part (b) proved problematic as well. It seems that the presence of the parameter <em>b</em> in the function may have contributed to this.</span></p>
<p>&nbsp;</p>
<div class="question_part_label">b.</div>
</div>
<div class="question" style="padding-left: 20px;">
<p><span style="font-size: medium; font-family: times new roman,times;">This question was moderately well answered. </span><span style="font-size: medium; font-family: times new roman,times;">In part (c) a great number of students substituted <em>b</em> = 5 in the equation of the function instead of substituting it in the equation of their derivative. </span></p>
<div class="question_part_label">c.</div>
</div>
<div class="question" style="padding-left: 20px;">
<p><span style="font-size: medium; font-family: times new roman,times;">This question was moderately well answered.&nbsp;</span><span style="font-size: medium; font-family: times new roman,times;">Very few students used the GDC to find the equation of the tangent at <em>x</em> = 1 in part (d). </span></p>
<div class="question_part_label">d.</div>
</div>
<div class="question" style="padding-left: 20px;">
<p><span style="font-size: medium; font-family: times new roman,times;">This question was moderately well answered.&nbsp;</span><span style="font-size: medium; font-family: times new roman,times;">Good use of the GDC was seen in part (e), although some students wrote the <em>x</em>-coordinates of the point of intersection and neglected to write the <em>y</em>-coordinate.</span></p>
<div class="question_part_label">e.</div>
</div>
<div class="question" style="padding-left: 20px;">
<p><span style="font-size: medium; font-family: times new roman,times;">This question was moderately well answered.</span><span style="font-size: medium; font-family: times new roman,times;"> The sketch in part (f) was, for the most part, not well done. Often the axes labels were missing. Very few tangents to the curve at the correct point were seen. Often the intended tangent lines intersected the curve, which shows that candidates either did not know what a tangent is or did not make sense of the sketch.</span></p>
<div class="question_part_label">f.</div>
</div>
<div class="question" style="padding-left: 20px;">
<p><span style="font-size: medium; font-family: times new roman,times;">This question was moderately well answered.&nbsp;</span><span style="font-size: medium; font-family: times new roman,times;">Good use of the GDC was shown in part (g) for finding the coordinates of the minimum point. </span></p>
<div class="question_part_label">g.</div>
</div>
<div class="question" style="padding-left: 20px;">
<p><span style="font-size: medium; font-family: times new roman,times;">This question was moderately well answered.</span><span style="font-size: medium; font-family: times new roman,times;"> Few acceptable answers were given in part (h).</span></p>
<div class="question_part_label">h.</div>
</div>
<br><hr><br><div class="specification">
<p><span style="font-family: times new roman,times; font-size: medium;">Consider the function <em>f </em>(<em>x</em>) = <em>x</em><sup>3 </sup><em>&ndash;</em> 3x&ndash; 24<em>x</em> + 30.</span></p>
</div>

<div class="question" style="padding-left: 20px; padding-right: 20px;">
<p><span>Write down <em>f</em> (0).</span></p>
<div class="marks">[1]</div>
<div class="question_part_label">a.</div>
</div>
<div class="question" style="padding-left: 20px; padding-right: 20px;">
<p><span>Find \(f'(x)\).</span></p>
<div class="marks">[3]</div>
<div class="question_part_label">b.</div>
</div>
<div class="question" style="padding-left: 20px; padding-right: 20px;">
<p><span>Find the gradient of the graph of <em>f</em> (<em>x</em>) at the point where <em>x</em> = 1.</span></p>
<div class="marks">[2]</div>
<div class="question_part_label">c.</div>
</div>
<div class="question" style="padding-left: 20px; padding-right: 20px;">
<p><span>(i) Use </span><em>f '</em><span>(</span><em>x</em><span>) to find the </span><em>x</em><span>-coordinate of M and of N.</span></p>
<p><span>(ii) Hence or otherwise write down the coordinates of M and of N.</span></p>
<div class="marks">[5]</div>
<div class="question_part_label">d.</div>
</div>
<div class="question" style="padding-left: 20px; padding-right: 20px;">
<p><span>Sketch the graph of <em>f</em> (<em>x</em>) for \( - 5 \leqslant x \leqslant 7\) and \( - 60 \leqslant y \leqslant 60\). Mark clearly M and N on your graph.</span></p>
<div class="marks">[4]</div>
<div class="question_part_label">e.</div>
</div>
<div class="question" style="padding-left: 20px; padding-right: 20px;">
<p><span>Lines <em>L</em><sub>1</sub> and <em>L</em><sub>2</sub> are parallel, and they are tangents to the graph of <em>f</em> (<em>x</em>) at points A and B respectively. <em>L<sub>1</sub></em> has equation <em>y</em> = 21<em>x</em> + 111.</span></p>
<p><span>(i) Find the <em>x</em>-coordinate of A and of B.</span></p>
<p><span>(ii) Find the <em>y</em>-coordinate of B.</span></p>
<div class="marks">[6]</div>
<div class="question_part_label">f.</div>
</div>
<h2 style="margin-top: 1em">Markscheme</h2>
<div class="question" style="padding-left: 20px;">
<p><span>30     <em><strong>(A1)</strong></em></span></p>
<p><span><em><strong>[1 mark]</strong></em></span></p>
<div class="question_part_label">a.</div>
</div>
<div class="question" style="padding-left: 20px;">
<p><span><em>f</em> <em>'</em>(<em>x</em>) = 3<em>x</em><sup>2</sup> – 6<em>x</em> – 24     <em><strong>(A1)(A1)(A1)</strong></em> </span></p>
<p><br><span><strong>Note:</strong> Award <em><strong>(A1)</strong></em> for each term. Award at most <em><strong>(A1)(A1)</strong></em> if extra terms present.</span></p>
<p><span><em><strong>[3 marks]</strong></em><br></span></p>
<div class="question_part_label">b.</div>
</div>
<div class="question" style="padding-left: 20px;">
<p><span><em>f '</em>(1) = –27     <em><strong>(M1)(A1)</strong></em><strong>(ft</strong><strong>)</strong><em><strong>(G2)</strong></em> </span></p>
<p><br><span><strong>Note:</strong> Award <em><strong>(M1)</strong></em> for substituting <em>x</em> = 1 into their derivative.</span></p>
<p><span><em><strong>[2 marks]</strong></em><br></span></p>
<div class="question_part_label">c.</div>
</div>
<div class="question" style="padding-left: 20px;">
<p><span>(i)<em> f '</em>(<em>x</em>) = 0</span></p>
<p><span>3<em>x</em><sup>2</sup> – 6<em>x</em> – 24 = 0     <em><strong>(M1)</strong></em></span></p>
<p><span><em>x</em> = 4; <em>x</em> = –2     <em><strong>(A1)</strong></em><strong>(ft)</strong><em><strong>(A1)</strong></em><strong>(ft)</strong></span></p>
<p><br><span><strong>Notes:</strong> Award <em><strong>(M1)</strong></em> for either <em>f '</em>(<em>x</em>) = 0 or 3<em>x</em><sup>2</sup> </span><span>– 6<em>x</em> </span><span>– 24 = 0 seen. Follow through from their derivative. Do not award the two answer marks if derivative not used.</span></p>
<p><br><span>(ii) M(–2, 58) accept <em>x</em> = –2, <em>y</em> = 58     <strong><em>(A1)</em>(ft)</strong></span></p>
<p><span>N(4, – 50) accept <em>x</em> = 4, <em>y</em> = –50     <strong><em>(A1)</em>(ft)</strong> </span></p>
<p><br><span><strong>Note:</strong> Follow through from their answer to part (d) (i).</span></p>
<p><span><em><strong>[5 marks]</strong></em><br></span></p>
<div class="question_part_label">d.</div>
</div>
<div class="question" style="padding-left: 20px;">
<p><span><img src="data:image/png;base64,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" alt></span></p>
<p><span><em><strong>(A1)</strong></em> for window</span></p>
<p><span><em><strong>(A1)</strong></em> for a smooth curve with the correct shape</span></p>
<p><span><em><strong>(A1)</strong></em> for axes intercepts in approximately the correct positions</span></p>
<p><span><em><strong>(A1)</strong></em> for M and N marked on diagram and in approximately</span><br><span>correct position     <em><strong>(A4)</strong></em><br><br></span></p>
<p><span><strong>Note:</strong> If window is not indicated award at most <em><strong>(A0)(A1)(A0)(A1)</strong></em><strong>(ft)</strong>.</span></p>
<p><span><em><strong>[4 marks]</strong></em><br></span></p>
<div class="question_part_label">e.</div>
</div>
<div class="question" style="padding-left: 20px;">
<p><span>(i) 3<em>x</em><sup>2</sup> – 6<em>x </em></span><span><span>–</span> 24 = 21     <em><strong>(M1)</strong></em></span></p>
<p><span>3<em>x</em><sup>2 </sup></span><span><span>–</span> 6<em>x </em></span><span><span>–</span> 45 = 0     <em><strong>(M1)</strong></em></span></p>
<p><span><em>x</em> = 5; <em>x</em> = </span><span><span>–</span>3     <em><strong>(A1)</strong></em><strong>(ft)</strong><em><strong>(A1)</strong></em><strong>(ft)</strong><em><strong>(G3)</strong></em></span></p>
<p><br><span><strong>Note:</strong> Follow through from their derivative.</span></p>
<p><br><strong><span>OR</span></strong></p>
<p><span>Award <em><strong>(A1)</strong></em> for <em>L</em><sub>1</sub> drawn tangent to the graph of <em>f</em> on their </span><span>sketch in approximately the correct position (<em>x</em> = </span><span><span>–</span>3), </span><span><em><strong>(A1)</strong></em> for a second tangent parallel to their <em>L</em><sub>1</sub>, </span><span><em><strong>(A1)</strong></em> for <em>x</em> = –3, <em><strong>(A1)</strong></em> for <em>x</em> = 5 .     </span><strong><em><span><span>(A1)</span></span></em><span><span>(ft)</span></span><em><span><span>(A1)</span></span></em><span><span><span>(ft)</span></span></span><em><span>(A1)(A1)</span></em></strong></p>
<p><br><span><strong>Note:</strong> If only <em>x</em> = </span><span>–3 is shown without working award </span><em><strong>(G2)</strong></em><strong>.</strong><span> </span><span>If both answers are shown irrespective of working</span><span>award <em><strong>(G3)</strong></em>.</span></p>
<p><br><span>(ii)<em> f</em> (5) = </span><span><span>–</span>40     <em><strong>(M1)(A1)</strong></em><strong>(ft)</strong><em><strong>(G2)</strong></em> </span></p>
<p><br><span><strong>Notes:</strong> Award <em><strong>(M1)</strong></em> for attempting to find the image of their <em>x</em> = 5. </span><span>Award <em><strong>(A1)</strong></em> only for (5, </span><span><span>–</span>40). </span><span>Follow through from their <em>x</em>-coordinate of B only if it has</span> <span><strong>been clearly identified</strong> in (f) (i).</span></p>
<p><span><em><strong>[6 marks]</strong></em><br></span></p>
<div class="question_part_label">f.</div>
</div>
<h2 style="margin-top: 1em">Examiners report</h2>
<div class="question" style="padding-left: 20px;">
<p><span style="font-size: medium; font-family: times new roman,times;">The value of <em>f</em> (0) and the derivative function, <em>f '</em>(<em>x</em>) were well done in parts (a) and (b). In part (c) many candidates found <em>f</em> (1) instead of <em>f</em> <em>'</em>(1) . In part (d) many students did not use their <em>f</em> (<em>x</em>) to find the <em>x</em>-coordinates of M and N and instead used their GDC. The sketch was generally well done although some students forgot to label M and N or did not use the specified window. The last part of the question was a clear discriminator. Examiners were pleased to see how this challenging question was solved using alternative methods.</span></p>
<div class="question_part_label">a.</div>
</div>
<div class="question" style="padding-left: 20px;">
<p><span style="font-size: medium; font-family: times new roman,times;">The value of <em>f</em> (0) and the derivative function, <em>f '</em>(<em>x</em>) were well done in parts (a) and (b). In part (c) many candidates found <em>f</em> (1) instead of <em>f</em> <em>'</em>(1) . In part (d) many students did not use their <em>f</em> (<em>x</em>) to find the <em>x</em>-coordinates of M and N and instead used their GDC. The sketch was generally well done although some students forgot to label M and N or did not use the specified window. The last part of the question was a clear discriminator. Examiners were pleased to see how this challenging question was solved using alternative methods.</span></p>
<div class="question_part_label">b.</div>
</div>
<div class="question" style="padding-left: 20px;">
<p><span style="font-size: medium; font-family: times new roman,times;">The value of <em>f</em> (0) and the derivative function, <em>f '</em>(<em>x</em>) were well done in parts (a) and (b). In part (c) many candidates found <em>f</em> (1) instead of <em>f</em> <em>'</em>(1) . In part (d) many students did not use their <em>f</em> (<em>x</em>) to find the <em>x</em>-coordinates of M and N and instead used their GDC. The sketch was generally well done although some students forgot to label M and N or did not use the specified window. The last part of the question was a clear discriminator. Examiners were pleased to see how this challenging question was solved using alternative methods.</span></p>
<div class="question_part_label">c.</div>
</div>
<div class="question" style="padding-left: 20px;">
<p><span style="font-size: medium; font-family: times new roman,times;">The value of <em>f</em> (0) and the derivative function, <em>f '</em>(<em>x</em>) were well done in parts (a) and (b). In part (c) many candidates found <em>f</em> (1) instead of <em>f</em> <em>'</em>(1) . In part (d) many students did not use their <em>f</em> (<em>x</em>) to find the <em>x</em>-coordinates of M and N and instead used their GDC. The sketch was generally well done although some students forgot to label M and N or did not use the specified window. The last part of the question was a clear discriminator. Examiners were pleased to see how this challenging question was solved using alternative methods.</span></p>
<div class="question_part_label">d.</div>
</div>
<div class="question" style="padding-left: 20px;">
<p><span style="font-size: medium; font-family: times new roman,times;">The value of <em>f</em> (0) and the derivative function, <em>f '</em>(<em>x</em>) were well done in parts (a) and (b). In part (c) many candidates found <em>f</em> (1) instead of <em>f</em> <em>'</em>(1) . In part (d) many students did not use their <em>f</em> (<em>x</em>) to find the <em>x</em>-coordinates of M and N and instead used their GDC. The sketch was generally well done although some students forgot to label M and N or did not use the specified window. The last part of the question was a clear discriminator. Examiners were pleased to see how this challenging question was solved using alternative methods.</span></p>
<div class="question_part_label">e.</div>
</div>
<div class="question" style="padding-left: 20px;">
<p><span style="font-size: medium; font-family: times new roman,times;">The value of <em>f</em> (0) and the derivative function, <em>f '</em>(<em>x</em>) were well done in parts (a) and (b). In part (c) many candidates found <em>f</em> (1) instead of <em>f</em> <em>'</em>(1) . In part (d) many students did not use their <em>f</em> (<em>x</em>) to find the <em>x</em>-coordinates of M and N and instead used their GDC. The sketch was generally well done although some students forgot to label M and N or did not use the specified window. The last part of the question was a clear discriminator. Examiners were pleased to see how this challenging question was solved using alternative methods.</span></p>
<div class="question_part_label">f.</div>
</div>
<br><hr><br><div class="specification">
<p><span style="font-family: times new roman,times; font-size: medium;">The diagram shows part of the graph of \(f(x) = {x^2} - 2x + \frac{9}{x}\) , where \(x \ne 0\) .</span></p>
<p><span style="font-family: times new roman,times; font-size: medium;"><img src="data:image/png;base64,iVBORw0KGgoAAAANSUhEUgAAAWsAAAFzCAIAAAB3ob/ZAAAgAElEQVR4nO2d32tbSZ725z+RQPTQNhj27Z6LwEtHuNkXsl7GbfpCKAkDMyQNkck0TdOhB8ntMMMMG3mWhGQbi03PoqTlYXrCdqSO0wRWGcTciCWDwAt7YQ7sRVgZXQQThEEhNId6Lyo+llWSztH5UfUcneeDL6IjWX7yrdKj+tap+taPBCGE+OVHpgUQQmIMHQScH3r1X2ZS6UxqceXmsyMhhLD7rc2lhU8aB69NayOEDhIL7P17q4uZ1aplCyGE3Xt6/dzP71mvTMsihA4SD446N/8h83c3Oz8IIYSw9+9d+bJzZBsWRQgdJCa8sqoXM6lfNno/CPH6oF76uP6c/kEQoIPEAjkb8stG7wdx9OzO5qMD+gfBgA4SC+x+a3Mp9Q+3nu23fn2Dc6gEBzpIPPihc/Pd1JlfXPrsNy2OPwgQdJB4YFvVtdSZj6r/fWRaCSHD0EHigW3VCptPexx+EDDoIHHg6Nmd0h/3T9++PTw8NCWHEAc6CDBHz26d+/tfVb+9U7qrrv5YLxTa7bYRXYQ40EGA6bc2FhZXSn/s9EZvvrTb7UwqvZzNDgYDI9IIkdBB4sdgMFjOZqWDNOp103JIoqGDxI9GvS4dpNlsZlJpTogQg9BBYsbh4aHjHUKIfC5XKhZNiyLJhQ4SMyrblXwuJ4SQDmJZViaV7na7pnWRhEIHiRmDwUCmLdJBhBC0D2IQOkhccRyEEIPQQeIKHYQgQAeJK3QQggAdJK7QQQgCdJC4QgchCNBBADiyWvVvbl1a3Wi9GH3K7nW+Lq2k0pmFy3eendqaSwchCNBBTGPv3/vw4kc/P5NJnVUc5GXnZn5l82nPlvXZ87c6L53n6CAEAToIBLZVXVMcxLaqawubrb4cedj91ubS8YEPgg5CMKCDQDDOQV5Z1YuZIcsQ/dbGwkXnmBg6CEGADgLBGAex9++tLi6VWn3nSr+1sbC4Vt2XlkIHIQiE5iA7tRqXV/tmjIOc9gv1imYH4Q5gMpYZHGT38ZPdx0+cfzgPJe8vv/+73/3T8EX1ZdOveH/ZJBlervj+xRA1qFceffXZe6l3Ltz488lrHmxdeOvH7336h0fO3xq6IoTIpNIjP8H/j5Ne9rvf/dNP3nl3PpojOqmqhoC9YqZfvH9/Z+R39TCbg0x5dqdWWy8UAusJKkMPoWsAz2LkDuCxw5C5bI7YaZBFp5rNJrSDTGdvb4/VbnwzaSZ12EFsq7qWMjaTmkml9/b2dP5F4h1ZdMpIycswZ1KXs1nW/vUH/t3c9UKBFRUxkQMQU60TWhYjhKhsV7bK5WB6QpChAR1ZjBBQK8oq25WxxdDmsjnipWF4AAKdxbiKkwXEox5KzV13edEqnT2ZEB1eACKEsHt/u315KZXOnCvVOiZXtTebzeVsVr0+d80RMw2Hh4fDA5B4O8hgMNCQLSe5uwyj2UG63e7YeS6EUCRZg1P10pSMMB1ECLFVLle2KwH0hCMjahA06F9RNvbrASEUidUgy24PTz5CO4gXJo11Sejod5BSsbhTq2n+o2QKIwMQI4TsINIULcsK922Jin4H2anVeLIEDuoAxAghZzFCiPVCIdJvqsQOWUfQ7yByyc/IRYRQJFPDVrmsGjp0FuNRXKNej3RklczuoqLfQeSX3sjuJ4RQJFCDXCWsDvbnwUHkpH10u+wS2F3GYmRvrlw6PXwFIRQJ1FAqFkGW54TvIEKIfC4X3Qq5BHaXsRhxEPVeG0IokqZh0gBEswxJJA4S6S67pHWXSRhxkGazOZKiIoQiaRryudykxd/QDuId7rLTgBEHmbJJl+hBrvzGaYJIxiBiXMIcFkn7wpmEqRplI+vKEEKRHA2DwSCfy01ZtAk9BhlbQ2XkWediZbty6dLlmaqqeH/ZJBlervj+xRA1+Phz6mvGVhgK+Ce8/GKpWLx27fP4Nkd0UlUNAfuAerFRr//knXe/ffidaztqI6oxiExkothlpz9GmBpMjUEa9frwJBdCKBKiwcsufmgHmQm5y874grk5xpSDRPfdQKYj17CjRT7CWu1b5bKGciGJxZSD6NmBTUYAWcOuElUWI4Rot9tR7LJLyJDVFYOnPQxvXEAIRRI0lIpFLyskoLOYWcVF9GWVhO7iBYMOMrzeByEUc69hyhIynTLGEqGDCCHWC4XQy4XMfXfxiEEHkR3aVFk9lbnXsF4oeNwVPW8OEkW5kLnvLh4xe2adM7pECMV8a2g2m96XkEE7iA/G7uYkoWDWQUrFYtTF6IgwXYfdC5Gfmxt1uZDEYtZBoq7hQCSYd3CHiTaLEULs1GrhdrX5HrJ6x6yDOIWXEUIxrxpkkGe6gwudxfgTF3q5kHntLrNi1kHE8dYnhFDMq4ZJRUA0y5hO5A4ihMjnciHuspvX7jIrxh1ErhhECMVcapB7cGf96p1PBwm3XMhcdhcfGHcQuWIQIRTzp0FOoPqYQIR2EN+wXEgUGHcQ1uWPjp1azdRJ2rOiYwwiQi0XMn9fOP4w7iBCiPVC4dq1z02rgGiOEDX4mECNQoZHoqoPMnJFntvspbyC61tNkeGv5oKPKwE1hHLFVH2Q4SvXrn3+/vL78WqO6KSqGvz9jz746QfOBKrvdtSGpjFIiFvC9ccIUwPCGGTSYbqaQWiOsDTIFai+711CO0hAMPcmxxcEBxFRlrNMIPgrUFX0OQjLhYQLiINUtivR1eVPGlvlMvgKVBVNWYw4vr8d5B1CkREKCBpAHORfvtw2XrIMoTmCa5CZfsB7W9BZTEBxYZULmY/uEhwQB9l9/GQ5mzWbnyI0R/BPx5RTYLTJ8IE+BxEhbeicg+4SCjgOYjw/RWiOgBrCWgAy5w4SSrmQOeguoYDjIDI/NZjIIDRHEA2yYlMo4zhoBwmOXMXIIr2hAOIggnX5gyHzl1l30OGg1UEEy4WEB46DCN5oC4DMX4yvqfGNbgdRj24m/oByEOOJTEyR+UusF9RonQcRYdQ9jHvSGxYgDiJDIZdCmUpkEJrDh4Yo8hfoeZCwxAVMZGLaXUIHykGEEHLrk1kNBvGhIYr8JREOErDuYUy7S+igOYjBGg4IzTGrhojyl0Q4SMC6h3HsLlGA5iBCiHwuZ2RPB0JzzKQhrPVjAWWEgu6ZVImprjZPgDjIMKFX1Z5XtsrluBQQcsXAGEQEq3sYuy+ciABxkOFQmKpahtAc3jXIdC+iVVHQY5ApdVzGXpxy5f79HeesAB81VCbJ8HLF9y+GqMHHn1Nfg1BhSJX3wU8/uHr1Y+TmiK7nqBrUl/3pmweyAGpEUod/Vw9mxiAiQF0J/THC1AA4BhGGFoYgNIdHDeuFwnqhEF18oB0kXFhXIiAgDqLCmkOTkLdv5+wQWGMOwgLuAYF1EM6njmUOlp+OxVgWI/x+WcVoyBopIA6ihkL//kmE5piuQa7Z1bB1CDqLCV1cZbviI6b43UUPsA4ihNgql3WuT0VojukaSsWinvKFyXIQfwXc8buLHpAdRI7YtaWoCM0xRUOjXg/36Gh/MiLCpIP4qysB3l20gewgQoh8Lhe8Hl1ADTqZpEF+TWqb/oB2kChgXQnfgDjIJORt3YTPlHe73eVsVpuTGsGwg4RVwD2B4MctdkefhMtgMIh69QcCJrMY4auAO/KQVScgDjI9/9ez+wOhOVQNcvOL5lEYdBYTkbitcnmmYR5md9EPvoNoO4ENoTlGNMjZ0yRsETLvIO12e6YC7oDdxQj4DiJ0DUMQmmNYg8zNjVRsS6KDzLoACa27mCIWDqJnGILQHI4Gs7OnSXQQIcR6oeA94lDdxSCxcBBxPAyJdDoAoTmkhsPDw+Vs1uDRDdAOEh2hnESVNEAcxAs614YYJCE3X0aAGIPMVJkG5wvHLCAO4iUUcl4gukWZCM2x+/jJeqFg/OQX6DHI2BoqI896L4UycuWDn35w7drnXn5xigwvV4JLDa7Bx59TX4NZYWjSy0rFYqlYRGiOiHrO1asf/+Sdd7vdbihR9S11+Hf1ADEGEbOcRKU/RpgaYjQGEcc7ZeamtN8Ile2Ktp0v04F2kEgJfhJV0gBxEO9UtitzU154GGkfiT02GGUMIoTI53JeTqIy/oUDogHEQbyHQt7ZjeLUZIPNIVeOtdtthC4hwMcgUYvzWNsKoakQNMTOQcTxlGroKzVNNcfwyjGELiES7iAeT6JCaCoEDXF0EBFNrR0jzTGy8BShS4iEO4jwlsggNBWChpg6iFxzFW4uo7851HXrCF1C0EG8JDIITYWgIaYOIiI4cklzc4ydOkXoEgLcQTQQ8EjdRAHiIP6Q92XiWH8o4XdeVLAcRAiRz+XmryJ+AOx+a3PJWTa2WrXsN0/E2kHiuAB8MBhI46N9DIOVxQgPR+oiDBf1abCfN66cOV51urhW3T82EBQH8R2KEPewamgOaXlTzotC6JYCPIvRI861zDdCU+nSYB917qyVWv1xz8XdQcTxhEjwvf8a7hLmc7l8Ljclv0boloIOIpl+EhVCU+nS8KJVOptJrW5U6y1r1EbmwEFESMV4Im2Ovb295WzWNeFC6JaCDiKZfqQuQlPp0WBb1bWTjXOrG/X9o6Fn58NBhBA7tVpAE4muOeSS08p2xXW+BqFbCnAH0QaP1B3G7nV2q6WVVDqT+vBW56VzHcRBQgHwBsdgMCgVi2iqAEEcg4ipiQyC2evXYPeeXj+3uDQ0JwLiIGGFIoiJhN4cMnOZPvERtQZ/QI9BnOoDoRQyUK8MX7x69eNLly5PetkkGV6uhCU1iAZfkbEf3PjZ22/97MaD73djWB/ES3NcvfqxnFiNVMP0l3378DuZVV29+vG3D7+bqeeoGgL2gSDtqA3QMciUI3X1xwhEg21V1/DWg4QbCjmx6mXeIQoNe3t7+VxuOZv1sV4WoVsK8DGIZrh05zR2v/X7660XzmMQBwkdy7LkvQ+dS5MPDw/lrMdWuRyjRW4I4DpI4o/Ufd3rPNnt9Gz572d3N379tGefPD2vDiKEODw8XC8U9BxYPRgM5A2XfC6n/4CoOQA0ixHHA1r1CwFhuKhFw+te67crqXQmlV66dPNhyzo6/TSIg0QXCvnBXi8UXD/Y/jRI71jOZqevP/IIQrcU4FmMZnHySF01kUFoKgQNc+8gYii5KBWLU3xkVg3dblfe+pGnYYWStiB0CUEHGWFsIoPQVAgakuAgEsuyHB8ZOzXmUcPh4WGz2ZT5UT6XC3eWDaFLCDrICGMTGYSmQtCQHAeRWJa1VS7LgUNlu9Jut501h1M0DAYDy7Ia9bo0Dvm7Ucx3IHQJAe4g+pmUyBAB4yCaGQwG7XZbWol0hFKxuFOrNer14Z+dWk2WU5Qvk7XvIjprIuFAO4gQYqtcTsKBiT5IpoMM0+122+12o16Xx1mN/EgrsSyLd2cjBd1B2u02j9QdCx2EIAA9DyKOE5nh8SdCwomgAcRBEEJBDQ7Q8yCmYlQqFocTGYSmQtBAB6EGFTrIGJrN5nAig9BUCBroINSgQgcZgzxS10lkEJoKQQMdhBpUoB3EIOuFAu/IjADiICThxGAMIk4nMghmj6ABxEEQQkENDtBjkCl1XMZe9F0cRb3yp28eZFLp+/d3hq9rq9oyVmoQDf4iM/Jw/ioM6dEQnVRVQ8A+EKQdtRGPMYgQYr1QkIetmpUhQdDAMQg1qEA7iFka9brrkbqJAsRBSMKJjYPIOzI8UteBDkIQiE0WI4SQ+6OMyxAAoRAwDoIQCmpwgM5ijMdop1bL53LGZQiAUAg6CDWMgw4yjW63K+/ImJUhAEIh6CDUMA46iAv5XO7atc9Nq4AIBR2EGlSgHQQBmciYVgEBiIOQhBMzB5GJDO/ICDoIwSBmWYwQ4v3l9zUcIzIdhFCAOAhCKKjBATqLAYnRtWufrxcKZjUghIIOQg0qdBB37t/fyaTSTp1uIyCEgg5CDSp0EHd2Hz8J5ZCxgBoM/nUJHYQaVKAdBIfKdsV4ImMcEAchCSeWDrK3t2c8kTEOHYQgEMssRghhNpFBCAWIgyCEghocoLOYKXVcxl70XRzF9WW7j59Utivnz1/QU7VlkoZQ3sr3FVYY8qchOqmqhoAtHqQdtRHXMYhMZEwdR4YQCo5BqEEF2kHQWM5mk3ykLoiDkIQT1zGIEKKyXdkql81qMAiIgyCEghocoMcgaDFqt9umEhmEUNBBqEGFDuKOI0MeqWskkUEIBR2EGlToIO4My9gql40kMgihoINQgwodxJ1hGTKRMavBFHQQalCBdhBAZCLjHKmbKEAchCSceDuIEGKrXE7mkbp0EIJAvLMYcfpIXVMajADiIAihoAYH6CwGM0byJCrNiQxCKOgg1KBCB3FHlbFeKGhOZBBCQQehBhU6iDuqDP2JDEIo6CDUoALtILAk80hdEAchCWcexiBCiPVCYadWM6tBMyAOghAKanCAHoOMrYAw8myQmgjeX6bKaDab8khd17cKS+qkUERxZezfYn0Qfxqik6pqCNgHgrSjNuZkDKL5JCqEUHAMQg0q0A4CTj6X05nIGAfEQUjCmR8HSdqRunQQgsCcZDFCbyKDEAoQB0EIBTU4QGcx+DHK53J6CrgjhIIOQg0qdBB3psjYqdX0nESFEAo6CDWo0EHcmSJD20lUCKGgg1CDCrSDxALjR+pqA8RBSMKZNwepbFdKxaJpFTqggxAE5iqLEbpOokIIBYiDIISCGhygs5i4xEhDAXeEUNBBqEGFDuKOqwwNBdwRQkEHoQYVOog7rjI0nESFEAo6CDWo0EHccZWhoYA7QijoINSgAu0gMSIJBdxBHIQknDkcg4jo6x4ihALEQRBCQQ0O0GOQsTVURp4NUlXF+8smyXAe/umbBzKRiU6qq4aor7DCkD8N0UlVNQRs8SDtqI35HIOIiOseIoSCYxBqUIF2kHgh6x6aVhEhIA5CEs7cjkEiLReCEAoQB0EIBTU4QI9BYhej6OoeIoSCDkINKnQQd7zLiK5cCEIo6CDUoEIHcce7DJnIRFEuBCEUdBBqUKGDuDOTjIjKhSCEgg5CDSrQDhJHKtsVPXUP9QPiICThzLmD6CkXYgQ6CEFgzrMYEU25EIRQgDgIQiiowQE6iwHpsrPKiKJcCEIoEDQIDBnU4KBfxvw7iCwXYlZDFCBoEBgyqMGBDuLOrDKiKBeCEAoEDQJDBjU4hCKj2+16T/zn30GEEKViMdxyIQihQNAgMGRQg0MoMprNZiaVLhWLlmW5vjgRDhJ6uRCEUCBoEBgyqMEhLBndbrdULGZS6cp2ZfqtzNkchD/84U8Cfxr1eggOEuv7VeHuskMIRQbjSw8hFNTgEK6MRr2+nM3mc7kp0yJJcZBwd9khhIIOQg0qYcnY29vL53LL2WyjXg8ti4l1jMLdZYcQCjoINUREo17PpNJb5bKXz4snB7F77TuXzmRS6cy5zYbVD6zQB/aR9R+3pIbU6sbXz3r2zG8Rzi67I6tV/+bWpdWN1ougb+UFu9f5urSSSmcWLt85/Z827SB9q/Xo4c3L75ZaRjqEEEIcWX+5eXkplc6kFldKf+z0XhtRAfDpcKQ8b1w5u1bdn/3DcYrDw0Mvd2EkHhzk6L8aX/9nzxbC7v3t9uWlhc1WP6DCmbEPnty5/R/WkS3E696zykcLiys3nx3N+CYhHMpt79/78OJHPz+TSZ3V4iAvOzfzK5tPe7awe0+vn8vf6rx0njPqIK+s6pVfXMovpdJLphzEfr57++5frL5wPsPn7nSOdPdMhE/HMa8P6p+8nfpxcAeZCVcHefU/f/3PA1sIOU7rtzYW9Hx4xmsQQuw+fmRVL2Zmb6qwdtnZVvW91DsagmBb1bWT/6bdb20urVat4/+06TGIEPb+vdXFt89vmRmU/k/7rwdvBh27j5/YVnVNk60Pg/DpkNhHnS8/+vzzf3wLzkFOeNNORpz+lIzv+61NH2YvF6cG32Wny0FeWdWLmSHLEP3WxsLFe9Yr+SjhDjKM6U/vGw0mPx1Hz25d+rJz8PRC+A7yv41L/yeTSmcWPmkcvH4z1Eql373Z+UEIMYuD9O/e+Gzj0o2WoWzTQTrIu1fqB7PHKZSz7DQ5iL1/b3XxVI7Qb20sLDr9gw7i8MZB/u6TxoGpzmn20/Gyc/OzW52Xot+KwEGEEMI+qBcW0kulB3+5/dt7+6ca3JuD9FsbC+k3s5j1/VknIMJl9/GfW6XLwzMC3gllcaomBzntF+oVOojD7uPv+63frM0+NRYOhj8d9lHny4/k/z0yBxHiRat0NpM6U6g/H3nzGbIYu/esVlod+y4aGYrX7BweHmZSae/zzOMV0EEk6ijJFHIMbzS5NvbpOHp2Z/PRm/G42mdCw+63NpdSJ0m0w49kP5iwmvWUmt3HTyLrNK+s6sWJi2qH5wKOnhXO/2o/QF+ZcpadbVXXJi7sPYkds5g3oIxBXt4u/GJkaK2fKD8dU3jZuf37k9wtwjGIdJAx9jTrirJXVvWi0Rt4D/71378P8h7BF6fqnEkdDrVtVddSnEkd4fXBo7uf/eu3JiUIIUx9Ok4SqIlfeKFg957+plT61bkx/jirg7xolVbNZDH2Qev23Vbv9ZvFf0f/Xau2fTRV8MWp2m4c8m6uG697rbt3WgeP3qwH7e9//ce/GlqOYfjTIYloDGI/b1wtNQ76b1ZRHPxX7fYT5z6Gm4PYzxtXzsoFf7uPd3ut365dqgVJInxytN8orU7JsGYi4OJUjUsPYFeUCSGMO0jfqm+uTEl4NQDy6XAI3UHkFMebtbb2UefOSmpxpVS3hv6PrmOQ/n71ypJsnoXLt+odH8vJg2I/b1w5E+I4rbJd8Vs5Vc5Ia+yvx7ffM+dKtQ7OqnaZFUc1ZvbA64P6J0ujXSKiScQpAHw6TsmJbiZ1IknZWTeMrJwaZHEqQijMj0GEEBihQAAkDtC12ncfP5H65D+ch8PPDl+c9Yr3l02S4eWKEOLbh99lUul/+XI7iNQgGvxFZuShOn+mIfhRNIdmDdFJVTUE7ANB2lEbSRyDiMCVUxFCwTEINahAO8g80Ww287mcaRWBAHEQknAS6iDynm632zUtxD90EIJAQrMYIUQ+l/N9TxchFCAOghAKanCAzmLmLEZBCg4hhIIOQg0qdBB3wpIRpOAQQijoINSgQgdxJywZQU7DRAgFHYQaVKAdZP4I/TRMnYA4CEk4iXaQ0E/D1AkdhCCQ3CxGHBcc8nFPFyEUIA6CEApqcIDOYuYyRvlcbsqZoHo0+IMOQg0qdBB3wpXhr+AQQijoINSgQgdxJ1wZ/u7pIoSCDkINKnQQd8KV4e+eLkIo6CDUoALtIPNKTO/pgjgISThJH4MIX/t0EUIB4iAIoaAGB+gxyNgaKiPPBqmq4v1lk2R4uaJelPt079/f0abBX2RGHrLCkD8N0UlVNQTsA0HaURscgwgx+z5dhFBwDEINKtAOMscE2adrChAHIQmHYxAhjmsvm9UwKyAOghAKanCAHoPMcYxmvaeLEAo6CDWo0EHciUjGlPN0tWmYCToINajQQdyJSEajXve+vB0hFHQQalChg7gTkQzLsryfp4sQCjoINahAO8jcs5zNtttt0yq8AuIgJOHQQU7YKpdjtLydDkIQYBZzgveSZQihAHEQhFBQgwN0FjP3MfJ+DBVCKOgg1KBCB3EnUhkel7cjhIIOQg0qdBB3IpVR2a5slctmNXiEDkINKtAOkgRmXd5ukLjoJPMNxyCnkNXbLcsyqMEjIA6CEApqcIAeg4ytgDDybJCaCN5fNkmGlyuuL5PV2yPV4C8yIw9ZH8SfhuikqhoC9oEg7agNjkFG2anVXHf6I4SCYxBqUIF2kIQgq7ebVuFOLESSuYcOMkqQE7l1QgchCDCLGYPrTn+EUIA4CEIoqMEBOotJToxcD7JDCAUdhBpU6CDuaJDhepAdQijoINSgQgdxR4+M6VMhCKGgg1CDCrSDJIpSsei96KERQByEJBw6yHhmKnpoBDoIQYAOMh5Z9HDKVIhx6CAEAc6DTGTKVAhCKEAcBCEU1OAAPQ+StBhNmQpBCAUdhBpU6CDuaJMxZSoEIRR0EGpQoYO4o03GlPMfEEJBB6EGFWgHSSDIG2RAHIQkHI5BpjFpKgQhFCAOghAKanCAHoOMraEy8myQqireXzZJRuhVW7744voHP/0gdA3+IjPykBWG/GmITqqqIWAfCNKO2uAYZBqTVoUghIJjEGpQgXaQZAI7FQLiICTh0EFcgN0gQwchCDCLcWFsrRCEUIA4CEIoqMEBOotJZozG1gpBCAUdhBpU6CDuaJYhy6aOnCCDEAo6CDWo0EHc0S9jvVBo1OtmNajQQahBBdpBEktlu+J6gox+QByEJBw6iDuYh+kCSiIJhFmMO/Iw3W63a0KD3W9tLjkLT1erlv3mCRAHQegV1OAAncUkNkZCiOVsttlsGtBgP29cOXO8bn1xrbp/bCB0EGoYAx3EHSMytsrlrXJZuwb7qHNnrdTqj3uODkINKnQQd4zIaDab+VxOu4YXrdLZTGp1o1pvWaM2QgehBhVoB0kyU6oNRYdtVddOtt6ubtT3j4aeBXEQknDoIF4xtcXO7nV2q6WVVDqT+vBW5+WwHv1iCBmBWYxXhrfY6ddg955eP7e4NDQnAuIgCL2CGhygs5ixNVRGng1SVcX7yybJiKhqi7zyxRfXz5+/EIoG+e9HX332nlIl6Pjn4qdfPTr9i/aDGz97+62f3Xjw/S4rDLHCkFs7aoNjEK/ILXYGNdhWdY3rQahhKtAOknDGbrHTiN1v/f5664XzGMRBSMLhGGQG8rmc3GKnRcPrXufJbqdny38/u7vx66c9++RpEAdB6BXU4AA9BklsjBwq2xW5rkyTg7R+u5JKZ1LppUs3H7aso9NP00GoQYUO4o5BGc66MoRQ0EGoQYUO4o5BGc66MoRQ0EGoQYUO4o5ZGXJdGUIo6CDUoALtIEQglW4HcQYLVgoAAAaISURBVBCScOggs7FTq4HUK6ODEASYxcyGXFeGEAoQB0EIBTU4QGcxiY3RMLJe2f37OwY1SOgg1KBCB3HHuIzlbPaf//mWWQ2CDkIN46CDuGNcRqlYvHr1Y7MaBB2EGsYB7SBE0qjX1XMw9QPiICThcAwyM8ObdA2CoEEANAc1DAM9BhlbAWHk2SA1Eby/bJIML1eCS/324XeZVPruV/8Wyv/ae2RGHrI+iD8N0UlVNQTsA0HaURscg/jh/eX3hw9/MALHINSgAu0gxGGrXK5sV8xqAHEQknDoIH5AmEylgxAEmMX44e5X/5ZJpQeDgUENIA6C0BzU4ACdxSQ2RipyMtVcxUMh6CDUMA46iDsIMnYfP8nncmYnU+kg1KBCB3EHQcbu4yfGJ1PpINSgAu0gZBjjk6kgDkISDh3EJ7LioUEBdBCCALMYnxpMHx+D4iAgzWFaAoQGAZ7FJDZGkzSYnUylg1CDCh3EHQQZUkOpWDQ4mUoHoQYVOog7CDKkhka9brBmKh2EGlToIO4gyJAazG7zp4NQgwq0g5ARZM3Ubrdr5K+DOAhJOByDBNKQSaXb7bYRDSAOAtUcCdcgwMcgY2uojDwbpKqK95dNkuHlSlhS5fXz5y988cX1IG/lPTIjD1lhyJ+G6KSqGgL2gSDtqA2OQQJpMHgAFccg1KAC7SBEpd1uL2ezRv40iIOQhEMHCYRc2354eKj/T9NBCALMYoJqMLW2HcRB0JojyRoEeBaT2BhN17BeKDTqdf0a6CDUoEIHcQdBxrCGrXJ5q1zWr4EOQg0qdBB3EGQMazBVKIQOQg0q0A5CxmKqUAiIg5CEQwcJiqm17XQQggCzmBA0GFnbDuIggM2RWA0CPItJbIxcNZSKRf23Y+gg1KBCB3EHQcaIBiNr2+kg1KBCB3EHQcaIhmazmc/lNGugg1CDCrSDkEkYuR0D4iAk4XAMEoIGI3XbQRwEsDkSq0GAj0HGVkAYeTZITQTvL5skI6KaC140/OSdd+XtmOBvPikyIw9ZH8SfhuikqhoC9oEg7agNjkHC0aD/dgzHINSgQgdxB0GGqkH/7Rg6CDWoQDsImYL+2zEgDkISDscg4WjQfzsGxEEwmyOZGgT4GCSxMfKiQf/tGDoINajQQdxBkDFWw3I2q3N3DB2EGlToIO4gyBirQfPtGDoINahAOwiZjubbMSAOQhIOHSQ0GvW6ztsxdBCCAB0kNDTfjqGDEAQ4DxKaBs3FykAcBLY5EqhBgM+DJDZG3jVkUum9vT09Gugg1KBCB3EHQcYkDTrPjqGDUIMKHcQdBBmTNGyVy5Xtih4NdBBqUIF2EOJKo17XdkMXxEFIwuEYJEwN7XZb2wcbxEGQmyNpGgT4GGRsDZWRZ4NUVfH+skkyvFwJS+pYDfKG7rcPvwvy5pMiM/KQFYb8aYhOqqohYB8I0o7a4BgkZA3a9tdxDEINKtAOQrygbX8diIOQhEMHCRlt++voIAQBZjEha6hsV/TcjgFxEPDmSJQGAZ7FJDZGM2lo1OvrhYIGDXQQalChg7iDIGOKBm376+gg1KBCB3EHQcYUDd1uN5NKHx4eRq2BDkINKtAOQjyi54YuiIOQhEMHCZ98LtdsNqP+K3QQggCzmPA1zHxD98hq1b+5dWl1o/Vi9Cm71/m6tJJKZxYu33nWs4eeAXEQ/OZIjgYBnsUkNkazapitYKq9f+/Dix/9/EwmdVZxkJedm/mVzac9W9i9p9fP5W91XjrP0UGoQYUO4g6CjOkafNzQta3qmuIgtlVdW9hs9eXIw+63NpdWq9bxOIQOQg0qdBB3EGRM1+Djhu44B3llVS9mhixD9FsbCxfvWa/kIzoINajQQdxBkDFdg48bumMcxN6/t7q4VGr1nSv91sbC4lp1X1oKHYQaVKAdhHhn1hu6YxzktF+oV0AchCQcjkEi0bCczc50Qze+DhKL5kiIBgE+BhlbQ2Xk2SBVVby/bJIML1fCkjpdw/nzF7744roQwraq7ymlgI5//t+nXz2Sv/Xoq8/eS71z4cafT97n0R8+/b8/fvv81gPnbz3YuvDWj9/79A+PJlcYCvh/jCIUCM0Ryl/0ImPsFc3NIfTCLCYSGvX6TFVCJs2kDs+D2FZ1LXUyk0oIAnQQCHzczSUEAToIBGMdZPqKMkIQoIMY50WrdPZkOmNklGH3/nb78lIqnTlXqnV6HH8QNOgghBD/0EEIIf6hgxBC/PP/AZvaezuOMJRIAAAAAElFTkSuQmCC" alt></span></p>
</div>

<div class="question" style="padding-left: 20px; padding-right: 20px;">
<p><span>Write down</span></p>
<p><span>(i)     the equation of the vertical asymptote to the graph of \(y = f (x)\) ;</span></p>
<p><span>(ii)    the solution to the equation \(f (x) = 0\) ;</span></p>
<p><span>(iii)   the coordinates of the local minimum point.</span></p>
<div class="marks">[5]</div>
<div class="question_part_label">a.</div>
</div>
<div class="question" style="padding-left: 20px; padding-right: 20px;">
<p><span>Find  \(f'(x)\) . </span></p>
<div class="marks">[4]</div>
<div class="question_part_label">b.</div>
</div>
<div class="question" style="padding-left: 20px; padding-right: 20px;">
<p><span>Show that \(f'(x)\) can be written as \(f'(x) = \frac{{2{x^3} - 2{x^2} - 9}}{{{x^2}}}\) .</span></p>
<div class="marks">[2]</div>
<div class="question_part_label">c.</div>
</div>
<div class="question" style="padding-left: 20px; padding-right: 20px;">
<p><span>Find the gradient of the tangent to \(y = f (x)\) at the point \({\text{A}}(1{\text{, }}8)\) .</span></p>
<div class="marks">[2]</div>
<div class="question_part_label">d.</div>
</div>
<div class="question" style="padding-left: 20px; padding-right: 20px;">
<p><span>The line, \(L\), passes through the point A and is perpendicular to the tangent at A. </span></p>
<p><span>Write down the gradient of \(L\) .</span></p>
<div class="marks">[1]</div>
<div class="question_part_label">e.</div>
</div>
<div class="question" style="padding-left: 20px; padding-right: 20px;">
<p><span>The line, \(L\) , passes through the point A and is perpendicular to the tangent at A. </span></p>
<p><span>Find the equation of \(L\) . Give your answer in the form \(y = mx + c\) .</span></p>
<div class="marks">[3]</div>
<div class="question_part_label">f.</div>
</div>
<div class="question" style="padding-left: 20px; padding-right: 20px;">
<p><span><span>The line, \(L\) , passes through the point A and is perpendicular to the tangent at A. </span></span></p>
<p><span>\(L\) also intersects the graph of \(y = f (x)\) at points B and C . Write down the <strong><em>x</em>-coordinate</strong> of B and of C .</span></p>
<div class="marks">[2]</div>
<div class="question_part_label">g.</div>
</div>
<h2 style="margin-top: 1em">Markscheme</h2>
<div class="question" style="padding-left: 20px;">
<p><span>(i)     \(x = 0\)     <em><strong>(A1)(A1)</strong></em></span></p>
<p><span><strong>Note:</strong> Award <em><strong>(A1)</strong></em> for \(x = \) a constant, <em><strong>(A1)</strong></em> for the constant in their equation being \(0\).</span></p>
<p> </p>
<p><span>(ii)    \( - 1.58\) (\( - 1.58454 \ldots \))     <em><strong>(G1)</strong></em></span></p>
<p><span><strong>Note:</strong> Accept \( - 1.6\), do not accept \( - 2\) or \( - 1.59\).</span></p>
<p> </p>
<p><span>(iii)   \((2.06{\text{, }}4.49)\) \((2.06020 \ldots {\text{, }}4.49253 \ldots )\)     <em><strong>(G1)(G1)</strong></em></span></p>
<p><span><strong>Note:</strong> Award at most <em><strong>(G1)(G0)</strong></em> if brackets not used. Award <strong><em>(G0)(G1)</em>(ft)</strong> if coordinates are reversed.</span></p>
<p><span><strong>Note:</strong> Accept \(x = 2.06\), \(y = 4.49\) .</span></p>
<p><span><strong>Note:</strong> Accept \(2.1\), do not accept \(2.0\) or \(2\). Accept \(4.5\), do not accept \(5\) or \(4.50\).</span></p>
<p> </p>
<p><em><strong><span>[5 marks]</span></strong></em></p>
<div class="question_part_label">a.</div>
</div>
<div class="question" style="padding-left: 20px;">
<p><span>\(f'(x) = 2x - 2 - \frac{9}{{{x^2}}}\)     <em><strong>(A1)(A1)(A1)(A1)</strong></em></span></p>
<p><span><strong>Notes:</strong> Award <em><strong>(A1)</strong></em> for \(2x\), <em><strong>(A1)</strong></em> for \( - 2\), <em><strong>(A1)</strong></em> for \( - 9\), <em><strong>(A1)</strong></em> for \({x^{ - 2}}\) . Award a maximum of <em><strong>(A1)(A1)(A1)(A0)</strong></em> if there are extra terms present.</span></p>
<p><em><strong><span>[4 marks]</span></strong></em></p>
<div class="question_part_label">b.</div>
</div>
<div class="question" style="padding-left: 20px;">
<p><span><strong></strong>\(f'(x) = \frac{{{x^2}(2x - 2)}}{{{x^2}}} - \frac{9}{{{x^2}}}\)     <em><strong>(M1)</strong></em></span></p>
<p><span><strong>Note:</strong> Award <em><strong>(M1)</strong></em> for taking the correct common denominator.</span></p>
<p> </p>
<p><span>\( = \frac{{(2{x^3} - 2{x^2})}}{{{x^2}}} - \frac{9}{{{x^2}}}\)     <em><strong>(M1)</strong></em></span></p>
<p><span><strong>Note:</strong> Award <em><strong>(M1)</strong></em> for multiplying brackets or equivalent.</span></p>
<p><span> </span></p>
<p><span>\( = \frac{{2{x^3} - 2{x^2} - 9}}{{{x^2}}}\)     <em><strong>(AG)</strong></em></span></p>
<p><span><strong>Note:</strong> The final <em><strong>(M1)</strong></em> is not awarded if the given answer is not seen.</span></p>
<p><em><strong><span>[2 marks]</span></strong></em></p>
<div class="question_part_label">c.</div>
</div>
<div class="question" style="padding-left: 20px;">
<p><span>\(f'(1) = \frac{{2{{(1)}^3} - 2(1) - 9}}{{{{(1)}^2}}}\)     <em><strong>(M1)</strong></em></span></p>
<p><span>\( = - 9\)     <em><strong>(A1)(G2)</strong></em></span></p>
<p><span><span><strong>Note:</strong> Award <em><strong>(M1)</strong></em> for substitution into <strong>given</strong> (or their correct) </span><span><span>\(f'(x)\)</span> . There is no follow through for use of their incorrect derivative.</span></span></p>
<p><em><strong><span><span>[2 marks]</span></span></strong></em></p>
<div class="question_part_label">d.</div>
</div>
<div class="question" style="padding-left: 20px;">
<p><span>\(\frac{1}{9}\)     <strong><em>(A1)</em>(ft)</strong></span></p>
<p><span><strong>Note:</strong> Follow through from part (d).</span></p>
<p><em><strong><span>[1 mark]</span></strong></em></p>
<div class="question_part_label">e.</div>
</div>
<div class="question" style="padding-left: 20px;">
<p><span>\(y - 8 = \frac{1}{9}(x - 1)\)     <em><strong>(M1)(M1)</strong></em></span></p>
<p><span><strong>Notes:</strong> Award <em><strong>(M1)</strong></em> for substitution of their gradient from (e), <em><strong>(M1)</strong></em> for substitution of given point. Accept all forms of straight line.</span></p>
<p> </p>
<p><span>\(y = \frac{1}{9}x + \frac{{71}}{9}\) (\(y = 0.111111 \ldots x + 7.88888 \ldots \))     <em><strong>(A1)</strong></em><strong>(ft)</strong><em><strong>(G3)</strong></em></span></p>
<p><span><strong>Note:</strong> Award the final <strong><em>(A1)</em>(ft)</strong> for a correctly rearranged formula of <strong>their</strong> straight line in (f). Accept \(0.11x\), do not accept \(0.1x\). Accept \(7.9\), do not accept \(7.88\), do not accept \(7.8\).</span></p>
<p><em><strong><span>[3 marks]</span></strong></em></p>
<div class="question_part_label">f.</div>
</div>
<div class="question" style="padding-left: 20px;">
<p><span>\( - 2.50\), \(3.61\) (\( - 2.49545 \ldots \), \(3.60656 \ldots \))     <em><strong>(A1)</strong></em><strong>(ft)</strong><em><strong>(A1)</strong></em><strong>(ft)</strong></span></p>
<p><span><strong>Notes:</strong> Follow through from their line \(L\) from part (f) even if no working shown. Award at most <em><strong>(A0)(A1)</strong></em><strong>(ft)</strong> if their correct coordinate pairs given.</span></p>
<p><span><strong>Note:</strong> Accept \( - 2.5\), do not accept \( - 2.49\). Accept \(3.6\), do not accept \(3.60\).</span></p>
<p><em><strong><span>[2 marks]</span></strong></em></p>
<div class="question_part_label">g.</div>
</div>
<h2 style="margin-top: 1em">Examiners report</h2>
<div class="question" style="padding-left: 20px;">
<p><span style="font-family: times new roman,times; font-size: medium;">As usual, the content in this question caused difficulty for many candidates. However, for those with a sound grasp of the topic, there were many very successful attempts. The curve was given so that a comparison could be made to a GDC version and the correct form of the derivative was also given to permit weaker candidates to progress to the latter stages. Unfortunately, some decided to proceed with their own incorrect versions, in which case <strong>very limited follow through accrued</strong>. It should be emphasized to candidates that when an answer is given in this way it should be used in subsequent parts of the question.</span></p>
<p><span style="font-family: times new roman,times; font-size: medium;">As in previous years, much of the question could have been answered successfully by using the GDC. However, it was also clear that a large number of candidates did not attempt either to verify their work with their GDC or to use it in place of an algebraic approach.</span></p>
<p><span style="font-family: times new roman,times; font-size: medium;">Differentiation of terms with negative indices remains a testing process for the majority; it will continue to be tested. Some centres still do not teach the differential calculus.</span></p>
<div class="question_part_label">a.</div>
</div>
<div class="question" style="padding-left: 20px;">
<p><span style="font-family: times new roman,times; font-size: medium;">As usual, the content in this question caused difficulty for many candidates. However, for those with a sound grasp of the topic, there were many very successful attempts. The curve was given so that a comparison could be made to a GDC version and the correct form of the derivative was also given to permit weaker candidates to progress to the latter stages. Unfortunately, some decided to proceed with their own incorrect versions, in which case <strong>very limited follow through accrued</strong>. It should be emphasized to candidates that when an answer is given in this way it should be used in subsequent parts of the question.</span></p>
<p><span style="font-family: times new roman,times; font-size: medium;">As in previous years, much of the question could have been answered successfully by using the GDC. However, it was also clear that a large number of candidates did not attempt either to verify their work with their GDC or to use it in place of an algebraic approach.</span></p>
<p><span style="font-family: times new roman,times; font-size: medium;">Differentiation of terms with negative indices remains a testing process for the majority; it will continue to be tested. Some centres still do not teach the differential calculus.</span></p>
<div class="question_part_label">b.</div>
</div>
<div class="question" style="padding-left: 20px;">
<p><span style="font-family: times new roman,times; font-size: medium;">As usual, the content in this question caused difficulty for many candidates. However, for those with a sound grasp of the topic, there were many very successful attempts. The curve was given so that a comparison could be made to a GDC version and the correct form of the derivative was also given to permit weaker candidates to progress to the latter stages. Unfortunately, some decided to proceed with their own incorrect versions, in which case <strong>very limited follow through accrued</strong>. It should be emphasized to candidates that when an answer is given in this way it should be used in subsequent parts of the question.</span></p>
<p><span style="font-family: times new roman,times; font-size: medium;">As in previous years, much of the question could have been answered successfully by using the GDC. However, it was also clear that a large number of candidates did not attempt either to verify their work with their GDC or to use it in place of an algebraic approach.</span></p>
<p><span style="font-family: times new roman,times; font-size: medium;">Differentiation of terms with negative indices remains a testing process for the majority; it will continue to be tested. Some centres still do not teach the differential calculus.</span></p>
<div class="question_part_label">c.</div>
</div>
<div class="question" style="padding-left: 20px;">
<p><span style="font-family: times new roman,times; font-size: medium;">As usual, the content in this question caused difficulty for many candidates. However, for those with a sound grasp of the topic, there were many very successful attempts. The curve was given so that a comparison could be made to a GDC version and the correct form of the derivative was also given to permit weaker candidates to progress to the latter stages. Unfortunately, some decided to proceed with their own incorrect versions, in which case<strong> very limited follow through accrued</strong>. It should be emphasized to candidates that when an answer is given in this way it should be used in subsequent parts of the question.</span></p>
<p><span style="font-family: times new roman,times; font-size: medium;">As in previous years, much of the question could have been answered successfully by using the GDC. However, it was also clear that a large number of candidates did not attempt either to verify their work with their GDC or to use it in place of an algebraic approach.</span></p>
<p><span style="font-family: times new roman,times; font-size: medium;">Differentiation of terms with negative indices remains a testing process for the majority; it will continue to be tested. Some centres still do not teach the differential calculus.</span></p>
<div class="question_part_label">d.</div>
</div>
<div class="question" style="padding-left: 20px;">
<p><span style="font-family: times new roman,times; font-size: medium;">As usual, the content in this question caused difficulty for many candidates. However, for those with a sound grasp of the topic, there were many very successful attempts. The curve was given so that a comparison could be made to a GDC version and the correct form of the derivative was also given to permit weaker candidates to progress to the latter stages. Unfortunately, some decided to proceed with their own incorrect versions, in which case <strong>very limited follow through accrued</strong>. It should be emphasized to candidates that when an answer is given in this way it should be used in subsequent parts of the question.</span></p>
<p><span style="font-family: times new roman,times; font-size: medium;">As in previous years, much of the question could have been answered successfully by using the GDC. However, it was also clear that a large number of candidates did not attempt either to verify their work with their GDC or to use it in place of an algebraic approach.</span></p>
<p><span style="font-family: times new roman,times; font-size: medium;">Differentiation of terms with negative indices remains a testing process for the majority; it will continue to be tested. Some centres still do not teach the differential calculus.</span></p>
<div class="question_part_label">e.</div>
</div>
<div class="question" style="padding-left: 20px;">
<p><span style="font-family: times new roman,times; font-size: medium;">As usual, the content in this question caused difficulty for many candidates. However, for those with a sound grasp of the topic, there were many very successful attempts. The curve was given so that a comparison could be made to a GDC version and the correct form of the derivative was also given to permit weaker candidates to progress to the latter stages. Unfortunately, some decided to proceed with their own incorrect versions, in which case <strong>very limited follow through accrued</strong>. It should be emphasized to candidates that when an answer is given in this way it should be used in subsequent parts of the question.</span></p>
<p><span style="font-family: times new roman,times; font-size: medium;">As in previous years, much of the question could have been answered successfully by using the GDC. However, it was also clear that a large number of candidates did not attempt either to verify their work with their GDC or to use it in place of an algebraic approach.</span></p>
<p><span style="font-family: times new roman,times; font-size: medium;">Differentiation of terms with negative indices remains a testing process for the majority; it will continue to be tested. Some centres still do not teach the differential calculus.</span></p>
<div class="question_part_label">f.</div>
</div>
<div class="question" style="padding-left: 20px;">
<p><span style="font-family: times new roman,times; font-size: medium;">As usual, the content in this question caused difficulty for many candidates. However, for those with a sound grasp of the topic, there were many very successful attempts. The curve was given so that a comparison could be made to a GDC version and the correct form of the derivative was also given to permit weaker candidates to progress to the latter stages. Unfortunately, some decided to proceed with their own incorrect versions, in which case <strong>very limited follow through accrued</strong>. It should be emphasized to candidates that when an answer is given in this way it should be used in subsequent parts of the question.</span></p>
<p><span style="font-family: times new roman,times; font-size: medium;">As in previous years, much of the question could have been answered successfully by using the GDC. However, it was also clear that a large number of candidates did not attempt either to verify their work with their GDC or to use it in place of an algebraic approach.</span></p>
<p><span style="font-family: times new roman,times; font-size: medium;">Differentiation of terms with negative indices remains a testing process for the majority; it will continue to be tested. Some centres still do not teach the differential calculus.</span></p>
<div class="question_part_label">g.</div>
</div>
<br><hr><br><div class="specification">
<p><span style="font-family: times new roman,times; font-size: medium;">Mal is shopping for a school trip. He buys \(50\) tins of beans and \(20\) packets of cereal. The total cost is \(260\) Australian dollars (\({\text{AUD}}\)).</span></p>
</div>

<div class="specification">
<p><span style="font-family: times new roman,times; font-size: medium;">The triangular faces of a square based pyramid, \({\text{ABCDE}}\), are all inclined at \({70^ \circ }\) to the base. The edges of the base \({\text{ABCD}}\) are all \(10{\text{ cm}}\) and \({\text{M}}\) is the centre. \({\text{G}}\) is the mid-point of \({\text{CD}}\).</span></p>
<p><span style="font-family: times new roman,times; font-size: medium;"><img src="data:image/png;base64,iVBORw0KGgoAAAANSUhEUgAAAZMAAAECCAIAAAC0RK3QAAAe8klEQVR4nO2d/2sj553H+3fcwY2u096ddFUvue3lnLaxMKE9xz+IzSYxLk7Itl6n1tKkIT1akHzeXljaeAs28YU17XGRiDa0zZGV2HRbqLcM2wSTOhHnH0wRA3ewVEY/LM5WCGRMGJ774TMajUajmZE0X55n9H7hH9a2LI211kvP83k+Xz7DAABAND4T9QUAAMDIwFwAAPGAuQAA4gFzAQDEA+YCAIgHzAUAEA+YCwAgHjAXAEA8YC4AgHjAXAAA8YC5AADiAXOJyKlaXJalhO1HuqC0or4+AIIG5hIWrV7Kpvo9pbXr5dwGzAXiD8wlLDbmYozdv1v5EOYCsQfmEpZBc2nqe7dULcJLAiAsYC5hGTCXdnzr1bfqMBeYBmAuYdHqpWyqPzyfOl+EucBUAHMJi3XNpbXrb1/BmgtMBzCXsCDOBaYYmEtY7M8WAZgKYC5hGWau9pFSO4nkigAIDZhLWOzMpTX3d9ZeU1rYMoKYA3OJCKp/wLQDcwEAxAPmAgCIB8wFABAPmAsAIB4wFwBAPGAuAIB4wFzC0+l0Pvjgg3d++cuoLwSA8IC5hOfff/hDyuT6xc9/HvW1ABASMJfYvHfrliwlvjwzI0uJRx/555MT1P2AqQDmEpijo6PPf/azf/e5z3/yySe07Crk81FfFABhAHOJSqfTeeTcl2Qp8fHHHzPGlhYXv/XNb8pSYn9/P+pLAyBwYC5ReeJf5mUpsbOzQ58W8vlnnnpqaXFxLpPpdDrRXhsAQQNzCcm/FdZlKfHtF14wvlKtVGQpoaqqLCWubW5GeG0AhADMJR43ymVZSnzl0UfNa6v9/X0yF3338PAwwisEIGhgLsEgQ8lSotFomL/eaDQoyNXpdLBnBLEH5hKJRqPxpYcflqWEbd6pLCWqlQpjjPaMN8rl0C8QgJCAuYTh5OTkKzOPylLiuy+9ZHuDy7mckRVxbXOTNo8hXiAA4QFziUGn0/n2Cy/IUuIrM48O2wZe29ycy2SM289lMkuLi9gzglgCc4kBraGcl1F0vGio6vDwEHtGEFdgLgHYvb5L2qIw1jBIVWa1Yc8I4grMxTvGYaJrZc/JyYksJfb29sxfmctkLudyAV8jAGEDc3ENnRI+lP5i5rFZLxGrwe3h3t6e62INAOGAufil0WjMZTJf/PsveM8sLeTzg0uzQj4/mP8FgNDAXJxCCaWPnDs3UpSdImKWL2LPCOIHzMUpl3M5Cm+NZBw6Xhzs0kV7RnMIDAChgbl4hJZOX3v88blMZqRmgRQXsz1MvJzLjXpvAHALzMUdtG5afPqZMQqnO53OsHg8FTai9SCIBzAXX1AOxHPPPjt2EulcJrN7fdf2W+RE7BlBDIC5OIIOEy8+/3zmsdmxA+q2x4sG2DOCeABz8QKdAGYee+zi88/PZTJjJzFQf65h30XrQRAPYC4u6HQ6dJj42o9/PGEveTpGdBAfWg+CGABzcQEVGP7nz34mS4lhUSqP0KrKQUxoPQhiAMwVPZQD8R87O5QvOqFQHI4XDbBnBKIDc0UMHSbuvL5DsXNfanSWFhddrUR7RrSRAIICc0UJrX0u53Jvvvmmj6MSC/m869EkWg8CoYG5IoNyIOYymT/84Q/+7t0ob8v1Zmg9CMQF5ooGI0x+dHTk+9rHmGDmeku0HgSCAnNFA+VAHB4e0j/8bUFDm1Ave0/sGYGgwFwRQIeJe3t7FCYPohzHezdB2jOi9SAQC5grbCgItXt9l5QRUGqCeYKZK2g9CIQD5goVikAV8nmq9Qlum3Ztc3NpcdHjjdF6EAgHzBUedJhIuaZBL3MsE8xcQbt6IBYwV0jQuoZyTUkrgWpicIKZKyRTjttInDVrb68XfrK18jB1i+37SF7afvc3teZZ30+0D7bnU+mVcr2tRXTNoJ9mZVX/L3ux2vx0yI0+bVaurr910HT8T4O5wsAoqG40GnTwF3SHP+ojOFLsn4ae8dp6sFUvrqXnryrNM8aYphbP0wtgpdJkjGnNj16/lJYScvLSjukP/tPa1jkpIUtPbNfa0V15yGjtWrnk9Ps+qL3xdm2YNAJHaykbaRdzMaYdKxtZ57ccmCsMKG1qf3+fshDCqXYeI8uU19aDZ8eVl9PJl6vH3SWV8dZN5mKMsQe1rQuylJDNN5vCNVf7YHs+O9zUWru2s/DQVnTmMt5OHM3FGNPuVddm0muV4yH/dTBX4FAOBEmEdmThZH46txgcBoetB7XjSi6ZWtg66L0cbczVW4ilC0orkguNnPZRaWVm+BpTa9fLq8mELIS59NXZTK5yz9ZdMFew0GEiNa4JIbxlZvf67lwmM+pP8deunhZTs+vK/d7X7MzFtHopm5Il/ZXZfYWYXyQtVXlne2VGlhKylFoovG2Ki2lt9bfdbxkfT2zXmrWtJ+jTc1sHf6bbJDeUlsaY1lbv3ty6lKYbzxfKNX2janrotZJyW7/b+atKs91Uri6QXoesBHs/+9DV6odvr8+nZCm1UKiovRu31Ds7q0n9QUuK2tavv7I+nzJdvEUNLbWysWD+7fSnzvbeBjD241JClh5erfzJ9mKMZ4C1VeXdLf3rUtYctBowl+mZN98DY6ylrCcTcrao2qkL5goQo6C60+mEE94yM+rxouUH/Sr/npRPa9sPDbwObc3F/lTVg/d040+blRdNn56qxWXdgNq96tqMLPVeFdpxJZekneZpu7azoIvmjLGeENMXX735P021uEyLGlripQtKizazUkKWlkvqKWOs+1gJWZpZLR61uw+Xnn/i68m10ofvricTspQ6X6zbvSSNn81eUY41evX2ZEGPRStQ+vfMavGorT9VznG9tm7h3prL6d5M0PKHfruWemd3+xZdTKteXEvTL6Jf53JJPdWfseSG0tL0J9b0y/abS18Gptcqx9qpWlzu2+zrz7zxrPYBcwWFUVDd6XQiaebnMMHMFY5aD+qSmtxcul/SBaVlfQ13ZUGf6q40ZNG922xR1ZimFs+bX5PJDaWlDSjD8tDdh0uuleot4w7P2W/auj/bdzHdG+uC0B9I3yDra8DRzeV4bwOXZFn6GT9OZnlQ27qge0c39ey6ct96/VZz3VcKs8ZTTd8yPS30RJmXeD1grkAwVEUZWxShD7mBMp0Vjrc55af1oP6Hbnk5edgtDuhDv1FbVUpbr63T9mQUc9mHz9qqUtz5SeFi2ou59Ksd31zWrZb+POiPO6q5nO+t/0cPtmkrmry0fUftX+INi1i1VKW8fa1Ae0Z7c3V/u76P3v+p0xMFcwWCUVDNukmekTSTkSfoDc1Ju/oRzKUvAQzFDJirrf5u61I6+XL1+MTyGnbaLQ59/VCIZyZXuXfmcc0lrrkYY1qz9lZhwdgC9/IbBm+vh67Sa5XjM8c1l/7bDds4w1zhYhRUs27AO6rCmvGOFwle2tV73S12g029QIlFH3rahN1ukelrsUJWD05tVUzBe9vXj6YLbqTd4sTm6t+gWbU+7m7R/t5s0Zp3rsyn6B66h7n9x77MWKB52C0aMUT77AfsFkPEKKhmphd/VEkGzhPMXKE9Y8StB/U/7j5zWTNRjYOzvkxUQx/LJfW0tzHJFtVWd+/TM1erXvzBFfPxpfUCLLvFrgWk5ZLa1FPJesowouzmaE73QMD+Dg26P0sG6S4ku698CorTqodc3Iupd5+W2XXl+Pju/v9aXWC65wf37t79P83x3kx82rz1xnal1tQsaVZGDh097Vpb/UBRW103pc4X/9giv/eu31ipkS6Nww39cbVm7dfG8SIi9KFhFFTTp5GEt8zQRnUSb3LQetB0JsiYKQzf/zFfKFXumuPHptQEMhS9Suns/0jVsxOy65V626xC40MP6BiGsizxurlRUna9qNTV314xpS/0P/Rr1XdeNO7h3NY7/927fpvVRN/Prry2bXNjUyKCKepE3+r+jsZWtx894Su1sHGnqweHe+tdVPPuB2qrXi1krdkkvS1kIr2yVSXpGI9SKCr1PyobWVlKyPMbVbVlqv4x3jaGXwCyIsLBXFDNuhaLdsHiOsHMFR5aD1IQKtD8Uhtz9dYFIBKQiRoK5oJqNmCxqPAywcwVDtrVD1T/+E93tdL3MVUFj5yB6p8QMBdUG5/6NYJsQuYymcmTGzjYM7bqxbVzK7sf2W6CJoVyIM2Raa1d21mY36lNT8EjV6DiOhyMgmr6lM4WOclB9zLBzBXaM0bdepC63Nxqut9yZLRmrWrU8VD8q3jb2jMHhAS63ISCuaCa9Rcq8oDHCWauoPUg4AqYayIsnuIkvGWGrtCXfSva1QN+gLnGh0LXZk/xE94y8D7BzBW0qwf8AHONibmgmr4S3AiyCfFxl0d7Rg5/RzBtwFzjYCmoZt31Fw8lyoMsLS762F2Hw9aDYAqBuUbGyIEwMjyDHkE2ISNNMHOFv9aDYBqBuUbGXFBNmJO5OMSv40XLHWLPCCIE5hoNc0E1wW14y2CMCWauYM8IogXmGgFLQTXjO7xlQPs7fzNj6T45/8VBjIG5vGJ02jKCWTxUI3skiMJDTloPgukE5vKEUVBt3h+FOYJsQi7ncr7H1HlpPQimEpjLHUtBNRHyCLIJuba5OcYEM1f4aVcPpg2Yyx1aW5njROGPIJuQsSeYuUJ7RiEWniBOwFwuUA6EeW1F4S2xdkmTTDBzRqBgH4gTMJcTto0fBApvGdAEs4BSNzhoPQimDphrKIMF1Uy08JaZQHvvcNB6EEwXMJc9VFBt2QQJXfgyyQQzV7BnBCEDc9kwWFDNOBhBNiEUsAvu/mmJKuJqFIgIzGVlsKCaiHwE2YRMPsHMFbQeBKEBc1mx7SJPL3uhg9DBHS8aoPUgCA2Yq4/BgmpmqvuJ6qp8wZcJZq6gXT0IB5irx2BBNRM/vGXGlwlmrmDPCEIA5tIZLKgmRA9vmfFlgpkrlDsm6AksEAWYi7EhBdWsuwoTOrxlhip1QnggtB4EQQNz2RdUMy5HkE2IjxPMXEHrQRAoMJdNQTXr6oy3EWQTQseL4ex8hc7aBfzDt7laynrSGJje+0ivbN1U1LYfjzBYUG3+ur99RHkgzIM/2jPG7zkEPMC3uRhjTGspG2lpuaSe6p83D8qFrCzNrBaPJpSXbUG1w9djwNLiYpjttNB6EASEeOZijDGtXsqm5OSG0tLGvl/bgmoWx/CWmUI+7+MEM1fQehAExJSay7agmohfeMuM7xPMXEG7ehAEAppLa9beKixMsFu0Lagm+B9BNiG0EQ6zHQ3a1YMgEMVcA0H6gtIa6+6GFVQzQUaQTUgQE8xcoT1jLOOGICpEMZc5Ql+rbl1KS4n0SrneHnm3OOzQkJJRp6HDVCR1hWg9CPxFPHMxxhg7VYvLspQ6X6yPpC7bgmrCNhk1lgQxwcwVtB4E/iKoufQt5Eh7RtuCaiL24S0zAU0wcwXt6oGPCGqus+PKy+lR1lzDCqrZdIS3zAQ3wcwV7BmBXwhoLq1Zq2ytJhPy/FWleeblLoYVVLOp3MWQqSPRBz3bonc6AzzAt7mGVP/IUna9eLvmTVvDCqoJEUeQTUigE8xcQetB4At8m8sPbAuqCXFHkE1ItPEmtB4EkxNzc1Ho3dZNlGQ0nc0MAp1g5gra1YPJibO5VFX9xtLSpW+t7O3tWfaDFHCZ2sTuoCeYuYI9I5iQOJtraXHREiC7nMvdKJf39/e/+9JL0xbeMkPb5Gjb/qH1IJiE2JqLXpw3ymVVVff29q5tbg6KrJDPk8imLeYSwgQzV9B6EExCPM1lpEFYNoOdTuf999+XpcT8179uK7JqpXJ4eBj7hUA4E8xcQbt6MDbxNNew80TLCLJOp6OqarVSobRys8XmMpl4i2wuk+GhBBp7RjAeMTQXZVrabkOcR5CdnJwcHh5WK5VCPj8osmubm9VKRVXVeAT1oz1eNKA94/QUMAC/iJu56NDQNvxM51ne85jMIrPsK6knstAiC22CmStoPQjGIG7mGpbAZdQtjn3PjUZjf3//Rrk8TGSDuRc8Qx7n4WgCrQfBGMTKXKSnwSJES3jLr8cikVFpkSX3Yvf6LuciC3OCmStoVw9GJVbmIokM+sI5vOULlHuxe313mMh4y73g5HjRAHtGMBLxMRdtfwbft6ktV8hleobIeE4iC3mCmTNT2LQDTEJMzGVU81j2gzyMIDPnXgwTWSS5F4V8nqviQbQeBN6JibloP2jJaaT+NryNIHMQWchJZOFPMHMFrQeBR+JgLorvDi4fhg3L4ArnJDISWUC5F+FPMHMFe0bgkTiYi1YullcgvSx5SBMfCQeR+Z5ERsbnzey0Z+Tn6ADwifDmMiqrzV/kIbzlC65JZCSyse+fT0eg9SBwRWxzUWPiwSRGDsNbvuAgsvGSyCKZYOYKWg8CV8Q2l21l9fSMIHNNItvb23PWN50ShHbB3kHrQeCMwOayrayethFkZhxENiyJLMIJZq5gzwgcENVctpXVtMvAyRThmkS2v7//+9//nrfjRQP63+RwMwt4QFRz2VZWO0wnm3Kcs2Gfe/ZZPjuRofUgGIaQ5rKtrJ6e8NbkdDqdYQ18zNmwPCxd0XoQ2CKkuQYrq6c5vDUhhXz+e6+8EloS2aigXT2wRTxzDVZWI/F6Enav785lMuavnJyceEkiC+3Zpj0jbxmzIFoEM5dtZTW9wPgMM/OP6/Gi70lkY4DWg8CCYOYarKymFx4Sf8Zm1AlmjUbDNYnMd5Gh9SCwIJK5Biur6SsIgkwC1SGMrf4xksjGA60HgRmRzGWprDZ2jthETIiPpenBzeVFu3pgRhhzDVZWI7zlFwFNMPO9ExktsYXr/wGCQAxzDVZWI7zlI+FMMDNE5tCJzFVk1HMNb1dADHNZKquR4+MvlGgScrbneHN5kQEDCAHMZamsDmIE2ZTDwwQzEpmXJDK0qweMf3MNVlaHMIJs2uBtghlza6m4+PQzspT4+OOPo75MEBm8m8tSWU37Grzf+g5t0KK+iqHYzuVFVv00w7W5LJXV9ClaZQYBbxPMHLDty2agNWvvvbu1miS7ZdeLt2vNlnrrt6oW8mWCYOHaXObKaoS3AoXDCWa20J/BkF5GZ82D3dXkzOpWpdY8079Sq2yvzMjZIswVM/g1l6WyGuGtQKFRSfy3NrPty0Zox5VcMrWwddC2fKN9sJ19VWlBXbGCU3NZKqvpdYXwVnDwOcHMgm1fti73lcKsnNywM5TWunv7LswVLzg1l7myOjYjyDiHt+PFQeic0XbdranF81ICu8LpgUdzmSurO51OXEeQ8cbS4iLPyb2DfdlMaC1lIy0l0gWlFfZ1gWjg0Vzmymqq9uB8FxMPuJ1gxob0ZTMBc00d3JnLXFlN4S1U2IYDz8eL9AbmsJml3SLMNT3wZS5zZbUx6BjhrXCgPCkOi5kH+7LZoNVL2dSQCD2IIXyZyxyCxQiykKGTOw435pa+bEOgDaNdVgQ7a9Y+VNswWqzgyFzm3GiMIIsEDlNPaA/rLWLQqhfX0lJqoVBU1O6usa0qb5WrdWwi4wYv5jJXVmMEWVRczuW4Ol6kiMEofVDPmrXbpUK2W9tI1T9nwV4liAJezGXkRtMfKxowRcK1zU3LBLNosfRlA8CAC3MZudEM4a1IcZ1gFibOldVgyuHCXEZltZjhrftKYVbfngzL4aaTLykhS6nzxTq3seJRJ5gFh1FZjQJ7YEv05jJyo4WeqacdV3LJhCxd2K49GPwm5UnyX5tCWSk8vHM4VFYDwCI3l5Eb/cknn4g9gqylrGcvrs6n0muVY4uetHvV71xavShGrxUeUn8dK6sBYCxycxmV1cKPIGsp6xd2f/fuy2lpuaSemr6htWtvrG7dqhZmhTBXQBPMRr0Gsf8YQPBEaS4jNzoOI8hayvqFovpAWU/216Bo96rfKVSPjxU3c3U6nWubm089+WS0GVVUZxPhBThWVgOgE6W5KAT7/vvvx+EIicylnarFZdm07NLUcm7roE1RfEdzUWTnc3/92WiPViOZYGbgVlkNgE5k5qJ1VvHNN8UObxno5mKspaz3OnPeVwovltRT5sFctNj5q7/4y2g3StEeL9KTwMMRAeCcaMxlVFb/4Pvfj0lEwzAXe1DbukClv5pafFrfObqbq9FoZB6bjXz5GeEEM0+V1QAwxqIyF4Vgf/rTnwof3jLomUtviH6+ePd3+oKLeTEXP0Q1wcxbZTUAjEViLsqN/u5LL8XqDdZkLiM31RSqF8lckUwwM/dlA8CVsM1lVFY/9eSTcQrEaseVXM9c1OjOnJUqkrnorCDMRxy9shpMO2Gbi14V37p4MUYjyEzVP9LsunKfMca0e++98ZtjjfUS6PUPrqt/iPAnmKGyGoxKqOai3OivPf449gU8Q5Hy0N5XUFkNxiBUc1FldazCWzEltJMTVFaD8QjPXJTiOPNPj8QpvBVXlhYXwzleRGU1GI+QzEWB+YfSX4xReCvOFPL5ECaYobIajE1I5qLKah76EAAvhDPBDJXVYGzCMBdFfFN/87cYQSYKdLwYqFNQWQ0mIXBzdTqdZ556SpYSX/3yl9GjWRSCnmCGymowIYGbi/YdyNYRjkAD56isBhMSrLmoslqWEjuvvx7oAwHfCW6CGSqrweQEa67vvfKKLCW+sbSE8JZwBDfBDJXVYHJGM5d2XMklLd2Kh0JRXowgE5SAJpihshr4wkjmooafnirvOp3OP/7DQ7KUuHnz5vhXB6KDinL8XRkZfdmwBgcTMoq52gfba1e3f+Cp58GPf/Qjqqye5OJAhAQxwQyV1cAvvJtLaymvPl3845+VjbTREWEIH330kSwlvpBM4a1VaPzd1qGyGviIZ3Np9dKFV5WWxloD420GoG4Q57PZ3eu7e3t7iHMJio8TzIy+bB4SuFqq8s72yozeF2i+ULpVaz74sHrX6c0STBtezdWdYcP0aFdyQ2nZ7xhPTk52Xt95bnlZ7jWlStCb7Y1yeX9/HyITBR8nmHmtrG4flVZm5PlCSVHb+ldUpVhYcFvmg2nDo7mMGTaM6Q0/Z3KVe65xelVVq5XKtc1NOgiHyMSCzgEnT3P3Wlmt3auuzcjzO7W25S/r7Ljyr09z35ERhIk3c7WU9WTCop5RexN3Op1hIpvLZAr5fLVSOTw8RDkIP/g1wYz6srndDzWPHbK20tT3bgnRChuEhBdznarFF/v/ns6OK4Nj6Eej0+kcHh5WK5VCPk8REIiMN3yZYOa5svq+Uph1iEIAYMaDudoH22tl6/tdS1lP+tlS/eTkZJjIqMtdtVJRVRWHlSEzl8lM0phohMpqWtdbzUUphMYfw0RvliBOuJlLO1Y2sjYniVq9lE3JUvaKchzEWySJ7Ea5TBlAEFlUTHi8SH3ZPCWF2ZuLMcZY+2B7PuV8nA2mDUdz6XoaiGqZvy65ZEj4QqPR2N/ftxXZ5VyOci9QBxcEk0wwG62yWv+jsltVafVSFuYCfUQz43pCSGS713eNkRwQWUBQlGq8898RK6v12rIFPfnGBMwFBhDSXBZUVd3b27MVGXIvJmTsCWbjVFa3D7bnU3LfhF3GGMwFbIiDuSyQyJBE5hdjHC+OXVmtNe9cmU/JUna9qKh6VldLVYrriHOBfmJoLjMOSWQkMuReuDLGBLOJKqu1Zu3WL3rVP1IivbJ181atiWQJYCLm5rJgiAxJZN4p5PMj9S9FZTUIgekylwWHJLK5TAa5F8RIE8xGqawGYHym2lwWzCJDEpnBSBPMMLMahAPMNRSHJLKlxcXpyb2g40UvQSvMrAahAXN5xRDZFCaReVxGeausBsAHYK4xcUgiI5HFKffCywQzzKwGYQJz+YMhslgmkVFOicMNMLMahAzM5T+uSWQ3ymWxci9cJ5iNUFkNgB/AXIETg5aKzhPMMLMahA/MFTbOSWQkMt5yL+jQcNiSCjOrQfjAXBEjSkvFYeXTmFkNIgHm4gvnJDJDZOFfmG2LQcysBlEBc3ENPy0Vd6/vzmUyli9iZjWICphLJFyTyIKbyzt4vIjKahAhMJfAhNlS0TLBDJXVIFpgrvjgZS7v2KKhkJZRA4TKahAtMFc8CSKJzJhghspqEDkw11TgcS6vs4mM40VUVoPIgbmmkfGSyGiHiMpqwAMwF2AnJydekshu/+pXspSY/epXUVkNIgfmAlYcksjoA5XVIHJgLuBCo9Gg3Itvv/CCLCWeW16O+ooAgLnAiGCfCHgA5gIAiAfMBQAQD5gLACAeMBdwRmspG2mbE8bUQuG/qorajvr6wHQCcwEv3FcKs3K2qGr0qdZWlVIhK0uJ9Eq53taivTgwhcBcwAsWcxGtenEtLaUWtg6w8gIhA3MBL9iaizHtXnVtRpaWS+ppNNcFphWYC3hhiLnYqVpclqXU+WIdO0YQJjAX8MIwc+nx+3RBaUVyXWBagbmAF2AuwBcwF/ACdouAL2Au4AXHCH3y5erxWTTXBaYVmAt4AVkRgC9gLuCFYZmoqYWNO01sFEHowFzAmWHVPzOrW794rwZrgWiAuQAA4gFzAQDEA+YCAIgHzAUAEA+YCwAgHjAXAEA8YC4AgHjAXAAA8YC5AADiAXMBAMQD5gIAiAfMBQAQj/8HYYO8dGGZk7EAAAAASUVORK5CYII=" alt></span></p>
</div>

<div class="question" style="padding-left: 20px; padding-right: 20px;">
<p><span>Write down an equation showing this information, taking \(b\) to be the cost of one tin of beans and \(c\) to be the cost of one packet of cereal in \({\text{AUD}}\).</span></p>
<div class="marks">[1]</div>
<div class="question_part_label">i.a.</div>
</div>
<div class="question" style="padding-left: 20px; padding-right: 20px;">
<p><span>Stephen thinks that Mal has not bought enough so he buys \(12\) more tins of beans and \(6\) more packets of cereal. He pays \(66{\text{ AUD}}\).</span></p>
<p><span>Write down another equation to represent this information.</span></p>
<div class="marks">[1]</div>
<div class="question_part_label">i.b.</div>
</div>
<div class="question" style="padding-left: 20px; padding-right: 20px;">
<p><span><span>Stephen thinks that Mal has not bought enough so he buys \(12\) more tins of beans and \(6\) more packets of cereal. He pays \(66{\text{ AUD}}\).</span></span></p>
<p><span>Find the cost of one tin of beans.</span></p>
<div class="marks">[2]</div>
<div class="question_part_label">i.c.</div>
</div>
<div class="question" style="padding-left: 20px; padding-right: 20px;">
<p><span>(i)     Sketch the graphs of the two equations from parts (a) and (b).</span></p>
<p><span>(ii)    Write down the coordinates of the point of intersection of the two graphs.</span></p>
<div class="marks">[4]</div>
<div class="question_part_label">i.d.</div>
</div>
<div class="question" style="padding-left: 20px; padding-right: 20px;">
<p><span>Using the letters on the diagram draw a triangle showing the position of a \({70^ \circ }\) angle.</span></p>
<div class="marks">[1]</div>
<div class="question_part_label">ii.a.</div>
</div>
<div class="question" style="padding-left: 20px; padding-right: 20px;">
<p><span>Show that the height of the pyramid is \(13.7{\text{ cm}}\), to 3 significant figures.</span></p>
<div class="marks">[2]</div>
<div class="question_part_label">ii.b.</div>
</div>
<div class="question" style="padding-left: 20px; padding-right: 20px;">
<p><span>Calculate</span></p>
<p><span>(i)     the length of \({\text{EG}}\);</span></p>
<p><span>(ii)    the size of angle \({\text{DEC}}\).</span></p>
<div class="marks">[4]</div>
<div class="question_part_label">ii.c.</div>
</div>
<div class="question" style="padding-left: 20px; padding-right: 20px;">
<p><span>Find the total surface area of the pyramid.</span></p>
<div class="marks">[2]</div>
<div class="question_part_label">ii.d.</div>
</div>
<div class="question" style="padding-left: 20px; padding-right: 20px;">
<p><span>Find the volume of the pyramid.</span></p>
<div class="marks">[2]</div>
<div class="question_part_label">ii.e.</div>
</div>
<h2 style="margin-top: 1em">Markscheme</h2>
<div class="question" style="padding-left: 20px;">
<p><span>\(50b + 20c = 260\)     <em><strong>(A1)</strong></em></span></p>
<p><span><em><strong>[1 mark]<br></strong></em></span></p>
<div class="question_part_label">i.a.</div>
</div>
<div class="question" style="padding-left: 20px;">
<p><span>\(12b + 6c = 66\)     <em><strong>(A1)</strong></em></span></p>
<p><span><em><strong>[1 mark]<br></strong></em></span></p>
<div class="question_part_label">i.b.</div>
</div>
<div class="question" style="padding-left: 20px;">
<p><span>Solve to get \(b = 4\)     <em><strong>(M1)(A1)</strong></em><strong>(ft)<em>(G2)</em></strong></span></p>
<p><span><strong>Note: <em>(M1)</em></strong> for attempting to solve the equations simultaneously.</span></p>
<p><span><strong><em>[2 marks]</em></strong><br></span></p>
<div class="question_part_label">i.c.</div>
</div>
<div class="question" style="padding-left: 20px;">
<p><span>(i)</span></p>
<p><span><img src="data:image/png;base64,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" alt>     <em><strong>(A1)(A1)(A1)</strong></em></span></p>
<p><span><em><strong><br></strong></em><span><strong>Notes:</strong> Award <em><strong>(A1)</strong></em> for labels and some idea of scale, <strong><em>(A1)</em>(ft)<em>(A1)</em>(ft)</strong> for each line.</span></span><span><strong><br></strong>The axis can be reversed.</span></p>
<p><span><br>(ii)    \((4,3)\) or \((3,4)\)     <strong><em>(A1)</em>(ft)</strong></span></p>
<p><span><strong><br>Note: </strong>Accept \(b = 4\), \(c = 3\)<br></span></p>
<p><span><em><strong>[4 marks]</strong></em><br></span></p>
<div class="question_part_label">i.d.</div>
</div>
<div class="question" style="padding-left: 20px;">
<p><span><img src="data:image/png;base64,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" alt></span><span><span>    </span><span> <em><strong>(A1)</strong></em></span></span></p>
<p><span><span><em><strong>[1 mark]<br></strong></em></span></span></p>
<div class="question_part_label">ii.a.</div>
</div>
<div class="question" style="padding-left: 20px;">
<p><span>\(\tan 70 = \frac{h}{5}\)     <em><strong>(M1)</strong></em></span></p>
<p><span>\(h = 5\tan 70 = 13.74\)     <em><strong>(A1)</strong></em></span></p>
<p><span>\(h = 13.7{\text{ cm}}\)     <em><strong>(AG)</strong></em></span></p>
<p><span><em><strong>[2 marks]<br></strong></em></span></p>
<div class="question_part_label">ii.b.</div>
</div>
<div class="question" style="padding-left: 20px;">
<p><span><em>Unit penalty <strong>(UP)</strong> is applicable in this part of the question where indicated in the left hand column.</em></span></p>
<p><span>(i)     \({\text{E}}{{\text{G}}^2} = {5^2} + {13.7^2}\) OR \({5^2} + {(5\tan 70)^2}\)     <em><strong>(M1)</strong></em></span></p>
<p><span><em><strong>(UP)</strong></em>     \({\text{EG}} = 14.6{\text{ cm}}\)     <em><strong>(A1)(G2)</strong></em></span></p>
<p><span>(ii)    \({\text{DEC}} = 2 \times {\tan ^{ - 1}}\left( {\frac{5}{{14.6}}} \right)\)     <em><strong>(M1)</strong></em></span></p>
<p><span>\( = {37.8^ \circ }\)     <em><strong>(A1)</strong></em><strong>(ft)<em>(G2)</em></strong></span></p>
<p><span><strong><em>[4 marks]<br></em></strong></span></p>
<div class="question_part_label">ii.c.</div>
</div>
<div class="question" style="padding-left: 20px;">
<p><span><em>Unit penalty <strong>(UP)</strong> is applicable in this part of the question where indicated in the left hand column.</em></span></p>
<p><span>\({\text{Area}} = 10 \times 10 + 4 \times 0.5 \times 10 \times 14.619\)     <em><strong>(M1)</strong></em><br></span></p>
<p><span><em><strong>(UP)</strong></em>     \( = 392{\text{ c}}{{\text{m}}^2}\)     <em><strong>(A1)</strong></em><strong>(ft)</strong><em><strong>(G2)</strong></em><br></span></p>
<p><span><em><strong>[2 marks]<br></strong></em></span></p>
<div class="question_part_label">ii.d.</div>
</div>
<div class="question" style="padding-left: 20px;">
<p><span><em>Unit penalty <strong>(UP)</strong> is applicable in this part of the question where indicated in the left hand column.</em></span></p>
<p><span>\({\text{Volume}} = \frac{1}{3} \times 10 \times 10 \times 13.7\)     <em><strong>(M1)</strong></em></span></p>
<p><span><em><strong>(UP)</strong></em>     \( = 457{\text{ c}}{{\text{m}}^3}\) (\(458{\text{ c}}{{\text{m}}^3}\))     <em><strong>(A1)(G2)</strong></em></span></p>
<p><span><em><strong>[2 marks]<br></strong></em></span></p>
<div class="question_part_label">ii.e.</div>
</div>
<h2 style="margin-top: 1em">Examiners report</h2>
<div class="question" style="padding-left: 20px;">
<p><span style="font-family: times new roman,times; font-size: medium;">Most candidates managed to write down the equation.</span></p>
<div class="question_part_label">i.a.</div>
</div>
<div class="question" style="padding-left: 20px;">
<p><span style="font-family: times new roman,times; font-size: medium;">Most candidates managed to write down the equation.</span></p>
<div class="question_part_label">i.b.</div>
</div>
<div class="question" style="padding-left: 20px;">
<p><span style="font-family: times new roman,times; font-size: medium;">Many managed to find the correct answer and the others tried their best but made some mistake in the process.</span></p>
<div class="question_part_label">i.c.</div>
</div>
<div class="question" style="padding-left: 20px;">
<p><span style="font-family: times new roman,times; font-size: medium;">(i)&nbsp;&nbsp;&nbsp;&nbsp; Few candidates sketched the graphs well. Few used a ruler.</span></p>
<p><span style="font-family: times new roman,times; font-size: medium;">(ii)&nbsp;&nbsp;&nbsp; Many candidates could not be awarded ft from their graph because the answer they gave was not possible.</span></p>
<div class="question_part_label">i.d.</div>
</div>
<div class="question" style="padding-left: 20px;">
<p><span style="font-family: times new roman,times; font-size: medium;">Very few correct drawings.</span></p>
<div class="question_part_label">ii.a.</div>
</div>
<div class="question" style="padding-left: 20px;">
<p><span style="font-family: times new roman,times; font-size: medium;">Some managed to show this more by good fortune and ignoring their original triangle than by good reasoning.</span></p>
<div class="question_part_label">ii.b.</div>
</div>
<div class="question" style="padding-left: 20px;">
<p><span style="font-family: times new roman,times; font-size: medium;">(i)&nbsp;&nbsp;&nbsp;&nbsp; Many found this as ft from the previous part. Some lost a UP here.</span></p>
<p><span style="font-family: times new roman,times; font-size: medium;">(ii)&nbsp;&nbsp;&nbsp; This was not well done. The most common answer was \({40^ \circ }\).</span></p>
<div class="question_part_label">ii.c.</div>
</div>
<div class="question" style="padding-left: 20px;">
<p><span style="font-family: times new roman,times; font-size: medium;">Many managed this or were awarded ft points.</span></p>
<div class="question_part_label">ii.d.</div>
</div>
<div class="question" style="padding-left: 20px;">
<p><span style="font-family: times new roman,times; font-size: medium;">This was well done and most candidates also remembered their units on this part.</span></p>
<div class="question_part_label">ii.e.</div>
</div>
<br><hr><br><div class="specification">
<p><span style="font-family: times new roman,times; font-size: medium;">Consider the curve \(y = {x^3} + \frac{3}{2}{x^2} - 6x - 2\)</span> .</p>
</div>

<div class="question" style="padding-left: 20px; padding-right: 20px;">
<p><span>(i)     Write down the value of \(y\) when \(x\) is \(2\).</span></p>
<p><span>(ii)    Write down the coordinates of the point where the curve intercepts the \(y\)-axis.</span></p>
<div class="marks">[3]</div>
<div class="question_part_label">a.</div>
</div>
<div class="question" style="padding-left: 20px; padding-right: 20px;">
<p><span>Sketch the curve for \( - 4 \leqslant x \leqslant 3\) and \( - 10 \leqslant y \leqslant 10\). Indicate clearly the information found in (a).</span></p>
<div class="marks">[4]</div>
<div class="question_part_label">b.</div>
</div>
<div class="question" style="padding-left: 20px; padding-right: 20px;">
<p><span>Find \(\frac{{{\text{d}}y}}{{{\text{d}}x}}\) .</span></p>
<div class="marks">[3]</div>
<div class="question_part_label">c.</div>
</div>
<div class="question" style="padding-left: 20px; padding-right: 20px;">
<p><span>Let \({L_1}\) be the tangent to the curve at \(x = 2\).</span></p>
<p><span>Let \({L_2}\) be a tangent to the curve, parallel to \({L_1}\).</span></p>
<p><span>(i)     Show that the gradient of \({L_1}\) is \(12\).</span></p>
<p><span>(ii)    Find the \(x\)-coordinate of the point at which \({L_2}\) and the curve meet.</span></p>
<p><span>(iii)   Sketch and label \({L_1}\) and \({L_2}\) on the diagram drawn in (b).</span></p>
<div class="marks">[8]</div>
<div class="question_part_label">d.</div>
</div>
<div class="question" style="padding-left: 20px; padding-right: 20px;">
<p><span>It is known that \(\frac{{{\text{d}}y}}{{{\text{d}}x}} &gt; 0\) for \(x &lt; - 2\) and \(x &gt; b\) where \(b\) is positive.</span></p>
<p><span>(i)     Using your graphic display calculator, or otherwise, find the value of \(b\).</span></p>
<p><span>(ii)    Describe the behaviour of the curve in the interval \( - 2 &lt; x &lt; b\) .</span></p>
<p><span>(iii)   Write down the equation of the tangent to the curve at \(x = - 2\).</span></p>
<div class="marks">[5]</div>
<div class="question_part_label">e.</div>
</div>
<h2 style="margin-top: 1em">Markscheme</h2>
<div class="question" style="padding-left: 20px;">
<p><span>(i)     \(y = 0\)     <em><strong>(A1)</strong></em></span></p>
<p><span>(ii)    \((0{\text{, }}{- 2})\)     <em><strong>(A1)(A1)</strong></em></span></p>
<p><br><span><strong>Note: </strong>Award <em><strong>(A1)(A0)</strong></em> if brackets missing.</span></p>
<p><br><span><strong>OR</strong></span></p>
<p><span>\(x = 0{\text{, }}y = - 2\)     <em><strong>(A1)(A1)</strong></em></span></p>
<p><br><span><strong>Note: </strong>If coordinates reversed award <strong><em>(A0)(A1)</em>(ft)</strong>. Two coordinates must be given.</span></p>
<p><span><em><strong>[3 marks]</strong></em><br></span></p>
<div class="question_part_label">a.</div>
</div>
<div class="question" style="padding-left: 20px;">
<p><span><img src="data:image/png;base64,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" alt></span><span>     <em><strong>(A4)</strong></em></span></p>
<p><span><strong>Notes: <em>(A1)</em></strong> for appropriate window. Some indication of scale on the \(x\)-axis must be present (for example ticks). Labels not required. <em><strong>(A1)</strong></em> for smooth curve and shape, <em><strong>(A1)</strong></em> for maximum and minimum in approximately correct position, <em><strong>(A1)</strong></em> for \(x\) and \(y\) intercepts found in (a) in approximately correct position.</span></p>
<p><span><em><strong>[4 marks]</strong></em><br></span></p>
<div class="question_part_label">b.</div>
</div>
<div class="question" style="padding-left: 20px;">
<p><span>\(\frac{{{\text{d}}y}}{{{\text{d}}x}} = 3{x^2} + 3x - 6\)     <em><strong>(A1)(A1)(A1)</strong></em></span></p>
<p><span><strong> Note: <em>(A1)</em></strong> for each correct term. Award <em><strong>(A1)(A1)(A0)</strong></em> at most if any other term is present.</span></p>
<p><span><em><strong>[3 marks]</strong></em><br></span></p>
<div class="question_part_label">c.</div>
</div>
<div class="question" style="padding-left: 20px;">
<p><span>(i)     \(3 \times 4 + 3 \times 2 - 6 = 12\)     <em><strong>(M1)(A1)(AG)</strong></em></span></p>
<p><br><span><strong>Note: <em>(M1)</em></strong> for using the derivative and substituting \(x = 2\) . <em><strong>(A1)</strong></em> for correct (and clear) substitution. The \(12\) must be seen.</span></p>
<p><br><span>(ii)    Gradient of \({L_2}\) is \(12\) (can be implied)     <em><strong>(A1)</strong></em></span></p>
<p><span>\(3{x^2} + 3x - 6 = 12\)     <em><strong>(M1)</strong></em></span></p>
<p><span>\(x = - 3\)     <em><strong>(A1)(G2)</strong></em></span><br><br></p>
<p><span><strong>Note: <em>(M1)</em></strong> for equating the derivative to \(12\) or showing a sketch of the derivative together with a line at \(y = 12\) or a table of values showing the \(12\) in the derivative column.</span></p>
<p><br><span>(iii)   <em><strong>(A1)</strong></em> for \({L_1}\) correctly drawn at approx the correct point     <em><strong>(A1)</strong></em></span></p>
<p><span><em><strong>(A1)</strong></em> for \({L_2}\) correctly drawn at approx the correct point</span><span>     <em><strong>(A1)</strong></em></span></p>
<p><span><em><strong>(A1)</strong></em> for 2 parallel lines</span><span>     <em><strong>(A1)</strong></em></span></p>
<p><br><span><strong>Note: </strong>If lines are not labelled award at most <em><strong>(A1)(A1)(A0)</strong></em>. Do not accept 2 horizontal or 2 vertical parallel lines.</span></p>
<p><span><em><strong>[8 marks]</strong></em><br></span></p>
<div class="question_part_label">d.</div>
</div>
<div class="question" style="padding-left: 20px;">
<p><span>(i)     \(b = 1\)     <em><strong>(G2)</strong></em></span></p>
<p><span>(ii)    The curve is decreasing.     <em><strong>(A1)</strong></em></span></p>
<p><br><span><strong>Note: </strong>Accept any valid description.</span></p>
<p><br><span>(iii)   \(y = 8\)     <em><strong>(A1)(A1)(G2)</strong></em></span></p>
<p><br><span><strong>Note: <em>(A1)</em></strong> for “\(y =\) a constant”, <em><strong>(A1)</strong></em> for \(8\).</span></p>
<p><span><em><strong>[5 marks]</strong></em><br></span></p>
<div class="question_part_label">e.</div>
</div>
<h2 style="margin-top: 1em">Examiners report</h2>
<div class="question" style="padding-left: 20px;">
<p><span style="font-family: times new roman,times; font-size: medium;">Many candidates managed to gain good marks in this question as they were able to answer the first three parts of the question. Good sketches were drawn with the required information shown on them. Very few candidates did not recognise the notation \(\frac{{{\text{d}}y}}{{{\text{d}}x}}\) but they showed that they knew how to differentiate as in (d)(i) they found the derivative to show that the gradient of \({L_1}\) was \(12\). Candidates found it difficult to find the other \(x\) for which the derivative was \(12\). However, some could draw both tangents without having found this value of \(x\) . In general, tangents were not well drawn. The last part question did act as a discriminating question. However, those candidates that had the function drawn either in their GDC or on paper recognised that at \(x = - 2\) there was a maximum and so wrote down the correct equation of the tangent at that point.</span></p>
<div class="question_part_label">a.</div>
</div>
<div class="question" style="padding-left: 20px;">
<p><span style="font-family: times new roman,times; font-size: medium;">Many candidates managed to gain good marks in this question as they were able to answer the first three parts of the question. Good sketches were drawn with the required information shown on them. Very few candidates did not recognise the notation \(\frac{{{\text{d}}y}}{{{\text{d}}x}}\) but they showed that they knew how to differentiate as in (d)(i) they found the derivative to show that the gradient of \({L_1}\) was \(12\). Candidates found it difficult to find the other \(x\) for which the derivative was \(12\). However, some could draw both tangents without having found this value of \(x\) . In general, tangents were not well drawn. The last part question did act as a discriminating question. However, those candidates that had the function drawn either in their GDC or on paper recognised that at \(x = - 2\) there was a maximum and so wrote down the correct equation of the tangent at that point.</span></p>
<div class="question_part_label">b.</div>
</div>
<div class="question" style="padding-left: 20px;">
<p><span style="font-family: times new roman,times; font-size: medium;">Many candidates managed to gain good marks in this question as they were able to answer the first three parts of the question. Good sketches were drawn with the required information shown on them. Very few candidates did not recognise the notation \(\frac{{{\text{d}}y}}{{{\text{d}}x}}\) but they showed that they knew how to differentiate as in (d)(i) they found the derivative to show that the gradient of \({L_1}\) was \(12\). Candidates found it difficult to find the other \(x\) for which the derivative was \(12\). However, some could draw both tangents without having found this value of \(x\) . In general, tangents were not well drawn. The last part question did act as a discriminating question. However, those candidates that had the function drawn either in their GDC or on paper recognised that at \(x = - 2\) there was a maximum and so wrote down the correct equation of the tangent at that point.</span></p>
<div class="question_part_label">c.</div>
</div>
<div class="question" style="padding-left: 20px;">
<p><span style="font-family: times new roman,times; font-size: medium;">Many candidates managed to gain good marks in this question as they were able to answer the first three parts of the question. Good sketches were drawn with the required information shown on them. Very few candidates did not recognise the notation \(\frac{{{\text{d}}y}}{{{\text{d}}x}}\) but they showed that they knew how to differentiate as in (d)(i) they found the derivative to show that the gradient of \({L_1}\) was \(12\). Candidates found it difficult to find the other \(x\) for which the derivative was \(12\). However, some could draw both tangents without having found this value of \(x\) . In general, tangents were not well drawn. The last part question did act as a discriminating question. However, those candidates that had the function drawn either in their GDC or on paper recognised that at \(x = - 2\) there was a maximum and so wrote down the correct equation of the tangent at that point.</span></p>
<div class="question_part_label">d.</div>
</div>
<div class="question" style="padding-left: 20px;">
<p><span style="font-family: times new roman,times; font-size: medium;">Many candidates managed to gain good marks in this question as they were able to answer the first three parts of the question. Good sketches were drawn with the required information shown on them. Very few candidates did not recognise the notation \(\frac{{{\text{d}}y}}{{{\text{d}}x}}\) but they showed that they knew how to differentiate as in (d)(i) they found the derivative to show that the gradient of \({L_1}\) was \(12\). Candidates found it difficult to find the other \(x\) for which the derivative was \(12\). However, some could draw both tangents without having found this value of \(x\) . In general, tangents were not well drawn. The last part question did act as a discriminating question. However, those candidates that had the function drawn either in their GDC or on paper recognised that at \(x = - 2\) there was a maximum and so wrote down the correct equation of the tangent at that point.</span></p>
<div class="question_part_label">e.</div>
</div>
<br><hr><br><div class="specification">
<p><span style="font-family: times new roman,times; font-size: medium;">The diagram shows a Ferris wheel that moves with constant speed and completes a rotation every 40 seconds. The wheel has a radius of \(12\) m and its lowest point is \(2\) m above the ground.</span></p>
<p><span style="font-family: times new roman,times; font-size: medium;"><img src="data:image/png;base64,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" alt></span></p>
<p>&nbsp;</p>
</div>

<div class="question" style="padding-left: 20px; padding-right: 20px;">
<p><span>Initially, a seat C is vertically below the centre of the wheel, O. It then rotates in an anticlockwise (counterclockwise) direction.</span></p>
<p><span>Write down</span></p>
<p><span>(i)     the height of O above the ground;</span></p>
<p><span>(ii)    the maximum height above the ground reached by C .</span></p>
<div class="marks">[2]</div>
<div class="question_part_label">a.</div>
</div>
<div class="question" style="padding-left: 20px; padding-right: 20px;">
<p><span>In a revolution, C reaches points A and B , which are at the same height above the ground as the centre of the wheel. Write down the number of seconds taken for C to first reach A and then B .</span></p>
<div class="marks">[2]</div>
<div class="question_part_label">b.</div>
</div>
<div class="question" style="padding-left: 20px; padding-right: 20px;">
<p><span>The sketch below shows the graph of the function, \(h(t)\) , for the height above ground of C, where \(h\) is measured in metres and \(t\) is the time in seconds, \(0 \leqslant t \leqslant 40\) .</span></p>
<p><span><img src="data:image/png;base64,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" alt></span></p>
<p><span><strong>Copy</strong> the sketch and show the results of part (a) and part (b) on your diagram. Label the points clearly with their coordinates.</span></p>
<div class="marks">[4]</div>
<div class="question_part_label">c.</div>
</div>
<h2 style="margin-top: 1em">Markscheme</h2>
<div class="question" style="padding-left: 20px;">
<p><span>(i)     \(14\) m     <em><strong>(A1)</strong></em></span></p>
<p> </p>
<p><span>(ii)    \(26\) m     <em><strong>(A1)</strong></em></span></p>
<p> </p>
<p><span><em><strong>[2 marks]</strong></em></span></p>
<div class="question_part_label">a.</div>
</div>
<div class="question" style="padding-left: 20px;">
<p><span>A:\(10\), B:\(30\)     <em><strong>(A1)(A1)</strong></em></span></p>
<p><span><em><strong>[2 marks]</strong></em></span></p>
<div class="question_part_label">b.</div>
</div>
<div class="question" style="padding-left: 20px;">
<p><span><img src="data:image/png;base64,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" alt width="771" height="217"><span>     <strong><em>(A1)</em>(ft)<em>(A1)</em>(ft)<em>(A1)</em>(ft)<em>(A1)</em>(ft)</strong></span></span></p>
<p><br><span><strong>Note:</strong> Award <strong><em>(A1)</em>(ft)</strong> for coordinates of each point clearly indicated either by scale or by coordinate pairs. Points need not be labelled A and B in the second diagram. Award a maximum of <strong><em>(A1)(A0)(A1)</em>(ft)<em>(A1)</em>(ft)</strong> if coordinates are reversed. Do not penalise reversed coordinates if this has already been penalised in Q4(a)(iii).</span></p>
<p><em><strong><span>[4 marks]</span></strong></em></p>
<div class="question_part_label">c.</div>
</div>
<h2 style="margin-top: 1em">Examiners report</h2>
<div class="question" style="padding-left: 20px;">
<p><span style="font-family: times new roman,times; font-size: medium;">Most candidates were able to start this question. Those of an average ability completed it to the end of part (c) and the best gained good success in the latter parts. Its purpose was to discriminate at the highest level and this it did.</span></p>
<p><span style="font-family: times new roman,times; font-size: medium;">Some concerns were raised on the G2 forms as to the appropriateness of this question. However, the MSSL course tries in part to link areas of the syllabus to &ldquo;real-life&rdquo; situations and address these. A look back to past years&rsquo; examination papers, and to the syllabus documentation, should yield similar examples.</span></p>
<div class="question_part_label">a.</div>
</div>
<div class="question" style="padding-left: 20px;">
<p><span style="font-family: times new roman,times; font-size: medium;">Most candidates were able to start this question. Those of an average ability completed it to the end of part (c) and the best gained good success in the latter parts. Its purpose was to discriminate at the highest level and this it did.</span></p>
<p><span style="font-family: times new roman,times; font-size: medium;">Some concerns were raised on the G2 forms as to the appropriateness of this question. However, the MSSL course tries in part to link areas of the syllabus to &ldquo;real-life&rdquo; situations and address these. A look back to past years&rsquo; examination papers, and to the syllabus documentation, should yield similar examples.</span></p>
<div class="question_part_label">b.</div>
</div>
<div class="question" style="padding-left: 20px;">
<p><span style="font-family: times new roman,times; font-size: medium;">Most candidates were able to start this question. Those of an average ability completed it to the end of part (c) and the best gained good success in the latter parts. Its purpose was to discriminate at the highest level and this it did.</span></p>
<p><span style="font-family: times new roman,times; font-size: medium;">Some concerns were raised on the G2 forms as to the appropriateness of this question. However, the MSSL course tries in part to link areas of the syllabus to &ldquo;real-life&rdquo; situations and address these. A look back to past years&rsquo; examination papers, and to the syllabus documentation, should yield similar examples.</span></p>
<div class="question_part_label">c.</div>
</div>
<br><hr><br><div class="specification">
<p><span style="font-size: medium; font-family: times new roman,times;">A manufacturer claims that fertilizer has an effect on the height of rice plants. He measures the height of fertilized and unfertilized plants. The results are given in the following table.</span></p>
<p><span style="font-size: medium; font-family: times new roman,times;"><img src="data:image/png;base64,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" alt></span></p>
<p><span style="font-size: medium; font-family: times new roman,times;">A chi-squared test is performed to decide if the manufacturer&rsquo;s claim is justified</span> <span style="font-size: medium; font-family: times new roman,times;">at the<strong> 1 %</strong> level of significance.</span></p>
</div>

<div class="specification">
<p><span style="font-size: medium; font-family: times new roman,times;">The population of fleas on a dog after <em>t</em> days, is modelled by</span></p>
<p><span style="font-size: medium; font-family: times new roman,times;">\[N = 4 \times {(2)^{\frac{t}{4}}},{\text{ }}t \geqslant 0\]</span></p>
<p><span style="font-size: medium; font-family: times new roman,times;">Some values of<em> N</em> are shown in the table below.</span></p>
<p><span style="font-size: medium; font-family: times new roman,times;"><img src="data:image/png;base64,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" alt></span></p>
</div>

<div class="question" style="padding-left: 20px; padding-right: 20px;">
<p><span>Write down the null and alternative hypotheses for this test.</span></p>
<div class="marks">[2]</div>
<div class="question_part_label">i, a.</div>
</div>
<div class="question" style="padding-left: 20px; padding-right: 20px;">
<p><span>For the number of fertilized plants with height greater than 75 cm,</span> <span>show that the expected value is 97.5.</span></p>
<div class="marks">[3]</div>
<div class="question_part_label">i, b.</div>
</div>
<div class="question" style="padding-left: 20px; padding-right: 20px;">
<p><span>Write down the value of \(\chi_{calc}^2\).</span></p>
<div class="marks">[2]</div>
<div class="question_part_label">i, c.</div>
</div>
<div class="question" style="padding-left: 20px; padding-right: 20px;">
<p><span>Write down the number of degrees of freedom.</span></p>
<div class="marks">[1]</div>
<div class="question_part_label">i, d.</div>
</div>
<div class="question" style="padding-left: 20px; padding-right: 20px;">
<p><span>Is the manufacturer’s claim justified? Give a reason for your answer.</span></p>
<div class="marks">[2]</div>
<div class="question_part_label">i, f.</div>
</div>
<div class="question" style="padding-left: 20px; padding-right: 20px;">
<p><span>Write down the value of </span><span><em>p</em>.</span></p>
<div class="marks">[1]</div>
<div class="question_part_label">ii, a, i.</div>
</div>
<div class="question" style="padding-left: 20px; padding-right: 20px;">
<p><span>Write down the value of <em>q</em>.</span></p>
<div class="marks">[2]</div>
<div class="question_part_label">ii, a, ii.</div>
</div>
<div class="question" style="padding-left: 20px; padding-right: 20px;">
<p><span>Using the values in the table above, draw the graph of <em>N</em> for 0 ≤ <em>t</em> ≤ 20. Use 1 cm to represent 2 days on the horizontal axis and 1 cm to represent 10 fleas on the vertical axis.</span></p>
<div class="marks">[6]</div>
<div class="question_part_label">ii, b.</div>
</div>
<div class="question" style="padding-left: 20px; padding-right: 20px;">
<p><span><strong>Use your graph</strong> to estimate the number of days for the population of fleas to reach 55.</span></p>
<div class="marks">[2]</div>
<div class="question_part_label">ii, c.</div>
</div>
<h2 style="margin-top: 1em">Markscheme</h2>
<div class="question" style="padding-left: 20px;">
<p><span>H<sub>0</sub>: The height of the rice plants is independent of the use of</span> <span>a fertilizer.     <em><strong>(A1)</strong></em></span></p>
<p><br><span><strong>Notes:</strong> For independent accept “not associated”, can accept “the use of a</span> <span>fertilizer has no effect on the height of the plants”.</span></p>
<p><span>Do not accept “not correlated”. </span></p>
<p><br><span>H<sub>1</sub>: The height of the rice plants is not independent (dependent)</span> <span>of the use of fertilizer.     <em><strong>(A1)</strong></em><strong>(ft)</strong><br></span></p>
<p><span><strong> </strong></span></p>
<p> </p>
<p><span><strong>Note:</strong> If H<sub>0</sub> and H<sub>1</sub> are reversed award <em><strong>(A0)(A1)</strong></em><strong>(ft)</strong>.</span></p>
<p><span> </span></p>
<p><em><strong><span>[2 marks]</span></strong></em></p>
<div class="question_part_label">i, a.</div>
</div>
<div class="question" style="padding-left: 20px;">
<p><span>\(\frac{{180 \times 195}}{{360}}\) <strong>or</strong> \(\frac{{180}}{{360}} \times \frac{{195}}{{360}} \times 360\)     <em><strong>(A1)(A1)(M1)</strong></em></span></p>
<p><span>= 97.5     <em><strong>(AG)</strong></em></span></p>
<p><strong><em> </em></strong></p>
<p><span><strong>Notes:</strong> Award <em><strong>(A1)</strong></em> for numerator, <em><strong>(A1)</strong></em> for denominator <em><strong>(M1)</strong></em> for division.</span></p>
<p><span>If final 97.5 is not seen award at most <em><strong>(A1)(A0)(M1)</strong></em>.</span></p>
<p><span> </span></p>
<p><em><strong><span>[3 marks]</span></strong></em></p>
<div class="question_part_label">i, b.</div>
</div>
<div class="question" style="padding-left: 20px;">
<p><span>\( \chi_{calc}^2 = 14.01 (14.0, 14)\)     <em><strong>(G2)</strong></em></span></p>
<p><strong><span>OR</span></strong></p>
<p><span>If worked out by hand award <em><strong>(M1)</strong></em> for correct substituted formula with correct values, <em><strong>(A1)</strong></em> for correct answer.     <em><strong>(M1)(A1)</strong></em></span></p>
<p><span><em><strong>[2 marks]</strong></em></span></p>
<div class="question_part_label">i, c.</div>
</div>
<div class="question" style="padding-left: 20px;">
<p><span>2     <em><strong>(A1)</strong></em></span></p>
<p><span><em><strong>[1 mark]</strong></em></span></p>
<div class="question_part_label">i, d.</div>
</div>
<div class="question" style="padding-left: 20px;">
<p><span>\( \chi_{calc}^2 &gt; \chi_{crit}^2\)     <em><strong>(R1)</strong></em></span></p>
<p><span>The manufacturer's claim is justified. (or equivalent statement)     <em><strong>(A1)</strong></em> </span></p>
<p><span><strong><em> </em></strong></span></p>
<p><span><strong>Note:</strong> Do not accept <em><strong>(R0)(A1)</strong></em>.</span></p>
<p><span> </span></p>
<p><em><strong><span>[2 marks]</span></strong></em></p>
<div class="question_part_label">i, f.</div>
</div>
<div class="question" style="padding-left: 20px;">
<p><span>\(p = 4\)     <em><strong>(G1)</strong></em></span></p>
<p><span><em><strong>[1 mark]<br></strong></em></span></p>
<div class="question_part_label">ii, a, i.</div>
</div>
<div class="question" style="padding-left: 20px;">
<p><span>\(q = 4(2)^{\frac{16}{4}}\)     <strong><em>(M1)</em></strong></span></p>
<p><span>\(= 64\)     <strong><em>(A1)(G2)</em></strong></span></p>
<p><span><strong><em>[2 marks]</em></strong></span></p>
<div class="question_part_label">ii, a, ii.</div>
</div>
<div class="question" style="padding-left: 20px;">
<p><span><img src="data:image/png;base64,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" alt>     <span><em><strong>(A1)(A1)(A1)</strong></em>     <em><strong>(A3)</strong></em> </span></span></p>
<p><strong><em> </em></strong></p>
<p><span><strong>Notes:</strong> Award <em><strong>(A1)</strong></em> for <em>x</em> axis with correct scale and label, <em><strong>(A1)</strong></em> for <em>y</em> axis</span> <span>with correct scale and label.</span></p>
<p><span>Accept <em>x</em> and <em>y</em> for labels.</span></p>
<p><span>If <em>x</em> and <em>y</em> axis reversed award at most <em><strong>(A0)(A1)</strong></em><strong>(ft)</strong>.</span></p>
<p><span><em><strong>(A1)</strong></em> for smooth curve.</span></p>
<p><span>Award <em><strong>(A3)</strong></em> for all 6 points correct, <em><strong>(A2)</strong></em> for 4 or 5 points correct, </span><span><em><strong>(A1)</strong></em> for 2 or 3 points correct, <em><strong>(A0)</strong></em> otherwise.</span></p>
<p><span> </span></p>
<p><em><strong><span>[6 marks]</span></strong></em></p>
<div class="question_part_label">ii, b.</div>
</div>
<div class="question" style="padding-left: 20px;">
<p><span>15   (±0.8)     <em><strong>(M1)(A1)</strong></em><strong>(ft)</strong><em><strong>(G2)</strong></em> </span></p>
<p><br><span><strong>Note:</strong> Award <em><strong>(M1)</strong></em> for line drawn shown on graph, <em><strong>(A1)</strong></em><strong>(ft)</strong> from</span> <span>candidate's graph.</span></p>
<p><span> </span></p>
<p><em><strong><span>[2 marks]</span></strong></em></p>
<div class="question_part_label">ii, c.</div>
</div>
<h2 style="margin-top: 1em">Examiners report</h2>
<div class="question" style="padding-left: 20px;">
<p><span style="font-size: medium; font-family: times new roman,times;">It was clear that the candidates who performed poorly in part (i) lacked the basic knowledge</span> <span style="font-size: medium; font-family: times new roman,times;">of chi-squared analysis. Some mixed up the null and alternate hypotheses and also were not </span><span style="font-size: medium; font-family: times new roman,times;">able to correctly demonstrate the way of finding the expected value. There were many errors </span><span style="font-size: medium; font-family: times new roman,times;">in finding the critical value of \(\chi^2\) at the 1% level of significance.</span></p>
<div class="question_part_label">i, a.</div>
</div>
<div class="question" style="padding-left: 20px;">
<p><span style="font-size: medium; font-family: times new roman,times;">It was clear that the candidates who performed poorly in part (i) lacked the basic knowledge</span> <span style="font-size: medium; font-family: times new roman,times;">of chi-squared analysis. Some mixed up the null and alternate hypotheses and also were not </span><span style="font-size: medium; font-family: times new roman,times;">able to correctly demonstrate the way of finding the expected value. There were many errors </span><span style="font-size: medium; font-family: times new roman,times;">in finding the critical value of \(\chi^2\) at the 1% level of significance.</span></p>
<div class="question_part_label">i, b.</div>
</div>
<div class="question" style="padding-left: 20px;">
<p><span style="font-size: medium; font-family: times new roman,times;">It was clear that the candidates who performed poorly in part (i) lacked the basic knowledge</span> <span style="font-size: medium; font-family: times new roman,times;">of chi-squared analysis. Some mixed up the null and alternate hypotheses and also were not </span><span style="font-size: medium; font-family: times new roman,times;">able to correctly demonstrate the way of finding the expected value. There were many errors </span><span style="font-size: medium; font-family: times new roman,times;">in finding the critical value of \(\chi^2\) at the 1% level of significance.</span></p>
<div class="question_part_label">i, c.</div>
</div>
<div class="question" style="padding-left: 20px;">
<p><span style="font-size: medium; font-family: times new roman,times;">It was clear that the candidates who performed poorly in part (i) lacked the basic knowledge</span> <span style="font-size: medium; font-family: times new roman,times;">of chi-squared analysis. Some mixed up the null and alternate hypotheses and also were not </span><span style="font-size: medium; font-family: times new roman,times;">able to correctly demonstrate the way of finding the expected value. There were many errors </span><span style="font-size: medium; font-family: times new roman,times;">in finding the critical value of \(\chi^2\) at the 1% level of significance.</span></p>
<div class="question_part_label">i, d.</div>
</div>
<div class="question" style="padding-left: 20px;">
<p><span style="font-size: medium; font-family: times new roman,times;">It was clear that the candidates who performed poorly in part (i) lacked the basic knowledge</span> <span style="font-size: medium; font-family: times new roman,times;">of chi-squared analysis. Some mixed up the null and alternate hypotheses and also were not </span><span style="font-size: medium; font-family: times new roman,times;">able to correctly demonstrate the way of finding the expected value. There were many errors </span><span style="font-size: medium; font-family: times new roman,times;">in finding the critical value of \(\chi^2\) at the 1% level of significance.</span></p>
<div class="question_part_label">i, f.</div>
</div>
<div class="question" style="padding-left: 20px;">
<p><span style="font-size: medium; font-family: times new roman,times;">Candidates found this part rather easy, with some making arithmetic mistakes and thus losing one or more marks. The graph was well done with a high percentage scoring full marks. Some candidates did not label the axes, others had an incorrect scale and a few lost one mark for not drawing a smooth curve.</span></p>
<div class="question_part_label">ii, a, i.</div>
</div>
<div class="question" style="padding-left: 20px;">
<p style="margin: 0.0px 0.0px 10.0px 0.0px; font: 16.0px Times; color: #3f3f3f;"><span style="font-family: 'times new roman', times; font-size: medium;">Candidates found this part rather easy, with some making arithmetic mistakes and thus losing one or more marks. The graph was well done with a high percentage scoring full marks. Some candidates did not label the axes, others had an incorrect scale and a few lost one mark for not drawing a smooth curve.</span></p>
<div class="question_part_label">ii, a, ii.</div>
</div>
<div class="question" style="padding-left: 20px;">
<p><span style="font-size: medium; font-family: times new roman,times;">Candidates found this part rather easy, with some making arithmetic mistakes and thus losing one or more marks. The graph was well done with a high percentage scoring full marks. Some candidates did not label the axes, others had an incorrect scale and a few lost one mark for not drawing a smooth curve.</span></p>
<div class="question_part_label">ii, b.</div>
</div>
<div class="question" style="padding-left: 20px;">
<p><span style="font-size: medium; font-family: times new roman,times;">Candidates found this part rather easy, with some making arithmetic mistakes and thus losing one or more marks. The graph was well done with a high percentage scoring full marks. Some candidates did not label the axes, others had an incorrect scale and a few lost one mark for not drawing a smooth curve.</span></p>
<div class="question_part_label">ii, c.</div>
</div>
<br><hr><br><div class="question" style="padding-left: 20px; padding-right: 20px;">
<p>Hugo is given a rectangular piece of thin cardboard, \(16\,{\text{cm}}\) by \(10\,{\text{cm}}\). He decides to design a tray with it.</p>
<p>He removes from each corner the shaded squares of side \(x\,{\text{cm}}\), as shown in the following diagram.</p>
<p><img src="data:image/png;base64,iVBORw0KGgoAAAANSUhEUgAAAngAAAFKCAYAAACOzSIBAAAgAElEQVR4Ae3dC3hU1b338Z95ORQj5Sh6elJoIWkSeVEJJfWG+gJ6EMUbEayKYAM9AbFSCeDtSCJYoBVbTcALCLFcGlCoQNIbihFSHhXrJYKiFBIk4gFjK0gxDZYHZ95nzWTCJJkkGzJZ7D357udpmcysvf9rfdY4+c2+5RS/3+8XCwIIIIAAAggggEDMCMTFzEgYCAIIIIAAAggggEBAgIDHGwEBBBBAAAEEEIgxAQJejE0ow0EAAQQQQAABBAh4vAcQQAABBBBAAIEYEyDgxdiEMhwEEEAAAQQQQICAx3sAAQQQQAABBBCIMQECXoxNKMNBAAEEEEAAAQQIeLwHEEAAAQQQQACBGBMg4MXYhDIcBBBAAAGXCBwtU36/JKUkXqP8smpJR1VVdLdSEpOUklWkKpd001vd8Km6fJPW5uXoiYBpM72vLlfpmjxNmlemo800c+dLrX+vEPDcObP0CgEEEEAAAQQaChzdooKbM3Xv3DJ93fC1ej9Xq2zRJGVNmafXmm9Yb61Y+qFDLA2GsSCAAAIIIOBegQ5KyJiniox57u0iPYsZAfbgxcxUMhAEEEAAgZMncEjlrzyh8eeYQ7JJSst6QhvKv2jQnaYOu9VfN7h+norLquSr24I5NPmy8rP6Bw/xDs1RYVm53s67JvDz+Xm1hyGrijTeHALuN1vFL8/WteZx4mgtLf9KUkt1qlVWt71XtL1uPOfq2pwilVcfjdCH8D7Wdbb2wf+qOCtdKYnpGr/mLyorygvzWaC3q47UX6H2kGrIMCVxmHKXlKq8ulbBjC1lhJ4MsG7Xk8P7KKVfnsoaHX81dQfo5rnbA9s/OHeE/m/dYXLz1CGVl/72mGWiGd8SlZYfqt+fhj+Z/i3JqTU1rk2s56uqN9aUxP4an/fysXEEttvSXDQsHvrZ9H2JcoeeG3wfNDQKNZPEHrwwDB4igAACCCBw/AJHtK9oukZkF6mmduWaksc1vsTJlhqva9aqKZmnqa/vkUrmaFi3jvLt+53uGzZZ60MFti/XjNG7dMVFDUNkbc0vCjR1XO3j+GT1/E816mPjOrXtJR2cm6Xr636s0Y7Caco++oa6/26lNtTrw//Rd96YrkFdmttf9IU2TLlVG+q2Z8Y3R7f+vZPWrR2jVLNq9Qdamp2lmSXhZya+p+dmjNVzK6do1W/vUnrY+if+8JB2LLlfP5zxYt1cSWZ8Dyur8PeauOZZZaefHmHzB1W26AFlzS0Le612vXe+DPavsxnIQZXNvUs312tXpQ1zx+uNnXl68akMdYtzMueRPCOtFzQqfnWmfpt/m3oF+hDsYqQthHWehwgggAACCCDQrMCh1zX/QRPu4tUrs0CvfrRbFR9t1vNTBiu+2RUl+Xbr5cXrVaN0TVzzrioqd2vn63kaYlas2aHdgb1cX2nX+heC4a53lgre2KGKyh169enztHdjeCAKLxavXpNWaYvZ3pv36sLOHzuoE7Z+/NXKfWmrKirf1apJJlrVaMfzf5bu+4O2VO7SljVT1Ms0r3lLWypCiS9s/YYPe/9Ij71strdDm/Izgi5by/Th38zut69U/sIvguGuru5u7XyjQKN7x0vbFyh30TuqTsjQworVmniG2XhvTVzzvirenaz0RruqvqNhBZu0alLvQC9On7Raf638k7LTO8tXvkYPBMJdgq6YYcZi5uo1FWSmSSrTk9N+o7LQHsPwMRz9SJuWmXDXQyOXvBWYp4ptqzSxtn8zXygP7G31lRdpZiDcpWn0kte00/jXjqNmXZ7mb/rc4ZyHF699HHqfhRlVbPuDcgcnqKZkkQrfPlBvJQJePQ5+QMCpgNlN/nutzRun83NK1eyOfbO7/vcraw8HDFRu6edOi9AOAQQ8IOD7rFLbAhnnvzT+zoFKML9Z4xJ0/phRGtZSwovrpcziD1Tx0VMasLdUxUW/1vT/nla7p+6f2n/oK8n3sV5f+66kePW55SYNSOgoqaMSBt2he0f3aEKon26+ro86m6507qx4J3XCthQ//Bbd2KuLpNOVNvAyBfZpxQ/RbcN7q7Pi1DntUl0ZCFphKzX50PT7Nl2farbXUd0uHqiLw9uGj+++bN0eqGsIL9eU+29SvAmXy17VzkaHYsM34uTxV9r1WoneN0373qF7f3RuwEdx3TRo6mSNNHO1fZ027YwQWOPOUtLFCZL26Lkx12t83koVl+xVyuxXtLPyA60d00txOqq/fVAW3P7gsZowqJuCb4X/0ox1H6ii8s+aOegs6TjnIjSyoxXvap3pWs2LmnlV3+Ah2vOuq93ruUfFJdvq/S5qlHtDG+JfBBBoSuArlS+5X7949aDeLnlDGj2qqYbyVW3WUzlTtEg36+HRy7SlIDX4gdLkGryAAAJeE/B9+YU+MZ2OP0P/Hh+23yS+i878RmDnVzNDinTIMNT8NJ3ZpZPk+6e+2GN+s/fQeYlnBkJDsEUndTnztFDj+v8GDsuaIBhaHNQJNTUx7Mwujfc+fuN0dQkfX1j75h9+Q/9x+mnH+n1WD51zhrQhdHS5bny9NTCt+7F2ilN8l9NlRlHzxW598vlRpZ/VfKXmXz2qL78I7uU6fVCaksKmSnVzVaUP9xyU0k00Dlvieuj66fP05ZmzNKPwPW2Y+0DdIedss3dy3lQNS43Tvt0VYSs19fD45iK4laP6fM9uHWxqk8ao6mDgsLOJ0WYJH17tU/yDAALNC3RS6pj5+vXCGZrat5mv59Vvad7YB/R+2uNav3CybhxEuGvelVcR8KZA3DfP0HdN12t26ePPwi4cqDmk/f9qfkzHDhmmaeSMx5S/ZrN21h2GrF037jSd0cN81nyubZX7wy68+EqH9v8zcoEGYcxRnchbavtn68b3sf783t6w8flUc+igAqJnJOm7Z7V2n1QHffOMroHxHCx9T7uPXcEi1c1Vgs7pEekcPLNH8QKNnlWsisqtWrdknh7Lf0QTBydI25dp6j1rVO7rpG5JKUGvvx/Ul+HbD1M8sbmIU/zpZwRD9xl3a1XF7uBhYnOIOfS/ggyZfYyhhYAXkuBfBKIqYE7IfUS/TpysGRP7Bw/ZRHX7bAwBBNwiEJfcXxmBL3vvalXha6oyv9h9VXp7yXIVRzjad6zfPlXv3a1d5on47+n8IdfpuvSu+tur6/RyaO+WeS2upy65sV9gV+D7K1/QpsB5eUdUVfqMflm459jmmnzksE6T67fxC+HjezRfv9kRPOnFV7VRj895IXhu448u09mtzXfqpORLB6uPGc7WZ/TLZR/I3H5avn0qfSxPz5m56j1UA85u/MXdt69IPwlcIT1MM0qrlTzoeg3LuFE3X3dx2J7ODvrWuem121+r5Zv2BcOquYAkcPXzubpxyXYdcjLnjcjj1CX98uAh/y9WaUFhsO++qlc0I3BFbePTfwh4jRB5AoHWCwRPtD2qYRfVaNX44G0NgrdNaOpsvfqXzKdlhd1CIHTrgH7TVXpwn96eN05p5tYHQ2ertOqIfFVvqTBnWOB8jLSsQu2IdIJw64fEFhBAoCmBuCRdOXZI8FyxpVm67HtJSvlef936eEnYlZqRVo5T57P76TKTJ2qKNPWSXkpJ7KXLxqyXepsT3GrPwTPBZMhNwQsvthco6+Ladj/Zpu6Xh++ziVTDPOe0TlPrt/XznZR606TgBQth55edfXGWCrfXSL0naOa4H9SeLxfam9ncbVJMf8P21oXdJiUuNUO5gYtGqrRhxnX6vvks/d6lylr6nmQudJl9u9LDrkQNjTyu2yBljTMXm7ynwjGX6uzA7Wd6aUDgyukEDRl7hZLjpGPbD2sXOk+u922adHWyujia81DlsH+7pGv0fVcrXsf6HjKKHzxOo88P7p0MrUHAC0nwLwJRE6g9kbd3H/VKu0J3F2yWudJpqlZo/N1LI1yhdVBleXcoe9M5+tk2c0XXnzXz35bq1jtWqNz3uUofyQrcjX3Dv/ZpS2mVvjNxkd4zV29phe7OeUrFfz1dGbOKA1dqDd/7uB6ovZorasNhQwgg0IJAR3XLeFirn52iK2p3/sQPnqKF6wpqr/hsevW4btfoocLZGmmuxjRL71GasWa5Ft1xWeCE/tCJ83HdbtCjxQuDhwTNDj+z/eLHNCHN2ZUOTus03dM2fqXzBcr+bZEKHr+7zlAyh60Xa525RUoodMUl6aqcnx5rk9BB+irSsdBOSr76Lk0yh1ADy6mBPxVnLhpJn7xY65Y8WmdpLl7pNXq6Cl5e3MQtUswGQutNPzZX5uneo5S7ZJkezehRe87b6Uqf9JRW5YePI0FXTMrTqsX3alBCR534XHRRrzFztHpJeB/MthdqdYNbpASG7GdBAIETE/j6r/4lN5zj7zNto/8f9bbwd//GaQMaPf/1zsX+jJ7n+DMW/9X/dXj7f2z053z/If/Gf4Se/Zf/0zfn+8fdMM//zpfmucP+nYtH+ZN7h7cJ1ki+YbF/Z2i1JvsTXozHCCDgPYEv/e88PtSf3DPRn9w721+091/BIXz5pj/v6nP8yT37+cet/cR7w6LHbSrAHrxAzOX/EIiigG+/Pn6/8a1QgufpSLsqPg2e9xEo6dOhso0qrncidkclXDBBC4t/euxbaxS7x6YQQMBrAvE6e+DQ2vvOhQ7lJinlvJv1pDmEGT9Q1174La8Niv62sQABr42B2Xw7FIg7Uz37nKWa9yv1WaMjB99Qcsq3G98qpWabtlQ0dX5eOzRkyAggECYQp87pd+jZNXlhhxXNy7WHFosfDvy1i7AVeIgAf6qM9wAC0RfoqvTBgxS/Zqs+rDqi1G6196Kq/lQV5eco49KeYfcnilPn7klK1jI9OS1PSYF7KdXexaj6A5Xu/LYGpTe+oiv6fWaLCCDgboGOSkjPUHZBhrLd3VF65xIB9uC5ZCLoRiwJxKnLgP/WzIFvKPehVcGrWgO3TFipbeMmaURqp3qDrbvqytxL6crau5ObK7R+uEFdzo58P6Z6G+AHBBBAAAEEGgicYs7wa/Bcq388fPiwDhw4oO7du7d6W2wAAfcJ+HSo9GFdNmZZ2C0QLlXuywXKDA9v1eXasOhRZc81t0pI08iZObpr1AWR74ln/pzZ8ieVm7tcOwKHXe7VvZnDNSj5Uy29cbhmbg3dTOtS5b50j3TfqGPPxf9IBesH6JUhWcH7OBkw81yLfwDcfbL0CIGWBMrLy5WamtpSM15HoN0LRD3gbd68WfPy83XryJEalpHR7oEBQAABBBCInkBKYpLuuf9+ZY7J1KmnmltfsCCAQCSBqB2iNd+qcqZN0+0jb9Nbf3kzUi2eQwABBBBAoNUCv5ozR9cOHarioqJWb4sNIBCrAq0OeOZw7IL5CzT0yiF6fvmKWHViXAgggAACLhLYU/mxpmZP1shbbtHWrVtd1DO6goA7BFoV8My3J/MtynybYkEAAQQQQMC2gDliNGJYRuAI0t69e22Xpx4CrhU4oYBnvi2Zb03m25P5FsWCAAIIIIDAyRQwR5AGXnqZlhcWyhxZYkGgvQscV8Az347MeXbm2xLn2bX3tw7jRwABBNwnMD0nN3BkqaSkxH2do0cIWBRwdBWt+Ta0ZvVqmf9wWBBAAAEEEPCCwAUXXaifzZrFbVW8MFn0MeoCjgLeLx99VM88PT/qxdkgAggggAACbSnQI7Gnnlm0iJDXlshs25UCjgKe6bk57+6Rn//c8aHZx/LzuA+eK6ecTiGAAALeFTD3wXO6PDxrpoaPGMH98pyC0S6mBByfg9e3b189t3KlTHAz34hYEEAAAQQQcKPAraNu05tl72jU6NGEOzdOEH2yIuA44IV6Y/46xR/XrQvcSTz0HP8igAACCCBwsgXMOXeri4s0a/Zsde3a9WR3h/oInFSB4w54prfmz8NMuHOC1r28XuabEgsCCCCAAAInS8AcVTJHl8xRJnO0iQUBBKQTCnghOPMHn803pd88t0LmmxMLAggggAACNgXM36U1R5X42+c21anlBYEO0ehk//799f3vf1/rX3pJc/PzuflxNFDZBgIIIIBAkwLm6FHmmDFcHdukEC+0d4GoBDyDaA7bmm9Q/2/AAD1bUNDeXRk/AggggEAbCJijRXdnZ8vsWGBBAIGmBRzfJqXpTfAKAggggAACCCCAgJsEWnUOnpsGQl8QQAABBBBAAAEEggIEPN4JCCCAAAIIIIBAjAkQ8GJsQhkOAggggAACCCBAwOM9gAACCCCAAAIIxJgAAS/GJpThIIAAAggggAACBDzeAwgggAACCCCAQIwJEPBibEIZDgIIIIAAAgggQMDjPYAAAggggAACCMSYAAEvxiaU4SCAAAIIIIAAAo7+VFlKYhJSCCCAAAIIeFagonJ31Pt++PBhbdmyhT+bFnVZNhgNAUcBzxRauHhpNOp5dhvjx2aqLT4gPAtCxxFAwBMC5gs6n9+ZbTJX6196SVOzJ+vNsnfUtWvXNqnBRhE4UQEO0Z6oHOshgAACCLRbgQMHDmhufn5g/M8WFLRbBwbuXgECnnvnhp4hgAACCLhUYNXKVdpT+XGgd888PV/l5eUu7Sndaq8CBLz2OvOMGwEEEEDghARMmPvVnDn11s3Py6v3Mz8gcLIFCHgnewaojwACCCDgKYFIYe6lP61TSUmJp8ZBZ2NbgIDncH7b+0nKDplohgACLhPgsyu6E7J582aZMBdp+fmsWTJX1rIg4AYBAp4bZoE+IIAAAgi4XsCEt2n/8z9N9tOck7dm9eomX+cFBGwKEPBsalMLAQQQQMCzAua2KKELK5oaxPScXJkrbFkQONkCBLyTPQPURwABBBBwvYAJbeaed06Wxx97zEkz2iDQpgIEvDblZeMIIIAAArEgcDz3unt++QpumxILk+7xMRDwPD6BdB8BBBBAoG0FzG1RzL3ujmd5KCfneJrTFoGoCxDwok7KBhFAAAEEYkngRMLaW395U8VFRbHEwFg8JuD4b9F6bFxR7y5/izbqpGwQAQQsCJjPLm6VcuLQZu9dckpK4H8Nt2IOxZrl1lG3NXwp8PPOnTsjPs+TCNgQIODZUKYGAggggIAnBVJTUzVr9uyIfQ8FvKZej7gSTyJgSYBDtJagKYMAAggggAACCNgSIODZkqYOAggggAACCCBgSYCAZwmaMggggAACCCCAgC0BAp4taeoggAACCCCAAAKWBAh4lqApgwACCCCAAAII2BIg4NmSpg4CCCCAAAIIIGBJgIDnEJr7SDmEohkCCLhKgM8uV00HnUHAmgABzxo1hRBAAAEEEEAAATsCBDw7zlRBAAEEEEAAAQSsCRDwrFFTCAEEEEAAAQQQsCNAwLPjTBUEEEAAAQQQQMCaAAHPGjWFEEAAAQQQQAABOwIEPDvOVEEAAQQQQAABBKwJEPAcUo8fm+mwJc0QQAAB9wjw2eWeuaAnCNgUIODZ1KYWAggggAACCCBgQYCAZwGZEggggAACCCCAgE0BAp5NbWohgAACCCCAAAIWBAh4FpApgQACCCCAAAII2BQg4NnUphYCCCCAAAIIIGBBgIBnAZkSCCCAAAIIIICATQECnk1taiGAAAIIIIAAAhYECHgWkCmBAAIIIIAAAgjYFCDgOdReuHipw5Y0QwABBNwjwGeXe+aCniBgU4CAZ1ObWggggAACCCCAgAUBAp4FZEoggAACCCCAAAI2BQh4NrWphQACCCCAAAIIWBAg4FlApgQCCCCAAAIIIGBTgIBnU5taCCCAAAIIIICABQECngVkSiCAAAIIIIAAAjYFCHgOtcePzXTYkmYIIICAewT47HLPXNATBGwKEPBsalMLAQQQQAABBBCwIEDAs4BMCQQQQAABBBBAwKYAAc+mNrUQQAABBBBAAAELAgQ8C8iUQAABBBBAAAEEbAoQ8GxqUwsBBBBAAAEEELAgQMCzgEwJBBBAAAEEEEDApgABz6Y2tRBAAAEEEEAAAQsCBDwLyJRAAAEEEEAAAQRsChDwHGovXLzUYUuaIYAAAu4R4LPLPXNBTxCwKUDAs6lNLQQQQAABBBBAwIIAAc8CMiUQQAABBBBAAAGbAgQ8m9rUQgABBBBAAAEELAgQ8CwgUwIBBBBAAAEEELApQMCzqU0tBBBAAAEEEEDAggABzwIyJRBAAAEEEEAAAZsCBDyH2uPHZjpsSTMEEEDAPQJ8drlnLugJAjYFCHg2tamFAAIIIIAAAghYECDgWUCmBAIIIIAAAgggYFOAgGdTm1oIIIAAAggggIAFAQKeBWRKIIAAAggggAACNgU6OC3W8ETd5v6+YcO2kWp4ff1IY+I5BBBAwI0CDT+Tvf7529r+u3GO6BMC0RY4xe/3+1vaaEpikioqd7fUjNcRQAABBBBwnUBb/Q4z2zULvx9dN+V0SBKHaHkbIIAAAggggAACMSZAwIuxCWU4CCCAAAIIIIAAAY/3AAIIIIAAAgggEGMCBLwYm1CGgwACCCCAAAIIEPB4DyCAAAIIIIAAAjEmQMCLsQllOAgggAACCCCAAAGP9wACCCCAAAIIIBBjAgS8GJtQhoMAAggggAACCBDweA8ggAACCCCAAAIxJkDAi7EJZTgIIIAAAggggAABj/cAAggggAACCCAQYwJtEvD27t2r8vJyV1IdOHBAmzdvdmXf6BQCCCCAAAIIIBANgagGvMOHD2t5YaEGXnqZPvzgg2j0L+rb2L9/v24feZtypk1zbQiN+qDZIAIIIIAAAgi0K4GoBbySkhJdO3SopufkegLw+eUrNPTKIVowf4HMXj0WBBBAAAEEEEAgVgRaHfDModiRt9yiCVnjtKfyY8+5/GrOHN00fLiKi4o813c6jAACCCCAAAIIRBI44YBn9nqZw5xmL9hbf3kz0rY985wJplOzJweC6tatWz3TbzqKAAIIIIAAAghEEjjugGfOszN7uy5M/4HMYc5YWkxQHTEsIxBczYUiLAgggAACCCCAgBcFjivgmatPzXl2Zm9XLC8muJoLRcz5eSbQsiCAAAIIIIAAAl4SOMXv9/tb6nBKYpKuumaoXvrTupaaxtzrPRJ76sGcHA0ePDjmxsaAEEAAgfYgYH6HVVTujvpQzXbN0hbbjnpn2WC7E3C8B2/7hx+2OxwzYHN+XunGjezJa5ezz6ARQAABBBDwpkAHp91+Yc0aPVtQoGeenu9olcfy8zQsI8NRW5uNzFW/5sIQJ8sFF12on82apdTUVCfNaYMAAggggAACCLhCwPEevK5du+re++7TupfXBw7XuqL3bdQJc1h2QcEiPbdyJeGujYzZLAIIIIAAAgi0nYDjgBfqgtmb9dTTT+s3z62QCUKxttxz//3647p1nHMXaxPLeBBAAAEEEGhHAscd8EI2/fv3DwQhcyg2FpZbR92mP7/2qibcOUGnnnpqLAyJMSCAAAIIIIBAOxU44YBnvEwQMufZvVn2jkxA8uJizrNbXVykWbNnq3v37l4cAn1GAAEEEEAAAQTqCbQq4IW2ZM7PMwHJnJ9nApMXFnN42ex9NOfZ9e3b1wtdpo8IIIAAAggggIAjgagEvFAlc36eCUzmAoXTOncOPX0C//6virPSZe4xlHLOZBXvOyL5qvT2vHFKS0zS+XllOnoCWw2tYs6zM1cFu/Eq31Af+RcBBBBAAAEEEDhRAcc3Oj4ZN3L07SvSxMGT9erwR5X/rZ365KqfKrNXlxMdK+shgAACCLRDAW503A4nnSErqnvwou0Z1+0y3Ty8h2oKH9cLPW7X7YS7aBOzPQQQQAABBBCIQQFXBzypq9IHD1K8knXRud9ydxqNwTcHQ0IAAQQQQAABbwq4POCFUN9V0Wsfyxf6kX8RQAABBBBAAAEEmhRwdcDzVW3U3JKvdW1vaVfFp6puchi8gAACCCCAAAIIIBAScG/A8+3R72dvVNpP7tOPb+mnmjUbVbbvPRXOe1H72JUXmj/+RQABBBBAAAEEGgm4L+D5dmjpsHOVcu2z0t33aVi3zvp22kXqVfOCfvn0R7rox0PUzX29bgTLEwgggAACCCCAwMkScPVtUk4WCnURQAABBGJHgNukxM5cMhLnAuwLc25FSwQQQAABBBBAwBMCBDxPTBOdRAABBBBAAAEEnAsQ8Jxb0RIBBBBAAAEEEPCEAAHPE9NEJxFAAAEEEEAAAecCBDznVrREAAEEEEAAAQQ8IUDA88Q00UkEEEAAAQQQQMC5AAHPuRUtEUAAAQQQQAABTwgQ8DwxTXQSAQQQQAABBBBwLkDAc25FSwQQQAABBBBAwBMCBDxPTBOdRAABBBBAAAEEnAsQ8Jxb0RIBBBBAAAEEEPCEAAHPE9NEJxFAAAEEEEAAAecCBDznVrREAAEEEEAAAQQ8IUDA88Q00UkEEEAAAQQQQMC5AAHPuRUtEUAAAQQQQAABTwgQ8DwxTXQSAQQQQAABBBBwLkDAc25FSwQQQAABBBBAwBMCBDxPTBOdRAABBBBAAAEEnAsQ8Jxb0RIBBBBAAAEEEPCEAAHPE9NEJxFAAAEEEEAAAecCBDznVrREAAEEEEAAAQQ8IUDA88Q00UkEEEAAAQQQQMC5AAHPuRUtEUAAAQQQQAABTwgQ8DwxTXQSAQQQQAABBBBwLkDAc25FSwQQQAABBBBAwBMCBDxPTBOdRAABBBBAAAEEnAsQ8Jxb0RIBBBBAAAEEEPCEAAHPE9NEJxFAAAEEPC1QXa7SopXKzxqm3NLPIw7FV/WWCnOGKSUxSWlZC/R21ZGI7XgSAScCBDwnSrRBAAEEEEDgRAV8O7Q082GtKM7XkyUHI2+l+i3NG/uIKgbP187KHVo/er+mj31GZdW+yO15FoEWBAh4LQDxMgIIIIAAAq0SiOulzNWFWvDgHeoTcUNfqfyFufr1D36qKYO6KU4dlTDoDt37g99p5gvlIuJFROPJFgQIeC0A8TICCCCAAAJtKuD7WK+v/VDJKd9W57pCXZU++BLtWrtZu0h4dSo8cC5AwJXOV2YAAA1WSURBVHNuRUsEEEAAAQSiLuDbtVlFW7+p8xLPVKNfyltL9PqurxrXrC7XhrxxSktMUkpif42ft1lVgSDoU3X5Jq3NG6fzc0r1j6rNeiKrv1ISz9W1019Rle+IqspWKHfoucH1lnyg6sZb55kYEGj0XoqBMTEEBBBAAAEEPCLgU/Xe3dql7yql+7H9d812vvot5f/wAW3q97C2VO7WztfvV4cFWbpz2Q75Dm3SnGGZunduiY5UvavSfUm6q+A1bVkzQVqao4fmFuuv37xOM9e9q1eXXKe9M36h1eURAmSzHeBFLwgQ8LwwS/QRAQQQQACBgIBPh97+gwrrzteT4hIu1o8nDJC+OKSaLoM0c9uLyu0bLyX00+XpCYpTnDqn9FF6/CH9/Yw0DUjtIqmjvtW9uzrqE1XsZR9eLL65OsTioBgTAggggAAC3hCIU+fuSUpWaTBopXZqodsHVFZSqiO6/Fi7uASdf/d8rT32DI8QaHy4HxMEEEAAAQQQsCcQl9xfGX07Nih4RJ9V7lJN38G6JLlx6Kt5531VcAuVBmb8GC7AIdpwDR4jgAACCCBgWyCupy658T9VXLJNh+pqV2tvxWfqc2N/Jdf7Td1Z3VO+K21foNxHfqfyupDnU/WW11R2iEtu6wjb+YN6b5t2bsHwEUAAAQQQOAkCnZR60yT9+J0n9HjpPvl0RFWlz+iX79yg3JtSGxxqC7ad2FvaUThZQ89LDvzli5TEPrplY0ed3YVf6ydhAl1Z8hS/3+9vqWfmz6ZUVO5uqRmvI4AAAggg4DqBtvodZrZrlpZ/P36u0pwRyircc8ym73StWztGqWF5zFe1WU/lTNHckkPqNXqaZk68SekJDQ/dBjdh/qzZiidnaUbhe5LSNHLGZP3opgFK/nSZRlz5sN4PVeo7XX98VHrgqmPPxY8u0J8Gb9I1Y5apprZd/OjFenXWIJnLL1hiQ4CAFxvzyCgQQAABBJoQOPkBr4mO8TQCbSgQ9t2hDauwaQQQQAABBBBAAAFrAgQ8a9QUQgABBBBAAAEE7AgQ8Ow4UwUBBBBAAAEEELAmQMCzRk0hBBBAAAEEEEDAjgABz44zVRBAAAEEEEAAAWsCBDxr1BRCAAEEEEAAAQTsCBDw7DhTBQEEEEAAAQQQsCZAwLNGTSEEEEAAAQQQQMCOAAHPjjNVEEAAAQQQQAABawIEPGvUFEIAAQQQQAABBOwIEPDsOFMFAQQQQAABBBCwJkDAs0ZNIQQQQAABBBBAwI4AAc+OM1UQQAABBBBAAAFrAgQ8a9QUQgABBBBAAAEE7AgQ8Ow4UwUBBBBAAAEEELAmQMCzRk0hBBBAAAEEEEDAjgABz44zVRBAAAEEEEAAAWsCBDxr1BRCAAEEEEAAAQTsCBDw7DhTBQEEEEAAAQQQsCZAwLNGTSEEEEAAAQQQQMCOAAHPjjNVEEAAAQQQQAABawIEPGvUFEIAAQQQQAABBOwIEPDsOFMFAQQQQAABBBCwJkDAs0ZNIQQQQAABBBBAwI4AAc+OM1UQQAABBBBAAAFrAgQ8a9QUQgABBBBAAAEE7AgQ8Ow4UwUBBBBAAAEEELAmQMCzRk0hBBBAAAEEEEDAjgABz44zVRBAAAEEEEAAAWsCBDxr1BRCAAEEEEAAAQTsCBDw7DhTBQEEEEAAAQQQsCZAwLNGTSEEEEAAAQQQQMCOAAHPjjNVEEAAAQQQQAABawIEPGvUFEIAAQQQQAABBOwIEPDsOFMFAQQQQAABBBCwJkDAs0ZNIQQQQAABBBBAwI4AAc+OM1UQQAABBBBAAAFrAgQ8a9QUQgABBBBAAAEE7AgQ8Ow4UwUBBBBAAAEEELAmQMCzRk0hBBBAAAEEEEDAjgABz44zVRBAAAEEEEAAAWsCBDxr1BRCAAEEEEAAAQTsCBDw7DhTBQEEEEAAAQQQsCZAwLNGTSEEEEAAAQQQQMCOAAHPjjNVEEAAAQQQQAABawIEPGvUFEIAAQQQQAABBOwIEPDsOFMFAQQQQAABBBCwJkDAs0ZNIQQQQAABBBBAwI4AAc+OM1UQQAABBBBAAAFrAgQ8a9QUQgABBBBAAAEE7AgQ8Ow4UwUBBBBAAAEEELAmQMCzRk0hBBBAAAEEEEDAjgABz44zVRBAAAEEEEAAAWsCBDxr1BRCAAEEEEAAAQTsCBDw7DhTBQEEEEAAAQQQsCZAwLNGTSEEEEAAAQQQQMCOAAHPjjNVEEAAAQQQQAABawIEPGvUFEIAAQQQQAABBOwIEPDsOFMFAQQQQAABBBCwJkDAs0ZNIQQQQAABBBBAwI4AAc+OM1UQQAABBBBAAAFrAgQ8a9QUQgABBBBAAAEE7AgQ8Ow4UwUBBBBAAAEEELAmQMCzRk0hBBBAAAEEEEDAjkAHp2VSEpPqNV24eGm9n8N/GD82M/zHiI+9uH5F5e6IY+FJBBBAwK0CDT+7TT+9+Pkb7tva/odvi8cIxKrAKX6/39/S4MwHRHP/QbW0fiy8bkIrAS8WZpIxINC+BPj8ltrq8zsUnvnd0L7+m/LKaDlE65WZop8IIIAAAggggIBDAQKeQyiaIYAAAggggAACXhEg4HllpugnAggggAACCCDgUICA5xCqvZ+D6JCJZggg4DIBPrtcNiF0BwFLAgQ8S9CUQQABBBBAAAEEbAkQ8GxJUwcBBBBAAAEEELAkQMCzBE0ZBBBAAAEEEEDAlgABz5Y0dRBAAAEEEEAAAUsCBDxL0JRBAAEEEEAAAQRsCRDwbElTBwEEEEAAAQQQsCRAwHMI7eTv6zrcFM0QQAABawJ8dlmjphACrhIg4LlqOugMAggggAACCCDQegECXusN2QICCCCAAAIIIOAqAQKeq6aDziCAAAIIIIAAAq0XIOC13pAtIIAAAggggAACrhIg4LlqOugMAggggAACCCDQegECXusN2QICCCCAAAIIIOAqAQKeq6aDziCAAAIIIIAAAq0XIOC13pAtIIAAAggggAACrhIg4DmcjoWLlzpsSTMEEEDAPQJ8drlnLugJAjYFCHg2tamFAAIIIIAAAghYECDgWUCmBAIIIIAAAgggYFOAgGdTm1oIIIAAAggggIAFAQKeBWRKIIAAAggggAACNgUIeDa1qYUAAggggAACCFgQIOBZQKYEAggggAACCCBgU4CA51B7/NhMhy1phgACCLhHgM8u98wFPUHApgABz6Y2tRBAAAEEEEAAAQsCBDwLyJRAAAEEEEAAAQRsChDwbGpTCwEEEEAAAQQQsCBAwLOATAkEEEAAAQQQQMCmAAHPpja1EEAAAQQQQAABCwIEPAvIlEAAAQQQQAABBGwKEPBsalMLAQQQQAABBBCwIEDAs4BMCQQQQAABBBBAwKYAAc+h9sLFSx22pBkCCCDgHgE+u9wzF/QEAZsCBDyb2tRCAAEEEEAAAQQsCBDwLCBTAgEEEEAAAQQQsClAwLOpTS0EEEAAAQQQQMCCAAHPAjIlEEAAAQQQQAABmwIEPJva1EIAAQQQQAABBCwIEPAsIFMCAQQQQAABBBCwKUDAc6g9fmymw5Y0QwABBNwjwGeXe+aCniBgU4CAZ1ObWggggAACCCCAgAUBAp4FZEoggAACCCCAAAI2BQh4NrWphQACCCCAAAIIWBAg4FlApgQCCCCAAAIIIGBTgIBnU5taCCCAAAIIIICABQECngVkSiCAAAIIIIAAAjYFOtgsRi0EEEAAAQS8JHDgwAHt37+/2S6Xl5c3+XpqamqTr/ECAm0pQMBrS122jQACCCDgaYHDhw9r6JVDmh1DU6/fOuo2zZo9u9l1eRGBthLgEK1D2YWLlzpsSTMEEEDAPQJ8drVuLrp376577r//hDaSOWbMCa3HSghEQ4CAFw1FtoEAAgggELMCmWMy1SOx53GNz4RCDs8eFxmNoyxAwIsyKJtDAAEEEIgtgVNPPVWTsrMdD8qEwZtvudlxexoi0BYCBLy2UGWbCCCAAAIxJTAsI0MXXHShozGZMNi1a1dHbWmEQFsJEPDaSpbtIoAAAgjElMDPZs1qcTwmBJowyILAyRYg4J3sGaA+AggggIAnBMw5debK2OaWBx58sLmXeQ0BawIEPGvUFEIAAQQQ8LrAlKlTmxyCCX99+/Zt8nVeQMCmAAHPofb4sZkOW9IMAQQQcI8An13RnQtzbt3Ds2ZG3OidP/lJxOd5EoGTIUDAOxnq1EQAAQQQ8KzA8BEjGt02xdwWxdwzjwUBtwgQ8NwyE/QDAQQQQMATAua2KbN/8Yu6vprboph75bEg4CYBAp6bZoO+IIAAAgh4QqB///666pqhgb4+mJMjE/pYEHCTAH+L1k2zQV8QQAABBDwjkD15sg7s36/Bgwd7ps90tP0InOL3+/0tDTclMamlJryOAAIIIICAawUqKne3Sd8OHDjATY3bRJaNtlbAUcBrbRHWRwABBBBAAAEEELAnwDl49qyphAACCCCAAAIIWBEg4FlhpggCCCCAAAIIIGBPgIBnz5pKCCCAAAIIIICAFQECnhVmiiCAAAIIIIAAAvYECHj2rKmEAAIIIIAAAghYEfj/pDBujbqaqAoAAAAASUVORK5CYII=" alt></p>
<p>The remainder of the cardboard is folded up to form the tray as shown in the following diagram.</p>
<p><img src="data:image/png;base64,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" alt></p>
<p>Write down, <strong>in terms of</strong> \(x\) , the length and the width of the tray.</p>
<div class="marks">[2]</div>
<div class="question_part_label">a.</div>
</div>
<div class="question" style="padding-left: 20px; padding-right: 20px;">
<p>(i)     State whether \(x\) can have a value of \(5\). Give a reason for your answer.</p>
<p>(ii)    Write down the interval for the possible values of \(x\) .</p>
<div class="marks">[4]</div>
<div class="question_part_label">b.</div>
</div>
<div class="question" style="padding-left: 20px; padding-right: 20px;">
<p>Show that the volume, \(V\,{\text{c}}{{\text{m}}^3}\), of this tray is given by</p>
<p>\[V = 4{x^3} - 52{x^2} + 160x.\]</p>
<div class="marks">[2]</div>
<div class="question_part_label">c.</div>
</div>
<div class="question" style="padding-left: 20px; padding-right: 20px;">
<p>Find \(\frac{{dV}}{{dx}}.\)</p>
<div class="marks">[3]</div>
<div class="question_part_label">d.</div>
</div>
<div class="question" style="padding-left: 20px; padding-right: 20px;">
<p><strong>Using your answer from part (d)</strong>, find the value of \(x\) that maximizes the volume of the tray.</p>
<div class="marks">[2]</div>
<div class="question_part_label">e.</div>
</div>
<div class="question" style="padding-left: 20px; padding-right: 20px;">
<p>Calculate the maximum volume of the tray.</p>
<div class="marks">[2]</div>
<div class="question_part_label">f.</div>
</div>
<div class="question" style="padding-left: 20px; padding-right: 20px;">
<p>Sketch the graph of \(V = 4{x^3} - 51{x^2} + 160x\) , for the possible values of \(x\) found in part (b)(ii), and \(0 \leqslant V \leqslant 200\) . Clearly label the maximum point.</p>
<div class="marks">[4]</div>
<div class="question_part_label">g.</div>
</div>
<h2 style="margin-top: 1em">Markscheme</h2>
<div class="question" style="padding-left: 20px;">
<p>\(16 - 2x,\,\,10 - 2x\)          <em><strong>(A1)(A1)</strong></em></p>
<div class="question_part_label">a.</div>
</div>
<div class="question" style="padding-left: 20px;">
<p>(i)     no          <em><strong>(A1)</strong></em></p>
<p>(when \(x\) is \(5\)) the width of the tray will be zero / there is no short edge to fold / \(10 - 2\,(5) = 0\)          <em><strong>(R1)</strong></em></p>
<p><strong>Note:</strong> Do not award <em><strong>(R0)(A1)</strong></em>. Award the <em><strong>(R1)</strong></em> for reasonable explanation.</p>
<p> </p>
<p>(ii)    \(0 &lt; x &lt; 5\)          <em><strong>(A1)</strong></em><em><strong>(A1)</strong></em></p>
<p><strong>Note:</strong> Award <em><strong>(A1)</strong></em> for \(0\) and \(5\) seen, <em><strong>(A1)</strong></em> for correct strict inequalities (accept alternative notation). Award <em><strong>(A1)(A0)</strong></em> for “between \(0\) and \(5\)” or “from \(0\) to \(5\)”.<br>Do not accept a list of integers.</p>
<div class="question_part_label">b.</div>
</div>
<div class="question" style="padding-left: 20px;">
<p>\(V = (16 - 2x)\,(10 - 2x)\,(x)\)          <em><strong>(M1)</strong></em></p>
<p><strong>Note:</strong> Award <em><strong>(M1)</strong></em> for their correct substitution in volume of cuboid formula.</p>
<p>\( = 160x - 32{x^2} - 20{x^2} + 4{x^3}\) (or equivalent)          <em><strong>(M1)</strong></em></p>
<p>\( = 160x - 52{x^2} + 4{x^3}\)          <em><strong>(AG)</strong></em></p>
<p><strong>Note:</strong> Award <em><strong>(M1)</strong></em> for showing clearly the expansion and for simplifying the expression, and this <strong>must</strong> be seen to award second <em><strong>(M1)</strong></em>. The <em><strong>(AG)</strong></em> line must be seen for the final <em><strong>(M1)</strong></em> to be awarded.</p>
<div class="question_part_label">c.</div>
</div>
<div class="question" style="padding-left: 20px;">
<p>\(12{x^2} - 104x + 160\)  (or equivalent)          <em><strong>(A1)(A1)(A1)</strong></em></p>
<p><strong>Note:</strong> Award <em><strong>(A1)</strong></em> for \(12{x^2}\),<em><strong> (A1)</strong></em> for \( - 104x\) and <em><strong>(A1)</strong></em> for \( + 160\). If extra terms are seen award at most <em><strong>(A1)(A1)(A0)</strong></em>.</p>
<div class="question_part_label">d.</div>
</div>
<div class="question" style="padding-left: 20px;">
<p>\(12{x^2} - 104x + 160 = 0\)          <em><strong>(M1)</strong></em></p>
<p><strong>Note:</strong> Award <em><strong>(M1)</strong></em> for equating <strong>their</strong> derivative to \(0\).</p>
<p>\(x = 2\)          <strong><em>(A1)</em>(ft)</strong></p>
<p><strong>Note:</strong> Award <em><strong>(M1)</strong></em> for a sketch of their derivative in part (d), <strong><em>(A1)</em>(ft)</strong> for reading the \(x\)-intercept from their graph.<br>Award <em><strong>(M0)(A0)</strong></em> for \(x = 2\) with no working seen.<br>Award at most <em><strong>(M1)(A0)</strong></em> if the answer is a pair of coordinates.<br>Award at most <em><strong>(M1)(A0)</strong></em> if the answer given is \(x = 2\) <strong>and </strong>\(x = \frac{{20}}{3}\)<br>Follow through from their derivative in part (d). Award <strong><em>(A1)</em>(ft)</strong> only if answer is positive and less than \(5\).</p>
<div class="question_part_label">e.</div>
</div>
<div class="question" style="padding-left: 20px;">
<p>\(4{(2)^3} - 52{(2)^2} + 160(2)\)          <em><strong>(M1)</strong></em></p>
<p><strong>Note:</strong> Award <em><strong>(M1)</strong></em> for correct substitution of their answer to part (e) into volume formula.</p>
<p>\(144\,({\text{c}}{{\text{m}}^3})\)          <em><strong>(A1)</strong></em><strong>(ft)<em>(G2)</em></strong></p>
<p><strong>Note:</strong> Follow through from part (d).</p>
<div class="question_part_label">f.</div>
</div>
<div class="question" style="padding-left: 20px;">
<p><img src="data:image/png;base64,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" alt></p>
<p><strong><em>(A1)(A1)</em>(ft)<em>(A1)(A1)</em>(ft)</strong></p>
<p><strong>Note:</strong> Award <strong><em>(A1)</em></strong> for correctly labelled axes and window for \(V\), <em>ie</em> \(0 \leqslant V \leqslant 200\).<br>Award <strong><em>(A1)</em>(ft)</strong> for the correct domain \((0 &lt; x &lt; 5)\). Follow through from part (b)(ii) if a different domain is shown on graph.<br>Award <strong><em>(A1)</em></strong> for smooth curve with correct shape.<br>Award <strong><em>(A1)</em>(ft)</strong> for <strong>their</strong> maximum point indicated (coordinates, cross or dot <em>etc</em>.) in approximately correct place.<br>Follow through from parts (e) and (f) only if the maximum on their graph is different from \((2,\,\,144)\).</p>
<div class="question_part_label">g.</div>
</div>
<h2 style="margin-top: 1em">Examiners report</h2>
<div class="question" style="padding-left: 20px;">
<p>Question 5: Differential calculus<br>In general, many candidates struggled in some parts. Most candidates who could state the dimensions of the box gave a reasonable justification of why \(x\) could not be \(5\). Very few candidates scored the two marks in part (d)(ii). Either their inequalities were not strict or their limits were incorrect or both. Some candidates stated the range of \(x\)&nbsp;as \(1,\,\,2,\,\,3,\,\,4\). The algebra in part (c) caused problems for a number of candidates. It seemed that there was a lack of understanding of what the question required. Some substituted \(x = 2\) in the volume formula. A few candidates wrote the product of the length, width, height, omitting the appropriate brackets. Part (d) was well answered by most candidates. However, its application in the following part was not as good in part (e). In part (e) some candidates left both solutions for \(x\), not appreciating the fact that one was outside the range. Others lost both marks as they did not show that they had used their derivative to part (d) as required by the question. Very few candidates scored full marks for the sketch in part (g). Not following the given instructions about the domain and range let most candidates down in this question.</p>
<div class="question_part_label">a.</div>
</div>
<div class="question" style="padding-left: 20px;">
<p>Question 5: Differential calculus<br>In general, many candidates struggled in some parts. Most candidates who could state the dimensions of the box gave a reasonable justification of why \(x\) could not be \(5\). Very few candidates scored the two marks in part (d)(ii). Either their inequalities were not strict or their limits were incorrect or both. Some candidates stated the range of \(x\)&nbsp;as \(1,\,\,2,\,\,3,\,\,4\). The algebra in part (c) caused problems for a number of candidates. It seemed that there was a lack of understanding of what the question required. Some substituted \(x = 2\) in the volume formula. A few candidates wrote the product of the length, width, height, omitting the appropriate brackets. Part (d) was well answered by most candidates. However, its application in the following part was not as good in part (e). In part (e) some candidates left both solutions for \(x\), not appreciating the fact that one was outside the range. Others lost both marks as they did not show that they had used their derivative to part (d) as required by the question. Very few candidates scored full marks for the sketch in part (g). Not following the given instructions about the domain and range let most candidates down in this question.</p>
<div class="question_part_label">b.</div>
</div>
<div class="question" style="padding-left: 20px;">
<p>Question 5: Differential calculus</p>
<p>In general, many candidates struggled in some parts. Most candidates who could state the dimensions of the box gave a reasonable justification of why \(x\) could not be \(5\). Very few candidates scored the two marks in part (d)(ii). Either their inequalities were not strict or their limits were incorrect or both. Some candidates stated the range of \(x\)&nbsp;as \(1,\,\,2,\,\,3,\,\,4\). The algebra in part (c) caused problems for a number of candidates. It seemed that there was a lack of understanding of what the question required. Some substituted \(x = 2\) in the volume formula. A few candidates wrote the product of the length, width, height, omitting the appropriate brackets. Part (d) was well answered by most candidates. However, its application in the following part was not as good in part (e). In part (e) some candidates left both solutions for \(x\), not appreciating the fact that one was outside the range. Others lost both marks as they did not show that they had used their derivative to part (d) as required by the question. Very few candidates scored full marks for the sketch in part (g). Not following the given instructions about the domain and range let most candidates down in this question.</p>
<div class="question_part_label">c.</div>
</div>
<div class="question" style="padding-left: 20px;">
<p>Question 5: Differential calculus</p>
<p>In general, many candidates struggled in some parts. Most candidates who could state the dimensions of the box gave a reasonable justification of why \(x\) could not be \(5\). Very few candidates scored the two marks in part (d)(ii). Either their inequalities were not strict or their limits were incorrect or both. Some candidates stated the range of \(x\)&nbsp;as \(1,\,\,2,\,\,3,\,\,4\). The algebra in part (c) caused problems for a number of candidates. It seemed that there was a lack of understanding of what the question required. Some substituted \(x = 2\) in the volume formula. A few candidates wrote the product of the length, width, height, omitting the appropriate brackets. Part (d) was well answered by most candidates. However, its application in the following part was not as good in part (e). In part (e) some candidates left both solutions for \(x\), not appreciating the fact that one was outside the range. Others lost both marks as they did not show that they had used their derivative to part (d) as required by the question. Very few candidates scored full marks for the sketch in part (g). Not following the given instructions about the domain and range let most candidates down in this question.</p>
<div class="question_part_label">d.</div>
</div>
<div class="question" style="padding-left: 20px;">
<p>Question 5: Differential calculus</p>
<p>In general, many candidates struggled in some parts. Most candidates who could state the dimensions of the box gave a reasonable justification of why \(x\) could not be \(5\). Very few candidates scored the two marks in part (d)(ii). Either their inequalities were not strict or their limits were incorrect or both. Some candidates stated the range of \(x\)&nbsp;as \(1,\,\,2,\,\,3,\,\,4\). The algebra in part (c) caused problems for a number of candidates. It seemed that there was a lack of understanding of what the question required. Some substituted \(x = 2\) in the volume formula. A few candidates wrote the product of the length, width, height, omitting the appropriate brackets. Part (d) was well answered by most candidates. However, its application in the following part was not as good in part (e). In part (e) some candidates left both solutions for \(x\), not appreciating the fact that one was outside the range. Others lost both marks as they did not show that they had used their derivative to part (d) as required by the question. Very few candidates scored full marks for the sketch in part (g). Not following the given instructions about the domain and range let most candidates down in this question.</p>
<div class="question_part_label">e.</div>
</div>
<div class="question" style="padding-left: 20px;">
<p>Question 5: Differential calculus</p>
<p>In general, many candidates struggled in some parts. Most candidates who could state the dimensions of the box gave a reasonable justification of why \(x\) could not be \(5\). Very few candidates scored the two marks in part (d)(ii). Either their inequalities were not strict or their limits were incorrect or both. Some candidates stated the range of \(x\)&nbsp;as \(1,\,\,2,\,\,3,\,\,4\). The algebra in part (c) caused problems for a number of candidates. It seemed that there was a lack of understanding of what the question required. Some substituted \(x = 2\) in the volume formula. A few candidates wrote the product of the length, width, height, omitting the appropriate brackets. Part (d) was well answered by most candidates. However, its application in the following part was not as good in part (e). In part (e) some candidates left both solutions for \(x\), not appreciating the fact that one was outside the range. Others lost both marks as they did not show that they had used their derivative to part (d) as required by the question. Very few candidates scored full marks for the sketch in part (g). Not following the given instructions about the domain and range let most candidates down in this question.</p>
<div class="question_part_label">f.</div>
</div>
<div class="question" style="padding-left: 20px;">
<p>Question 5: Differential calculus</p>
<p>In general, many candidates struggled in some parts. Most candidates who could state the dimensions of the box gave a reasonable justification of why \(x\) could not be \(5\). Very few candidates scored the two marks in part (d)(ii). Either their inequalities were not strict or their limits were incorrect or both. Some candidates stated the range of \(x\)&nbsp;as \(1,\,\,2,\,\,3,\,\,4\). The algebra in part (c) caused problems for a number of candidates. It seemed that there was a lack of understanding of what the question required. Some substituted \(x = 2\) in the volume formula. A few candidates wrote the product of the length, width, height, omitting the appropriate brackets. Part (d) was well answered by most candidates. However, its application in the following part was not as good in part (e). In part (e) some candidates left both solutions for \(x\), not appreciating the fact that one was outside the range. Others lost both marks as they did not show that they had used their derivative to part (d) as required by the question. Very few candidates scored full marks for the sketch in part (g). Not following the given instructions about the domain and range let most candidates down in this question.</p>
<div class="question_part_label">g.</div>
</div>
<br><hr><br><div class="specification">
<p><span style="font-family: times new roman,times; font-size: medium;">The temperature in \(^ \circ {\text{C}}\) of a pot of water removed from the cooker is given by \(T(m) = 20 + 70 \times {2.72^{ - 0.4m}}\), where \(m\) is the number of minutes after the pot is removed from the cooker.</span></p>
</div>

<div class="question" style="padding-left: 20px; padding-right: 20px;">
<p><span>Show that the temperature of the water when it is removed from the cooker is \({90^ \circ }{\text{C}}\).</span></p>
<div class="marks">[2]</div>
<div class="question_part_label">a.</div>
</div>
<div class="question" style="padding-left: 20px; padding-right: 20px;">
<p><span>The following table shows values for \(m\) and \(T(m)\).</span></p>
<p><span><img src="data:image/png;base64,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" alt></span></p>
<p><span>(i)     Write down the value of \(s\).</span></p>
<p><span>(ii)    Draw the graph of \(T(m)\) for \(0 \leqslant m \leqslant 10\) . Use a scale of \(1{\text{ cm}}\) to represent \(1\) minute on the horizontal axis and a scale of \(1{\text{ cm}}\) to represent \({10^ \circ }{\text{C}}\) on the vertical axis.</span></p>
<p><span>(iii)   <strong>Use your graph</strong> to find how long it takes for the temperature to reach \({56^ \circ }{\text{C}}\). Show your method clearly.</span></p>
<p><span>(iv)   Write down the temperature approached by the water after a long time. Justify your answer.<strong><br></strong></span></p>
<div class="marks">[9]</div>
<div class="question_part_label">b.</div>
</div>
<div class="question" style="padding-left: 20px; padding-right: 20px;">
<p><span>Consider the function \(S(m) = 20m - 40\) for \(2 \leqslant m \leqslant 6\) .</span></p>
<p><span>The function \(S(m)\) represents the temperature of soup in a pot placed on the cooker two minutes after the water has been removed. The soup is then heated.</span></p>
<p><span>Draw the graph of \(S(m)\) on the same set of axes used for part (b).</span></p>
<div class="marks">[2]</div>
<div class="question_part_label">c.</div>
</div>
<div class="question" style="padding-left: 20px; padding-right: 20px;">
<p><span>Consider the function \(S(m) = 20m - 40\) for \(2 \leqslant m \leqslant 6\) .</span></p>
<p><span>The function \(S(m)\) represents the temperature of soup in a pot placed on the cooker two minutes after the water has been removed. The soup is then heated.</span></p>
<p><span>Comment on the meaning of the constant \(20\) in the formula for \(S(m)\) in relation to the temperature of the soup.</span></p>
<div class="marks">[1]</div>
<div class="question_part_label">d.</div>
</div>
<div class="question" style="padding-left: 20px; padding-right: 20px;">
<p><span>Consider the function \(S(m) = 20m - 40\) for \(2 \leqslant m \leqslant 6\) .</span></p>
<p><span>The function \(S(m)\) represents the temperature of soup in a pot placed on the cooker two minutes after the water has been removed. The soup is then heated.</span></p>
<p><span>(i)     <strong>Use your graph</strong> to solve the equation \(S(m) = T(m)\) . Show your method clearly.</span></p>
<p><span>(ii)    Hence describe by using inequalities the set of values of \(m\) for which \(S(m) &gt; T(m)\).</span></p>
<div class="marks">[4]</div>
<div class="question_part_label">e.</div>
</div>
<h2 style="margin-top: 1em">Markscheme</h2>
<div class="question" style="padding-left: 20px;">
<p><span>\(T(0) = 20 + 70 \times {2.72^{ - 0.4 \times 0}} = 90\)     <em><strong>(M1)(A1)(AG)</strong></em></span></p>
<p><span><strong>Note: <em>(M1)</em></strong> for taking \(m = 0\) , <em><strong>(A1)</strong></em> for substituting \(0\) into the formula. For the <em><strong>A</strong></em> mark to be awarded \(90\) must be justified by correct method.</span></p>
<p><span><em><strong>[2 marks]</strong></em><br></span></p>
<div class="question_part_label">a.</div>
</div>
<div class="question" style="padding-left: 20px;">
<p><span>(i)    21.3     <em><strong>(A1)</strong></em></span></p>
<p><span>(ii)  </span></p>
<p><span><img src="data:image/png;base64,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" alt>     <em><strong>(A4)</strong></em><strong>(ft)</strong></span></p>
<p><span><strong>Note:</strong> Scales and labels <em><strong>(A1)</strong></em>. Smooth curve <em><strong>(A1)</strong></em>. All points correct including the \(y\)-intercept <em><strong>(A2)</strong></em>, 1 point incorrect <em><strong>(A1)</strong></em>, otherwise <em><strong>(A0)</strong></em>. Follow through from their value of \(s\).</span></p>
<p><span> </span></p>
<p><span>(iii)    \(m = 1.7{\text{ minutes}}\) <em>(Accept</em> \( \pm 0.2\) <em>)</em>     <em><strong>(A2)</strong></em><strong>(ft)</strong></span></p>
<p><span><strong><br>Note:</strong> Follow through from candidate's graph. Accept answers in minutes and seconds if consistent with graph. If answer incorrect and correct line(s) seen on graph award <em><strong>(M1)(A0)</strong></em>.</span></p>
<p><span> </span></p>
<p><span>(iv)    \({20^ \circ }{\text{C}}\)     <em><strong>(A1)</strong></em><strong>(ft)</strong></span></p>
<p><span>The curve behaves asymptotically to the line \(y = 20\) or similar.     <em><strong>(A1)</strong></em></span></p>
<p><span><strong>OR</strong></span></p>
<p><span>The room temperature is 20 or similar</span></p>
<p><span><strong>OR</strong></span></p>
<p><span>When \({\text{m}}\) is a very large number the term \(70 \times {2.72^{ - 0.4{\text{m}}}}\)</span><span> tends to zero or similar.</span></p>
<p><br><span><strong>Note: </strong>Follow through from their graph if appropriate.</span></p>
<p><span><em><strong>[9 marks]<br></strong></em></span></p>
<div class="question_part_label">b.</div>
</div>
<div class="question" style="padding-left: 20px;">
<p><span><img src="data:image/png;base64,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" alt><span>     <em><strong>(A1)(A1)</strong></em></span></span></p>
<p><em><strong><br></strong></em><span><strong>Notes: <em>(A1)</em></strong> for correct line, <em><strong>(A1)</strong></em> for domain. If line not drawn on same set of axes award at most <em><strong>(A1)(A0)</strong></em>.</span></p>
<p><span><em><strong>[2 marks]</strong></em><br></span></p>
<div class="question_part_label">c.</div>
</div>
<div class="question" style="padding-left: 20px;">
<p><span>It indicates by how much the temperature increases per minute.     <em><strong>(A1)</strong></em></span></p>
<p><span><em><strong>[1 mark]<br></strong></em></span></p>
<div class="question_part_label">d.</div>
</div>
<div class="question" style="padding-left: 20px;">
<p><span>(i)     \(m = 3.8\)<em> (Accept</em> \( \pm 0.1\) <em>)</em>     <em><strong>(A2)</strong></em><strong>(ft)</strong></span></p>
<p><br><span><strong>Note: </strong>Follow through from candidate's graph. Accept answers in minutes and seconds if consistent with graph. If answer incorrect and correct line(s) seen on graph award <em><strong>(M1)(A0)</strong></em>.</span></p>
<p><br><span>(ii)    \(3.8 &lt; m \leqslant 6\)     <em><strong>(A1)(A1)</strong></em><strong>(ft)</strong></span></p>
<p><br><span><strong>Note: <em>(A1)</em></strong> for \(m &gt; 3.8\) and <em><strong>(A1)</strong></em> for \(m \leqslant 6\). Follow through from candidate's answer to part (e)(i). If candidate was already penalized in (c) for domain and does not state \(m \leqslant 6\) then award <strong><em>(A2)</em>(ft)</strong>.</span></p>
<div class="question_part_label">e.</div>
</div>
<h2 style="margin-top: 1em">Examiners report</h2>
<div class="question" style="padding-left: 20px;">
<p><span style="font-family: times new roman,times; font-size: medium;">Good marks were gained in this question, mainly from parts (a) to (d). Very few students answered the show that question using a backwards process. These students did not gain full marks. Labelled and neat exponential graphs were drawn. Marks were lost sometimes for starting the curve at point \((1{\text{, }}66.9)\) instead of at \((0{\text{, }}90)\). Also there were students using a ruler to help them joined up the points for which they lost one mark as the graph must be smooth. Candidates managed to find the time at which the temperature was \(56\). However, those students that gave their answer as a coordinate pair lost the answer mark. Some students justified the behaviour of the curve by mentioning the asymptote but the big majority said that the room temperature was \(20\) and it was awarded full marks. Most of the students drew the straight line correct and in the given domain. Those students that drew the line in a separate set of axes could not answer the last part of the question. This part question proved to be difficult for many students and so worked as a discriminating one. One of the most common errors seen in part (b)(iii) and (e)(i) was to give the answer as a point instead of giving just the first coordinate of that point.</span></p>
<div class="question_part_label">a.</div>
</div>
<div class="question" style="padding-left: 20px;">
<p><span style="font-family: times new roman,times; font-size: medium;">Good marks were gained in this question, mainly from parts (a) to (d). Very few students answered the show that question using a backwards process. These students did not gain full marks. Labelled and neat exponential graphs were drawn. Marks were lost sometimes for starting the curve at point \((1{\text{, }}66.9)\) instead of at \((0{\text{, }}90)\). Also there were students using a ruler to help them joined up the points for which they lost one mark as the graph must be smooth. Candidates managed to find the time at which the temperature was \(56\). However, those students that gave their answer as a coordinate pair lost the answer mark. Some students justified the behaviour of the curve by mentioning the asymptote but the big majority said that the room temperature was \(20\) and it was awarded full marks. Most of the students drew the straight line correct and in the given domain. Those students that drew the line in a separate set of axes could not answer the last part of the question. This part question proved to be difficult for many students and so worked as a discriminating one. One of the most common errors seen in part (b)(iii) and (e)(i) was to give the answer as a point instead of giving just the first coordinate of that point.</span></p>
<div class="question_part_label">b.</div>
</div>
<div class="question" style="padding-left: 20px;">
<p><span style="font-family: times new roman,times; font-size: medium;">Good marks were gained in this question, mainly from parts (a) to (d). Very few students answered the show that question using a backwards process. These students did not gain full marks. Labelled and neat exponential graphs were drawn. Marks were lost sometimes for starting the curve at point \((1{\text{, }}66.9)\) instead of at \((0{\text{, }}90)\). Also there were students using a ruler to help them joined up the points for which they lost one mark as the graph must be smooth. Candidates managed to find the time at which the temperature was \(56\). However, those students that gave their answer as a coordinate pair lost the answer mark. Some students justified the behaviour of the curve by mentioning the asymptote but the big majority said that the room temperature was \(20\) and it was awarded full marks. Most of the students drew the straight line correct and in the given domain. Those students that drew the line in a separate set of axes could not answer the last part of the question. This part question proved to be difficult for many students and so worked as a discriminating one. One of the most common errors seen in part (b)(iii) and (e)(i) was to give the answer as a point instead of giving just the first coordinate of that point.</span></p>
<div class="question_part_label">c.</div>
</div>
<div class="question" style="padding-left: 20px;">
<p><span style="font-family: times new roman,times; font-size: medium;">Good marks were gained in this question, mainly from parts (a) to (d). Very few students answered the show that question using a backwards process. These students did not gain full marks. Labelled and neat exponential graphs were drawn. Marks were lost sometimes for starting the curve at point \((1{\text{, }}66.9)\) instead of at \((0{\text{, }}90)\). Also there were students using a ruler to help them joined up the points for which they lost one mark as the graph must be smooth. Candidates managed to find the time at which the temperature was \(56\). However, those students that gave their answer as a coordinate pair lost the answer mark. Some students justified the behaviour of the curve by mentioning the asymptote but the big majority said that the room temperature was \(20\) and it was awarded full marks. Most of the students drew the straight line correct and in the given domain. Those students that drew the line in a separate set of axes could not answer the last part of the question. This part question proved to be difficult for many students and so worked as a discriminating one. One of the most common errors seen in part (b)(iii) and (e)(i) was to give the answer as a point instead of giving just the first coordinate of that point.</span></p>
<div class="question_part_label">d.</div>
</div>
<div class="question" style="padding-left: 20px;">
<p><span style="font-family: times new roman,times; font-size: medium;">Good marks were gained in this question, mainly from parts (a) to (d). Very few students answered the show that question using a backwards process. These students did not gain full marks. Labelled and neat exponential graphs were drawn. Marks were lost sometimes for starting the curve at point \((1{\text{, }}66.9)\) instead of at \((0{\text{, }}90)\). Also there were students using a ruler to help them joined up the points for which they lost one mark as the graph must be smooth. Candidates managed to find the time at which the temperature was \(56\). However, those students that gave their answer as a coordinate pair lost the answer mark. Some students justified the behaviour of the curve by mentioning the asymptote but the big majority said that the room temperature was \(20\) and it was awarded full marks. Most of the students drew the straight line correct and in the given domain. Those students that drew the line in a separate set of axes could not answer the last part of the question. This part question proved to be difficult for many students and so worked as a discriminating one. One of the most common errors seen in part (b)(iii) and (e)(i) was to give the answer as a point instead of giving just the first coordinate of that point.</span></p>
<div class="question_part_label">e.</div>
</div>
<br><hr><br><div class="specification">
<p><span style="font-family: times new roman,times; font-size: medium;">A deep sea diver notices that the intensity of light, \(I\) , below the surface of the ocean decreases with depth, \(d\) , according to the formula</span><br><span style="font-family: times new roman,times; font-size: medium;">\[I = k{(1.05)^{ - d}}{\text{,}}\]where \(I\) is expressed as a percentage, \(d\) is the depth in metres below the surface and \(k\) is a constant.</span></p>
<p><span style="font-family: times new roman,times; font-size: medium;">The intensity of light at the surface is \(100\% \).</span></p>
</div>

<div class="question" style="padding-left: 20px; padding-right: 20px;">
<p><span>Calculate the value of \(k\) .</span></p>
<div class="marks">[2]</div>
<div class="question_part_label">a.</div>
</div>
<div class="question" style="padding-left: 20px; padding-right: 20px;">
<p><span>Find the intensity of light at a depth \(25{\text{ m}}\) below the surface.</span></p>
<div class="marks">[2]</div>
<div class="question_part_label">b.</div>
</div>
<div class="question" style="padding-left: 20px; padding-right: 20px;">
<p><span>To be able to see clearly, a diver needs the intensity of light to be at least \(65\% \).</span></p>
<p><span>Using your graphic display calculator, find the greatest depth below the surface at which she can see clearly.</span></p>
<div class="marks">[2]</div>
<div class="question_part_label">c.</div>
</div>
<div class="question" style="padding-left: 20px; padding-right: 20px;">
<p><span>The table below gives the intensity of light (correct to the nearest integer) at different depths.</span></p>
<p><span><img src="data:image/png;base64,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" alt></span></p>
<p><span>Using this information draw the graph of \(I\) against \(d\) for \(0 \leqslant d \leqslant 100\) . Use a scale of \(1{\text{ cm}}\) to represent 10 metres on the horizontal axis and 1 cm to represent \(10\% \) on the vertical axis.</span></p>
<div class="marks">[4]</div>
<div class="question_part_label">d.</div>
</div>
<div class="question" style="padding-left: 20px; padding-right: 20px;">
<p><span>Some sea creatures have adapted so they can see in low intensity light and cannot tolerate too much light.<br></span></p>
<p><span>Indicate clearly on your graph the range of depths sea creatures could inhabit if they can tolerate between \(5\% \) and \(35\% \) of the light intensity at the surface.</span></p>
<div class="marks">[2]</div>
<div class="question_part_label">e.</div>
</div>
<h2 style="margin-top: 1em">Markscheme</h2>
<div class="question" style="padding-left: 20px;">
<p><span>\(d = 0\), \(k = 100\)     <em><strong>(M1)(A1)(G2)</strong></em></span></p>
<p><br><span><strong>Note:</strong> Award <em><strong>(M1)</strong></em> for \(d = 0\) seen.</span></p>
<div class="question_part_label">a.</div>
</div>
<div class="question" style="padding-left: 20px;">
<p><span><span>\(I = 100 \times {(1.05)^{ - 25}} = 29.5(\% )\) (\(29.5302 \ldots \))    </span> <span><em><strong>(M1)(A1)</strong></em><strong>(ft)</strong><em><strong>(G2)</strong></em></span></span></p>
<div class="question_part_label">b.</div>
</div>
<div class="question" style="padding-left: 20px;">
<p><span>\(65 = 100 \times {(1.05)^{ - d}}\)     <em><strong>(M1)</strong></em></span></p>
<p><br><span><strong>Note:</strong> Award <em><strong>(M1)</strong></em> for sketch with line drawn at \(y = 65\) .</span></p>
<p> </p>
<p><span><span>\(d = 8.83{\text{ (m)}}\) (\(8.82929 \ldots \))    </span> <span><em><strong>(A1)</strong></em><strong>(ft)</strong><em><strong>(G2)</strong></em></span></span></p>
<div class="question_part_label">c.</div>
</div>
<div class="question" style="padding-left: 20px;">
<p><span><img src="data:image/png;base64,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" alt></span></p>
<p><span><em><strong>(A1)</strong></em> for labels and scales</span><br><span><em><strong>(A2)</strong></em> for all points correct, <em><strong>(A1)</strong></em> for 3 or 4 points correct</span><br><span><em><strong>(A1)</strong></em> for smooth curve asymptotic to the \(x\)-axis     <em><strong>(A4)</strong></em></span></p>
<div class="question_part_label">d.</div>
</div>
<div class="question" style="padding-left: 20px;">
<p><span>Lines in approx correct positions <strong>on</strong> graph     <em><strong>(M1)</strong></em></span><br><span>The range of values indicated (arrows or shading) \(22\)</span><span></span><span><span>–</span>\(60{\text{ m}}\)     <em><strong>(A1)</strong></em></span></p>
<div class="question_part_label">e.</div>
</div>
<h2 style="margin-top: 1em">Examiners report</h2>
<div class="question" style="padding-left: 20px;">
[N/A]
<div class="question_part_label">a.</div>
</div>
<div class="question" style="padding-left: 20px;">
[N/A]
<div class="question_part_label">b.</div>
</div>
<div class="question" style="padding-left: 20px;">
[N/A]
<div class="question_part_label">c.</div>
</div>
<div class="question" style="padding-left: 20px;">
[N/A]
<div class="question_part_label">d.</div>
</div>
<div class="question" style="padding-left: 20px;">
[N/A]
<div class="question_part_label">e.</div>
</div>
<br><hr><br><div class="specification">
<p>Consider the function&nbsp;\(f\left( x \right) = \frac{{48}}{x} + k{x^2} - 58\), where <em>x</em> &gt; 0 and <em>k</em> is a constant.</p>
<p>The graph of the function passes through the point with coordinates (4 , 2).</p>
</div>

<div class="specification">
<p>P is the minimum point of the graph of <em>f&thinsp;</em>(<em>x</em>).</p>
</div>

<div class="question" style="padding-left: 20px; padding-right: 20px;">
<p>Find the value of <em>k</em>.</p>
<div class="marks">[2]</div>
<div class="question_part_label">a.</div>
</div>
<div class="question" style="padding-left: 20px; padding-right: 20px;">
<p>Using your value of <em>k</em> , find <em>f</em> ′(<em>x</em>).</p>
<div class="marks">[3]</div>
<div class="question_part_label">b.</div>
</div>
<div class="question" style="padding-left: 20px; padding-right: 20px;">
<p><strong>Use your answer</strong> to part (b) to show that the minimum value of <em>f</em>(<em>x</em>) is −22 .</p>
<div class="marks">[3]</div>
<div class="question_part_label">c.</div>
</div>
<div class="question" style="padding-left: 20px; padding-right: 20px;">
<p>Write down the <strong>two</strong> values of<em> x</em> which satisfy<em> f </em>(<em>x</em>) = 0.</p>
<div class="marks">[2]</div>
<div class="question_part_label">d.</div>
</div>
<div class="question" style="padding-left: 20px; padding-right: 20px;">
<p>Sketch the graph of <em>y</em> = <em>f</em> (<em>x</em>) for 0 &lt; <em>x</em> ≤ 6 and −30 ≤ <em>y</em> ≤ 60.<br>Clearly indicate the minimum point P and the <em>x</em>-intercepts on your graph.</p>
<div class="marks">[4]</div>
<div class="question_part_label">e.</div>
</div>
<h2 style="margin-top: 1em">Markscheme</h2>
<div class="question" style="padding-left: 20px;">
<p>\(\frac{{48}}{4} + k \times {4^2} - 58 = 2\)    <em><strong>(M1)</strong></em><br><strong>Note:</strong> Award <em><strong>(M1)</strong></em> for correct substitution of <em>x</em> = 4 and <em>y</em> = 2 into the function.</p>
<p><em>k</em> = 3     <em><strong>(A1) (G2)</strong></em></p>
<p><em><strong>[2 marks]</strong></em></p>
<div class="question_part_label">a.</div>
</div>
<div class="question" style="padding-left: 20px;">
<p>\(\frac{{ - 48}}{{{x^2}}} + 6x\)     <strong><em>(A1)(A1)(A1)</em>(ft)<em> (G3)</em></strong></p>
<p><strong>Note:</strong> Award <strong><em>(A1)</em></strong> for −48 , (A1) for <em>x</em><sup>−2</sup>, <strong><em>(A1)</em>(ft)</strong> for their 6<em>x</em>. Follow through from part (a). Award at most <strong><em>(A1)(A1)(A0)</em></strong> if additional terms are seen.</p>
<p><em><strong>[3 marks]</strong></em></p>
<div class="question_part_label">b.</div>
</div>
<div class="question" style="padding-left: 20px;">
<p>\(\frac{{ - 48}}{{{x^2}}} + 6x = 0\)     <em><strong>(M1)</strong></em></p>
<p><strong>Note:</strong> Award <em><strong>(M1)</strong></em> for equating their part (b) to zero.</p>
<p><em>x</em> = 2     <em><strong>(A1)</strong></em><strong>(ft)</strong></p>
<p><strong>Note:</strong> Follow through from part (b). Award (<em><strong>M1)(A1)</strong></em> for \(\frac{{ - 48}}{{{{\left( 2 \right)}^2}}} + 6\left( 2 \right) = 0\) seen.</p>
<p>Award <em><strong>(M0)(A0)</strong></em> for <em>x</em> = 2 seen either from a graphical method or without working.</p>
<p>\(\frac{{48}}{2} + 3 \times {2^2} - 58\,\,\,\left( { =  - 22} \right)\)   <em><strong>(M1)</strong></em></p>
<p><strong>Note:</strong> Award <em><strong>(M1)</strong></em> for substituting their 2 into their function, but only if the final answer is −22. Substitution of the known result invalidates the process; award <em><strong>(M0)(A0)(M0)</strong></em>.</p>
<p>−22     <em><strong>(AG)</strong></em></p>
<p><em><strong>[3 marks]</strong></em></p>
<div class="question_part_label">c.</div>
</div>
<div class="question" style="padding-left: 20px;">
<p>0.861  (0.860548…), 3.90  (3.90307…)     <em><strong>(A1)</strong></em><strong>(ft)</strong><em><strong>(A1)</strong></em><strong>(ft)</strong><em><strong> (G2)</strong></em></p>
<p><strong>Note:</strong> Follow through from part (a) but only if the answer is positive. Award at most <em><strong>(A1)</strong></em><strong>(ft)</strong><em><strong>(A0)</strong></em> if answers are given as coordinate pairs or if extra values are seen. The function <em>f </em>(<em>x</em>) only has two <em>x</em>-intercepts within the domain. Do not accept a negative <em>x</em>-intercept.</p>
<p><em><strong>[2 marks]</strong></em></p>
<div class="question_part_label">d.</div>
</div>
<div class="question" style="padding-left: 20px;">
<p><img src="data:image/png;base64,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"><em><strong>(A1)(A1)</strong></em><strong>(ft)</strong><em><strong>(A1)</strong></em><strong>(ft)</strong><em><strong>(A1)</strong></em><strong>(ft)</strong></p>
<p><strong>Note:</strong> Award <em><strong>(A1)</strong></em> for correct window. Axes must be labelled.<br><em><strong>(A1)</strong></em><strong>(ft)</strong> for a smooth curve with correct shape and zeros in approximately correct positions relative to each other.<br><em><strong>(A1)</strong></em><strong>(ft)</strong> for point P indicated in approximately the correct position. Follow through from their <em>x</em>-coordinate in part (c). <em><strong>(A1)</strong></em><strong>(ft)</strong> for two <em>x</em>-intercepts identified on the graph and curve reflecting asymptotic properties.</p>
<p><em><strong>[4 marks]</strong></em></p>
<div class="question_part_label">e.</div>
</div>
<h2 style="margin-top: 1em">Examiners report</h2>
<div class="question" style="padding-left: 20px;">
[N/A]
<div class="question_part_label">a.</div>
</div>
<div class="question" style="padding-left: 20px;">
[N/A]
<div class="question_part_label">b.</div>
</div>
<div class="question" style="padding-left: 20px;">
[N/A]
<div class="question_part_label">c.</div>
</div>
<div class="question" style="padding-left: 20px;">
[N/A]
<div class="question_part_label">d.</div>
</div>
<div class="question" style="padding-left: 20px;">
[N/A]
<div class="question_part_label">e.</div>
</div>
<br><hr><br><div class="question" style="padding-left: 20px; padding-right: 20px;">
<p>A function, \(f\) , is given by</p>
<p>\[f(x) = 4 \times {2^{ - x}} + 1.5x - 5.\]</p>
<p>Calculate \(f(0)\)</p>
<p> </p>
<div class="marks">[2]</div>
<div class="question_part_label">a.</div>
</div>
<div class="question" style="padding-left: 20px; padding-right: 20px;">
<p>Use your graphic display calculator to solve \(f(x) = 0.\)</p>
<div class="marks">[2]</div>
<div class="question_part_label">b.</div>
</div>
<div class="question" style="padding-left: 20px; padding-right: 20px;">
<p>Sketch the graph of \(y = f(x)\) for \( - 2 \leqslant x \leqslant 6\) and \( - 4 \leqslant y \leqslant 10\) , showing the \(x\) and \(y\) intercepts. Use a scale of \(2\,{\text{cm}}\) to represent \(2\) units on both the horizontal axis, \(x\) , and the vertical axis, \(y\) .</p>
<div class="marks">[4]</div>
<div class="question_part_label">c.</div>
</div>
<div class="question" style="padding-left: 20px; padding-right: 20px;">
<p>The function \(f\) is the derivative of a function \(g\) . It is known that \(g(1) = 3.\)</p>
<p>i)     Calculate \(g'(1).\)</p>
<p>ii)    Find the equation of the tangent to the graph of \(y = g(x)\) at \(x = 1.\) Give your answer in the form \(y = mx + c.\)</p>
<div class="marks">[4]</div>
<div class="question_part_label">d.</div>
</div>
<h2 style="margin-top: 1em">Markscheme</h2>
<div class="question" style="padding-left: 20px;">
<p>\(4 \times {2^{ - 0}} + 1.5 \times 0 - 5\)       <em><strong>(M1)</strong></em></p>
<p><strong>Note:</strong> Award <em><strong>(</strong><strong>M1)</strong></em> for substitution of \(0\) into the expression for \(f(x)\) .</p>
<p>\( =  - 1\)        <em><strong>(A1)(G2)</strong></em></p>
<div class="question_part_label">a.</div>
</div>
<div class="question" style="padding-left: 20px;">
<p>\( - 0.538\,\,\,( - 0.537670...)\) and \(3\)        <em><strong>(A1)(A1)</strong></em></p>
<p><strong>Note:</strong> Award at most<strong><em> (A0)(A1)</em>(ft)</strong> if answer is given as pairs of coordinates.</p>
<div class="question_part_label">b.</div>
</div>
<div class="question" style="padding-left: 20px;">
<p><img src="data:image/png;base64,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" alt></p>
<p><strong><em>(A1)(A1)(A1)</em>(ft)<em>(A1)</em>(ft)</strong></p>
<p><strong>Note:</strong> Award <em><strong>(A1)</strong></em> for labels and some indication of scale in the correct given window.</p>
<p>Award <em><strong>(A1)</strong></em> for smooth curve with correct general shape with \(f( - 2) &gt; f(6)\) and minimum to the right of the \(y\)-axis.</p>
<p>Award <em><strong>(A1)</strong></em><strong>(ft)</strong> for correct \(y\)-intercept (consistent with their part (a)).</p>
<p>Award <em><strong>(A1)</strong></em><strong>(ft)</strong> for approximately correct \(x\)-intercepts (consistent with their part (b), one zero between \( - 1\) and \(0\), the other between \(2.5\) and \(3.5\)).</p>
<div class="question_part_label">c.</div>
</div>
<div class="question" style="padding-left: 20px;">
<p>i)     \(g'(1) = f(1) = 4 \times {2^{ - 1}} + 1.5 - 5\)       <em><strong>(M1)</strong></em></p>
<p><strong>Note:</strong> Award <em><strong>(M1)</strong></em> for substitution of \(1\) into \(f(x)\).</p>
<p>\( =  - 1.5\)        <em><strong>(A1)(G2)</strong></em></p>
<p> </p>
<p>ii)    \(3 =  - 1.5 \times 1 + c\) <strong>OR</strong> \((y - 3) =  - 1.5\,(x - 1)\)       <em><strong>(M1)</strong></em></p>
<p><strong>Note:</strong> Award <em><strong>(M1)</strong></em> for correct substitution of gradient and the point \((1,\,\,3)\) into the equation of a line. Follow through from (d)(i).</p>
<p>\(y =  - 1.5x + 4.5\)        <em><strong>(A1)</strong></em><strong>(ft)</strong><em><strong>(G2)</strong></em></p>
<div class="question_part_label">d.</div>
</div>
<h2 style="margin-top: 1em">Examiners report</h2>
<div class="question" style="padding-left: 20px;">
<p>Question 6: Functions and Calculus<br>Many candidates were able to calculate \(f(0)\) &nbsp;although some had a problem with \({2^0}\). Very few were able to find both roots in part (b), even when they represented correctly both zeros in their sketch. Many sketches were reasonably well drawn, presenting a smooth curve with correct points of intersection. Many lost a mark for not labelling their axes and the given scale was ignored by some, which was not penalized but often resulted in sketches which were difficult to read. The stronger candidates calculated \(g'(1)\) correctly, but very few represented the gradient of the function at \(x = 1\) correctly.</p>
<div class="question_part_label">a.</div>
</div>
<div class="question" style="padding-left: 20px;">
<p>Question 6: Functions and Calculus</p>
<p>Many candidates were able to calculate \(f(0)\) &nbsp;although some had a problem with \({2^0}\). Very few were able to find both roots in part (b), even when they represented correctly both zeros in their sketch. Many sketches were reasonably well drawn, presenting a smooth curve with correct points of intersection. Many lost a mark for not labelling their axes and the given scale was ignored by some, which was not penalized but often resulted in sketches which were difficult to read. The stronger candidates calculated \(g'(1)\) correctly, but very few represented the gradient of the function at \(x = 1\) correctly.</p>
<div class="question_part_label">b.</div>
</div>
<div class="question" style="padding-left: 20px;">
<p>Question 6: Functions and Calculus Many candidates were able to calculate \(f(0)\) &nbsp;although some had a problem with \({2^0}\). Very few were able to find both roots in part (b), even when they represented correctly both zeros in their sketch. Many sketches were reasonably well drawn, presenting a smooth curve with correct points of intersection. Many lost a mark for not labelling their axes and the given scale was ignored by some, which was not penalized but often resulted in sketches which were difficult to read. The stronger candidates calculated \(g'(1)\) correctly, but very few represented the gradient of the function at \(x = 1\) correctly.</p>
<div class="question_part_label">c.</div>
</div>
<div class="question" style="padding-left: 20px;">
<p>Question 6: Functions and Calculus</p>
<p>Many candidates were able to calculate \(f(0)\) &nbsp;although some had a problem with \({2^0}\). Very few were able to find both roots in part (b), even when they represented correctly both zeros in their sketch. Many sketches were reasonably well drawn, presenting a smooth curve with correct points of intersection. Many lost a mark for not labelling their axes and the given scale was ignored by some, which was not penalized but often resulted in sketches which were difficult to read. The stronger candidates calculated \(g'(1)\) correctly, but very few represented the gradient of the function at \(x = 1\) correctly.</p>
<div class="question_part_label">d.</div>
</div>
<br><hr><br><div class="question" style="padding-left: 20px; padding-right: 20px;">
<p>A distress flare is fired into the air from a ship at sea. The height, \(h\) , in metres, of the flare above sea level is modelled by the quadratic function</p>
<p>\[h\,(t) =  - 0.2{t^2} + 16t + 12\,,\,t \geqslant 0\,,\]</p>
<p>where \(t\) is the time, in seconds, and \(t = 0\,\) at the moment the flare was fired.</p>
<p>Write down the height from which the flare was fired.</p>
<div class="marks">[1]</div>
<div class="question_part_label">a.</div>
</div>
<div class="question" style="padding-left: 20px; padding-right: 20px;">
<p>Find the height of the flare \(15\) seconds after it was fired.</p>
<div class="marks">[2]</div>
<div class="question_part_label">b.</div>
</div>
<div class="question" style="padding-left: 20px; padding-right: 20px;">
<p>The flare fell into the sea \(k\) seconds after it was fired.</p>
<p>Find the value of \(k\) .</p>
<div class="marks">[2]</div>
<div class="question_part_label">c.</div>
</div>
<div class="question" style="padding-left: 20px; padding-right: 20px;">
<p>Find \(h'\,(t)\,.\)</p>
<div class="marks">[2]</div>
<div class="question_part_label">d.</div>
</div>
<div class="question" style="padding-left: 20px; padding-right: 20px;">
<p>i)     Show that the flare reached its maximum height \(40\) seconds after being fired.</p>
<p>ii)    Calculate the maximum height reached by the flare.</p>
<div class="marks">[3]</div>
<div class="question_part_label">e.</div>
</div>
<div class="question" style="padding-left: 20px; padding-right: 20px;">
<p>The nearest coastguard can see the flare when its height is more than \(40\) metres above sea level.</p>
<p>Determine the total length of time the flare can be seen by the coastguard.</p>
<div class="marks">[3]</div>
<div class="question_part_label">f.</div>
</div>
<h2 style="margin-top: 1em">Markscheme</h2>
<div class="question" style="padding-left: 20px;">
<p>\(12\,({\text{m}})\)       <em><strong>(A1)</strong></em></p>
<div class="question_part_label">a.</div>
</div>
<div class="question" style="padding-left: 20px;">
<p>\((h\,(15) = ) - 0.2 \times {15^2} + 16 \times 15 + 12\)       <em><strong>(M1)</strong></em></p>
<p><strong>Note:</strong> Award <em><strong>(M1)</strong></em> for substitution of \(15\) in expression for \(h\).</p>
<p>\( = 207\,({\text{m}})\)       <em><strong>(A1)(G2)</strong></em></p>
<div class="question_part_label">b.</div>
</div>
<div class="question" style="padding-left: 20px;">
<p>\(h\,(k) = 0\)       <strong><em>(M1)</em></strong></p>
<p><strong>Note:</strong> Award <em><strong>(M1)</strong></em> for setting \(h\) to zero.</p>
<p>\((k = )\,\,\,80.7\,({\text{s}})\,\,\,(80.7430)\)       <strong><em>(A1)(G2)</em></strong></p>
<p><strong>Note:</strong> Award at most <strong><em>(M1)(A0)</em></strong> for an answer including \(K =  - 0.743\) .<br> Award <strong><em>(A0)</em></strong> for an answer of \(80\) without working.</p>
<div class="question_part_label">c.</div>
</div>
<div class="question" style="padding-left: 20px;">
<p>\(h'\,(t) =  - 0.4t + 16\)        <em><strong>(A1)(A1)</strong></em></p>
<p><strong>Note:</strong> Award <em><strong>(A1)</strong></em> for \( - 0.4t\), (<em><strong>A1)</strong></em> for \(16\). Award at most <em><strong>(A1)(A0)</strong></em> if extra terms seen. Do not accept \(x\) for \(t\).</p>
<div class="question_part_label">d.</div>
</div>
<div class="question" style="padding-left: 20px;">
<p>i)     \( - 0.4t + 16 = 0\)       <em><strong>(M1)</strong></em></p>
<p><strong>Note:</strong> Award <em><strong>(M1)</strong></em> for setting their derivative, from part (d), to zero, provided the correct conclusion is stated and consistent with their \(h'\,(t)\).</p>
<p><strong>OR</strong></p>
<p>\(t = \frac{{ - 16}}{{2 \times ( - 0.2)}}\)       <em><strong>(M1)</strong></em></p>
<p><strong>Note:</strong> Award (<em><strong>M1)</strong></em> for correct substitution into axis of symmetry formula, provided the correct conclusion is stated.</p>
<p>\(t = \,\,40\,({\text{s}})\)       <em><strong>(AG)</strong></em></p>
<p> </p>
<p> </p>
<p>ii)    \( - 0.2 \times {40^2} + 16 \times 40 + 12\)       <em><strong>(M1)</strong></em></p>
<p><strong>Note:</strong> Award <em><strong>(M1)</strong></em> for substitution of \(40\) in expression for \(h\).</p>
<p>\( = 332\,({\text{m}})\)       <em><strong>(A1)(G2)</strong></em></p>
<div class="question_part_label">e.</div>
</div>
<div class="question" style="padding-left: 20px;">
<p>\(h\,(t) = 40\)       <em><strong>(M1)</strong></em></p>
<p><strong>Note:</strong> Award <em><strong>(M1)</strong></em> for setting \(h\) to \(40\). Accept inequality sign.</p>
<p><strong>OR</strong></p>
<p><img src="data:image/png;base64,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" alt></p>
<p><em><strong>M1</strong></em></p>
<p><strong>Note:</strong> Award <em><strong>(M1)</strong></em> for correct sketch. Indication of scale is not required.</p>
<p>\(78.2 - 1.17\,\,(78.2099...\,\, - 1.79005...)\)       <em><strong>(A1)</strong></em></p>
<p><strong>Note:</strong> Award <em><strong>(A1)</strong></em> for \(1.79\) and \(78.2\) seen.</p>
<p>(total time \( = \)) \(76.4\,({\text{s}})\,\,\,(76.4198...)\)       <em><strong>(A1)(G2)</strong></em></p>
<p><strong>Note:</strong> Award <em><strong>(G1)</strong></em> if the two endpoints are given as the final answer with no working.</p>
<div class="question_part_label">f.</div>
</div>
<h2 style="margin-top: 1em">Examiners report</h2>
<div class="question" style="padding-left: 20px;">
<p>Question 3: Quadratic function, problem solving.<br>Parts (a) (finding the initial height) and (b) (finding the height after 15 seconds), were done very well by the majority of candidates. Many struggled to translate question (c) to find the (positive) zeros of the function, or did not write that down, losing a possible method mark. The derivative in part (d) was no problem for most; only very few used \(x\)&nbsp;instead of \(t\). The maximum height reached was calculated correctly by the majority of candidates, but many lost the mark in part (e)(i) as they simply substituted 40 into their derivative or calculated the height at points close to 40. Only a few candidates showed correct method for part (f). Several were still able to obtain 2 marks as a result of &ldquo;trial and error&rdquo; of integer values for \(t\). Some candidate seem to have a problem with the notation &ldquo;\(h\,(t) = \,...\)&rdquo;, where this is interpreted as \(h \times t\), resulting in incorrect answers throughout.</p>
<div class="question_part_label">a.</div>
</div>
<div class="question" style="padding-left: 20px;">
<p>Question 3: Quadratic function, problem solving.</p>
<p>Parts (a) (finding the initial height) and (b) (finding the height after 15 seconds), were done very well by the majority of candidates. Many struggled to translate question (c) to find the (positive) zeros of the function, or did not write that down, losing a possible method mark. The derivative in part (d) was no problem for most; only very few used&nbsp;\(x\)&nbsp;instead of&nbsp;\(t\). The maximum height reached was calculated correctly by the majority of candidates, but many lost the mark in part (e)(i) as they simply substituted 40 into their derivative or calculated the height at points close to 40. Only a few candidates showed correct method for part (f). Several were still able to obtain 2 marks as a result of &ldquo;trial and error&rdquo; of integer values for&nbsp;\(t\). Some candidate seem to have a problem with the notation &ldquo;\(h\,(t) = \,...\)&rdquo;, where this is interpreted as&nbsp;\(h \times t\), resulting in incorrect answers throughout.</p>
<div class="question_part_label">b.</div>
</div>
<div class="question" style="padding-left: 20px;">
<p>Question 3: Quadratic function, problem solving.</p>
<p>Parts (a) (finding the initial height) and (b) (finding the height after 15 seconds), were done very well by the majority of candidates. Many struggled to translate question (c) to find the (positive) zeros of the function, or did not write that down, losing a possible method mark. The derivative in part (d) was no problem for most; only very few used&nbsp;\(x\)&nbsp;instead of&nbsp;\(t\). The maximum height reached was calculated correctly by the majority of candidates, but many lost the mark in part (e)(i) as they simply substituted 40 into their derivative or calculated the height at points close to 40. Only a few candidates showed correct method for part (f). Several were still able to obtain 2 marks as a result of &ldquo;trial and error&rdquo; of integer values for&nbsp;\(t\). Some candidate seem to have a problem with the notation &ldquo;\(h\,(t) = \,...\)&rdquo;, where this is interpreted as&nbsp;\(h \times t\), resulting in incorrect answers throughout.</p>
<div class="question_part_label">c.</div>
</div>
<div class="question" style="padding-left: 20px;">
<p>Question 3: Quadratic function, problem solving.</p>
<p>Parts (a) (finding the initial height) and (b) (finding the height after 15 seconds), were done very well by the majority of candidates. Many struggled to translate question (c) to find the (positive) zeros of the function, or did not write that down, losing a possible method mark. The derivative in part (d) was no problem for most; only very few used&nbsp;\(x\)&nbsp;instead of&nbsp;\(t\). The maximum height reached was calculated correctly by the majority of candidates, but many lost the mark in part (e)(i) as they simply substituted 40 into their derivative or calculated the height at points close to 40. Only a few candidates showed correct method for part (f). Several were still able to obtain 2 marks as a result of &ldquo;trial and error&rdquo; of integer values for&nbsp;\(t\). Some candidate seem to have a problem with the notation &ldquo;\(h\,(t) = \,...\)&rdquo;, where this is interpreted as&nbsp;\(h \times t\), resulting in incorrect answers throughout.</p>
<div class="question_part_label">d.</div>
</div>
<div class="question" style="padding-left: 20px;">
<p>Question 3: Quadratic function, problem solving.</p>
<p>Parts (a) (finding the initial height) and (b) (finding the height after 15 seconds), were done very well by the majority of candidates. Many struggled to translate question (c) to find the (positive) zeros of the function, or did not write that down, losing a possible method mark. The derivative in part (d) was no problem for most; only very few used&nbsp;\(x\)&nbsp;instead of&nbsp;\(t\). The maximum height reached was calculated correctly by the majority of candidates, but many lost the mark in part (e)(i) as they simply substituted 40 into their derivative or calculated the height at points close to 40. Only a few candidates showed correct method for part (f). Several were still able to obtain 2 marks as a result of &ldquo;trial and error&rdquo; of integer values for&nbsp;\(t\). Some candidate seem to have a problem with the notation &ldquo;\(h\,(t) = \,...\)&rdquo;, where this is interpreted as&nbsp;\(h \times t\), resulting in incorrect answers throughout.</p>
<div class="question_part_label">e.</div>
</div>
<div class="question" style="padding-left: 20px;">
<p>Question 3: Quadratic function, problem solving.</p>
<p>Parts (a) (finding the initial height) and (b) (finding the height after 15 seconds), were done very well by the majority of candidates. Many struggled to translate question (c) to find the (positive) zeros of the function, or did not write that down, losing a possible method mark. The derivative in part (d) was no problem for most; only very few used&nbsp;\(x\)&nbsp;instead of&nbsp;\(t\). The maximum height reached was calculated correctly by the majority of candidates, but many lost the mark in part (e)(i) as they simply substituted 40 into their derivative or calculated the height at points close to 40. Only a few candidates showed correct method for part (f). Several were still able to obtain 2 marks as a result of &ldquo;trial and error&rdquo; of integer values for&nbsp;\(t\). Some candidate seem to have a problem with the notation &ldquo;\(h\,(t) = \,...\)&rdquo;, where this is interpreted as&nbsp;\(h \times t\), resulting in incorrect answers throughout.</p>
<div class="question_part_label">f.</div>
</div>
<br><hr><br><div class="specification">
<p><span style="font-size: medium; font-family: times new roman,times;">Consider the function \(f(x) =&nbsp; - \frac{1}{3}{x^3} + \frac{5}{3}{x^2} - x - 3\).</span></p>
</div>

<div class="question" style="padding-left: 20px; padding-right: 20px;">
<p><span>Sketch the graph of <em>y</em> = <em>f</em> (<em>x</em>) for −3 ≤ <em>x</em> </span><span><span>≤</span> 6 and −10 </span><span><span>≤</span> <em>y</em> </span><span><span>≤ </span>10 showing clearly the axes intercepts and local maximum and minimum points. Use a scale of 2 cm to represent 1 unit on the <em>x</em>-axis, and a scale of 1 cm to represent 1 unit on the <em>y</em>-axis.</span></p>
<div class="marks">[4]</div>
<div class="question_part_label">a.</div>
</div>
<div class="question" style="padding-left: 20px; padding-right: 20px;">
<p><span>Find the value of <em>f</em> (−1).</span></p>
<div class="marks">[2]</div>
<div class="question_part_label">b.</div>
</div>
<div class="question" style="padding-left: 20px; padding-right: 20px;">
<p><span>Write down the coordinates of the <em>y</em>-intercept of the graph of <em>f</em> (<em>x</em>).</span></p>
<div class="marks">[1]</div>
<div class="question_part_label">c.</div>
</div>
<div class="question" style="padding-left: 20px; padding-right: 20px;">
<p><span>Find <em>f '</em>(<em>x</em>).</span></p>
<div class="marks">[3]</div>
<div class="question_part_label">d.</div>
</div>
<div class="question" style="padding-left: 20px; padding-right: 20px;">
<p><span>Show that \(f'( - 1) =  - \frac{{16}}{3}\).</span></p>
<div class="marks">[1]</div>
<div class="question_part_label">e.</div>
</div>
<div class="question" style="padding-left: 20px; padding-right: 20px;">
<p><span>Explain what <em>f</em> <em>'</em>(−1) represents.</span></p>
<div class="marks">[2]</div>
<div class="question_part_label">f.</div>
</div>
<div class="question" style="padding-left: 20px; padding-right: 20px;">
<p><span>Find the equation of the tangent to the graph of <em>f</em> (<em>x</em>) at the point where <em>x</em> is –1.</span></p>
<div class="marks">[2]</div>
<div class="question_part_label">g.</div>
</div>
<div class="question" style="padding-left: 20px; padding-right: 20px;">
<p><span>Sketch the tangent to the graph of <em>f</em> (<em>x</em>) at <em>x</em> = −1 on your diagram for (a).</span></p>
<div class="marks">[2]</div>
<div class="question_part_label">h.</div>
</div>
<div class="question" style="padding-left: 20px; padding-right: 20px;">
<p><span>P and Q are points on the curve such that the tangents to the curve at these points are horizontal. The <em>x</em>-coordinate of P is <em>a</em>, and the <em>x</em>-coordinate of Q is <em>b</em>, <em>b</em> &gt; <em>a</em>.</span></p>
<p><span>Write down the value of</span></p>
<p><span>(i) <em>a</em> ;</span></p>
<p><span>(ii) <em>b</em> .</span></p>
<div class="marks">[2]</div>
<div class="question_part_label">i.</div>
</div>
<div class="question" style="padding-left: 20px; padding-right: 20px;">
<p><span><span>P and Q are points on the curve such that the tangents to the curve at these points are horizontal. The <em>x</em>-coordinate of P is <em>a</em>, and the <em>x</em>-coordinate of Q is <em>b</em>, <em>b</em> &gt; <em>a</em>.</span></span></p>
<p><span>Describe the behaviour of <em>f</em> (<em>x</em>) for <em>a</em> &lt; <em>x</em> &lt; <em>b</em>.</span></p>
<div class="marks">[1]</div>
<div class="question_part_label">j.</div>
</div>
<h2 style="margin-top: 1em">Markscheme</h2>
<div class="question" style="padding-left: 20px;">
<p><span><img src="data:image/png;base64,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" alt></span></p>
<p><span><em><strong>(A1)</strong></em> for indication of window and labels. <em><strong>(A1)</strong></em> for smooth curve that does not </span><span>enter the first quadrant, the curve must consist of one line only.</span></p>
<p><span><em><strong>(A1)</strong></em> for <em>x</em> and <em>y</em> intercepts in approximately correct positions (allow ±0.5).</span></p>
<p><span><em><strong>(A1)</strong></em> for local maximum and minimum in approximately correct position.</span> <span>(minimum should be 0 ≤ <em>x</em> ≤ 1 and –2 ≤ <em>y</em> ≤ –4 ), the <em>y</em>-coordinate of the </span><span>maximum should be 0 ± 0.5.     <em><strong>(A4)</strong></em></span></p>
<p><span><em><strong>[4 marks]<br></strong></em></span></p>
<div class="question_part_label">a.</div>
</div>
<div class="question" style="padding-left: 20px;">
<p><span>\(-\frac{1}{3}(-1)^3 + \frac{5}{3}(-1)^2 - (-1) - 3 \)     <em><strong>(M1)</strong></em></span></p>
<p><span><strong>Note:</strong> Award <em><strong>(M1)</strong></em> for substitution of –1 into <em>f</em> (<em>x</em>)</span></p>
<p><br><span>= 0     <em><strong>(A1)(G2)</strong></em></span></p>
<p><span><em><strong>[2 marks]<br></strong></em></span></p>
<div class="question_part_label">b.</div>
</div>
<div class="question" style="padding-left: 20px;">
<p><span>(0, –3)     <em><strong>(A1)</strong></em></span></p>
<p><strong><span>OR</span></strong></p>
<p><span><em>x</em> = 0, <em>y</em> = –3     <em><strong>(A1)</strong></em></span></p>
<p><span><strong>Note:</strong> Award <em><strong>(A0)</strong></em> if brackets are omitted.</span></p>
<p><span><em><strong>[1 mark]</strong></em><br></span></p>
<div class="question_part_label">c.</div>
</div>
<div class="question" style="padding-left: 20px;">
<p><span>\(f'(x) =  - {x^2} + \frac{{10}}{3}x - 1\)     <em><strong>(A1)(A1)(A1)</strong></em></span></p>
<p><span><strong>Note:</strong> Award <em><strong>(A1)</strong></em> for each correct term. Award <em><strong>(A1)(A1)(A0)</strong></em> at most if there are extra terms.</span></p>
<p><span><em><strong>[3 marks]</strong></em><br></span></p>
<div class="question_part_label">d.</div>
</div>
<div class="question" style="padding-left: 20px;">
<p><span>\(f'( - 1) =  - {( - 1)^2} + \frac{{10}}{3}( - 1) - 1\)     <em><strong>(M1)</strong></em></span></p>
<p><span>\(= -\frac{16}{3}\)     <em><strong>(AG)</strong></em></span></p>
<p><span><strong>Note:</strong> Award <em><strong>(M1)</strong></em> for substitution of <em>x</em> = –1 into correct derivative only. The final answer must be seen.</span></p>
<p><span><em><strong>[1 mark]</strong></em><br></span></p>
<div class="question_part_label">e.</div>
</div>
<div class="question" style="padding-left: 20px;">
<p><span><em>f</em> <em>'</em>(–1) gives the gradient of the tangent to the curve at the point with <em>x</em> = –1.     <em><strong>(A1)(A1)</strong></em></span></p>
<p><span><strong>Note:</strong> Award <em><strong>(A1)</strong></em> for “gradient (of curve)”, <em><strong>(A1)</strong></em> for “at the point with <em>x</em> = –1”. Accept “the instantaneous rate of change of <em>y</em>” or “the (first) derivative”.</span></p>
<p><span><em><strong>[2 marks]</strong></em><br></span></p>
<div class="question_part_label">f.</div>
</div>
<div class="question" style="padding-left: 20px;">
<p><span>\(y = - \frac{16}{3} x + c\)     <em><strong>(M1)</strong></em></span></p>
<p><span><strong>Note:</strong> Award <em><strong>(M1)</strong></em> for \(-\frac{16}{3}\) substituted in equation.</span><br><br></p>
<p><span>\(0 = - \frac{16}{3} \times (-1) + c \)</span></p>
<p><span>\(c = - \frac{16}{3}\)</span></p>
<p><span>\(y =  - \frac{{16}}{3}x - \frac{{16}}{3}\)</span><span>     <em><strong>(A1)(G2)</strong></em></span></p>
<p><span><strong>Note:</strong> Accept <em>y</em> = –5.33<em>x</em> – 5.33.</span></p>
<p><br><strong><span>OR</span></strong></p>
<p><span>\((y - 0) = \frac{{-16}}{3}(x + 1)\)     <em><strong>(M1)(A1)(G2)</strong></em></span></p>
<p><span><strong>Note:</strong> Award <em><strong>(M1)</strong></em> for</span> <span>\( - \frac{{16}}{3}\)</span><span> substituted in equation, <em><strong>(A1)</strong></em> for correct equation. Follow through from their answer to part (b). Accept <em>y </em></span><span><span>= –</span>5.33 (<em>x</em> +1). Accept equivalent equations.</span></p>
<p><span><em><strong>[2 marks]</strong></em><br></span></p>
<div class="question_part_label">g.</div>
</div>
<div class="question" style="padding-left: 20px;">
<p><span><em><strong>(A1)</strong></em><strong>(ft)</strong> for a tangent to their curve drawn.</span></p>
<p><span><em><strong>(A1)</strong></em><strong>(ft)</strong> for their tangent drawn at the point <em>x</em> = –1.     <em><strong>(A1)</strong></em><strong>(ft)</strong><em><strong>(A1)</strong></em><strong>(ft)</strong></span></p>
<p><span><strong>Note:</strong> Follow through from their graph. The tangent must be a straight line otherwise award at most <em><strong>(A0)(A1)</strong></em>.</span></p>
<p><span><em><strong>[2 marks]</strong></em><br></span></p>
<div class="question_part_label">h.</div>
</div>
<div class="question" style="padding-left: 20px;">
<p><span>(i) \(a = \frac{1}{3}\)</span>     <em><strong><span>(G1)</span></strong></em></p>
<p><em><strong><span> </span></strong></em></p>
<p><span>(ii) \(b = 3\)     <em><strong>(G1)</strong></em></span></p>
<p><span><strong>Note:</strong> If <em>a</em> and <em>b</em> are reversed award <em><strong>(A0)(A1)</strong></em>.</span></p>
<p><span> </span></p>
<p><span><em><strong>[2 marks]</strong></em><br></span></p>
<div class="question_part_label">i.</div>
</div>
<div class="question" style="padding-left: 20px;">
<p><span><em>f</em> (<em>x</em>) is increasing     <em><strong>(A1)</strong></em></span></p>
<p><span><em><strong>[1 mark]<br></strong></em></span></p>
<div class="question_part_label">j.</div>
</div>
<h2 style="margin-top: 1em">Examiners report</h2>
<div class="question" style="padding-left: 20px;">
<p><span style="font-size: medium; font-family: times new roman,times;">This question caused the most difficulty to candidates for two reasons; its content and perhaps lack of time.</span></p>
<p><span style="font-size: medium; font-family: times new roman,times;">Drawing/sketching graphs is perhaps the area of the course that results in the poorest responses. It is also the area of the course that results in the best. It is therefore the area of the course that good teaching can influence the most.</span></p>
<p><span style="font-size: medium; font-family: times new roman,times;">Candidates should:</span></p>
<ul>
<li><span style="font-size: medium; font-family: times new roman,times;">Use the correct scale and window. Label the axes.</span></li>
<li><span style="font-size: medium; font-family: times new roman,times;">Enter the formula into the GDC and use the table function to determine the points to be plotted.</span></li>
<li><span style="font-size: medium; font-family: times new roman,times;">Refer to the graph on the GDC when drawing the curve.</span></li>
<li><span style="font-size: medium; font-family: times new roman,times;">Draw a curve rather than line segments; ensure that the curve is smooth.</span></li>
<li><span style="font-size: medium; font-family: times new roman,times;">Use a pencil rather than a pen so that required changes once further information has been gathered (the turning points, for example) can be made.</span></li>
</ul>
<p><em><strong></strong></em></p>
<div class="question_part_label">a.</div>
</div>
<div class="question" style="padding-left: 20px;">
<p><span style="font-size: medium; font-family: times new roman,times;">This question caused the most difficulty to candidates for two reasons; its content and perhaps lack of time.</span></p>
<p><span style="font-size: medium; font-family: times new roman,times;">Drawing/sketching graphs is perhaps the area of the course that results in the poorest responses. It is also the area of the course that results in the best. It is therefore the area of the course that good teaching can influence the most.</span></p>
<p><span style="font-size: medium; font-family: times new roman,times;">Candidates should:</span></p>
<ul>
<li><span style="font-size: medium; font-family: times new roman,times;">Use the correct scale and window. Label the axes.</span></li>
<li><span style="font-size: medium; font-family: times new roman,times;">Enter the formula into the GDC and use the table function to determine the points to be plotted.</span></li>
<li><span style="font-size: medium; font-family: times new roman,times;">Refer to the graph on the GDC when drawing the curve.</span></li>
<li><span style="font-size: medium; font-family: times new roman,times;">Draw a curve rather than line segments; ensure that the curve is smooth.</span></li>
<li><span style="font-size: medium; font-family: times new roman,times;">Use a pencil rather than a pen so that required changes once further information has been gathered (the turning points, for example) can be made.</span></li>
</ul>
<p><span style="font-size: medium; font-family: times new roman,times;">In part (b) the answer could have been checked using the table on the GDC.</span></p>
<p>&nbsp;</p>
<div class="question_part_label">b.</div>
</div>
<div class="question" style="padding-left: 20px;">
<p><span style="font-size: medium; font-family: times new roman,times;">This question caused the most difficulty to candidates for two reasons; its content and perhaps lack of time.</span></p>
<p><span style="font-size: medium; font-family: times new roman,times;">Drawing/sketching graphs is perhaps the area of the course that results in the poorest responses. It is also the area of the course that results in the best. It is therefore the area of the course that good teaching can influence the most.</span></p>
<p><span style="font-size: medium; font-family: times new roman,times;">Candidates should:</span></p>
<ul>
<li><span style="font-size: medium; font-family: times new roman,times;">Use the correct scale and window. Label the axes.</span></li>
<li><span style="font-size: medium; font-family: times new roman,times;">Enter the formula into the GDC and use the table function to determine the points to be plotted.</span></li>
<li><span style="font-size: medium; font-family: times new roman,times;">Refer to the graph on the GDC when drawing the curve.</span></li>
<li><span style="font-size: medium; font-family: times new roman,times;">Draw a curve rather than line segments; ensure that the curve is smooth.</span></li>
<li><span style="font-size: medium; font-family: times new roman,times;">Use a pencil rather than a pen so that required changes once further information has been gathered (the turning points, for example) can be made.</span></li>
</ul>
<p><span style="font-size: medium; font-family: times new roman,times;">In part (c) <strong>coordinates</strong> were required.</span></p>
<p>&nbsp;</p>
<div class="question_part_label">c.</div>
</div>
<div class="question" style="padding-left: 20px;">
<p><span style="font-size: medium; font-family: times new roman,times;">This question caused the most difficulty to candidates for two reasons; its content and perhaps lack of time.</span></p>
<p><span style="font-size: medium; font-family: times new roman,times;">Drawing/sketching graphs is perhaps the area of the course that results in the poorest responses. It is also the area of the course that results in the best. It is therefore the area of the course that good teaching can influence the most.</span></p>
<p><span style="font-size: medium; font-family: times new roman,times;">Candidates should:</span></p>
<ul>
<li><span style="font-size: medium; font-family: times new roman,times;">Use the correct scale and window. Label the axes.</span></li>
<li><span style="font-size: medium; font-family: times new roman,times;">Enter the formula into the GDC and use the table function to determine the points to be plotted.</span></li>
<li><span style="font-size: medium; font-family: times new roman,times;">Refer to the graph on the GDC when drawing the curve.</span></li>
<li><span style="font-size: medium; font-family: times new roman,times;">Draw a curve rather than line segments; ensure that the curve is smooth.</span></li>
<li><span style="font-size: medium; font-family: times new roman,times;">Use a pencil rather than a pen so that required changes once further information has been gathered (the turning points, for example) can be made.</span></li>
</ul>
<p><span style="font-size: medium; font-family: times new roman,times;">The responses to part (d) were generally correct.</span></p>
<p>&nbsp;</p>
<p><span style="font-size: medium; font-family: times new roman,times;">&nbsp;</span></p>
<div class="question_part_label">d.</div>
</div>
<div class="question" style="padding-left: 20px;">
<p><span style="font-size: medium; font-family: times new roman,times;">This question caused the most difficulty to candidates for two reasons; its content and perhaps lack of time.</span></p>
<p><span style="font-size: medium; font-family: times new roman,times;">Drawing/sketching graphs is perhaps the area of the course that results in the poorest responses. It is also the area of the course that results in the best. It is therefore the area of the course that good teaching can influence the most.</span></p>
<p><span style="font-size: medium; font-family: times new roman,times;">Candidates should:</span></p>
<ul>
<li><span style="font-size: medium; font-family: times new roman,times;">Use the correct scale and window. Label the axes.</span></li>
<li><span style="font-size: medium; font-family: times new roman,times;">Enter the formula into the GDC and use the table function to determine the points to be plotted.</span></li>
<li><span style="font-size: medium; font-family: times new roman,times;">Refer to the graph on the GDC when drawing the curve.</span></li>
<li><span style="font-size: medium; font-family: times new roman,times;">Draw a curve rather than line segments; ensure that the curve is smooth.</span></li>
<li><span style="font-size: medium; font-family: times new roman,times;">Use a pencil rather than a pen so that required changes once further information has been gathered (the turning points, for example) can be made.</span></li>
</ul>
<p><span style="font-size: medium; font-family: times new roman,times;">The &ldquo;show that&rdquo; nature of part (e) meant that the final answer had to be stated.</span></p>
<p><span style="font-size: medium; font-family: times new roman,times;">&nbsp;</span></p>
<div class="question_part_label">e.</div>
</div>
<div class="question" style="padding-left: 20px;">
<p><span style="font-size: medium; font-family: times new roman,times;">This question caused the most difficulty to candidates for two reasons; its content and perhaps lack of time.</span></p>
<p><span style="font-size: medium; font-family: times new roman,times;">Drawing/sketching graphs is perhaps the area of the course that results in the poorest responses. It is also the area of the course that results in the best. It is therefore the area of the course that good teaching can influence the most.</span></p>
<p><span style="font-size: medium; font-family: times new roman,times;">Candidates should:</span></p>
<ul>
<li><span style="font-size: medium; font-family: times new roman,times;">Use the correct scale and window. Label the axes.</span></li>
<li><span style="font-size: medium; font-family: times new roman,times;">Enter the formula into the GDC and use the table function to determine the points to be plotted.</span></li>
<li><span style="font-size: medium; font-family: times new roman,times;">Refer to the graph on the GDC when drawing the curve.</span></li>
<li><span style="font-size: medium; font-family: times new roman,times;">Draw a curve rather than line segments; ensure that the curve is smooth.</span></li>
<li><span style="font-size: medium; font-family: times new roman,times;">Use a pencil rather than a pen so that required changes once further information has been gathered (the turning points, for example) can be made.</span></li>
</ul>
<p><span style="font-size: medium; font-family: times new roman,times;">The interpretive nature of part (f) was not understood by the majority.</span></p>
<p><span style="font-size: medium; font-family: times new roman,times;">&nbsp;</span></p>
<div class="question_part_label">f.</div>
</div>
<div class="question" style="padding-left: 20px;">
<p><span style="font-size: medium; font-family: times new roman,times;">This question caused the most difficulty to candidates for two reasons; its content and perhaps lack of time.</span></p>
<p><span style="font-size: medium; font-family: times new roman,times;">Drawing/sketching graphs is perhaps the area of the course that results in the poorest responses. It is also the area of the course that results in the best. It is therefore the area of the course that good teaching can influence the most.</span></p>
<p><span style="font-size: medium; font-family: times new roman,times;">Candidates should:</span></p>
<ul>
<li><span style="font-size: medium; font-family: times new roman,times;">Use the correct scale and window. Label the axes.</span></li>
<li><span style="font-size: medium; font-family: times new roman,times;">Enter the formula into the GDC and use the table function to determine the points to be plotted.</span></li>
<li><span style="font-size: medium; font-family: times new roman,times;">Refer to the graph on the GDC when drawing the curve.</span></li>
<li><span style="font-size: medium; font-family: times new roman,times;">Draw a curve rather than line segments; ensure that the curve is smooth.</span></li>
<li><span style="font-size: medium; font-family: times new roman,times;">Use a pencil rather than a pen so that required changes once further information has been gathered (the turning points, for example) can be made.</span></li>
</ul>
<p>&nbsp;</p>
<div class="question_part_label">g.</div>
</div>
<div class="question" style="padding-left: 20px;">
<p><span style="font-size: medium; font-family: times new roman,times;">This question caused the most difficulty to candidates for two reasons; its content and perhaps lack of time.</span></p>
<p><span style="font-size: medium; font-family: times new roman,times;">Drawing/sketching graphs is perhaps the area of the course that results in the poorest responses. It is also the area of the course that results in the best. It is therefore the area of the course that good teaching can influence the most.</span></p>
<p><span style="font-size: medium; font-family: times new roman,times;">Candidates should:</span></p>
<ul>
<li><span style="font-size: medium; font-family: times new roman,times;">Use the correct scale and window. Label the axes.</span></li>
<li><span style="font-size: medium; font-family: times new roman,times;">Enter the formula into the GDC and use the table function to determine the points to be plotted.</span></li>
<li><span style="font-size: medium; font-family: times new roman,times;">Refer to the graph on the GDC when drawing the curve.</span></li>
<li><span style="font-size: medium; font-family: times new roman,times;">Draw a curve rather than line segments; ensure that the curve is smooth.</span></li>
<li><span style="font-size: medium; font-family: times new roman,times;">Use a pencil rather than a pen so that required changes once further information has been gathered (the turning points, for example) can be made.</span></li>
</ul>
<p>&nbsp;</p>
<div class="question_part_label">h.</div>
</div>
<div class="question" style="padding-left: 20px;">
<p><span style="font-size: medium; font-family: times new roman,times;">This question caused the most difficulty to candidates for two reasons; its content and perhaps lack of time.</span></p>
<p><span style="font-size: medium; font-family: times new roman,times;">Drawing/sketching graphs is perhaps the area of the course that results in the poorest responses. It is also the area of the course that results in the best. It is therefore the area of the course that good teaching can influence the most.</span></p>
<p><span style="font-size: medium; font-family: times new roman,times;">Candidates should:</span></p>
<ul>
<li><span style="font-size: medium; font-family: times new roman,times;">Use the correct scale and window. Label the axes.</span></li>
<li><span style="font-size: medium; font-family: times new roman,times;">Enter the formula into the GDC and use the table function to determine the points to be plotted.</span></li>
<li><span style="font-size: medium; font-family: times new roman,times;">Refer to the graph on the GDC when drawing the curve.</span></li>
<li><span style="font-size: medium; font-family: times new roman,times;">Draw a curve rather than line segments; ensure that the curve is smooth.</span></li>
<li><span style="font-size: medium; font-family: times new roman,times;">Use a pencil rather than a pen so that required changes once further information has been gathered (the turning points, for example) can be made.</span></li>
</ul>
<p><span style="font-size: medium; font-family: times new roman,times;">Parts (i) and (j) had many candidates floundering; there were few good responses to these parts.</span></p>
<div class="question_part_label">i.</div>
</div>
<div class="question" style="padding-left: 20px;">
<p><span style="font-size: medium; font-family: times new roman,times;">This question caused the most difficulty to candidates for two reasons; its content and perhaps lack of time.</span></p>
<p><span style="font-size: medium; font-family: times new roman,times;">Drawing/sketching graphs is perhaps the area of the course that results in the poorest responses. It is also the area of the course that results in the best. It is therefore the area of the course that good teaching can influence the most.</span></p>
<p><span style="font-size: medium; font-family: times new roman,times;">Candidates should:</span></p>
<ul>
<li><span style="font-size: medium; font-family: times new roman,times;">Use the correct scale and window. Label the axes.</span></li>
<li><span style="font-size: medium; font-family: times new roman,times;">Enter the formula into the GDC and use the table function to determine the points to be plotted.</span></li>
<li><span style="font-size: medium; font-family: times new roman,times;">Refer to the graph on the GDC when drawing the curve.</span></li>
<li><span style="font-size: medium; font-family: times new roman,times;">Draw a curve rather than line segments; ensure that the curve is smooth.</span></li>
<li><span style="font-size: medium; font-family: times new roman,times;">Use a pencil rather than a pen so that required changes once further information has been gathered (the turning points, for example) can be made.</span></li>
</ul>
<p><span style="font-size: medium; font-family: times new roman,times;">Parts (i) and (j) had many candidates floundering; there were few good responses to these parts.</span></p>
<div class="question_part_label">j.</div>
</div>
<br><hr><br><div class="specification">
<p><span style="font-family: times new roman,times; font-size: medium;">George leaves a cup of hot coffee to cool and measures its temperature every minute. His results are shown in the table below.</span></p>
<p><span style="font-family: times new roman,times; font-size: medium;"><img src="data:image/png;base64,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" alt></span></p>
</div>

<div class="question" style="padding-left: 20px; padding-right: 20px;">
<p><span>Write down the decrease in the temperature of the coffee</span></p>
<p><span>(i) during the first minute (between <em>t</em> = 0 and <em>t</em> =1) ;</span></p>
<p><span>(ii) during the second minute;</span></p>
<p><span>(iii) during the third minute.</span></p>
<div class="marks">[3]</div>
<div class="question_part_label">a.</div>
</div>
<div class="question" style="padding-left: 20px; padding-right: 20px;">
<p><span>Assuming the pattern in the answers to part (a) continues, show that \(k = 19\).</span></p>
<div class="marks">[2]</div>
<div class="question_part_label">b.</div>
</div>
<div class="question" style="padding-left: 20px; padding-right: 20px;">
<p><span>Use the <strong>seven</strong> results in the table to draw a graph that shows how the temperature of the coffee changes during the first six minutes.</span></p>
<p><span>Use a scale of 2 cm to represent 1 minute on the horizontal axis and 1 cm to represent 10 °C on the vertical axis.</span></p>
<div class="marks">[4]</div>
<div class="question_part_label">c.</div>
</div>
<div class="question" style="padding-left: 20px; padding-right: 20px;">
<p><span><span>The function that models the change in temperature of the coffee is <em>y</em> = <em>p</em> (2<sup>−<em>t</em></sup> )+ <em>q</em>.</span></span></p>
<p><span>(i) Use the values <em>t</em> = 0 and <em>y</em> = 94 to form an equation in <em>p</em> and <em>q</em>.</span></p>
<p><span>(ii) Use the values <em>t</em> =1 and <em>y</em> = 54 to form a second equation in <em>p</em> and <em>q</em>.</span></p>
<div class="marks">[2]</div>
<div class="question_part_label">d.</div>
</div>
<div class="question" style="padding-left: 20px; padding-right: 20px;">
<p><span>Solve the equations found in part (d) to find the value of <em>p</em> and the value of <em>q</em>.</span></p>
<div class="marks">[2]</div>
<div class="question_part_label">e.</div>
</div>
<div class="question" style="padding-left: 20px; padding-right: 20px;">
<p><span>The graph of this function has a horizontal asymptote.</span></p>
<p><span>Write down the equation of this asymptote.</span></p>
<div class="marks">[2]</div>
<div class="question_part_label">f.</div>
</div>
<div class="question" style="padding-left: 20px; padding-right: 20px;">
<p><span>George decides to model the change in temperature of the coffee with a linear function using correlation and linear regression.</span></p>
<p><span>Use the <strong>seven</strong> results in the table to write down</span></p>
<p><span>(i) the correlation coefficient;</span></p>
<p><span>(ii) the equation of the regression line <em>y</em> on <em>t</em>.</span></p>
<div class="marks">[4]</div>
<div class="question_part_label">g.</div>
</div>
<div class="question" style="padding-left: 20px; padding-right: 20px;">
<p><span>Use the equation of the regression line to estimate the temperature of the coffee at <em>t</em> = 3.</span></p>
<div class="marks">[2]</div>
<div class="question_part_label">h.</div>
</div>
<div class="question" style="padding-left: 20px; padding-right: 20px;">
<p><span>Find the percentage error in this estimate of the temperature of the coffee at <em>t</em> = 3.</span></p>
<div class="marks">[2]</div>
<div class="question_part_label">i.</div>
</div>
<h2 style="margin-top: 1em">Markscheme</h2>
<div class="question" style="padding-left: 20px;">
<p><span>(i) 40</span></p>
<p><span>(ii) 20</span></p>
<p><span>(iii) 10     <em><strong>(A3)</strong></em></span></p>
<p><br><span><strong>Notes:</strong> Award <em><strong>(A0)(A1)</strong></em><strong>(ft)</strong><em><strong>(A1)</strong></em><strong>(ft)</strong> for −40, </span><span><span>−</span>20, </span><span><span>−</span>10.</span></p>
<p><span>    Award <em><strong>(A1)(A0)(A1)</strong></em><strong>(ft)</strong> for 40, 60, 70 seen.</span></p>
<p><span>    Award <em><strong>(A0)(A0)(A1)</strong></em><strong>(ft)</strong> for </span><span><span>−</span>40, </span><span><span>−</span>60, </span><span><span>−</span>70 seen.</span></p>
<div class="question_part_label">a.</div>
</div>
<div class="question" style="padding-left: 20px;">
<p><span>\(24 - k = 5\) or equivalent     <em><strong>(A1)(M1)</strong></em></span></p>
<p><br><span><strong>Note:</strong> Award <em><strong>(A1)</strong></em> for 5 seen, <em><strong>(M1)</strong></em> for difference from 24 indicated.</span></p>
<p><br><span>\(k = 19\)     <em><strong>(AG)</strong></em></span></p>
<p><br><span><strong>Note:</strong> If 19 is not seen award at most <em><strong>(A1)(M0)</strong></em>.</span></p>
<div class="question_part_label">b.</div>
</div>
<div class="question" style="padding-left: 20px;">
<p><span><img src="data:image/png;base64,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" alt><span>     <em><strong>(A1)(A1)(A1)(A1)</strong></em></span></span></p>
<p><br><span><strong>Note:</strong> Award <em><strong>(A1)</strong></em> for scales and labelled axes (<em>t</em> or “time” and <em>y</em> or “temperature”).</span></p>
<p><span>   Accept the use of <em>x</em> on the horizontal axis only if “time” is also seen as the label. </span></p>
<p><span>   Award <em><strong>(A2)</strong></em> for all seven points accurately plotted, award <em><strong>(A1)</strong></em> for <strong>5 or 6</strong> points accurately plotted, award <em><strong>(A0)</strong></em> for 4 points or fewer accurately plotted.</span></p>
<p><span>   Award <em><strong>(A1)</strong></em> for smooth curve that passes through all points on domain [0, 6]. </span></p>
<p><span>   If graph paper is not used or one or more scales is missing, award a maximum of <em><strong>(A0)(A0)(A0)(A1)</strong></em>.</span></p>
<div class="question_part_label">c.</div>
</div>
<div class="question" style="padding-left: 20px;">
<p><span>(i) \(94 = p + q\)     <em><strong>(A1)</strong></em></span></p>
<p><span>(ii) \(54 = 0.5p + q\)     <em><strong>(A1)<br><br></strong></em></span></p>
<p><span><strong>Note:</strong> The equations need not be simplified; accept, for example \(94 = p(2^{-0}) + q\).</span></p>
<div class="question_part_label">d.</div>
</div>
<div class="question" style="padding-left: 20px;">
<p><span><em>p</em> = 80, <em>q</em> = 14     <em><strong>(G1)(G1)</strong></em><strong>(ft)</strong></span></p>
<p><br><span><strong>Note:</strong> If the equations have been incorrectly simplified, follow through even if no working is shown.</span></p>
<div class="question_part_label">e.</div>
</div>
<div class="question" style="padding-left: 20px;">
<p><span><em>y</em> = 14     <em><strong>(A1)(A1)</strong></em><strong>(ft)</strong></span></p>
<p><br><span><strong>Note:</strong> Award <em><strong>(A1)</strong></em> for <em>y</em> = a constant, <em><strong>(A1)</strong></em> for their 14. Follow through from part (e) only if their <em>q</em> lies between 0 and 15.25 inclusive.</span></p>
<div class="question_part_label">f.</div>
</div>
<div class="question" style="padding-left: 20px;">
<p><span>(i) </span><span><span>–</span>0.878 (</span><span><span>–</span>0.87787...)     <em><strong>(G2)</strong></em><br></span></p>
<p><span><strong>Note:</strong> Award <em><strong>(G1)</strong></em> if –0.877 seen only. If negative sign omitted award a maximum of <em><strong>(A1)(A0)</strong></em>.<br></span></p>
<p><span> </span></p>
<p><span>(ii) y = </span><span><span>–</span>11.7<em>t</em> + 71.6 (y = </span><span><span>–</span>11.6517...<em>t</em> + 71.6336...)     <em><strong>(G1)(G1)</strong></em> <br></span></p>
<p><span><strong>Note:</strong> Award <em><strong>(G1)</strong></em> for </span><span><span>–</span>11.7<em>t</em>, <em><strong>(G1)</strong></em> for 71.6. </span></p>
<p><span>   If <em>y</em> = is omitted award at most <em><strong>(G0)(G1)</strong></em>.</span></p>
<p><span>   If the use of <em>x</em> in part (c) has <strong>not</strong> been penalized (the axis has been labelled “time”) then award at most <em><strong>(G0)(G1)</strong></em>.</span></p>
<div class="question_part_label">g.</div>
</div>
<div class="question" style="padding-left: 20px;">
<p><span>−11.6517...(3) + 71.6339...     <em><strong>(M1)</strong></em></span></p>
<p><br><span><strong>Note:</strong> Award <em><strong>(M1)</strong></em> for correct substitution in their part (g)(ii).</span></p>
<p><br><span>= 36.7 (36.6785...)     <em><strong>(A1)</strong></em><strong>(ft)</strong><em><strong>(G2)</strong></em></span></p>
<p><br><span><strong>Note:</strong> Follow through from part (g). Accept 36.5 for use of the 3sf answers from part (g).</span></p>
<div class="question_part_label">h.</div>
</div>
<div class="question" style="padding-left: 20px;">
<p><span>\(\frac{{36.6785... - 24}}{{24}} \times 100\)     <em><strong>(M1)</strong></em></span></p>
<p><br><span><strong>Note:</strong> Award <em><strong>(M1)</strong></em> for their correct substitution in percentage error formula.</span></p>
<p><br><span>= 52.8% (52.82738...)     <em><strong>(A1)</strong></em><strong>(ft)</strong><em><strong>(G2)</strong></em></span></p>
<p><br><span><strong>Note:</strong> Follow through from part (h). Accept 52.1% for use of 36.5. </span></p>
<p><span>   Accept 52.9 % for use of 36.7. If partial working (\(\times 100\) omitted) is followed by their correct answer award <em><strong>(M1)(A1)</strong></em>. If partial working is followed by an incorrect answer award <em><strong>(M0)(A0)</strong></em>. The percentage sign is not required.</span></p>
<div class="question_part_label">i.</div>
</div>
<h2 style="margin-top: 1em">Examiners report</h2>
<div class="question" style="padding-left: 20px;">
<p><span style="font-family: times new roman,times; font-size: medium;">Almost all candidates were able to score on the first parts of this question; errors occurring only when insufficient care was taken in reading what the question was asking for. The graph was usually well drawn, other than for those who have no idea what centimetres are.</span></p>
<p><span style="font-family: times new roman,times; font-size: medium;">The majority were able to determine the simultaneous equations, if only in unsimplified form; there was less success in solving these &ndash; though this is easily done via the GDC (the preferred approach) and the equation of the asymptote proved a discriminating task. The final parts, involving correlation and regression were largely independent of the previous parts and were accessible to most. Hopefully, contrasting the large percentage error with the value of the correlation coefficient will be valuable in class discussions. Given the many scripts that gave the value of the coefficient of determination as that of \(r\) , it seems better that the former is simply not taught.</span></p>
<div class="question_part_label">a.</div>
</div>
<div class="question" style="padding-left: 20px;">
<p><span style="font-family: times new roman,times; font-size: medium;">Almost all candidates were able to score on the first parts of this question; errors occurring only when insufficient care was taken in reading what the question was asking for. The graph was usually well drawn, other than for those who have no idea what centimetres are.</span></p>
<p><span style="font-family: times new roman,times; font-size: medium;">The majority were able to determine the simultaneous equations, if only in unsimplified form; there was less success in solving these &ndash; though this is easily done via the GDC (the preferred approach) and the equation of the asymptote proved a discriminating task. The final parts, involving correlation and regression were largely independent of the previous parts and were accessible to most. Hopefully, contrasting the large percentage error with the value of the correlation coefficient will be valuable in class discussions. Given the many scripts that gave the value of the coefficient of determination as that of \(r\) , it seems better that the former is simply not taught.</span></p>
<div class="question_part_label">b.</div>
</div>
<div class="question" style="padding-left: 20px;">
<p><span style="font-family: times new roman,times; font-size: medium;">Almost all candidates were able to score on the first parts of this question; errors occurring only when insufficient care was taken in reading what the question was asking for. The graph was usually well drawn, other than for those who have no idea what centimetres are.</span></p>
<p><span style="font-family: times new roman,times; font-size: medium;">The majority were able to determine the simultaneous equations, if only in unsimplified form; there was less success in solving these &ndash; though this is easily done via the GDC (the preferred approach) and the equation of the asymptote proved a discriminating task. The final parts, involving correlation and regression were largely independent of the previous parts and were accessible to most. Hopefully, contrasting the large percentage error with the value of the correlation coefficient will be valuable in class discussions. Given the many scripts that gave the value of the coefficient of determination as that of \(r\) , it seems better that the former is simply not taught.</span></p>
<div class="question_part_label">c.</div>
</div>
<div class="question" style="padding-left: 20px;">
<p><span style="font-family: times new roman,times; font-size: medium;">Almost all candidates were able to score on the first parts of this question; errors occurring only when insufficient care was taken in reading what the question was asking for. The graph was usually well drawn, other than for those who have no idea what centimetres are.</span></p>
<p><span style="font-family: times new roman,times; font-size: medium;">The majority were able to determine the simultaneous equations, if only in unsimplified form; there was less success in solving these &ndash; though this is easily done via the GDC (the preferred approach) and the equation of the asymptote proved a discriminating task. The final parts, involving correlation and regression were largely independent of the previous parts and were accessible to most. Hopefully, contrasting the large percentage error with the value of the correlation coefficient will be valuable in class discussions. Given the many scripts that gave the value of the coefficient of determination as that of \(r\) , it seems better that the former is simply not taught.</span></p>
<div class="question_part_label">d.</div>
</div>
<div class="question" style="padding-left: 20px;">
<p><span style="font-family: times new roman,times; font-size: medium;">Almost all candidates were able to score on the first parts of this question; errors occurring only when insufficient care was taken in reading what the question was asking for. The graph was usually well drawn, other than for those who have no idea what centimetres are.</span></p>
<p><span style="font-family: times new roman,times; font-size: medium;">The majority were able to determine the simultaneous equations, if only in unsimplified form; there was less success in solving these &ndash; though this is easily done via the GDC (the preferred approach) and the equation of the asymptote proved a discriminating task. The final parts, involving correlation and regression were largely independent of the previous parts and were accessible to most. Hopefully, contrasting the large percentage error with the value of the correlation coefficient will be valuable in class discussions. Given the many scripts that gave the value of the coefficient of determination as that of \(r\) , it seems better that the former is simply not taught.</span></p>
<div class="question_part_label">e.</div>
</div>
<div class="question" style="padding-left: 20px;">
<p><span style="font-family: times new roman,times; font-size: medium;">Almost all candidates were able to score on the first parts of this question; errors occurring only when insufficient care was taken in reading what the question was asking for. The graph was usually well drawn, other than for those who have no idea what centimetres are.</span></p>
<p><span style="font-family: times new roman,times; font-size: medium;">The majority were able to determine the simultaneous equations, if only in unsimplified form; there was less success in solving these &ndash; though this is easily done via the GDC (the preferred approach) and the equation of the asymptote proved a discriminating task. The final parts, involving correlation and regression were largely independent of the previous parts and were accessible to most. Hopefully, contrasting the large percentage error with the value of the correlation coefficient will be valuable in class discussions. Given the many scripts that gave the value of the coefficient of determination as that of \(r\) , it seems better that the former is simply not taught.</span></p>
<div class="question_part_label">f.</div>
</div>
<div class="question" style="padding-left: 20px;">
<p><span style="font-family: times new roman,times; font-size: medium;">Almost all candidates were able to score on the first parts of this question; errors occurring only when insufficient care was taken in reading what the question was asking for. The graph was usually well drawn, other than for those who have no idea what centimetres are.</span></p>
<p><span style="font-family: times new roman,times; font-size: medium;">The majority were able to determine the simultaneous equations, if only in unsimplified form; there was less success in solving these &ndash; though this is easily done via the GDC (the preferred approach) and the equation of the asymptote proved a discriminating task. The final parts, involving correlation and regression were largely independent of the previous parts and were accessible to most. Hopefully, contrasting the large percentage error with the value of the correlation coefficient will be valuable in class discussions. Given the many scripts that gave the value of the coefficient of determination as that of \(r\) , it seems better that the former is simply not taught.</span></p>
<div class="question_part_label">g.</div>
</div>
<div class="question" style="padding-left: 20px;">
<p><span style="font-family: times new roman,times; font-size: medium;">Almost all candidates were able to score on the first parts of this question; errors occurring only when insufficient care was taken in reading what the question was asking for. The graph was usually well drawn, other than for those who have no idea what centimetres are.</span></p>
<p><span style="font-family: times new roman,times; font-size: medium;">The majority were able to determine the simultaneous equations, if only in unsimplified form; there was less success in solving these &ndash; though this is easily done via the GDC (the preferred approach) and the equation of the asymptote proved a discriminating task. The final parts, involving correlation and regression were largely independent of the previous parts and were accessible to most. Hopefully, contrasting the large percentage error with the value of the correlation coefficient will be valuable in class discussions. Given the many scripts that gave the value of the coefficient of determination as that of \(r\) , it seems better that the former is simply not taught.</span></p>
<div class="question_part_label">h.</div>
</div>
<div class="question" style="padding-left: 20px;">
<p><span style="font-family: times new roman,times; font-size: medium;">Almost all candidates were able to score on the first parts of this question; errors occurring only when insufficient care was taken in reading what the question was asking for. The graph was usually well drawn, other than for those who have no idea what centimetres are.</span></p>
<p><span style="font-family: times new roman,times; font-size: medium;">The majority were able to determine the simultaneous equations, if only in unsimplified form; there was less success in solving these &ndash; though this is easily done via the GDC (the preferred approach) and the equation of the asymptote proved a discriminating task. The final parts, involving correlation and regression were largely independent of the previous parts and were accessible to most. Hopefully, contrasting the large percentage error with the value of the correlation coefficient will be valuable in class discussions. Given the many scripts that gave the value of the coefficient of determination as that of \(r\) , it seems better that the former is simply not taught.</span></p>
<div class="question_part_label">i.</div>
</div>
<br><hr><br><div class="specification">
<p class="p1">A biologist is studying the relationship between the number of chirps of the Snowy Tree cricket and the air temperature. He records the chirp rate, \(x\), of a cricket, and the corresponding air temperature, \(T\), in degrees Celsius.</p>
<p class="p1">The following table gives the recorded values.</p>
<p class="p1" style="text-align: center;"><img src="images/Schermafbeelding_2015-12-04_om_08.39.25.png" alt></p>
</div>

<div class="question" style="padding-left: 20px; padding-right: 20px;">
<p class="p1">Draw the scatter diagram for the above data. Use a scale of 2 cm for 20 chirps on the horizontal axis and 2 cm for 4<span class="s1"><strong>°</strong></span>C on the vertical axis.</p>
<div class="marks">[4]</div>
<div class="question_part_label">a.</div>
</div>
<div class="question" style="padding-left: 20px; padding-right: 20px;">
<p class="p1">Use your graphic display calculator to write down the Pearson’s product–moment correlation <span class="s1">coefficient, \(r\)</span>, between \(x\) and \(T\).</p>
<div class="marks">[2]</div>
<div class="question_part_label">b.</div>
</div>
<div class="question" style="padding-left: 20px; padding-right: 20px;">
<p class="p1">Interpret the relationship between \(x\) and \(T\) using your value of \(r\).</p>
<div class="marks">[2]</div>
<div class="question_part_label">c.</div>
</div>
<div class="question" style="padding-left: 20px; padding-right: 20px;">
<p class="p1">Use your graphic display calculator to write down the equation of the regression line \(T\) on \(x\). Give the equation in the form \(T = ax + b\).</p>
<div class="marks">[2]</div>
<div class="question_part_label">d.</div>
</div>
<div class="question" style="padding-left: 20px; padding-right: 20px;">
<p class="p1">Calculate the air temperature when the cricket’s chirp rate is \(70\).</p>
<div class="marks">[2]</div>
<div class="question_part_label">e.</div>
</div>
<div class="question" style="padding-left: 20px; padding-right: 20px;">
<p class="p1">Given that \(\bar x = 70\), draw the regression line \(T\) on \(x\) on your scatter diagram.</p>
<div class="marks">[2]</div>
<div class="question_part_label">f.</div>
</div>
<div class="question" style="padding-left: 20px; padding-right: 20px;">
<p class="p1">A forest ranger uses her own formula for estimating the air temperature. She counts the number of chirps in 15 seconds, \(z\), multiplies this number by \(0.45\) and then she adds \(10\).</p>
<p class="p1">Write down the formula that the forest ranger uses for estimating the temperature, \(T\).</p>
<p class="p1">Give the equation in the form \(T = mz + n\).</p>
<div class="marks">[1]</div>
<div class="question_part_label">g.</div>
</div>
<div class="question" style="padding-left: 20px; padding-right: 20px;">
<p class="p1">A cricket makes 20 chirps in <strong>15</strong> seconds.</p>
<p class="p1">For this chirp rate</p>
<p class="p1">(i) <span class="Apple-converted-space">    </span>calculate an estimate for the temperature, \(T\), <strong>using the forest ranger’s formula</strong>;</p>
<p class="p1">(ii) <span class="Apple-converted-space">    </span>determine the actual temperature recorded by the biologist, <strong>using the table above</strong>;</p>
<p class="p1">(iii) <span class="Apple-converted-space">    </span>calculate the percentage error in the forest ranger’s estimate for the temperature, compared to the actual temperature recorded by the biologist.</p>
<div class="marks">[6]</div>
<div class="question_part_label">h.</div>
</div>
<h2 style="margin-top: 1em">Markscheme</h2>
<div class="question" style="padding-left: 20px;">
<p class="p1"><img src="data:image/png;base64,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" alt></p>
<p class="p2"><strong><em><span class="Apple-converted-space">     </span>(A4)</em></strong></p>
<p class="p2"><strong>Notes:<span class="Apple-converted-space"> </span></strong>Award <strong><em>(A1) </em></strong>for correct scales and labels.</p>
<p class="p2">Award <strong><em>(A3) </em></strong>for all six points correctly plotted,</p>
<p class="p2"><strong><em>   (A2) </em></strong>for four or five points correctly plotted,</p>
<p class="p2"><strong><em>   (A1) </em></strong>for two or three points correctly plotted.</p>
<p class="p2">Award at most <strong><em>(A0)(A3) </em></strong>if axes reversed.</p>
<p class="p2">Accept tolerance for \(T\)-axis.</p>
<div class="question_part_label">a.</div>
</div>
<div class="question" style="padding-left: 20px;">
<p class="p1">\({\text{0.977}}\;\;\;{\text{(0.977324}} \ldots {\text{)}}\) <span class="Apple-converted-space">    </span><strong><em>(G2)</em></strong></p>
<p class="p1"><strong>Notes: </strong>Award <strong><em>(G1) </em></strong>for \(0.97\).</p>
<div class="question_part_label">b.</div>
</div>
<div class="question" style="padding-left: 20px;">
<p class="p1">(Very) strong positive correlation <span class="Apple-converted-space">    </span><strong><em>(A1)</em>(ft)<em>(A1)</em>(ft)</strong></p>
<p class="p1"><strong>Notes: </strong>Award <strong><em>(A1) </em></strong>for (very) strong, <strong><em>(A1) </em></strong>for positive.</p>
<p class="p1">Follow through from part (b).</p>
<div class="question_part_label">c.</div>
</div>
<div class="question" style="padding-left: 20px;">
<p class="p1">\(T = 0.129x + 6.82\) <span class="Apple-converted-space">    </span><strong><em>(G2)</em></strong></p>
<p class="p1"><strong>Notes: </strong>Award <strong><em>(G1) </em></strong>for \(0.129x\), <strong><em>(G1) </em></strong>for \( + 6.82\).</p>
<p class="p1">Award a maximum of <strong><em>(G0)(G1) </em></strong>if the answer is not an equation.</p>
<div class="question_part_label">d.</div>
</div>
<div class="question" style="padding-left: 20px;">
<p>\(0.129 \times 70 + 6.82\)     <strong><em>(M1)</em></strong></p>
<p><strong>Note: </strong>Award <strong><em>(M1) </em></strong>for substitution of 70 into their equation of regression line.</p>
<p> </p>
<p><strong>OR</strong></p>
<p>\(\frac{{8 + 12.8 +  \ldots  + 21.1}}{6}\)     <strong><em>(M1)</em></strong></p>
<p>\( = 15.9{\text{ }}(15.85)\)     <strong><em>(A1)</em>(ft)<em>(G2)</em></strong></p>
<p><strong>Note: </strong>Follow through from part (d) without working.</p>
<div class="question_part_label">e.</div>
</div>
<div class="question" style="padding-left: 20px;">
<p class="p1">regression line through \((70,{\text{ }}15.9)\) <span class="Apple-converted-space">    </span><strong><em>(A1)</em>(ft)</strong></p>
<p class="p1"><strong>Note: </strong>Accept \(15.9 \pm 0.2\).</p>
<p class="p1">Follow through from part (e).</p>
<p class="p2"> </p>
<p class="p1">with \(T\)-intercept, \(6.82\) <span class="Apple-converted-space">    </span><strong><em>(A1)</em>(ft)</strong></p>
<p class="p1"><strong>Note: </strong>Follow through from part (d). Accept \(6.82 \pm 0.2\).</p>
<p class="p1">In case the regression line is not straight (ruler not used), award <strong><em>(A0)(A1)</em>(ft) </strong>if line passes through both their \((70,{\text{ }}15.9)\) and \((0,{\text{ }}6.82)\), otherwise award <strong><em>(A0)(A0)</em></strong>.</p>
<p class="p1">Do not penalize if line does not intersect the \(T\)-axis.</p>
<div class="question_part_label">f.</div>
</div>
<div class="question" style="padding-left: 20px;">
<p class="p1">\(T = 0.45z + 10\) <span class="Apple-converted-space">    </span><strong><em>(A1)</em></strong></p>
<div class="question_part_label">g.</div>
</div>
<div class="question" style="padding-left: 20px;">
<p class="p1">(i) <span class="Apple-converted-space">    </span>\(0.45(20) + 10\) <span class="Apple-converted-space">    </span><strong><em>(M1)</em></strong></p>
<p class="p1"><strong>Note: </strong>Award <strong><em>(M1) </em></strong>for correct substitution of \(20\) into their formula from part (g).</p>
<p class="p2"> </p>
<p class="p1">\( = 19\;\;\;(^\circ {\text{C}})\) <span class="Apple-converted-space">    </span><strong><em>(A1)</em>(ft)<em>(G2)</em></strong></p>
<p class="p1"><strong>Note: </strong>Follow through from part (g).</p>
<p class="p2"> </p>
<p class="p1">(ii) <span class="Apple-converted-space">    </span>\( = 18.2\;\;\;(^\circ {\text{C}})\) <span class="Apple-converted-space">    </span><strong><em>(A1)</em></strong></p>
<p class="p1"> </p>
<p class="p1">(iii) <span class="Apple-converted-space">    </span>\(\left| {\frac{{19 - 18.2}}{{18.2}}} \right| \times 100\% \) <span class="Apple-converted-space">    </span><strong><em>(M1)(A1)</em>(ft)</strong></p>
<p class="p1"><strong>Note: </strong>Award <strong><em>(M1) </em></strong>for substitution in the percentage error formula, <strong><em>(A1) </em></strong>for correct substitution.</p>
<p class="p2"> </p>
<p class="p1">\({\text{4.40% }}\;\;\;{\text{(4.39560}} \ldots {\text{)}}\) <span class="Apple-converted-space">    </span><strong><em>(A1)</em>(ft)<em>(G2)</em></strong></p>
<p class="p1"><strong>Notes: </strong>Follow through from parts (h)(i) and (h)(ii).</p>
<div class="question_part_label">h.</div>
</div>
<h2 style="margin-top: 1em">Examiners report</h2>
<div class="question" style="padding-left: 20px;">
[N/A]
<div class="question_part_label">a.</div>
</div>
<div class="question" style="padding-left: 20px;">
[N/A]
<div class="question_part_label">b.</div>
</div>
<div class="question" style="padding-left: 20px;">
[N/A]
<div class="question_part_label">c.</div>
</div>
<div class="question" style="padding-left: 20px;">
[N/A]
<div class="question_part_label">d.</div>
</div>
<div class="question" style="padding-left: 20px;">
[N/A]
<div class="question_part_label">e.</div>
</div>
<div class="question" style="padding-left: 20px;">
[N/A]
<div class="question_part_label">f.</div>
</div>
<div class="question" style="padding-left: 20px;">
[N/A]
<div class="question_part_label">g.</div>
</div>
<div class="question" style="padding-left: 20px;">
[N/A]
<div class="question_part_label">h.</div>
</div>
<br><hr><br><div class="specification">
<p><span style="font-size: medium; font-family: times new roman,times;">Consider the function \(f(x) = 3x + \frac{{12}}{{{x^2}}},{\text{ }}x \ne 0\).</span></p>
</div>

<div class="question" style="padding-left: 20px; padding-right: 20px;">
<p><span>Differentiate \(f (x)\) with respect to \(x\).</span></p>
<div class="marks">[3]</div>
<div class="question_part_label">a.</div>
</div>
<div class="question" style="padding-left: 20px; padding-right: 20px;">
<p><span>Calculate \(f ′(x)\) when \(x = 1\).</span></p>
<div class="marks">[2]</div>
<div class="question_part_label">b.</div>
</div>
<div class="question" style="padding-left: 20px; padding-right: 20px;">
<p><span>Use your answer to part (b) to decide whether the function, \(f\) , is increasing or decreasing at \(x = 1\). Justify your answer.</span></p>
<div class="marks">[2]</div>
<div class="question_part_label">c.</div>
</div>
<div class="question" style="padding-left: 20px; padding-right: 20px;">
<p><span>Solve the equation \(f ′(x) = 0\).</span></p>
<div class="marks">[3]</div>
<div class="question_part_label">d.</div>
</div>
<div class="question" style="padding-left: 20px; padding-right: 20px;">
<p><span>The graph of <em>f</em> has a local minimum at point P. Let <em>T</em> be the tangent to the graph of <em>f</em> at P.</span></p>
<p><span>Write down the coordinates of P.</span></p>
<div class="marks">[2]</div>
<div class="question_part_label">e, i.</div>
</div>
<div class="question" style="padding-left: 20px; padding-right: 20px;">
<p><span>The graph of <em>f</em> has a local minimum at point P. Let <em>T</em> be the tangent to the graph of <em>f</em> at P.</span></p>
<p><span>Write down the gradient of <em>T</em>.</span></p>
<div class="marks">[1]</div>
<div class="question_part_label">e, ii.</div>
</div>
<div class="question" style="padding-left: 20px; padding-right: 20px;">
<p><span>The graph of <em>f</em> has a local minimum at point P. Let <em>T</em> be the tangent to the graph of <em>f</em> at P.</span></p>
<p><span>Write down the equation of <em>T</em>.</span></p>
<div class="marks">[2]</div>
<div class="question_part_label">e, iii.</div>
</div>
<div class="question" style="padding-left: 20px; padding-right: 20px;">
<p><span>Sketch the graph of the function <em>f</em>, for −3 ≤ <em>x</em> ≤ 6 and −7 ≤ <em>y</em> ≤ 15. Indicate clearly the point P and any intercepts of the curve with the axes.</span></p>
<div class="marks">[4]</div>
<div class="question_part_label">f.</div>
</div>
<div class="question" style="padding-left: 20px; padding-right: 20px;">
<p><span>On your graph draw and label the tangent <em>T</em>.<br></span></p>
<div class="marks">[2]</div>
<div class="question_part_label">g, i.</div>
</div>
<div class="question" style="padding-left: 20px; padding-right: 20px;">
<p><span><em>T</em> intersects the graph of <em>f</em> at a second point. Write down the<em> x</em>-coordinate of this point of intersection.</span></p>
<div class="marks">[1]</div>
<div class="question_part_label">g, ii.</div>
</div>
<h2 style="margin-top: 1em">Markscheme</h2>
<div class="question" style="padding-left: 20px;">
<p><span>\(f' (x) = 3 - \frac{24}{x^3}\)     <em><strong>(A1)(A1)(A1)</strong></em></span></p>
<p><strong><em> </em></strong></p>
<p><span><strong>Note:</strong> Award <em><strong>(A1)</strong></em> for 3, <em><strong>(A1)</strong></em> for –24, <em><strong>(A1)</strong></em> for <em>x</em><sup>3</sup> (or <em>x</em><sup>−3</sup>). If extra terms present award at most <em><strong>(A1)(A1)(A0)</strong></em>.</span></p>
<p><span> </span></p>
<p><em><strong><span>[3 marks]</span></strong></em></p>
<div class="question_part_label">a.</div>
</div>
<div class="question" style="padding-left: 20px;">
<p><span>\(f '(1) = -21\)     <em><strong>(M1)(A1)</strong></em><strong>(ft)</strong><em><strong>(G2)</strong></em> </span></p>
<p><strong><em> </em></strong></p>
<p><span><strong>Note: (ft)</strong> from their derivative only if working seen.</span></p>
<p><span> </span></p>
<p><em><strong><span>[2 marks]</span></strong></em></p>
<div class="question_part_label">b.</div>
</div>
<div class="question" style="padding-left: 20px;">
<p><span>Derivative (gradient, slope) is negative. Decreasing.     <em><strong>(R1)(A1)</strong></em><strong>(ft)</strong> </span></p>
<p><strong><em> </em></strong></p>
<p><span><strong>Note:</strong> Do not award <em><strong>(R0)(A1)</strong></em>.</span></p>
<p><span> </span></p>
<p><em><strong><span>[2 marks]</span></strong></em></p>
<div class="question_part_label">c.</div>
</div>
<div class="question" style="padding-left: 20px;">
<p><span>\(3 - \frac{{24}}{{{x^3}}} = 0\)     <em><strong>(M1)</strong></em></span></p>
<p><span>\(x^3 = 8\)     <em><strong>(A1)</strong></em></span></p>
<p><span>\(x = 2\)     <em><strong>(A1)</strong></em><strong>(ft)</strong><em><strong>(G2)</strong></em></span></p>
<p><span><em><strong>[3 marks]</strong></em></span></p>
<div class="question_part_label">d.</div>
</div>
<div class="question" style="padding-left: 20px;">
<p><span>(2, 9) (Accept <em>x</em> = 2, <em>y</em> = 9)     <em><strong>(A1)(A1)(G2)</strong></em></span></p>
<p><span><strong>Notes: (ft)</strong> from their answer in (d).</span></p>
<p><span>Award <em><strong>(A1)(A0)</strong></em> if brackets not included and not previously</span> <span>penalized.</span></p>
<p><em><strong><span>[2 marks]<br></span></strong></em></p>
<div class="question_part_label">e, i.</div>
</div>
<div class="question" style="padding-left: 20px;">
<p><span>0     <strong><em>(A1)</em></strong></span></p>
<p><span><strong><em>[1 mark]</em></strong></span></p>
<div class="question_part_label">e, ii.</div>
</div>
<div class="question" style="padding-left: 20px;">
<p><span><em>y</em> = 9     <strong><em>(A1)(A1)</em>(ft)<em>(G2)</em></strong></span></p>
<p><span> </span></p>
<p><span><strong>Notes:</strong> Award <strong><em>(A1)</em></strong> for<em> y</em> = constant, <strong><em>(A1)</em></strong> for 9.</span></p>
<p><span>Award <strong><em>(A1)</em>(ft)</strong> for their value of <em>y</em> in (e)(i).</span></p>
<p><span> </span></p>
<p><span><strong><em>[2 marks]</em></strong></span></p>
<div class="question_part_label">e, iii.</div>
</div>
<div class="question" style="padding-left: 20px;">
<p><span><img src="data:image/png;base64,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" alt><span>     <em><strong>(A4)</strong></em></span></span></p>
<p><span><strong><em> </em></strong></span></p>
<p><span><strong>Notes:</strong> Award <em><strong>(A1)</strong></em> for labels and some indication of scale in the stated window.</span></p>
<p><span>Award <em><strong>(A1)</strong></em> for correct general shape (curve must be smooth and must not cross the <em>y</em>-axis).</span></p>
<p><span>Award <em><strong>(A1)</strong></em> for <em>x</em>-intercept seen in roughly the correct position.</span></p>
<p><span>Award <em><strong>(A1)</strong></em> for minimum (P).</span></p>
<p><span> </span></p>
<p><em><strong><span>[4 marks]</span></strong></em></p>
<div class="question_part_label">f.</div>
</div>
<div class="question" style="padding-left: 20px;">
<p><span>Tangent drawn at P (line must be a tangent and horizontal).     <em><strong>(A1)</strong></em></span></p>
<p><span>Tangent labeled <em>T</em>.     <em><strong>(A1)</strong></em></span></p>
<p><span> </span></p>
<p><span><strong>Note: (ft)</strong> from their tangent equation only if tangent is drawn and answer is consistent with graph.</span></p>
<p><span> </span></p>
<p><span><em><strong>[2 marks]</strong></em></span></p>
<p> </p>
<div class="question_part_label">g, i.</div>
</div>
<div class="question" style="padding-left: 20px;">
<p><span><em>x</em> = −1     <strong><em>(G1)</em>(ft)</strong></span></p>
<p><span><em><strong>[1 mark]</strong></em></span></p>
<div class="question_part_label">g, ii.</div>
</div>
<h2 style="margin-top: 1em">Examiners report</h2>
<div class="question" style="padding-left: 20px;">
<p><span style="font-family: times new roman,times; font-size: medium;">Many students did not know the term &ldquo;differentiate&rdquo; and did not answer part (a).</span></p>
<div class="question_part_label">a.</div>
</div>
<div class="question" style="padding-left: 20px;">
<p><span style="font-family: times new roman,times; font-size: medium;">However, the derivative was seen in (b) when finding the gradient at <em>x</em> = 1. The negative index of the formula did cause problems for many when finding the derivative. The meaning of the derivative was not clear for a number of students.</span></p>
<div class="question_part_label">b.</div>
</div>
<div class="question" style="padding-left: 20px;">
[N/A]
<div class="question_part_label">c.</div>
</div>
<div class="question" style="padding-left: 20px;">
<p><span style="font-family: times new roman,times; font-size: medium;">Part (d) was handled well by some but many substituted <em>x</em> = 0 into <em>f</em> <em>'</em>(<em>x</em>).</span></p>
<div class="question_part_label">d.</div>
</div>
<div class="question" style="padding-left: 20px;">
<p><span style="font-family: times new roman,times; font-size: medium;">It was clear that most candidates neither knew that the tangent at a minimum is horizontal nor that its gradient is zero.</span></p>
<div class="question_part_label">e, i.</div>
</div>
<div class="question" style="padding-left: 20px;">
<p style="margin: 0.0px 0.0px 10.0px 0.0px; font: 16.0px Times; color: #3f3f3f;"><span style="font-family: 'times new roman', times; font-size: medium;">It was clear that most candidates neither knew that the tangent at a minimum is horizontal nor that its gradient is zero.</span></p>
<div class="question_part_label">e, ii.</div>
</div>
<div class="question" style="padding-left: 20px;">
<p style="margin: 0.0px 0.0px 10.0px 0.0px; font: 16.0px Times; color: #3f3f3f;"><span style="font-family: 'times new roman', times; font-size: medium;">It was clear that most candidates neither knew that the tangent at a minimum is horizontal nor that its gradient is zero.</span></p>
<div class="question_part_label">e, iii.</div>
</div>
<div class="question" style="padding-left: 20px;">
<p><span style="font-family: times new roman,times; font-size: medium;">There were good answers to the sketch though setting out axes and a scale seemed not to have had enough practise.</span></p>
<div class="question_part_label">f.</div>
</div>
<div class="question" style="padding-left: 20px;">
<p><span style="font-family: times new roman,times; font-size: medium;"><span style="font-family: times new roman,times; font-size: medium;">Those who were able to sketch the function were often able to correctly place and label the tangent and also to find the second intersection point with the graph of the function.</span></span></p>
<div class="question_part_label">g, i.</div>
</div>
<div class="question" style="padding-left: 20px;">
<p style="margin: 0.0px 0.0px 10.0px 0.0px; font: 16.0px Times; color: #3f3f3f;"><span style="font-family: 'times new roman', times; font-size: medium;">Those who were able to sketch the function were often able to correctly place and label the tangent and also to find the second intersection point with the graph of the function.</span></p>
<div class="question_part_label">g, ii.</div>
</div>
<br><hr><br>